You are on page 1of 678

State Exam 17.09.

2015
Made by: Dr. Alhag Abu Anzeh Muhammad (baghdats@yahoo.com)

Part A

1) 70 kilo man with pyloric obstruction secondary to ulcer disease admitted to


hospital for resuscitation after 1 week of prolonged vomiting.
Which metabolic disturbance is expected?

a. hypokalemic / hyperchloremic metabolic acidosis

b. hyperkalemic / hypochloremic metabolic alkalosis

c. hyperkalemic / hyperchloremic metabolic acidosis

d. hypokalemic / hypochloremic metabolic alkalosis

2) 35 years’ women experience acute onset of epigastric pain several hours


after a large dinner, she had similar episodes in past that resolved. after few
hours the episode persist, and she has fever, non-bilious vomiting.
What most likely source of the abdominal pain?

a. perforated ulcer

b. acute appendicitis

c. perforation following bowel obstruction

d. cholecystitis

e. diverticulitis
3) Which of the following vitamins is water-soluble?

a. vitamin a

b. vitamin d

c. vitamin e

d. vitamin c

e. vitamin k

4) 70 years old mans with chronic alcoholism awakens at 3:00 am with severe,
sharp, epigastric pain that 3 hours later become diffuse abdominal pain.
What is the most likely source of abdominal pain?

a. perforated ulcer

b. acute appendicitis

c. perforation following bowel obstruction

d. cholecystitis

e. diverticulitis

5) 55 years old man with 2-day history of abdominal distention, vomiting,


crampy abdominal pain, constipation is now experiencing severe diffuse
abdominal pain. What is the most likely source of abdominal pain?

a. perforated ulcer

b. acute appendicitis

c. perforation following bowel obstruction

d. cholecystitis

e. diverticulitis
6) Patient with known diverticular disease of colon has 5-day history of
worsening pain in left quadrant, he now has fever and diarrhea this morning,
on examination he is found to have fullness in left lower quadrant with
guarding. What best diagnosis in his condition?

a. Diagnostic Laparoscopy

b. Immediate operative Exploration

c. Air-Contrast Enema

d. Colonoscopy

e. CT scan of the abdomen and pelvis

7) Head and face of a 45 years old man were assaulted with fists and feet. he
arrives at emergency department with incomprehensible response to
questions. eye opening to pain only and a flexor response to pain.
What is the patient Glasgow's Coma Scale score?

a. 5

b. 6

c. 7

d. 8
8) A 32 years old woman is restrained passenger at high speed motor vehicle
collision. In the emergency room she is found to be non tachycardic with a
systolic blood pressure 100 mmhg. Hemoglobin level 12.2 on fast. A moderate
amount of fluid as seen in the right upper quadrant between the liver, kidney,
and diaphragm. What is the most appropriate step in management?

a. Laparotomy

b. CT scan of abdomen and pelvis with intravenous contrast enhancement.

c. Angiography for embolization of the liver laceration

d. Observation with serial abdominal examination

9) All of the following are components of Hasselbach's triangle, EXCEPT:

a. Femoral vein

b. Medial border of the rectus sheath

c. Cooper ligament

d. Inguinal ligament

e. Inferior epigastric vessels

10) Which of the following hernias represent incarceration of a limited


portion of the small bowel wall?

a. Spregelian hernia

b. Grynfeltt hernia

c. Petit hernia

d. Richter hernia

e. Littre hernia
11) Which of the following is not a germline mutation associated with a
higher incidence of breast cancer?

A. APC

B. BRCA1

C. BRCA2

D. P53

E. PTEN

12) In which patient should MRI be used as an adjacent to mammography for


breast cancer?

A. A 27 years old women in whose mother’s breast cancer was diagnosed at age 52

B. A 52 years old women with dense breast

C. A 72 years old women with history of DCIS

D. A 31 years old whose sister arrives the BRCA mutation but has a declined
genetic testing for herself

E. A 55 who received radiation treatment at age 50 for urine cancer

13) Calcitonin is produced by parafollicular cells of the thyroid gland. In what


disease is measurement of calcitonin essential?

A. Graves’ disease

B. Follicular thyroid cancer

C. Hashimoto disease

D. Medullary thyroid cancer

E. Papillary thyroid cancer


14) A 42 years old women complain of symptoms associated with
hyperthyroidism. On examination she has palpable nodule but without
exophthalmos, she doesn’t have pretibial myxedema, her laboratory work up
reveals a suppression TSH level with elevation of free T3.
What is the next step in management of this patient?

A. Radioactive 123 I uptake scan

B. Neck ultrasound

C. PTU

D. FNA

E. Observation

15) During an operation for presumed appendicitis, the appendicitis found to


be normal, the terminal ileum however is markedly thickened palpable
rubbery to firm, its serosa is erythematous and inflamed and several loops of
apparently normal small intestine are adherent to it. the terminal ileum
mesentery is thickened with fat growing about the bowl circumference.
Which of the following is the most likely the diagnosis?

A. Crohn disease of terminal ileum

B. Perforated Meckel’s diverticulum

C. Ulcerative colitis

D. Ileocecal tuberculosis

E. Acute ileitis
16) Which of the following is common reason for massive colonic bleeding?

A. Cancer

B. Ulcerative colitis

C. Diverticulosis

D. Diverticulitis

E. Granulomatous colitis

17) Hepatocellular carcinoma epedimologigally associated with of the


following EXCEPT?

a. Hepatitis A virus infection

b. Hepatitis B virus infection

c. Hepatitis c virus infection

d. Wilson disease

e. Alcoholic cirrhosis

18) Ultrasound imagining reveals gallstones for asymptomatic 50 years old


women which of the following is the recommended treatment?

A. Observation

B. Laparoscopic cholecystectomy

C. Open cholecystectomy

D. Ursodeoxycholic acid

E. Extracorporeal shock wave lithotripsy (ESWL)


19) What is preferral treatment of acute mild calculus cholecystitis?

A. Laparoscopic cholecystectomy within 5 days of admission

B. Laparoscopic cholecystectomy in 6 week

C. Open cholecystectomy within 5 days of admission

D. Open cholecystectomy in 6 week

E. There is no indication for cholecystectomy

20) A 24 years old women pregnant is hospitalized with gallstone pancreatitis


and recover after 2 days of non-operative management.
Which of the following is recommendation most appropriate in the
management of this patient?

A. Laparoscopic cholecystectomy before discharge

B. Open cholecystectomy before discharge

C. Laparoscopic cholecystectomy in 4 weeks

D. Open cholecystectomy in 4 weeks

E. Non operative management until term and postpartum laparoscopic


cholecystectomy

21) Which of the following organisms is most commonly isolated from bile?

a. Escherichia coli

b. Clostridium spp

c. Bactroides fragilis

d. Pseudomonas spp

e. Enterococcus spp.
22) A patient undergoes laparoscopic Roux-en- Y gastric bypass. On
postoperative day 5 she is tachycardic, with a pulse of 130 beats/min,
tachypneic, and oliguric.
What clinical scenario are you most concerned about?

a. Hypovolemia

b. Postoperative bleeding

c. Pulmonary embolism

d. Anastomotic leak

e. Inadequate pain control

23) A 49-year-old man was the restrained drive in a motor vehicle collision.
He decelerated rapidly in order to avoid hitting another car and swerved into
a ditch. He complains of chest pain.
Which of the following findings on chest x-ray would be most suspicious for
an aortic injury?

a. Multiple-right sided rib fractures

b. A left pulmonary contusion

c. A left pneumothorax

d. Widening of the mediastinum greater than 8 cm

e. Pneumomediastinum
24) A 58 –year old man presents to the ER after falling 3m from a ladder.
Examination reveals stable vital signs, no evidence of respiratory distress, and
multiple right-sided rib fractures. Chest x-ray shows a hemothorax on the
right side and a right tube thoracostomy is performed. Approximately 700 mL
of blood is immediately drained. Over the next 4 hours he continues to drain
300ml/h after the original evacuation.
Which of the following is the definitive treatment of this patient?

a. Platelets

b. Fresh-frozen plasma

c. Second tube thoracostomy

d. Thoracotomy

e. Observation

25) An 80-year old is admitted to the hospital complaining of nausea.


Abdominal pain, distention, and diarrhea. A cautiously performed transanal
contrast study reveals an apple-core configuration in the recto-sigmoid area.
Which of the following is the most appropriate next step in his management?

a. Colonoscopic decompression and rectal tube placement

b. Saline enemas and digital disimpaction of fecal matter from the rectum

c. Colon resection and proximal colostomy

d. Oral administration of metronidazole and checking a Clostridium difficile titer

e. Evaluation of an ECG and obtaining an angiogram to evaluate for colonic


mesenteric ischemia
26) A 46-year-old woman who was recently diagnosed with crohns disease
asks about the need for surgery. Which of the following findings would be an
indication for immediate exploratory laparoscopy?

a. Intestinal obstruction

b. Enterovesical fistula

c. ileum-ascending colon fistula

d. Enterovaginal fistula

e. Free perforation

27) A 35-year-old man presents with right upper quadrant pain, fever,
jaundice, and shaking chills. Ultrasound of the abdomen demonstrates
gallstones, normal gallbladder wall thickness, and common bile duct of 1.0cm.
The patient is admitted to the hospital and given IV fluids and antibiotics. He
continues to be febrile with increasing WBCs.
Which of the following is the most appropriate next step in the patient’s
management?

a. ERCP

b. Placement of a cholecystectomy tube

c. Laparoscopic cholecystectomy

d. Open cholecystectomy

e. Emergent operation and decompression of the common bile duct with a T tube
28) A 61-year-old woman with a history of unstable angina complains of
hematemesis after retching and vomiting following a night of binge drinking.
Endoscopy reveals a longitudinal mucosal tear at the gastroesophageal
junction, which is not actively bleeding.
Which of the following is the next recommended step in the management of
this patient?

a. Angiography with embolization

b. Balloon Tamponade

c. Exploratory laparotomy, gastrostomy, and oversewing of the tear

d. Systemic vasopressin infusion

e. Expectant management

29) A 52-year-old woman is referred to you with a 2-cm mass in the region of
the axilla and a normal breast exam. Her family history reveals a paternal
aunt with breast cancer diagnosed at age 34.
Which of the following is the most likely etiology of the mass?

a. Lymphoma

b. Melanoma

c. Breast cancer

d. Gastric carcinoma

e. Reactive lymphadenopathy
30) A 44-year-old premenopausal woman has right upper-quadrant breast
calcifications noted on her yearly screening mammogram. She has no family
history of breast cancer. She had her menarche at 14 years and delivered her
first child at age 26 years. She has had no prior breast biopsies. Stereotactic
core needle biopsy reveals lobular carcinoma in situ.
Which of the following is the next most appropriate step?

a. Needle-localized excisional biopsy

b. prophylactic bilateral mastectomy

c. Raloxifene treatment for 5 years

d. “mirror image” biopsy of the left breast

e. Annual mammography

31) 55 years old women presenting with erythema of the right breast treated 2
weeks with antibiotic, is erythema still present. Mammogram reveal diffusely
increased density, ultrasonography reveals no fluid compartment.
What is the most appropriate next step in management?

a. Antibiotic for 2 more weeks

b. Obtain an MRI

c. Change to border spectrum antibiotic

d. Perform right breast core biopsy

e. Incision and drainage of the right breast


32) 84 years old nursing home resident admitted to ED with acute abdominal
pain nausea and vomiting, his heart rate 90/min, blood pressure 130/70,
distended, tympanic abdomen without peritonitis. WBC 11,000, radiograph
showed in the image (picture). What is the best management of this patient?

a. Exploratory laparoscopy

b. Sigmoidoscopy

c. Cecostomy

d. Neostigmine

e. Water soluble contrast enema

33) 34 years old women is treated 4 months with steroids for ITP, platelet
count 40,000 and she is asymptomatic. What is the most appropriate therapy?

a. Rituximab

b. I.V Ig

c Splenectomy

d. Observation
34) 36 years old man undergo abdominal CT after M.V.C no injuries are
found, blood pressure 160/100. 3 cm adrenal mass is found in the CT.
Appropriate initial biochemistry evaluation should include all of the following
EXCEPT?

a. plasma metanephrines

b. plasma aldosterone level

c. low dose overnight dexamethasone suppression test

d. plasma renin level

e. serum adrenocorticotropic hormone level

35) 33 years old man had total thyroidectomy at age 20 for medullary thyroid
cancer. Father and sister have both treated for medullary thyroid cancer. The
father died in a hypertensive crisis and the sister during biopsy of the adrenal.
Patient CT-scan for flank pain (picture).
Which of the following is the most likely diagnosis?

a. Familiarly medullary thyroid carcinoma

b. Conn syndrome

c. Cushing disease

d. M.E.N 2a

e. Metastasis medullary cancer of the thyroid


36) 45 years old woman with history of hypertension undergoes CT of
abdomen and pelvis after presenting to the ER with RLQ pain the study is
negative except for an incidentally found 3cm mass in right adrenal evaluation
reveal elevated urine metanephrine the lesion is located in the?

a. Zona glumerulasa

b. Zona fasciculata

c. Zona reticulata

d. Medulla

e. Pera adrenal tissues

37) 23 years old women has 2.0 cm right thyroid nodule, she is symptomatic.
Her thyroid-stimulating-hormone levels is normal, fine needle aspiration show
papillary thyroid cancer, at the of surgery a 5 mm black lymph node is
detected in the right paratracheal area. frozen section of lymph node confirm
metastatic papillary thyroid cancer.
What is the most appropriate surgical management of this patient?

a. Total thyroidectomy

b. Right thyroid lobectomy with ipsilateral paratracheal node dissection

c. Right thyroidectomy with central neck dissection

d. Total thyroidectomy with right modified radical neck dissection

e. Right thyroidectomy
38) Which of the following the appropriate method to evaluate cervical spine
in an alert unintoxicated neurologically normal patient without distracting
injuries after a motor vehicle crash?

a. If the patient denies neck pain nothing more is needed

b. Clinical exam to assess midline tenderness and range of motion

c. View spine radiographs

d. CT scan of cervical spine

e. MRI

39) An emaciated hypovolemic 68 years old man with partially obstructive


midesophageal adenocarcinoma is resuscitated with 0,9% NaCl and then
started an 20% dextrose based TPN at Kal/kg/day.
Which of following condition can be anticipated in the first 24h?

a. Hypophosphatemia

b. Hyperkalemia

c. Hypermagnesemia

d. Hypocalcemia

e. Sodium wasting
40) 50 years old woman is hospitalized with an acute MI. she develops acute
cholecystitis with a T 39,2c WBC 25,000 bilirubin 1, U/S demonstrate 10cm
gallbladder pericholecystic fluid and 3cm wall thickening with gallstones.
In addition to antibiotics which next step?

a. Percutaneous drainage of the gallbladder

b. ERCP and stenting

c. Laparoscopic cholecystectomy

d. Open cholecystectomy

41) A 22 years old women undergoes uncomplicated laparoscopic


appendectomy for perforated appendicitis. The final pathology reveals
carcinoid tumor of the tip of the appendix measuring 3 cm.
What is the appropriate next step?

a. Observation

b. Chemotherapy

c. Cecectomy

d. Right hemicolectomy

e. Radiation
42. 32 years old man is referred to the ER due to calcium level of 16 mg/dl.
(NL 8,5-10,8). Evaluation reveals a PTH level of 380 ng/l (NL 15-65) and 24
hours’ urinary calcium of 350 mg (NL 100-250).
What is the first treatment to be administered to this patient?

a. Normal saline

b. Bisphosphonates

c. Steroids

d. Calcitonin

e. Magnesium

43) 35 years old women complains of muscle weakness, her blood pressure is
160/85 mmHg, heart rate 95. She is obese with hirsutism and abdominal
striae. What is the first diagnostic test?

A. Serum ACTH

B. MRI for pituitary evaluation

C. Adrenal CT

D. Somatostatin receptor scan


44) A 52 years old women complains of headaches, palpitations, and excessive
sweating. Her blood pressure is 182/91 mmHg, her Heart rate of 110 with no
other findings. 24 hours urinary and plasma metanephrines are elevated.
Abdominal CT scan demonstrates a 5-cm right adrenal mass. She is scheduled
for a laparoscopic right adrenalectomy.
What medical therapy should be initiated in this patient?

a. Atenolol

b. Prednisone

c. Spironolactone

d. Furosemide

e. Phenoxybenzamine

45) A 43 years old women that is usually healthy is referred to ER


complaining of right upper quadrant pain, nausea, with no vomiting, no fever,
no diarrhea, and no urinary complaints. The pain started the night before her
admission following an especially large meal. On examination her abdomen is
soft with right upper quadrant tenderness. WBC is increased with normal
liver function tests.
What is the next diagnostic test?

a. CT scan of the abdomen and pelvis

b. Endoscopic ultrasound

c. Diagnostic laparoscopy

d. HIDA Scan
46) 43 years old patient with hepatic cirrhosis second to HCV, was vomiting,
stable respiratory, a nasogastric tube is inserted and drain 200 treated with
medical therapy, present in the ER with bloody ml of blood.
What is the appropriate treatment?

a. Insertion of a black tube

b. Endoscopy by administration of glypressin

c. Administration of glypressin and I.V PPI

d. TIPS

e. Urgent surgery

47) 31 years old patient, present for a breast exam after her sister was
diagnosed with breast cancer at age 35, her aunt had breast cancer at 40
years old, the patient has a normal physical exam with no palpable mass,
What is the next step?

a. breast and axilla ultrasound

b. follow up in 6 month

c. prophelactic bilateral mastectomy

d. tamoxifine prophylactic

e. genetic council
51) A 22-year old couple is married and having unprotected sex for a year.
The couple comes for infertility workup and is referred for a post coital test.
Which of the following is true regarding this test?

A. it is a worthy substitute for a spermatogram if the couple is younger


than 30 years old.

B. it is performed by testing the cervical mucus 10 hours post coitus.

C. it is normal if 5 spermocytes are visualized in a high power field.

D. it`s value in infertility workup for this couple is limited and controversial.

E. it is performed immediately after ovulation.

52) A 35-year-old couple wishes to conceive. The husband works away from
home and does not return home every day. They wish to follow ovulation
carefully in order to have sex and conceive.
Which of the following tests is the most exact in timing ovulation?

A. serum progesterone level over 3 ng/ml.

B. rising of 1.5 degrees in body temperature.

C. presence of fluid in the pouch of Douglas in a sonogram.

D. viscous and turbid cervical mucus.

E. a rise in serum or urine LH.


53) Results of an initial infertility workup for a 32-year-old couple
demonstrates: the woman has a menstrual cycle of 29 days a progesterone
level of 5 mg/ml on day 23 of the cycle and normal uterine cavity and contrast
material passage to the pelvis on a hysterogram. sperm analysis reveals a
sperm count of 25 million cells /ml with 55% motility and 15% normal kruger
morphology. What is the most likely diagnosis for infertility in this case?

A. infertility due to luteal insufficiency.

B. infertility due to male factor.

C. infertility due to a mechanical factor.

D. unexplained infertility.

E. infertility due to a combination of an ovulation disorder and the woman’s age.

54) A single 35-year-old woman wishes to evaluate her ovarian reserve in


order to plan a pregnancy.
Which of the following results would indicate a recommendation for an earlier
pregnancy due to a decreased ovarian reserve?

A. A luteal body demonstrated on ultrasound on the 22 day of the menstrual cycle.

B. A menstrual cycle of 26 days.

C. Increased estradiol and FSH on day 3 of the menstrual cycle.

D. LH level that in higher then FSH level on day 14 of the menstrual cycle.

E. A progesterone level of 3 -6 mg/ml on day 22 of the menstrual cycle.


55) A 28 year -old woman with an irregular menstrual cycle is treated with
clomiphene citrate of induction of ovulation.
Which of the following may develop due to this treatment?

A. 25% chance of a twin pregnancy and 50% chance of a triplet or more


pregnancy.

B. 8%chance of a twin pregnancy, blurred vision, and emotional instability.

C. 80% chance of evolution a dosage of 50 mg

D. 75% chance of conception with a dosage of 100 mg.

E. a multiple pregnancy will develop only with addition of HCG.

56) A 12 year ago girl present with her mother that complains of not having
menstruation yet. The mother says all the women in her family had
menstruation at age of 10 years, and she is therefore worried about this delay.
Detailed history reveals a healthy asymptomatic girl. Exam shows a girl with a
normal size for her age, primary pubic hair, and breast buds.
Which of the following is the most common diagnosis and what does it mean?

a. it is a case of primary amenorrhea and it warrants workup.

b. it is a case of primary amenorrhea, but the workup can be delayed for 6 months.

c. this is not amenorrhea and there is no need for further workup

d. the secondary sex signs are due to external estrogen, and the possibility of
exposure should be investigated thoroughly.

e. pelvis imaging to rule out mullerian agenesis should be performed.


57) which of the following is the most common etiology for hirsutism in
women?

a. polycystic ovary syndrome

b. 21-hydroxylase deficiency

c. idiopathic hirsutisms

d. hypothyroidism

e. hyperprolactinemia

58) which of the following is the most appropriate treatment for a 38 y.o.
woman with endometriosis who wishes to conceive?

a. GnRH agonist for 6 months.

b. Combined oral contraceptives for 12 months.

c. Laparoscopic ablation and adhesion lysis.

d) Danazol for 6 months.

59) which of the following is true regarding intracytoplasmic sperm injection


(ICSI)?

a. ICSI is impossible to perform with defrosted sperm.

b. Performed only after testicular biopsy or epididymis.

c. Performed only if there is no sperm mobility.

d. Performed in case of infertility due to male factor.

e. Fertilization rate is lower compared to IVF.


60) Which of the following is true regarding in vitro fertilization (IVF)?

A. The most important prognostic factor is the women’s age

B. Fertilization and ovum division rate is about 50%

C. The rate of conception in IVF is not higher than in natural conception.

D. Implanting more than one embryo in IVF does not increase pregnancy rates

E. IVF is the only option for a woman over the age of 40

61) A 38-year-old women uses DMPA for birth control for the last 5 years.
She is worried about continuing using the drug.
Which of the following is associated with or caused by DMPA?

a. reduction of bone density by 15%

b. transient amenorrhea, lasting a maximum of one month after ceasing the drug.

c. increased rate (about 5%) of ectopic pregnancies.

d. after cessation of the drug returning to basic fertility is not immediate

62) Which of the following descriptions of a non-stress test define normal


result?

a. One 15bpm acceleration over a 20 minute period.

b. 30 minute monitoring with no accelerations or decelerations.

c. Two accelerations of 15 bpm lasting at least 15 seconds over a 20-minute period.

d. Decelerations that do not exceed 15 bpm for 15 seconds.

e. A minimum of one acceleration of at least 30 bpm lasting 30 seconds over a


20-minute period.
63) A 32-year-old primipara women is beginning active labor. Her physical
examination shows dilation of 6 cm and the fetus head is at the ischial spines.
Fetal heart monitors monitor shows a basic pulse of 140 bpm with decreased
variability and a sinusoidal pattern on monitoring.
Which fetal heart beat category is described in this question?

a. category 1, since the basic pulse is 140 bpm

b. category 2, since there is a sinusoidal pattern

c. category 3, due to the variability and the sinusoidal pattern.

d. category 4, due to lack of decelerations.

e. category 5, due to the decreased variability in fetal pulse.

64) A woman in her 3rd pregnancy and 29 weeks’ postmenstrual age is


admitted due to premature contractions starting early morning.
Vaginal examination shows she is dilated to 2 cm with 80% effacement.
Which of the following treatments would have the best effect on the baby if it
were to be born over the following week?

a. Treatment with antibiotics

b. Treatment with calcium blockers

c. Treatment with magnesium sulfate.

d. IV fluids and pethidine.

e. Treatment with corticosteroids.


65) A 39-year-old women in her 3rd pregnancy. She is in her 24th week and
her blood type is B (-) . In her previous pregnancy she has delivered a baby
with hydrops fetalis.
Which of the following is the best follow up method for fetal anemia?

a. measuring fetal brain blood flow is the most sensitive method.

b. she should be administrated anti D in her 28th week.

c. bilirubin levels in the amniotic fluid should be checked weekly starting from the
24th week.

d. measuring blood flow in the uterine arteries is the most sensitive method.

e. biophysical profile is the most sensitive method.

66) a mother of 3 years old child is on her 30th week of the pregnancy is seen
for decreased fetal movement, her 3 years old had afebrile illness 3w ago
followed by a rash over the thorax and cheeks (slapped check)
The mother was asymptomatic’ ultrasound reveled an accumulation of fluid
in the fetal peritoneal and pleural cavities.
Which of the following is the most likely infectious agent causing the fetus
condition?

a. Cytomegalovirus.

b. Parvoviruses.

c. Toxoplasma.

d. Rubella.

e. Varicella.
67) Which of the following environmental or genetic disorder may be
associated with macrosomic fetus?

a. Patau syndrome

b. Turner syndrome

c. Fetal alcohol syndrome

d. Congenital rubella infection

e. Beckwith wiedman syndrome

68) Which of the following is the most correct regarding twin pregnancies?

a. complication rate is higher in monochorionic twins vs bichorionic twins

b. the chance of twin pregnancies increases by 30% after treatment with


clomiphene citrate

c. the rate of twin pregnancies has decreased over the last decades due to
recommendation for fetal reduction

d. if one of the twin is in breech presentation the both should by delivered by


cesarean section
69) A woman on her 3rd pregnancy 34+4 weeks PMA with A- blood type is
admitted due to moderate virginal bleeding, she is hemodynamic. Stable she
feel fetal movement well the bleeding was not post coital.
Which of the following is the first and most important step in the diagnosis
and management of this woman?

a. Anti D imnuongloplin.

b. Vaginal examination for placental planting.

c. Biophysical profile of the fetus by ultrasound.

d. Ultrasound for determining placental position.

e. Corticosteroid for fetal lung maturation.

70) Which of the following is correct regarding prolapse of cord during labor?

a. this condition is rare in preterm vs term

b. the cord should be returned into the uterus immediately after the diagnosis of
prolapse is made

c. more prevalent in breech presentation vs transverse

d. perinatal mortality reaches 20%


71) A woman is on her 40 week pma of her second pregnancy. The fetus is at
spin +2 and she is dilated to 10 cm. Fetal monitor reads decreased variability
and late deceleration. Which of the following is true recording the timing and
method of delivery?

A. an instrumental delivery is contraindication

B. this monimonitor potion pattern warrant delivery within 60 minutes

C. instrumental delivery is possible after a normal fetal platelet count

D. vacuum delivery is possibility on for is an option for complication with forms if


it fails

E. the fetus shod be delivered as soon as passible

72) A primipara woman on her 38th week PMA present to the delivery room
with headache and epigastric pain.
Which of the following should be first step?

a. CBC and liver function tests

b. Measuring blood pressure

c. Urine protein test

d. Non stress test for fetus

e. Sending blood for type and cross


73) A woman on her 38 week PMA com of headache. She reports she fell fetal
movement blood pressure 145/110 and urine protein stick show +3.
What would you do next?

A. Labor indication withe treatment with magnesium sulfate

B. Labetalol for blood pressure management and stabilize delivery no late than 40
week PMA

C. Emergent caesarian section

D. Treat wit iv sodium nitroprusside and delivery as soon as possible

E. Magnesium with aclever

74) A 14-year-old girl one year after her first menarche, present with vaginal
bleeding during menarche. She is hemodynamic stable and lab results include
a hemoglobin level of 10.2 and normal coagulation studies ultrasound thin
endometrium recommend accord of patient?

A. Order estrogen preparation

B. A high dose of progestational agent

C. A preparation including vitamin k for hence the intrinsic coagulation Pathway

D. FFP to supplementation of coagulate feats

E. There is no room for intervention in this patient


75) A 49-year-old complains of post coital vaginal bleeding for the last 6
months. On examination the cervix appears normal and the IUD string ms in
place. The IUD was place at age of 46.
How would you progress in diagnosis and treatment?

A. These are symptoms of menopause and hence there is no need for intervention

B. A cervix tumor should be violet out in colposcopy or biopsy

C. This is a complication of IUD device and it should be removed

D. The patient is suffering from Adenomyosis with cervical involvement and there
is no room for intervention unless there are additional symptoms

E. A PAP smear should be performed and if it is normal follow up in one year

76) 25 years old woman present with fever up to 39°C, left lower abdominal
pain, nausea and general weakness, on examination she appears ill,
temperature is 39°C, her pulse is 120 and blood pressure is 110/70. Her left
lower abdominal pain is tender with rebound. On vaginal examination there is
tenderness in mobilization of the cervix and uterus. A left extremely tender
6 cm mass is palpated on the left, laboratory result reveal WBC of 17000,
elevated ESR, CRP 32, and a normal urine sample. Ultrasound exam shoes a
heterogeneous mass in the left adnexa with increase blood flow and no pelvic
fluid. What is the best course for diagnosis and treatment?

A. There as high suspicion for torsion and the patient should be prepared for
laparoscopy

B. The diagnosis is tubo-ovarian abscess and the patient should be operated


immediately

C. Surgery should be offered due to suspicion of malignancy

D. The initial recommended management include admission IV fluids, and wide


spectrum antibiotics

E. The infection is most commonly viral and supportive cure should be offered
77) Which of the following is true regarding HIV infection?

A. Breast-feeding is recommended for HIV carrier women world-wide

B. The presence of genital Condyloma increase the risk of infection

C. There is no radical difference in HIV rates in USA

D. These days, HIV infection in women is more associated with IV drugs use than
exposure during sexual intercourse

E. Men and women have the same risk for infection in heterosexual relationships

78) Which of the following is true regarding bacterial vaginosis?

A. It is associated with increased risk for preterm labor

B. Penicillin is the treatment of choice

C. Warrants treatment of the sexual partner

D. Vaginal lavages reduce the infection rate

E. Diagnosed by serologic testing

79) Which of the following warrants treatment of the sexual partner?

A. Candida albicans

B. Bacterial vaginosis

C. Candida Glabrata

D. N-Gonorrhea

E. HSV
80) Which of the following is true regarding endometrial myoma during
pregnancy?

A. It is hard to establish the diagnosis by ultrasound and therefore MRI is required

B. Immediate surgery is indicated in case of torsion or degeneration of the myoma

C. Increases the risk for cesarean section and placenta previa

D. Most pregnant women with myoma experience pain

E. More prevalent in Caucasians in comparison to African American women

81) Which of the following is true regarding hysteroscopy using bipolar


energy?

A. normal saline should not be used as a liquid medium when using bipolar energy

B. cervical cancer is a complete contraindication for hysteroscopy

C. perforation of the uterus is a common complication

D. hysteroscopy is a typical complication of glycine dosage

E. while performing a surgical hysteroscopy it is important to mention the fluid


pressure higher the mean arterial pressure (100 mmHg)
82) A 30-year-old woman complains of years of severe dysmenorrhea and
dyspareunia follow by lower abdominal pain. She has been unsuccessfully
trying to conceive for the last 2 years. Ultrasound is normal.
Which of following is the most likely diagnosis?

a. Ovarian cancer

b. Cervix cancer

c. Endometriosis

d. Fallopian tube cancer

e. Psychopathic disorder

83) Which anatomical features compose the pelvic floor?

A. pelvic bones, endopelvic fascia, levator ani

B. perianal muscles, vaginal wall, uterine cervix

C. coccyx, vaginal wall, perianal body

D. endopelvic fascia, internal anal sphincter, external anal sphincter

E. perianal muscles, urethra, vaginal wall

84) 82year diabetic woman with diabetic renal failure and an ischemic heart
disease, present with pelvic organ prolapse (POP) grade 3 that affects her
quality of life. What is the most appropriate initial management?

A. anterior colporophy surgery

B. hysterectomy in a vaginal approach

C. hysterectomy in an abdominal approach

D. combined surgery with anterior and posterior repair

E. local estrogen treatment and Pessary adjustment


85) Which of the following is used as a reference point for grading pelvic
organ prolapse?

A. pelvic muscles

B. the bladder neck

C. the hymen

D. labia major

E. the external cervical os

86) Which of the following would be expected in a 6 month old with a large
ventricular septal defect?

A. Cyanosis

B. AN enlarged heart on chest roentgenogram

C. A continuous cardiac murmur

D. Decreased pulmonary vasculature on roentgenogram

E. Right ventricular hypertrophy on ECG

87) A 12-year-old girl complains of decreasing visual acuity and a slight


feeling of discomfort on both eyes. During examination reveals anterior
uveitis. You suspect that the child may have which of the following?

A. Leukemia

B. Toxoplasmosis

C. Toxocara infection

D. Hypoparathyroidism

E. Juvenile idiopathic arthritis


88) It is recommended that young infants should sleep in the supine rather
than in the prone position. This is based on data suggesting that the prone
position is associated with an increased incidence of which of the following?

A. Delayed eruption of the first deciduous teeth

B. Gastroesophageal reflux and aspiration

C. Micrognathia

D. Strabismus

E. Sudden infant death

89) A full term newborn has episode of cyanosis and apnea which is worse
when attempted to feeding and become will when is cries.
Which of the most important next step quickly establish the diagnosis?

A. Echocardiogram

B. Ventilation perfusion scan

C. Passage of catheter into nose

D. Hemoglobin electrophoresis

E. Bronchoscopy evaluation of palate and larynx

90) Which of the following maternal condition is an absolute contraindication


for breast feeding?

A. Alcohol intake

B. Cigarette smoking

C. Hepatitis B infection

D. Hepatitis C infection

E. Chemotherapy
91) A 2 years old child drinks gasoline that have been left in a glass after the
first swallow she cries and drops the glass she is most likely to develop which
of the following?

A. Aplastic anemia

B. Coma and convulsions

C. Chemical pneumonitis

D. Hepatitis

E. Peripheral neuritis

92) Congenital hypothyroidism should be included in the differential diagnosis


of a newborn with which of the following?

A. Coma

B. Prolonged jaundice

C. Increase blood presser

D. Renal failure

E. Microcytic anemia

93) Infants typically double their birth weight by what age?

A. 2 WEEKS

B. 2 MONTHS

C. 4 MONTHS

D. 8 MONTHS

E. 1 MONTH
94) An 8-year-old child develops an intensely pruritic rash on the legs. There
are patches of erythematous papules and vesicles and several liner streaks of
erythematous vesiculation. The child is afebrile and otherwise well.
The most likely diagnosis is?

A. Eczema

B. Henoch-Schönlein Purpura

C. Poison ivy dermatitis

D. Scabies

E. Varicella

95) Which of the following in an 8-year-old child is most likely to indicate an


underlying psychologic or behavioral problem?

A. Enuresis

B. Encopresis

C. Motion illness

D. Migraine headache

E. Recurrent pharyngitis

96) Chronic upper airway obstruction from enlarged tonsils and adenoids in a
child may cause which of the following?

a. Convulsions

b. Core pulmonale

c. Pneumothorax

d. Thymic hyperplasia

e. Reactive airway disease


97) Febrile seizures most frequently at what age?

a. In the first month of life

b. Between 2-5-month age

c. Between 5 month and 6 years

d. Between 6-10 years

e. Around the time of puberty

98) A 3 years old child is unresponsive present to the ER with weakness,


hypersalivation, bradycardia and constricted pupils. The parents are so
distraught it is difficult to get information. However, an astute emergency
room physician realized the most likely drug or toxin to cause these signs is
which of the following?

A. Diphenhydramine

B. Phenobarbital

C. Ethyl alcohol

D. Organophosphate

E. Hydrocarbon
99) A 2 years old boy seen in your office because of fever, ear pain,
postauricular swelling, erythema, and tenderness. The pina protrudes out on
the involved side. The tympanic membrane is red and bulging, with decreased
mobility seen on pneumatic otoscopy. the angle of the jaw is easily palpated
and the opening to stensen's duct appears normal.
The most likely diagnosis is?

a. Bacterial Parotitis

b. Mumps

c. External otitis

d. Acute mastoiditis

e. Chronic mastoiditis

100) Early indicators of Cushing syndrome in children are which of the


following?

a. Weight gain and growth arrest

b. Growth arrest and acne

c. Acne and hypertension

d. Hypertension and striae

e. Acne and striae


101) A 10 years old boy is evaluated for recurrent headaches that started 6
months ago and occur about once a month. He his asymptomatic between
episodes. Each headache starts with blurred vision and abdominal pain
followed by right side throbbing pain. it lasts about 60 minutes, during which
he feels better if he takes ibuprofen and rests in a dark room.
Most likely diagnosis is?

a. Brain tumor

b. Seizure disorder

c. Migraine

d. Todd’s paralysis

e. Heterophoria

102) A 16 years old male brought to the emergency room by his friends.
They relate they were drinking alcohol and that their friend passed out about
2 hours ago and is increasingly difficult to arouse.
Which of the following is the most important step in management of this
patient?

a. Serum sodium

b. Serum glucose

c. Blood alcohol level

d. Serum calcium

e. Serum drug screen


103) Among children, short stature and delayed puberty most likely
associated with which of the following?

a. Klinefelter syndrome

b. Beckwith-wiedemann syndrome

c. Turner syndrome

d. Marfan syndrome

e. Pierre robin sequence

104) A 30kg child you are taking care of in the hospital is receiving IV fluids
in preparation for surgery.
You are trying to determine if the fluids are running at an appropriate rate
for the daily maintenance.
What do you determine is the approximate daily fluid requirement for this
child?

a. 3000ml

b. 2500ml

c. 1700ml

d. 600ml

e. 400ml
105) On a discharge examination from a nursery you hear heart murmur and
consult the cardiologist. An echocardiogram was notable for small ventricular
septal defect and a patent foramen ovale. You are notifying the parents of the
results and explain to them the role of the foramen ovale in fetal life.
Which of the following statements is true?

a. blood flows through the foramen ovale from the right ventricle to the left
ventricle

b. blood flows through the foramen ovale from the left ventricle to the right
ventricle

c. blood flows through the foramen ovale from the left atrium to the right atrium

d. blood flows through the foramen ovale from the right atrium to the left atrium

e. blood must pass from foramen ovale for blood to enter right atrium from
umbilical vein.

106) A group of students is learning about children's language development.


Their instructor explains that language is a critical barometer of both
cognitive and emotional development.
Which of the following is speech delay most closely associated?

A. 22q11 deletion syndrome

B. Spina bifida

C. Diabetes

D. Child abuse

E. Asthma
107) Prevention of bilirubin encephalopathy or kernicterus is one of the goals
for the appropriate diagnosis and treatment of hyperbilirubenia.
Which of the following mechanism has a role in preventing these adverse
outcomes?

A. Unconjugated bilirubin is not lipid soluble

B. Unconjugated bilirubin is tightly bound to albumin

C. Unconjugated bilirubin is tightly bound to hemoglobin

D. The blood brain barrier is impermeable to Unconjugated bilirubin

E. Unconjugated bilirubin is rapidly metabolized by cerebrospinal fluid

108) 2 weeks old infant present to the pediatric intensive care unit in shock
with vesicular rash, pneumonitis, hepatitis and coagulopathy.
The prenatal and birth history were uneventful and the mother was healthy.
What is the most likely etiology for this illness?

A. Group B streptococcus

B. Herpes simplex virus

C. Varicella virus

D. Treponema pallidum

E. HIV
109) A 3-year-old child presents with 7 days of fever, conjunctivitis, red
cracked lips, polymorphous rash, and an isolated cervical lymph node.
Which of the following sequel of disease diagnosed in this child is the most
common?

A. Fulminant hepatitis

B. Coronary artery disease

C. Recurrent pericarditis

D. Cerebral edema

E. Renal failure

110) 16-year-old adolescent presents with temperature of 39.5c and purpuric


skin lesions. The child is found to have a blood pressure of 60/30 and a heart
rate of 180 Bpm.
What infectious agent is the most likely to cause these phenomena?

A. Neisseria meningitides

B. Hemophilus influenza

C. Streptococcus pneumonia

D. Staphylococcus aureus

E. Group B beta hemolytic streptococcus


Part B

1) A 8-year-old boy rectal t 38c bilateral tender parotid swelling, and pain
when you flex his neck he has been complaining of headaches his
immunization history is unknown.
What is the most likely in this child infection?

A. Adenovirus

B. Parvovirus

C. Herpes 6 virus

D. Mumps virus

E. Epstein Barr virus

2) During the delivery room resuscitation of a vigorous term newborn.


Which of the following should be performed first?

A. Verify the airway is clear, dry, and stimulate the infant.

B. The heart rate should be auscultated.

C. Breath sounds should be auscultated.

D. The mouth and trachea should be suctioned.

E. Assess color and administer oxygen if necessary.


3) A newborn infant with hemolytic jaundice. The mother did not receive
prenatal care with this pregnancy or her prior pregnancy. The direct Coombs
test is positive. The mother’s blood type is A- and the baby’s blood type is O+.
Her first baby did not have hemolytic jaundice.
What is the cause of the hemolytic jaundice?

A. ABO incompatibility

B. Toxoplasmosis

C. Rh incompatibility

D. Rubella

E. Hereditary spherocytosis

4) You are formulating a differential diagnosis for a newborn infant with


respiratory distress. Which of the following is most closely associated with the
development of neonatal respiratory syndrome (hyaline membrane disease)?

A. Gestational age

B. Birth weight

C. Cesarean section delivery

D. Maternal diabetes

E. Meconium in the amniotic fluid


5) A 14-year-old boy has an acutely painful and swollen scrotum.
What should be the next step in management?

A. Fine needle aspiration

B. Bone marrow aspiration

C. Surgical exploration

D. Oral antibiotics

E. Bed rest and analgesia

6) You are seeing an 8-month-old child in your office for recurrent thrush,
candida diaper dermatitis, and multiple recent skin pustules. In addition, he
was recently hospitalized with a hepatic abscess. You suspect that this child
may have chronic granulomatous disease.
Which of the following defense mechanisms is defective and the cause for
recurrent infections in chronic granulomatous disease?

A. Leukocyte migration

B. Synthesis of collagen

C. Capillary permeability

D. Tissue repair following injury

E. Phagocyte oxidative burst


7) A 2-month-old male presents with a cough that has persisted for over 2
weeks. Mother has noted he coughs in spells and at the end of these spells, he
vomits. She feels he has lost weight. On examination, you note a thin male
infant who otherwise appears normal. Complete blood count reveals 30,000
white blood cells with 95% lymphocytes.
Which of the following is the most likely diagnosis?

A. Respiratory syncytial virus bronchiolitis

B. Pneumococcal pneumonia

C. Pertussis

D. Laryngomalacia

E. Vascular ring

8) A 4 -year-old child presents to your office with fever, and increased work of
breathing manifested by tachypnea and retractions. A chest x-ray confirms
lobar pneumonia. What is the most likely etiology of pneumonia in this child?

A. Mycoplasma pneumoniae

B. Streptococcus pyogenes

C. Chlamydophila pneumoniae

D. Streptococcus pneumoniae

E. Staphylococcus epidermidis
9) A 12-year-old presents to the emergency department in August with fever
102.2°F (39°C) and intense headache. A lumbar puncture (LP) is performed,
and your suspicion of aseptic meningitis is confirmed.
Which of the following CSF findings is most likely 48 hours into the course of
enteroviral meningitis?

A. 5,000 WBC, 90% polymorphonuclear leukocytes

B. 100 WBC, 90% eosinophils

C. 150 WBC, 80% lymphocytes

D. 50 WBC, 90% polymorphonuclear leukocytes

E. 50 WBC, 70% monocytes

10) A 7-year-old child who recently traveled with her family to India presents
to your office with a 2-day history of fever, diarrhea, and tenesmus. Stool
examination reveals blood and leukocytes. You suspect infection with
Salmonella typhi as the cause of her symptoms. What additional
signs/symptoms are typically found with this infection?

A. Rectal prolapse

B. Hepatosplenomegaly and abdominal pain

C. Intensely pruritic skin rash

D. Cough and lymphadenopathy

E. Toxic megacolon and perforation


11) A 2-month-old infant is noted to have bilateral cataracts. The remainder
of the infant’s examination is normal; and developmentally she is on target.
What early dietary adjustment could have prevented the development of
cataracts in this child?

A. Eliminating gluten

B. Providing low iron formula

C. Eliminating milk and milk products

D. Providing increased amounts of milk and milk products

E. Providing increased amounts of folate

12) The mother of a 6-month-old child seeing you for a well visit, requests
immunization against measles as she has heard it is present in the community.
At what age should the first dose of live attenuated measles vaccine (as MMR)
be routinely administered in the absence of a community measles outbreak?

A. 6 months of age

B. 9 months of age

C. 12–15 months of age

D. 18–24 months of age

E. At the time of school entry


13) A 10-month-old male infant with a history of umbilical hernia repair is
noted to have coarse facial features and is diagnosed with Hurler disease.
This condition is the most severe of which of the following groups of inherited
diseases?

A. Glycogen storage diseases

B. Glycoproteinoses

C. Monopolysaccharidoses

D. Sphingolipidoses

E. Mucolipidoses

14) An infant is diagnosed with a given disorder below.


Which of these poses the greatest recurrence risk for this patient’s future
siblings?

A. Hirschsprung disease

B. Cystic fibrosis

C. Ventricular septal defect

D. Biliary atresia

E. Trisomy 13
15) A 2-year-old infant has acquired sensorineural hearing loss.
His mother is asking what the most likely cause could be.
What is the least likely etiology of acquired sensorineural hearing loss in a
toddler?

A. Hyperbilirubinemia

B. Congenital infection

C. Aminoglycoside therapy

D. Neuroblastoma

E. Late-onset group B streptococcal meningitis

16) Which of the following symptoms in a newborn infant would prompt to


test for cystic fibrosis?

a. Pneumonia

b. Intrauterine growth retardation

c. Meconium ileus

d. Wheezing

e. Hypochloremic alkalosis
17) A 1-week term newborn has on examination oral candidiasis (thrush).
Which of the following is correct statement regarding thrush in a term
newborn?

a. responds well to topical therapy with nystatin

b. requires systemic therapy with amphotericin B

c. it responds only to both topical and systemic therapy

d. it requires investigation to rule out immunodeficiency

e. requires no treatment

18) A 12-year-old present with fever sore throat and general malaise.
On physical examination generalized lymphadenopathy, exudative
pharyngitis and splenomegaly. Laboratory evaluation reveals elevated serum
transaminase levels.
What is the organism most likely?

a. Varicella virus

b. Hepatitis B virus

c. Rubella virus

d. Epstein Barr virus

e. Measles
19) A 10-month old child has a temperature of 40 c for 4days without other
signs on the 4 day a rose pink maculopapular rash appears and the
temperature returns to normal.
What the most likely organism that cause disease?

a. Echovirus

b. Human herpesvirus 6

c. Measles virus

d. Group A streptococcus

e. Typhus

20) A 16-month old girl present with acute onset of truncal ataxia with
vomiting, nystagmus and dysarthria she is afebrile and has no nuchal rigidity.
Which of the following historical data would help y to identify a cause for
these symptoms?

a. elevated lead levels at age of 1year

b. febrile seizure episode at age of 13 year

c. sore throat with blisters on palates 3 week

d. febrile illness with rash 3 month ago

e. first MMR vaccination 2 month ago


21) A 2 years old toddler has a large abdominal mass and pancytopenia.
On examination opsomyoclonus.
Which of the following diagnosis would most likely be established by bone
marrow aspiration?

a. Hepatoblastoma

b. Neuroblastoma

c. Renal cell carcinoma

d. Rhabdomyosarcoma

e. Wilm’s tumor

22) A 14-month boy with severe eczema, recurrent sinus and ear infection and
thrombocytopenia. You suspect immunodeficiency. What is the diagnosis?

a. Chronic granulomatous disease

b. DiGeorge syndrome

c. Leukocyte adhesion defect

d. Ataxia telangiectasia

e. Wiskott-aldrich syndrome
23) 2 years old girl is listless and pale, you obtain complete blood count and
found patient with megaloblastic anemia,
What additional history explain this?

a. She eats only organically grown product

b. She had a long- term treatment with phenytoin

c. Has required phototherapy after birth

d. Has required multiple courses of antibiotics over otitis media

e. Is an infant of a diabetic woman

24) A 12 month old with poor gain, loose stool at age 18 month, poor appetite.
On examination irritable child with very little subcutaneous fat?

a. celiac screen

b. urine analysis

c. sweat chloride

d. quantitation immunoglobulins

e. fecal blood

25) Classic radiologic finding in duodenal atresia?

a. totally gasless abdomen

b. free air below the diaphragm

c. the double bubble sign

d. the anchor sign

e. the sting sing


26) A 56-year-old man presented to the ER with new onset chest pain. He
describes a sharp pain, which improves when he bends forward. Auscultation
did not reveal pathological heart sounds, and the pain didn’t improve with
nitroglycerin. Which of the following is the most likely diagnosis?

a. angina due to coronary blockage.

b. angina due to aortic stenosis.

c. dissection of the ascending aorta.

d. pericarditis.

e. pulmonary embolism causing pulmonary infract.

27) A 61-year-old presents to the ER complaining of “the worst headache of


her life”, she is generally well, except for arterial HTN treated medically for
the last 2 years. Physical examination revealed no neurological deficits.
Which of the following treatments would be the most appropriate for this
patient?

a. discharge with analgesics and follow up by her family physician.

b. discharge with effective analgesics and a referral for an ambulatory neurological


work up.

c. admit for observation without urgent or immediate further work up.

d. discharge after a lumber puncture to rule out meningitis.

e. admit for urgent work up diagnosis of the severe pain cause.


28) A 24-year-old women is admitted due to fever, rash, abdominal pain, and
diarrhea upon her return from a trip to Thailand. She has no additional
complains. The individuals from the same group experienced the same
symptoms. She denies swimming or exposure to water sources.
Which of the following is the most likely diagnosis?

a. Typhoid fever

b. Dengue fever

c. SLE

d. Leptospirosis

e. Infectious mononucleosis

29) A man 71-year-old come to the internal department with double vision
and instable gait he has a suspect (retro – vestibular brain infarct) after 5
days his temperature 38.8 C and severe fatigue the physical exam does not
reveal source of fever.
Which of the following is the most likely etiology of the fever?

A. Drug fever

B. Nosocomial infection

C. Central fever

D. An undiagnosed malignancy

E. Factitious fever
30) A woman 80 years old come to the clinic department for observation after
an episode of loss of consciousness. Attached is the ECG upon admission.
Based on the clinical story and ECG. (Picture)
Which if the most appropriate treatment for her condition?

A. trial of IV atropine in the ER

B. admitting for observation and an in house holder ECG

C. admitting for insertion of a pacemaker

D. rapid preparation for a coronary angiography

E. admitting for insertion of a defibrillator – pacemaker

31) A 55-year-old man with atherosclerosis is admitted due to acute


rotational dizziness, unstable gait, nausea and vomiting, and A positive
Romberg sign. Injury of which of following CNS structures would a brain
CT with perfusion scan demonstrate?

A. basal ganglia

B. cerebellum

C. brain stem

D. frontal lobe

E. the thalamus
32) A patient with shortness of breath is admitted to the ER. CBC
reveals hemoglobin of 7,5 gr %.
Which of the following in true regarding the clinical evaluation of patients
with anemia?

A. cyanosis will appear sooner in an anemic patient thus if the patient is acyanotic,
his saturation in normal.

B. anemia has no impact on oxygen saturation, thus is the patient is acyanotic he is


not hypoxic.

C. in an anemic patient cyanosis will appear later, and truss the patient may be
hypoxic even in the absence of central cyanosis.

D. anemia influences the appearance of peripheral but not central cyanosis.

33) A 69-year-old man who is a heavy smoker complains difficulty in


swallowing solids in the past two months, cough, and a weight loss of 4 kg over
the last month. He denies difficulty in swallowing fluids or pain while
swallowing. Which of the following is the most likely diagnosis?

A. candida esophagitis

B. a malignant laryngeal tumor

C. a cerebellar infarct

D. diaphragmatic hernia

E. esophageal malignancy
34) An 82-year-p;d woman is brought to the ER in severe condition She has
massive rectal bleeding. And her systolic blood pressure is only 73mmHg.
Which of the following should be the first diagnostic test?

a. Flexible sigmoidoscopy

b. Colonoscopy

c. Gastroscopy

d. Abdominal and pelvic CT

e. Urgent explorative laparoscopy

35) A 62 years old woman has been diagnosis with cirrhosis due to viral
hepatitis, 5 years ago she recently developed ascites (SAAG<1.1gr/dl).
Which of should be the initial therapeutic approach in trying to reduce the
ascites?

A. A low sodium diet

B. Furosemide

C. Spironolactone

D. Zaroxolyn

E. Peritoneal drainage of over 3 liters’ ascites

36. A 44 years old woman with polydipsia was diagnosed as suffering from
diabetes insipidus. Which of the following is the best diagnostic test in order to
differentiate central from nephrogenic diabetes insipidus?

A. Serum osmolality

B. Urine osmolality

C. Water deprivation test

D. Serum vasopressin levels


37) During a night call in an internal medicine wards you admit a 91 years old
patient with nausea and vomiting, sever dizziness and deterioration in
consciousness. Serum sodium is 102 mmol, you are instructed by the attending
physical to treat with hypertonic saline.
How would you predict the rate of increase in sodium in the patient?

A. There is a direct relationship between the volume of hypertonic solution and the
rise in sodium rate can be predicted

B. There isn’t a direct relationship between the volume of hypertonic solution and
the increase sodium. But there is a formula, which enables exact calculation of the
sodium increase rate

C. There is no relationship between the hypertonic saline volume and the increase
in sodium and thus it is impossible to predict the increase in sodium rate

D. The relation between hypertonic saline volume and serum sodium increase rate
is complex and individualized. Therefore, you need to monitor serum sodium
levels closely

E. There are formulas in the literature describing the relationship between


hypertonic saline volume and serum sodium increase rate. And thus there is no
need for frequent sodium levels

38) A 22-year-old male was admitted due to abdominal pain and decreased
consciousness. His lab results included pH 7.25 HCO3 4mmol/l,
PCO2 17mmHg, Na 135 meq/l, K 3 meq/l, Cl 88 meq/l.
Which of the following is the most likely diagnosis?

a. Combined respiratory and metabolic acidosis with a normal anion gap

b. Metabolic acidosis with a normal anion gap

c. Metabolic acidosis with increased anion gap

d. Combined respiratory and metabolic acidosis with increased anion gap


39) 32-year-woman complains of small joint pain of the hands, rigidity in the
morning, a facial rash not involving the bridge of the nose,
and new onset of the mouth ulcers.
Which of the following tests will be positive in over 90% of these Px?

A. ANA

B. Anti CCP

C. RF

D. Anti SM

E. Anti YO

40) A young woman with a new diagnosis of SLE is admitted to the ward.
Which of the following is the best first line treatment?

A. Plaquenil

B. Colchicine

C. Anti TNF

D. NSAIDs

E. cyclophosphamide
41) 65year man complains of right temple pain, pain during chewing shoulder
and hip griddle, pain and prolonged morning rigidity. Lab results include
increased ESR of 80mm in the first hour.
Which of the following would be the best initial treatment for this Px?

A. Azathioprine

B. Prednisone

C. NSAIDs

D. Anti TNF

E. Cyclophosphamide.

42) 45 years old woman complain of joint pain and morning rigidity lasting
about an hour the assume diagnosis is rheumatoid arthritis
Which of the following joint involvement patterns would support this DX?

A. Symmetrical involvement of the MCPs, PIPs, of the wrist

B. Symmetrical involvement of the DIPs of the wrist

C. INVOLVMENT of the sacroiliac joint and the right knee

D. Involvement of the PIP of the right finger and a '' sausage " toe

43) 61-years old suffer from symptomatic anemia has hemoglobin level is 8.2,
reticulocyte index 3.6%, and the RBCs show normocytic chromocyte.
Which of the following can explain the patient’s anemia?

A. Drug induce bone marrow suppression

B. Malignant infiltration of the bone marrow

C. Chronic renal disease

D. Chronic hemolytic

E. Thalassemia
44) An 82-year-old man complains of fatigue, loss of appetite, weight loss, and
abdominal pain. CT revealed a malignant looking mass in the abdominal
cavity, which measures about 12cm. Around the main mass there are several
smaller masses.
Which of the following is true regarding the pathophysiology of the tumor
described?

a. The tumor has most probably developed over the last month

b. The growth rate of the tumor is lower compared to the growth rate several weeks
ago.

c. Cure rate is high since the diagnosis was made recently

d. The diagnostic threshold for tumors is 10cm. so this mass has just crossed
diagnostic threshold

45) A 50-year-old man has gone through extensive diagnostic work-up for
bone pain. A bone marrow biopsy revealed 7% mononuclear plasma cells.
The rest of the work-up was normal.
Which of the following is the most likely diagnosis?

a. Monoclonal Gammopathy of undetermined significance (MGUS)

b. Asymptomatic (smoldering) myeloma

c. Non-secretory myeloma

d. Solitary plasmacytoma
46) A 83-year-old woman was admitted for surgical repair of a femur neck
fracture and was treated with heparin on the days following surgery. 8 days
later she developed thrombocytopenia of 72,000, and swelling and pain of the
left calf. The doctor suspects heparin induced thrombocytopenia (HIT).
Which of the following statements regarding HIT is true?

a. HIT is not a likely diagnosis in this case as it has been 8 days past surgery and
the platelet count is too high

b. HIT is not a likely diagnosis in this case since platelet count is too high,
although the time frame fits the diagnosis

c. The diagnosis is most probably HIT and lower extremity Doppler ultrasound to
rule out DVT should be performed

d. The diagnosis is most probably HIT but there is no point in performing lower
extremity Doppler ultrasound to rule out DVT

e. The diagnosis is most probably HIT and the patient should be treated with IV
platelets with or without lower extremity Doppler ultrasound

47) An 18 years old male present to the ER with a temperature of 40' C and
hypotension (SBP 70 mph),
he had undergone a splenectomy 2 years ago after a MVA.
Which of the following should be the first antibiotic treatment regimen in this
patient?

A. An initiated combination of Vancomycin and Clindamycin

B. A combination of Cefepime and Ceftriaxone

C. A combination of Vancomycin and Gentamycin

D. A combination of Vancomycin and Ceftriaxone

E. A combination of Ceftriaxone and Doxycycline


48) A 24 years old present with fever and malaise, headaches and rush, she
reports find in a tick on her self-2 days earlier, that has most probably come
from the family dog.
Which best initial therapy for this patient?

A. Ceftriaxone

B. Vancomycin

C. Doxycycline

D. Augmentin

E. Azithromycin

49) A 51-year-old woman was admitted with fever, fatigue, and lesions on the
fingertips, acute renal failure and splenomegaly. Repeated blood cultures
grew Enterococcus and in TEE there was suspected vegetation on the aortic
valve during the evening the patient has lost consciousness and demonstrated
clinical symptoms of acute heart failure. Urgent TEE demonstrated severe
aortic insufficiency.
Which of the following would be the best treatment at this point?

A. Emergent valve replacement

B. Antibiotic treatment for a week and then definite valve replacement

C. IV antibiotics for 4-6 weeks and then definite valve replacement

D. IV antibiotics for 4-6 for 4-6 weeks and then repeat TEE and revaluation of the
need for valve replacement
50) What is the correct sequence of treatments (medical and others) for
patients with heart failure and fluid retention?

A. Diuretics, defibrillator insertion, β blockers

B. ACE inhibitors, diuretics, β blockers

C. Diuretics, ACE inhibitors, β blockers

D. Insertion of a biventricular Pacemaker and management with ACE inhibitors


and β blocker according to its influence

51) 72 years old man complaining of recurrent episodes of syncope, shortness


of breath during exercise and difficulty in lying flat, a coronary
echocardiograph showed aortic stenosis with valve area of 0.66 cm2 and
pressure gradient of 66 mmHg.
Which of the following is true regarding the need of this patient for valve
replacement?

A. There is no indication for valve replacement, but since the patient will need
bypass surgery the replacement should be performed during the same surgery

B. There is an indication for valve replacement, and the replacement should be


done during the bypass surgery

C. There is an indication for conservative follow up, and if the pressure gradient
continues to increase the valve should be replaced

D. Valve replacement indicated only in the presence of angina


52) A 50 years old man presents to the ER with chest pain, shortness of
breath, nausea, diaphoresis and low blood pressure, the patient ECG
attached, which of the following is the correct diagnosis of this patient?
(Picture shows ST elevation MI in leads II, III, aVF )

A. Anterior wall MI

B. Anterior and lateral wall MI

C. Inferior wall MI

D. Lower and lateral wall MI

53) A 38-year-old woman was diagnosed with arterial hypertension she was
healthy until now and does not take any regular medication. Laboratory
results revealed serum NA of 139 and serum K of 2.8. The treating physician
subscribed Amlodipine 5mg once daily. But there was no change in blood
pressure.
Which of the following is the most likely diagnosis?

a. Essential hypertension

b. Reno-vascular hypertension

c. Pheochromocytoma

d. Primary hyperaldosteronism

e. White coast hypertension


54) A 71-year-old woman complains of pain and numbness of the right foot.
She has chronic atrial fibrillation but does not receive anticoagulation due to
recurrent falls. Attached Is a photograph of the patient’s feet.
Which of the following is the most reasonable next step in diagnosing this
patient? (Picture)

a. Doppler of the veins in order to rule out lower extremity deep vein thrombosis

b. Doppler of the arteries in order to rule out an arterial clot originating in the heart

c. CT of the legs in order to rule out a mass compressing the blood vessel

d. Spine CT in order to rule out a mass compressing the spinal cord

e. TTE in order to rule out a clot in the heart chambers

55) A 34-year-old woman is admitted due to a severe asthma attack.


Which of the following describes the most appropriate management of these
cases?

a. If the pCO2 level is normal it would be considered a mild asthma exacerbation

b. As opposing to the past. Nowadays IV magnesium is no longer considered part


of the treatment of asthma exacerbation

c. As opposing to the past IV aminophylline is considered an essential component


of the treatment of asthma exacerbation

d. Antibiotics are not component of the treatment of an acute asthma exacerbation


unless there is evidence of pneumonia

e. There is no point in ipratropium inhalation for bronchodilation in patients


already receiving beta agonists
56) A 31 years old man hospitalized with the diagnosis of community acquired
pneumonia. after 5 days of iv treatment there is no improvement in this
condition and chest x ray reveals plural effusion. Thoracic tap was performed.
Which of the following would be an indication for inserting a chest tube in this
patient?

A. ph. 6.6

B. Glucose 89 mg/dl

C. LDH 166 iu/l

D. Negative pleural effusion culture

57) A 70 years old man is about to be discharge after hospitalization for an


exacerbation of COPD. Which of the following treatment in the community
has been proved beyond doubt to prolong his life expectancy?

a. chronic use of bronchodilators

b. chronic use of steroid

c. daily respiratory physiotherapy

d. regular prophylactic antibiotics

e. regular use of supplemental oxygen


58) A 78- years old women is admitted due to a urinary tract infection. She
deteriorates over the first 24 hours and becomes severely septic with
respiratory deterioration and Pa O2/FiO2 is 180. Chest x-ray shows bilateral
infiltrates consistent with bilateral interstitial pneumonia. A right angiogram
reveals PCWP of 14 mmHg. The pulmonary diagnosis is most likely?

a. Bilateral bacterial pneumonia

b. Pulmonary edema due to acute heart failure

c. ARDS

d. Massive PE

59) 80 years old is found home unconscious. blood pressure is undetectable.


Pulse is weak and irregular and there is prominent finding of maximal neck
vein distension without tracheal deviation. breath sounds are superficial and
equal. Which of the following medical condition would best explain the
clinical picture described above?

a. Ventricular fibrillation

b. Pulseless ventricular tachycardia

c. Tension pneumothorax

d. Hypovolemic shock

e. Tamponade
60) A 35 years old male is known to suffer from locally advanced lung cancer
in the right lung. He hasn't received any treatment yet. the patient has come
to the ER complaining of headache and on physical examination the face is
darker with bluish discoloration compared to the abdomen. He has dilated
veins on the right chest and is confused and disoriented regarding time and
place. Which of the following is the most likely diagnosis?

a. Lung cancer brain metastases

b. Pulmonary embolism

c. Supra vena cava syndrome

d. Tension pneumothorax

e. Tamponade

61) patient with progressive renal failure may require emergency treatment.
Which of the following describe the emergency and the treatment?

a. hyperkalemia, calcium gluconate infusion

b. pulmonary edema; ACE -inhibitor

c. metabolic acidosis; positive pressure ventilation

d. hypernatremia; emergency dialysis

e. uremic pericarditis; allopurinol administration


62) A 71-year-old man has a permanent urinary catheter for the last 3 weeks
following prostatectomy. He now complains of a fever, urethral pain and a
burning sensation when urinating.
Which of the following is the best treatment in this case?

a. The catheter should be replaced or removed, and a urinary culture should be


sent. There is no need for antibiotic treatment

b. The catheter should be replaced or removed and a urinary culture should be sent.
Antibiotic treatment should be started immediately

c. There is no need to remove or replace the catheter. A urinary culture should be


sent. There is no need for antibiotic treatment

d. There is no need to remove or replace the catheter. A urinary culture should be


sent and antibiotic treatment should be started

63) A 61-year old man complains of pain and discomfort in the upper
abdomen over the last two months. He describes some difficulty in swallowing
solids. And had unintentionally lost 4kg over the last month. Which of the
following would be the next in the diagnosis of this patient?

A. A breath test

B. Treatment with a proton pump inhibitor

C. Treatment with H2 blocker.

D. Referral to a gastroenterologist at gastroscopy.

E. X-ray of the esophagus with barium contrast.


64) A 39-year-old woman complains of abdominal pain and diarrhea over the
past 5 months. On physical examination a lower right quadrant mass is
palpated and a small opening in the skin with secretion of a small amount of
puss is observed. Past antibiotic treatment has led to a clinical improvement
but the effect has stopped as soon as the antibiotics were ceased.
Which of the following is the most likely diagnosis in this patient?

a. Ulcerative colitis (UC)

b. Crohn's disease (CD)

c. Irritable bowel syndrome (IBS)

d. Infectious enterocolitis

e. Pseudomembranous colitis

65) An 81- years old was brought to the emergency room with deteriorated
consciousness following a few episodes of bloody diarrhea. Before the diarrhea
she had complained of severe abdominal pain. She is known to have chronic
atrial fibrillation, treated with beta blocker, but without anti-coagulants due
to recurrent falls in her past. Laboratory results in the ER revealed severe
metabolic acidosis with increased LDH and lactate. She died shortly after in
the Emergency room. Which of the following is the most likely cause of death?

a. Bleeding from duodenal or gastric ulcer

b. Bleeding from a malignant tumor

c. Mesenteric event due to embolization from a heart clot

d. Small bowel obstruction


66) A 41 years old chronic IV drug user for abdominal pain, nausea, vomiting,
loss of appetite and fever. Laboratory results revealed: HBsAg+, anti-HBs-,
HBeAg+, anti-HBe-, IgM anti-HBc.
Which of the following is the most likely diagnosis?

a. Acute hepatitis B

b. Chronic hepatitis B, high infectivity

c. Chronic hepatitis B, low infectivity

d. Status post immunization for hepatitis B

e. Recovery from acute hepatitis B infection

67) Attached is a photograph of a skin lesion, characteristic of patients with


chronic disease.
Which of the following will be found on physical examination of these
patients?

a. Butterfly rash over the facial areas

b. Exophthalmos

c. Hirsutism

d. Cardiomegaly

e. Palmar erythema
68) A 27-year-old with chronic cough and rash (photo) chest X ray
demonstrate bilateral hilum. laboratory negative beside hypercalcemia
What is the most likely diagnosis?

a. Rickettsia infection

b. Q fever

c. Lung cancer

d. Sarcoidosis

69) A 90- year old patient is diagnosed with type 2 diabetes, and is known to
have suffered multiple episodes of hypoglycemia in the past.
His HbA1c is 5.2gr%.
Which of the following would be the best medicine for this patient's
management?

a. Sulfonylurea

b. Basal insulin plus a sulfonylurea

c. Basal insulin combined with a short acting insulin

d. Basal insulin combined with metformin

e. There is no indication for the treatment with the above regimens in this patient
70) An 83 y/o woman is complaining of long standing dizziness and has even
fallen twice over past month. dizziness is increased about half an hour after
breakfast with palpitations, she is known that have hypertension and diabetes
and take Ramipril amlodipine doxycin, metformin and insulin glargine. blood
pressure 100/60 fasting glucose 110%, HbA1C 7%. physical examination
control, pulse 80 and regular. What is the diagnosis?

a. Hypoglycemia

b. Hyperglycemia

c. Atrial fibrillation

d. Atrial tachycardia

e. Orthostatic hypotension

71) A 90 y/o woman is admitted with an acute urinary infection and was
treated with IV ceftriaxone. due urinary retention and deteriorating renal
function a catheter was inserted, IV fluids were started. At night the patient
has begun yelling that she wants return home and medical and nursing staff
plot to keep her from her family. the patient with advanced dementia and
arterial hypertension.
Which of the following treatments would go offer this patient?

a. Paracetamol

b. Haloperidol

c. Diazepam

d. Promethazine

e. Risperidone
72) A 78-year- old man was admitted to neurology due to a stroke with left
hemiparesis, dropping of the left angle of the mouth and blurred speech.
Aspirin treatment was initiated. Neurological examination revealed a
preserved gag reflex. But his wife reports cough during swallowing of fluids
and solids. The next morning the patient develops a temperature of 39c and
his consciousness deteriorates. Chest x-ray reveals a large right middle lobe
infiltrate and antibiotic therapy is initiated.
Which of the following is the best treatment for this patient?

a. Follow-up only, since the aspiration is most probably a one-time event.

b. Thickening of fluids and soft solid foods

c. Insertion of a nasogastric tube

d. Insertion of a percutaneous endoscopic gastrostomy (PEG)

73) a 93-year-old man is admitted due to lower abdominal pain. He notes an


increase in urination frequency, but no fever or increased urgency in
urination. He also notes prolonged constipation and being gassy over the last
24 hours. His medical history includes benign prostate hyperplasia and
hypertension. On examination the patient seems in pain, he is afebrile and has
lower central abdominal swelling and tenderness. Rectal examination shows a
severely increased. Smooth non-tender prostate and fecal stones are removed.
Abdominal ultrasound increased bladder. Put a urinary catheter, immediately
stood out with 700 ml of urine. In the analysis of urine isolated leukocytes.
What is the most likely diagnosis?

А. Urinary tract infection

B. Acute prostatitis

C. Overflow urinary incontinence

D. Stress urinary incontinence

E. Urge urinary incontinence


74) An 83-year-old man complains of malaise, nausea, and headache.
His medical history includes diabetes, chronic back pain, and depression.
His regular medications include metformin, paracetamol, and escitalopram
(cipralex). On examination his blood pressure is 120/80 mmHg with no
jugular vein distention or peripheral edema.
Laboratory results show serum sodium of 123 mEq/l, urine sodium of 5 mg/dl
with osmolarity of 90 mOsm/l.
Which of the following is the best recommendation for this patient?

a. Adding furosemide

b. Increasing the dose of paracetamol

c. Increasing the dose of cipralex

d. Addition of sodium chloride pills without limitation of fluids

e. Limitation of fluids

75) A 50-year-old man is admitted for investigation of fever and night sweats,
pruritus, loss of appetite and weight loss. On physical examination he has an
enlarged spleen with no lymphadenopathy.
Abdominal CT demonstrated a few enlarged lymph nodes in the groin and
retro peritoneum.
Which of the following is the most appropriate next step in the diagnosis of
this patient?

a. Since the spleen is enlarged, the next step would be a CT guided biopsy of the
spleen

b. Since the spleen is enlarged, the next step would be a diagnostic and therapeutic
splenectomy

c. The patient should be referred for a PET CT; which ma identify suspicious
lymph nodes more suitable for biopsy

d. Since a hematologic disease is suspected, the next step would be a bone marrow
biopsy
76) In bipolar disorder amniotic state can induce exposure of drugs and
substance. Which of the following would not induce a manic sate?

A. A tricyclic antidepressant

B. Cocaine

C. Propranolol

D. Methylphenidate

E. Prednisone

77) Which of the following mental and cognitive symptoms isn’t in high
correlation with Parkinson disease?

A. Psychotic state with delusional and visual hallucination

B. Subcortical dementia

C. Depression symptoms

D. Intellectual disabilities

E. Anxious state

78) Alcohol withdrawal can appear in individual with alcohol dependence


following withdrawal cessation of alcohol consumption.
What is true regarding delirium tremens (DT)?

A. DT doesn’t significantly change the course of alcohol withdrawal

B. DT isn’t one of the first symptom of alcohol withdrawal

C. The occurrence of DT wasn’t providing and wore gradual cessation

D. DT doesn’t occur in absence of other withdrawal symptoms

E. The occurrence of DT increased significantly the complication of alcohol


withdrawal
79) Which of the following results (lab results and vitals’ signs), won't be
abnormal in this neuroleptic syndrome?

A. White blood count

B. CPK

C. Lives function tests

D. Thyroid function tests

E. Body temperature, blood pressure

80) Which of the following is the most likely presentation of a 30 years old
man with OCD?

A. Need of order and symmetry

B. Need to use lucky numbers

C. Fear of infection and mashing his hands

D. Avoiding stepping on lines on his floor at home

E. Pathological storage of worthless items


81) A man of 45 years old work as a security guarding in a parking lot for 10
years, over 3 years he has been experiencing persecution delusions, that he has
been followed and his mobile phone is tapped, he is very distraught by these
thoughts and is tense and alert over the last 2 months, he is very sad and he
doesn’t enjoy watching TV, he lost 7 KG, missed work, he has ideas of suicide.
Which of the following is the relevant diagnosis of this case?

A. The man has schizophrenia or delusional disorder, and has now developed an
episode of major depression

B. The man has paranoid type bipolar disorder, and has now developed of an
episode of major depression

C. The man has absent insight/delusional type of obsessive compulsive disorder


and has now developed of major depression

D. He has schizotypal personality disorder or a schizoid personality disorder and


has developed now double depression

E. He has had a major chronic depression disorder and now he has developed
double depression

82) Which of the following drugs or clinical states might increase serum
lithium value and lead to toxicity?

A. Ibuprofen

B. Rich diet in salt

C. Acetaminophen

D. Theophylline

E. Water increase consumption


83) Which of the following regarding drugs therapy of major depressive in the
context of bipolar disorder?

A. Treatment with SVRI without lithium as mood stabilizer increase the risk of
manic transition

B. Treatment with SSRI without lithium as mood stabilizer increase the risk of
manic transition

C. Treatment with SNRI antidepressant combined with Olanzapine doesn’t


increase the risk of manic transition

D. The SNRI and SSRI have an equal risk of inducing manic transition.

E. Antidepressant therapy cannot induce manic transition

84) Which of the following is true regarding a drug induced psychotic episode
compared to a psychotic episode in context of schizophrenia?

A. The two states can be differentiated by the classic visual hallucinations


appearing in drugs induced psychosis.

B. A diagnosis of drugs induced psychosis doesn’t exist, it is schizophrenia


revealed due to drugs exposure.

C. It is impossible to differentiate between the 2 diagnoses by the classic visual


hallucinations appearing in the drugs inducing psychosis.

D. Drug induced psychotic episode, does not warrants treatment since it is


transitory and is relieved once the drug is cleared, vs, a schizophrenic
psychotic episode which warrants treatment.

E. Drugs induced psychotic episode has an equal risk to a family history of


schizophrenia vs a psychotic episode in the context of schizophrenia.
85) A 25 years old woman present to the emergency room with complaints of
shortness of breath, palpations and feel about go to die, and it is a 4th episode
in the last 2 months, every time she come to the emergency room. Her father
has a history of heart attack, as he died. Over 2 last months, she is very
concerned about the next episode, and she feel her life has significantly
changed, she is afraid to go to crowded places where she might not reach help
in time. Over the last week, she is sad and tear.
Which is the most appropriate diagnosis in this patient?

A. Panic attack in the context of a panic disorder.

B. Panic attack of the context of social anxiety disease.

C. Panic attack of the context of background disease.

D. Panic attack of the context of major depression episode.

E. Panic attack of the context of illness anxiety disorder.

86) In which of the following diagnosis suicidal thoughts are diagnostic


criterion by DSM?

a. Major depression

b. Schizophrenia

c. Delusional disorder

d. Obsessive-Compulsive Disorder

e. Paranoid personality disorder


87) A 45-year-old man in conversational therapy on weekly basis, the patient
and the therapist are seating across each other, the duration of the meeting
changes (15 to 60 min). The therapist is active with significant counseling and
guidance. Regarding every issues, the most probably Diagnosis?

a. Psychoanalysis

b. Psychodynamic psychotherapy

c. Hypotonic treatment

d. Supportive psychotherapy

e. Relaxation psychotherapy

88) A 48-year-old female treated at psychiatric clinic for last month while
going through divorce process, she is disturbed and thinks a lot about the
divorce, she is nervous having trouble falling asleep suffers from headaches
and stress, she continues to work and partly functions at home.
What is the most appropriate diagnosis?

a. A stress disorder

b. Post traumatic disorder

c. Adjustment disorder

d. Somatization disorder

e. Major depression episode


89) A 31-year-old man complains of severe agitation anxiety and muscle pain
in the past 4 days, he didn’t sleep for two days, on examination dilated pupils
tearful sweaty,
What is the most probably diagnosis the best explains his status?

a. White out fluid withdrawal

b. Opioid withdrawal

c. Cocaine withdrawal

d. Amphetamine withdrawal

e. Alcohol withdrawal

90) In the month of August a man in his mid-60s is brought to the ER by his
family members due to unusual and weird behavior, the episode lasted for
only several hours, there is no History of mental disorders, on examination
disoriented blurred Consusnes complaining of weakness perceptive
disturbances, headaches and extremity numbness, decreased turgor, rapid
heart rate and weak pulse are noted. What is the least likely diagnosis?

a. Sedative drug overdose

b. Acute renal failure

c. Dehydration

d. Acute psychiatric episode

e. Stroke
91) A 23 y/o man is brought to the ER due to weird behavior, blurred speech,
false thoughts of persecution, and auditory illusions in the past 2 months, the
diagnosis of schizophrenia.
All of the following are important for establishing this diagnosis EXCEPT?

a. Sleep lap

b. Detailed history

c. Mental status

d. Personal and personality history

92) All of the following are physical complications of anorexia nervosa,


EXCEPT?

a. Loss of muscle mass

b. Decreased height than expected

c. Leukopenia

d. Hypocholesterolemia

e. Hypomagnesemia
93) A single 38 y/o male complains of no success in establishing the women he
contacted left after few dates, one claimed that he in stingy and another said
he is rigid, he describes himself as clean, neat, organized that cannot “cut any
corners and does not understand the difficulties.
Which of the following is he also likely to be found?

a. Suspicious

b. Weird

c. Very sensitive to criticism and rejection

d. Very moral

e. Impulsive

94) Which of the following is correct regarding suicide?

a. the chances for suicide are lower in psychiatric patients as compared to the
general population

b. schizophrenia is the mental disorder most commonly causing suicide as


compared to all other mental pathologies

c. the most common suicide cause for men is overdosing and jump from height in
women

d. divorced people are at increased risk for suicide as compared to singles

e. the risk for suicide increases with age


95) A 30 y/o male arrives to the ER at midnight complaining of severe
agitation and uncontrollable repeated diarrhea, a month ago he was
diagnosed with depression and was started on escitalopram 20mg once daily.
What is the next most important step?

a. discontinue escitalopram immediately and treat serotonergic syndrome

b. reassure the patient and explain that anxiety attacks are common comorbidities
in depression

c. reassure the patient and explain that those are common symptoms as depression
begins to improve

d. reassure the patient and explain what is an anxiety attack and add a sedative for
several days

e. reassure the patient and increase the dose of escitalopram to 60mg

96) A 25 years old male is brought to the ER by his family due to deterioration
in the last month, the family describes that most of the time he sits still and
cannot be moved, does not talk, negativist and refuses to eat or drink, several
hours ago he suddenly and unexpectedly became agitated, ran around
purposelessly, screamed, threw things around,
tried to jump out of the window and then returned to his "frozen status”.
What is the most likely Diagnosis?

A. Catatonia

B. NMS

C. Tardive dyskinesia

D. Serotonin syndrome

E. Delirium
97) All of the following are side effects of clozapine treatment, EXCEPT?

A. Increased threshold seizures

B. Drooling

C. Myocarditis

D. Agranulocytosis

E. Sedation

98) A 32 years old female, suffers from severe bipolar disorder, she recently
got married and interested in getting pregnant, she and her partner are
asking regarding the harm of medications during pregnancy.
Which of the following is the most teratogenic of this patient?

A. Valproate + Carbamazepine

B. Lithium Lamotrigine

C. Lamotrigine + Lithium

D. Olanzapine

99) A 40 years old male who has generalized anxiety disorder brought to the
ER with symptoms of benzodiazepine intoxication, who is treated in the ER
with i. v Anexate {flumazenil).
All of the following are expected side effects EXCEPT?

a. visual disturbances

b. seizures

c. vomiting

d. sedation

e. skin vasodilatation
100) An 84 years old male who until recently was coherent and well is brought
to the ER by his family who describe that in the last month " he became really
demented, doesn't remember a thing, not doing anything, speak very little and
mumbles a lot about being a burden on the family ". on examination mostly
sits with his head down and answers " don't know, don't remember ".
Which of the following will most likely be found in this patient?

A. True distinct disturbances in short-term but not long-term, memory

B. Depressive symptoms

C. Good cooperation

D. An attempt to cover difficulties

E. Confabulations

101) Of the following medications which one is most likely to possibly cause
metabolic syndrome as a side effect?

a. Paroxetine

b. Fluoxetine

c. Haloperidol

d. Clozapine

e. Olanzapine
102) Schizophrenia is a disorder with grave prognosis, which of the following
statements best describe the prognosis of this disorder?

A. About 50% of schizophrenia patients achieve good functionality in 5-10 years


from the first admission.

B. As many as 10% of the patients will require repeat admissions over time

C. About 20-30% of the patients continue to have moderate intensity of active


symptoms all the time.

D. The prognosis of schizophrenic patients is in general better than those with


mood disorder.

103) Which of the following statements best describes the clinical picture of
panic disorder?

a. The symptoms of panic attack significantly ameliorate after the first five
minutes.

b. 20% of the patient’s experience loss of consciousness during the attack.

c. Attack length is actually between 2-3 hours.

d. The disorder is limited to the attacks and there is no distress between the attacks.

104) Patient with mood disorder are at high risk to have other disorders.
Which of the following is most highly associated with mood disorder?

a. Panic disorder

b. Gender identity disorder

c. ADD

d. Schizophrenia
105) Bruxism is a movement disorder associated with sleep and involves the
teeth. Which of the following is highly likely to cause this?

a. Clonazepam

b. Propranolol

c. L-dopa

d. Fluoxetine

106) Yoel is diagnosed as having paranoid personality disorder.


Which of the following characteristic made the diagnosis?

a. Yoel was very verbal during their meeting and varied between anger and fear.

b. Yoel often used projection defense mechanism during their conversation.

c. Yoel described how he felt when other children insulted him at early childhood

d. Yoel told the psychiatrist that he can read is thoughts and that he possesses other
powers.

107) Alcohol dependency is a risk factor of suicide.


Which of the following is the best to describe the association between the two?

a. Over 30% of the people who are alcohol dependent commit suicide.

b. A large portion of the alcohol dependents who committed suicide had anti-social
personality disorder

c. Most people who were alcohol dependent and committed suicide were in
psychiatry admission in the year preceding the suicide

d. Most people who were alcohol dependent and committed suicide were female
gender.
108) Which of the following statements best describes the efficacy of
biofeedback treatment?

a. The treatment was found to significantly decrease the frequency of the attacks in
chronic asthma patients.

b. Treatment was found effective in hyperactivity children with ADHD.

c. Treatment improves symptoms in 40% of patients with Raynaud's syndrome.

d. Biofeedback treatment can prevent grand mal seizures in patients with epilepsy
that is medication resistant.

109) Patients with autistic spectrum disorder often also have?

a. Disturbed reaction intensity to sensual stimulus.

b. Bipolar mood disorder

c. Prone to focal reaction of the GI tract

d. Narcolepsy

110) ADD is a disorder that begins during childhood and can continue even
after adolescence. Which of the following statements describes the course of
the disorder after the age of 18?

a. Less than half of patients will have any symptoms during adulthood.

b. Most of the adult patients will present severe hyperactivity.

c. Many patients with adult attention deficit disorder will also have depression due
to low self-esteem.

d. Significant percent of patients with adult ADD had psychotic symptoms during
adolescence.
State Exam 03.03.2015
Made by: Dr. Alhag Abu Anzeh Muhammad (baghdats@yahoo.com)

Part A

1) A 10 year old boy develops pancreatitis following a blow to the abdomen.


During a 5 day hospitalization, he is treated with intravenous fluid and analgesics
and is discharged home on a low fat diet. 4 weeks later he has reoccurrence of
abdominal pain and vomiting. The most likely diagnosis is:
a) Perforated gallbladder
b) Functional abdominal pain
c) Hemorrhagic pancreatitis
d) Pancreatic pseudochyst
e) Chronic pancreatitis
2) A 10 year boy has been having "bellyaches" for about 2 years.
They occur mostly on awakening and before meals. Occasionally he vomits after
the onset of pain. Occult blood has been found in his stool. His father also gets
frequent stomachaches. Which of the following is the most like diagnosis:
a) Porphyria
b) Cow milk protein allergy
c) Meckel diverticulum
d) Functional abdominal pain
e) Peptic ulcer

3) Which of the following is regarded as a known complication of treatment with


total parenteral nutrition (TPN)?
a) Gout
b) Fulminant hepatic failure
c) Dysentery
d) Acute pancreatitis
e) Sepsis
4) A 12 month old girl has been spitting up her meals since 1 month of age.
Her growth is at the ninety five percentile, and she is otherwise asymptomatic and
without findings on physical examination.
Which of the following is the best treatment:
a) Pyloroplasty
b) Duodenal atresia repair
c) Eltroxine
d) Sleep with 30% inclination
e) Traheoesofagial fistula repair

5) New parents ask you how to reduce the chance of their baby to suffer from
sudden death syndrome.
You tell them to place the child in which of the following positions to sleep?
a) Supine position
b) Prone position
c) Seated
d) Sleeping on right side
e) Sleeping on the left side

6) A 2 month old boy is admitted to the hospital for nonbilious vomiting after
breast feeding. On physical examination you can see gastric peristaltic wave and
you can palpate a small mass in the upper abdomen.
The electrolytes show hypokalemic, hypochloremic alkalosis.
Which of the following will be the most expected imaging result?
a) Pneumatosis intestinalis on plain abdominal X ray
b) Double bubble on plain abdominal X ray
c) Intussusception on abdominal X ray
d) pyloric-thickening on abdominal ultrasound
e) Intestinal malrotation on abdominal CT scan
7) A 6 year old girl is sent home from summer camp with a fever of 38.5C,
stiff neck, photophobia and headache. Lumbar puncture reveals: white cell count
1500c/mm, with 75% polymorph nucleate and 25% lymphocytes, high protein and
very low glucose. Which of the following pathogens is most likely?
a) Neisseria meningitides
b) Haemophilus influenzae
c) Enterovirus
d) Borrelia burgdorferi
e) Mycobacterium tuberculosis
8) A parent is awakened in the night by his 2 year old son, who has developed
noisy breathing on inspiration, marked retraction of the chest wall, flaring of the
nostrils and barking cough. He has had a mild upper respiratory infection for 2
days. Which of the following is the most likely diagnosis?
a) Cystic fibrosis
b) Epiglottitis
c) Adenoidal hypertrophy
d) Foreign body in the esophagus
e) Viral croup

9) Which of the following tumors is associated with Epstein Barr virus infection?
a) Adrenal carcinoma
b) Lymphoma
c) Neuroblastoma
d) Chronic leukemia
e) Carcinoma of the colon
10) Which of the following cancer occurs primarily during childhood?

a) Neuroblastoma
b) Renal cell cancer
c) Breast cancer
d) prostate cancer
e) colon cancer

11) The breast fed infant of a mother who is a strict vegan may experience which
kind of anemia?

a) Megaloblastic

b) Microcytic

c) Sideroblastic

d) Pure red cell

e) Aplastic anemia

12) A 6 year old girl who was previously healthy presents with a 1 week history of
nocturnal perineal itching. There are no other symptoms, and findings on physical
examination are normal.
Which of the following conditions will most probably cause the problem?

a) Crohn's disease

b) Oxiuriasis

c) Cow's milk allergy

d) Ulcerative colitis

e) Zinc deficiency
13) A child suffers a provoked bite from a stray dog that was captured by animal
control and appears healthy. The most appropriate action would be to?

a) Confine the dog for 10 days for signs suggestive of rabies.

b) Submit the dog head for examination for rabies.

c) Begin rabies vaccination.

d) Administer human rabies immune globulin (HRIG) and begin rabies vaccination.

e) None of the above, no postexposur prophylaxis is necessary because it was


provoked attack.

14) A 5 year boy presents with a history of grossly bloody urine, puffy eyes, and
headache for one day. He was been a well-child, but he did have a fever and sore
throat about 10 days ago, which resolved without treatment.
Which of the following is the most diagnostic laboratory test result in this child?

a) CMV growth in urine culture.

b) Low IgA in serum.

c) High anti double stranded DNA in serum.

d) Low complement C3 in serum.

e) Large number of leukocytes and bacteria in urine.

15) A 3 year old boy presents to an urgent care clinic with a 3 day history of
abdominal pain and difficulty walking. Abnormal findings include diffuse
abdominal tenderness and swelling of the ankles. The most likely diagnosis is?

a) Periartritis nodosa.

b) Familial Mediterranean fever.

c) Henoch – schonlein purpura.

d) Serum sickness.

e) Crohn's disease.
16) What is the best way to find and evaluate the most significant serious
complication arising from Kawasaki disease?

A) Echocardiography.

B) MRI study of the abdomen.

C) CT scan of the head.

D) Nuclear scan of the lungs.

E) Barium swallow radiography.

17) A 12 month old male infant of Mediterranean origin is noted to have some
pallor. He has been fed similac with iron since birth. His examination is otherwise
normal except for a palpable spleen. A blood count reveals the following:
Hb 8.9g/dl, MCV 67, MCHC 32, RDW 12, reticulocyte count 1,5 %,
platelet count 240,000 mm3.
Which is the most probable diagnosis?

A) Iron deficiency anemia.

B) Folic acid deficiency.

C) Thalassemia .

D) Vitamin B12 deficincy.

E) Normal child.

18) A 15 year old female present to the clinic complaining of fever and abdominal
pain, she was sexually active with two partners and complains of dyspareunia.
On examination she has yellow, thick vaginal discharge, but no visible genital
lesions. She is the most probably infected with which of the following organisms?

A) Candida albicans.

B) Gonorrhea.

C) Mycobacterium tuberculosis.

D) Human papilloma virus.

E) Chlamydia trachomatis.
19) A 4 year old child is observed to hold his eyelids open with his fingers, and to
close one eye periodically in the evening. He has some trouble swallowing his
food, he can throw a ball and runs well.
Which of the following is the most likely to aid in the diagnosis?

A) Muscle biopsy.

B) Creatin phosphokinase (CPK).

C) Effect of test dose of erdophonium.

D) MRI of abdomen.

E) Anti-nuclear antibody (ANA).

20) A 4 year old girl is seen 10 days following an upper respiratory infection.
Her knees are swollen and painful, and her temperature is 39c, the cardiac
examination reveals a grade 4/6 systolic murmur at the apex.
The antistreptolysin titer is elevated.
Which is the treatment of choice for this girl?

A) IV penicillin

B) Oral aspirin .

C) O ral corticosteroids.

D) IM ceftriaxone.

E) IV methotrexate.

21) A 12 year old boy came to the ER complaining of sharp pleuritic chest pain
worse on supine position with a temperature 38.5 on auscultation he had
pericardial rub. What is the most likely diagnosis?

A) Musculoskeletal pain
B) Pericarditis
C) Bacterial endocarditis
D) Mycoplasma pneumonia
E) Pulmonary embolism
22) What is the most likely cause of meconium aspiration?

A) Intrauterine infection
B) Congenital heart disease
C) Cord around the neck
D) Preterm
E) Tracheosophageal fistula

23) A 3 week old girl came for 5 day duration of with rapid heart rate and has
evidence of dehydration with ambiguous genitalia
Electrolytes are NA 120 K 7.5 Hco3 12 Bun 20 an IV was placed and given normal
saline. On admission what’s the most important next step in management:

A) Diuretics
B) Potassium exchange resin
C) Glucose and insulin
D) Antibiotics
E) Corticosteroids

24) A 2 week infant presents with apnea born at term after an uncomplicated
pregnancy mother had rhinorrhea and cough for the past 3 weeks
Now she has severe persisting cough and posttussive emesis.
What is the most likely diagnosis?

A) Bronchitis
B) Asthma
C) CM allergy
D) Pertussis
E) Foreign body aspiration

25) Mother came to the hospital with high blood pressure at 35 weeks and L-P
ratio was done. Which system signifies maturity with high L-P RATIO?

A) Lung
B) Skin
C) CNS
D) Gastro intestinal
E) Immunological system
26) A 3 week gestation age baby with respiratory distress syndrome weaning from
the ventilator with an oxygen saturation of 100% at 50% minimal
Ventilator settings premature infants with respiratory distress syndrome, damage
to which of the following organs/systems following oxygen supply therapy?

A) Alveoli
B) Bronchus
C) Kidney
D) Liver
E) Retina

27) New born presenting with cyanosis and mild tachypnea placed on 95% oxygen
hood and then normalizes. What is the most likely diagnosis?

A) Transposition of great arteries


B) Pulmonary disease
C) Asphyxia
D) Liver disease
E) Methemoglobinemia

28) Infant with white cheese patches on tongue and buccal mucosa with mild
inflammation of mucosa. What is the treatment?

A) Nystatin
B) Ceftriaxone
C) Corticosteroids
D) Penicillin G
E) Aspirin

29) A 3 week infant with microcephaly cerebral calcification on skull x-ray and
blindness. What is the most likely cause?

A) Bilateral subdural hemorrhage


B) Agenesis of corpus callosum
C) Intrauterine infection
D) Erythroblastosis
E) Trisomy 18
30) A 2 month infant was noted to develop constipation for the past week then
flaccid diplegia difficulty suckling and swallowing been colicky as well the grandma
been treating the baby with tea and HONEY. What is the most likely cause?

A) Spinus atrophy
B) Myasthenia gravis
C) Congenital myotonic dystrophy
D) Duchene
E) Botulism toxin

31) Newborn has delayed passage of meconium stools and barium enema
radiograph shows dilated proximal colon and small obstruction of distal colon.
What should be next diagnosis test?

A) Biopsy of rectal mucosae membrane


B) Barium swallow and passage in small intestine
C) Ultrasound study of upper abdomen
D) Sweat chloride test
E) TSH level in blood

32) Newborn has diagnosis with aniridia.


Which of the following tumors could be associated with this finding:

A) Neuroblastoma
B) Wilms tumor
C) Acute leukemia
D) Dysgerminoma
E) Brain tumor

33) 7 year-old girl develops secondary nocturnal enuresis. What is best next test?

A) Serum and urine osmolality


B) Voiding cystourethrogram
C) Ultrasound of kidney
D) Urine analysis
E) Hba1c
34) you are evaluating a 9 month old infant for recurrent fractures following
relatively minor trauma. You note deep blue sclera and bowing of the lower
extremities. X ray examination reveals generalized osteopenia.
Which of the following is most likely diagnosis.

A) Osteogenesis imperfecta
B) Hypoparatirodism
C) Vitamin E deficiency
D) Child abuse
E) Vitamin D deficiency

35) 10 months old female is seen because of wheezing. This is the child's third visit
in the past 2 months for the same complaint. The physician notice that her weight
is less than 3% while her length is at 25th%. What is the most likely Diagnosis?

A) Severe asthma
B) Adenoidal hypertrophy
C) CF
D) Celiac disease
E) Congenital laringomalacia

36) What is the major source of immunity conferred in breast milk?

A) T cell mediate
B) Complement mediated
C) IgM
D) IgA
E) IgD
37) A 5 year -old, 6 days fever, non-purulent conjunctivitis, "strawberry tongue",
swelling of the hands, diffuse rash and enlarged cervical lymph nodes.
Which medication should be included in initial treatment regimen to loosen the
risk of complication?

A) Oral acetaminophen
B) IV antibiotics
C) IVIG
D) Anti TNF monoclonal antibodies
E) Corticosteroid

38) 6 months old infant, chronic lung disease is treated with thiazide diuretic in
order to decrease airway resistance and improve lung compliance.
Which of the following electrolyte abnormal may result from use thiazide type
diuretic:

A) Hypoglycemia
B) Hyperuriemia
C) Hyperkalemia
D) Hypernatremia
E) Hypocalcemia

39) previously healthy 12 year -old boy presents with nausea and vomiting for
2 days. Has no fever or meningeal signs, after IV rehydration he is treated with
antiemetic, half an hour later begins to slowly turn his head to the right and
extreme arching of his back. He is awake and alert.
What most likely cause this reaction?

A) Hypocalcemia
B) Tetanus
C) Focal seizure disorder
D) Metoclopramide use
E) Bacterial meningitis
40) Staphylococcus food poisoning is caused by heat stable, performed
enterotoxin. What are the clinical characteristics associated with this disease:

A) Chills and spiky fever


B) Seizure
C) Pneumonia
D) Abdominal cramps and vomiting
E) Appearance of diffuse impetigo
41) The best treatment for a 4 Year-old boy with uncomplicated
thrombocytopenia who presents with 45,000 platetes and has normal physical
examination is:

A) I.V immunaglobin

B) Amoxicillin

C) Anti-D immunoglobin

D) Platetet transfusion

E) Observation

42) A 16-year-old girl presents to the ED after ingesting 30 tablets of


acetaminophen which of the following is true regarding the toxicity of
acetaminophen overdose?

A) Primarily involves the liver

B) Primarily involves the CNS

C) Is the most severe in pat. Less than 2 years of age

D) Is the best treated with sodium bicarbonat

E) Is the best treatet with dialysis


43) A 1 year old female is seen with a history of fever,arthritis and oral
ulcers,malar rash and photosensitivity.
Which test would be most sensitive of systemic lupus erythmatosus?

A) Anti-nuclear antibody

B) Antidouble stranded DNA

C) C3

D) Anti Ro

E) Antihistone antibody

44) A 5 Mo. Old infant with failure to gain weight has mild degree of clubbing of
hisfinger .During phesical examination he becames agitated and strats to turn
blue.five min.later, after calming drown,his color improves and he seems
comfortable agin.
Which of the following diagnoses best explains his particular symptoms?

A) Aortic stenosis

B) Pericarditis

C) Hypoplastic left heart syndrome

D) Patent ductus arteriosus

E) Tetralogy of fallot

45) Conductive hearing loss is:

A) An uncommon complication of otitis media

B) A permanent condition related to nerve damage

C) Frequently associated with ossicular abnormalities

D) Common in Childern with chronic ear fluid

E) Unlikly when an intact tympanic membrane is present


46) A 15 year old girl short stature,neck webblings,and sexual infantilism is found
to have coarctation of aorta.
Which of the following diagnoses will most likely explain this condition?

A) Celiac disease

B) Chron´s disease

C) Turner syndrom

D) Trisomy 18

E) Heart failure

47) A 12 mo old female infant with failure has to thrive loose foul-smelling stools
each day.serologic testing reveals elevated serum tissue tranglutaminase
antibody.
Which of the following foods can she eat safely without aggravating or inducing
her symptoms?

A) Rice

B) Wheat

C) Oats

D) Barly

E) Rye

48) Which of the following should be included in the differential diagnosis of


infant who is not thriving?

A) Small atrial septal defect

B ) Acute bacterial meningitis

C) Cow´s milk allergy

D) Recurrent otitis media

E) Mild intermittent asthma


49) The developmental appropriate age of a child whorolls back to front,
has a thumb-finger grasp,self-inhibits to ´`no`´,and bangs tow cubes is:

A) 3 mo

B) 8 mo

C) 12 mo

D) 15 mo

E) 18 mo

50) A 15 mo-old girl is evaluatedfor a.10-min-long generalized seizure associated


with a temperature of 40 C.
Which of the following factors in the history is most likely to increase the risk of
future seizures:

A) APGAR score of 3 at 5

B) Family history of epilepsy

C) Clinical evidence of roseola

D) Female gender

E) Presence of 2 Cafe-au-late spots

51) A 55 years old woman presents to the office with erythema of the right breast.
After 2 weeks course of oral antibiotics the erythema still present and right breast
mammography reveals diffusely increased density. Ultrasonography show no fluid
component. Which of the following would be the most appropriate next step?

a) Continue the antibiotics for 2 more weeks

b) Change to broader spectrum antibiotics

c) Obtain an MRI

d) Perform a right breast core biopsy

e) Perform an incision and drainage of the right breast


52) In 75 years old woman with unilateral, bloody nipple discharge.
Which of the following is the most common etiology?

a) Phylloides

b) Paget disease

c) Lobular carcinoma in situ

d) Invasive ductal carcinoma

e) Intraductal papilloma

53) A 60 years old postmenopausal woman has left breast microcalcifications


noted on her yearly screening mammogram. She has no family history of breast or
ovarian cancer. Stereotactic core needle biopsy reveals atypical ductal hyperplasia.
Which of the following is the next most appropriate step?

a) Simple mastectomy

b) Lumpectomy with sentinel lymph node biopsy

c) Tamoxifen treatment for 5 years

d) Needle localized excisional biopsy

e) Annual mammography

54) Which of the following conditions is suitable for consideration of partial breast
radiation?

a) Young age

b) Aymphovascular invasion

c) Tumor larger than 3 cm

d) Estrogen receptor negative tumor

e) Medial quadrant lesion


55) A 65 years old woman is seen in the surgery clinic. It has been 6 weeks since a
recent hospitalization in which she received nonoperative treatment of presumed
perforated appendicitis (intravenous and oral antibiotics for 10 days). She is now
feeling well and is tolerating a regular diet. On physical exam, she has normal vital
signs and an unremarkable abdominal examination.
Which of the following is the next step in management of this patient:

a) Interval appendectomy

b) Abdominal CT scan

c) Diagnostic laparoscopy

d) Colonoscopy

e) C-reactive protein measurement

56) A 65 years old woman had a right colectomy for T1N1M0 adenocarcinoma of
the colon 2 years ago. She received postoperative chemotherapy. On follow up CT
she has a 5 cm biopsy proven recurrence in the left lobe of the liver. The rest of
the CT scan is negative. Her CEA is 10ng/ml. Her colonoscopy is negative.
Which of the following treatment is associated with the best 5 years survival?

a) Hepatic resection

b) Additional systemic chemotherapy

c) Radiofrequency ablation

d) Chemoembolization

e) Hepatic artery infusion


57) A 60 year old man present for elective laparoscopic cholecystectomy. On
examination of his abdomen after insertion of the laparoscope, an incidental
Meckel’s diverticulum is noted.
What is the most appropriate management for this condition?

a) Diverticulectomy

b) Segmental bowel resection

c) Inversion of the diverticulum

d) Enteroscopy

e) There is no need to resect the diverticulum

58) For colon cancer, which of the following factors is associated with lower
survival rates?

a) Bleeding presentation

b) Longer duration of symptoms

c) Intestinal obstruction at presentation

d) Family history of colon cancer

e) Peut's jegher's syndrome

59) A 57 years old woman was recently found to have an invasive adenocarcinoma
of the cecum, her preoperative workup should include PET-CT under which of the
circumstances?
A) Routinely
B) When initial abdominal CT scan fails to show metastatic disease
C) When initial abdominal CT scan shows a single metastasis of the right lobe of the
liver
D) When initial abdominal CT scan shows more than 5 metastatic lesions to the liver
E) When the patient has ulcerative colitis
60) 45 years old man has a 5 year history of idiopathic ulcerative colitis. Although
the past few years he has been asymptomatic from his colitis, he was recently
diagnosed with early primary cholangitis.
Which of the following managements should this patient undergo?
A) Total proctocolectomy with ileal pouch anal anastomosis to prevent progression
of primary sclerosing cholangitis
B) Annual screening colonoscopy to assess for the presence of dysplasia beginning at
10 years from the diagnosis of ulcerative colitis
C) Annual screening colonoscopy to assess for the presence of dysplasia beginning at
5 years from the diagnosis of primary sclerosing cholangitis
D) Annual screening colonoscopy to assess for the presence of dysplasia beginning
now
E) Screening colonoscopy beginning of the age of 55

61) 68 years old woman with NIDDM and myocardial infarction 3.5 weeks ago
presents with right upper quadrant pain nausea and vomiting for 5 days, heart
rate 90 BPM blood pressure 170\100, temperature 38.0 c, tender right upper
quadrant, bilirubin 1.0 mg\dl lipase 48 white blood cells 18,000, ultrasound shows
cholilithiasis, gallbladder wall 6 mm, and common bile duct 6 mm. After 24 hours
of antibiotics therapy her abdominal pain does not improve.
Which is the best next step?
a) Endoscopic retrograde cholingiopancreatography (ERCP)
b) Laparoscopic cholecystectomy
c) Percutaneous cholecystectomy
d) Percutaneous cholecystectomy tube
e) Percutaneous transhepatic biliary tube

62) Which of these conditions is associated with the greatest risk of developing
hepatocellular carcinoma?
a) Alcoholism
b) Primary biliary cirrhosis
c) Non-alcoholic fatty liver disease
d) Alfa-1 anti-trypsin deficiency
e) Diabetes
63) What is true regarding the routine use of nasogastric tube decompression after
abdominal surgery?
a) Reduces the incidence of anastomotic leak
b) Reduces pulmonary complication
c) Reduces the hospital length of stay
d) Increases the incidence of wound infection
e) None of the above

64) 35 years old is admitted to the intensive care unit with a diagnosis of
pancreatitis after an episode of hypotension. In the emergency department over
the next 18 hours he requires intubation and receives 14 L of crystalloid
resuscitation. On morning rounds he has peak airway pressure of 50 cm H2O.
His blood pressure is 80\50 mm Hg and his bladder pressure is 25 cm H2O.
He is placed on norepinephrine infusion. Abdominal ultrasound evaluation for
oliguric renal failure reveals no hydronephrosis but marked ascites.
Which of the following is the next most appropriate step in the patient's
management?
a) decompressive laparotomy
b) addition of vasopressin infusion
c) ultrasound guided percutaneous drain placement
d) evaluation of adrenal insufficiency
e) 2 L crystalloid infusion

65) After an uncomplicated appendectomy for acute appendicitis, pathologic


examination reveals carcinoid tumor in the specimen.
All of the following are indicated for repeat operation and right hemicolectomy
except?
a) Tumor size smaller than 1 cm
b) Lymphovascular invasion
c) Present of goblet-cell features
d) Invasion of the appendiceal mesentery
e) Tumor location at the base of the appendix
66) 53 years old smoker presents with a 3-cm symptomatic umbilical hernia.
Which of the following herniorraphies is least likely to lead to recurrence?
a) Figure of -8 suture
b) Simple interrupted suture
c) Simple continuous
d) Mesh
e) Vest over pants

67) 63 years old man with a-50 pack per year history of smoking cigarettes
undergoes an endovascular aneurysm repair to exclude his 6.5 cm infrarenal
abdominal aortic aneurysm. On post-operative day 2, the patient has a bloody
bowel movement.
Which of the following is the most appropriate next step?
a) CT scan of the abdomen and pelvis
b) Abdominal angiogram
c) Tagged RBC nuclear medicine scan
d) Flexible sigmoidoscopy
e) Exploratory laparotomy

68) A 54 years old obese but otherwise healthy woman is postoperative day 3
following total abdominal hysterectomy for uterine cancer. She develops
tachypnea, tachycardia and hypotension. You suspect pulmonary embolus.
After resuscitation with IV fluid and IV heparin.
Which of the following is the most appropriate next step?
a) D-dimer testing
b) Echocardiogram
c) Pulmonary arteriogram
d) Ventilation-perfusion Lung scan
e) CT angiogram of the chest
69) A high-school football player sustains a closed tibia fracture. He undergoes
repair of the fracture with intramedullary nailing of the tibia. Approximately 8
hours later, the patient complains of ipsilateral severe pain in the injured leg.
His distal pulses are detectable by Doppler exam only.
Which of the following is the most appropriate next step?
a) Elevate the lower extremity
b) Administer narcotics
c) Angiogram
d) Assess arterial pressure indexes
e) Measure compartment pressure

70) A 25-years old woman is brought to the Emergency room after a rollover
motor vehicle crash, she has a femur fracture, her Glasgow coma scale score is 7,
her right pupil is dilated, her systolic blood pressure is 80 mm Hg, and her heart
rate is 120 beats per minute.
Which of the following is the least important initial intervention in the care of this
patient?
a) Intubation
b) CT scan of the head
c) Chest x-ray
d) FAST
e) Fluid resuscitation

71) A 45 y/o woman with history of HTN comes to the ER with abdominal pain.
She undergoes CT scan of the abdomen that rules out an acute pathology.
An incidental 3 cm mass is seen in the right adrenal. Further investigation reveals
high level of urine metanephrines.
Base on those finding, the adrenal mass is most probably located in the:

a) zona glomerulosa

b) zona fasciculate

c) zona reticularis

d) medulla

e) para adrenal tissues


72) A 53 years old woman complaints of weakness, anorexia, constipation and
back ache. Laboratory reveals: albumin-3.2mg/dl, calcium 14mg/dl,
phosphate- 2.6mg/dl, chloride-108mg/dl, BUN – 32.
Which of the following would be the initial treatment in this patient.

A) Administration of IV saline

B) Thiazide diuretics

C) IV phosphor

D) Mithramycin

E) Neck exploration and removal of parathyroid gland

73) 44 y/o was diagnosed with 2.5 cm nodule in the left thyroid lobe.
She underwent an FNA of the nodule which was compatible with a follicular
neoplasm. Which of the following is true regarding this cytologic findings?

A) Confirm follicular thyroid carcinoma.

B) Rules out papillary thyroid carcinoma.

C) Associated with 20-30% risk of malignancy.

D) Necessitates a repeat FNA.

E) Given this findings the pz may be followed with a repeat neck exploration US in 6
month.

74) Which of the following is primary treatment for Barrett’s esophagus with low
grade dysplasia:

A) Follow up.

B) Antireflux operation.

C) Endoscopic radiofrequency ablation.

D) Photodynamic therapy.

E) Esophagectomy.
75) 45 y/o comes to the ER with abdominal pain. He is taken to the OR.
And is found to have perforated duodenal ulcer. What is the treatment of choice?

A) Simple closure with omental patch.

B) Truncle vagotomy and pyloroplasty.

C) Truncle vagotomy and anterectomy.

D) Highly selective vagotomy with omental patch.

E) Hemigastrectomy

76) A 62 years old man treated with NSAIDs in past for month for backache comes
to emergency room with melena. On the examination his blood pressure is 105/70
and heart rate 110/min, his abdomen is soft and non-tender and on rectal
examination he has melena.
Examination and treatment in this patient will include all of the following except:

A) Plain abdominal film

B) Gastrectomy

C) Treatment with PPI's

D) IV liquids

E) NG tube insertion
77) A 43 years old man with history of long standing umbilical hernia comes to the
emergency room with abdominal pain, vomiting and a painful protrusion at the
umbilical hernia sight that appeared 24 hours ago. On exam he is tachycardic
120/min, fever 38, his abdomen is distended with peritoneal signs and there is a
protrusion at the umbilical sight with redness.
Which of the following is true regarding the treatment of this patient?

A) The patient needs an emergent surgery

B) A manual attempt to reduce hernia can be done and if hernia returns the patient
can be operated in elective manner

C) The patient may be treated conservatively with NPO, nasogastric tube liquids and
antibiotics

D) An aspiration of the protrusion should be done to rule out an abscess

E) The patient should be admitted to the department and surgery should be planned
for next day.

78) A 45 years old man with ITP is being prepared to the surgery.
Which one of the following sentences is true regarding the patient's need for
immunization to prevent OPSI?

A) The patient should be immunized against atypical bacteries such as Hemophillus,


Legionella and Pneumococcus

B) The patient should be immunized against encapsulated bacteries such as


Pneumococcus and Meningococcus

C) Only female patients planning pregnancy should be immunized for measles

D) There is no need in immunization, only prophylactic antibiotic is indicated

E) There is no need in immunization or antibiotic therapy since the risk for OPSI is
very low
79) Young female is injured at a motor vehicle excitant, as seat belted passenger.
She is hemodynamically and respiratory stable and has a seat belt sign on her
abdomen. Her abdomen is soft with mild right sided tenderness. CT scan
demonstrates free air in proximity to ascending colon. What is the next step?

A) Admission to surgical ward with close monitoring and antibiotics.

B) Exploratory laparotomy/ laparoscopy.

C) Colonoscopy to search for perforation and clip it.

D) Repeat CT scan with rectal margins.

E) Diagnostic peritoneal lavage.

80) 35 y/o male who rides cross country bies, have an abdomen trauma and is
brought to the ER. He is alert and hemodynamically and respiratory stable. There
are abrasions and tenderness under left costal margins. CT scan shows 3rd stage
splenic rupture, no active bleeding. Mild moderate amount of abdominal blood.
Next step:

A) Exploratory laparoctomy.

B) Angiography and splenic artery embolization.

C) DLP to rule out bowel injury.

D) Admission and monitoring in ICU.

E) Admission 3 vaccines to prevent OPSI.


81) A 51 years old female comes to the ER and complains of RUQ pain that started
the night before. The pain began following a fatty meal and was accompanied by.
A fever 38.5c and nausea. On exam she has clear RUQ tenderness with positive
murphy's sign, her WBC count is 11,000 and full chemistry with liver enzymes are
normal. What is the most common etiology for the patient's condition?

A) Excessive alcohol consumption


B) Autoimmune disease
C) Gallstones
D) Use of isothiazide
E) Hepatitis

82) All the following are indications for cholecystectomy except:

A) Biliary colic.
B) 3cm gallstone.
C) 7mm polyp within the gallbladder.
D) Pancreatitis in face off cholelithiasis.
E) Hepatitis.

83) 61 year old male is at the ER complaining of scleral and skin jaundice, dark
urine, acholic stool, and weight loss of 10kg in the past 2 months. On exam he
looks severely jaundiced at the skin and sclera with no other findings. His bilirubin
is 12, mostly direct, with substantial cholestatic elevated liver enzymes and
increase CA 9-19 to 280. On US there is significant extra and intra hepatic bile duct
dilatation. What is the most likely diagnosis for this patient?

A) Bile obstruction by gallstones.


B) Periampular tumor.
C) Hepatitis.
D) Hemolytic anemia.
E) Gilbert syndrome.
84) Which of the following fits one of the criteria for the diagnosis of SIRS?

A) Fever 37.5c.
B) WBC 12,500.
C) Respiratory rate of 18.
D) Heart rate of 85
E) Systolic blood pressure<90.

85) 27 year old female is admitted for over a week to ICU following multi organ
trauma has potassium 6.2 mmol/l.
Which of the following ECG changes will be a marker for possible development of
fatal arrhythmia secondary to hyperkalemia?

A) Wide T wave with prolonged QT.


B) Spiked T waves that are higher than R wave.
C) Absent P wave with development of U wave.
D) ST depression with no relation to T wave.
E) Shortened QT interval with flattening of T wave.

86) What is the most common tumor of the small bowel?

A) Adenocarcinoma.
B) Squamous cell carcinoma.
C) Carcinoid.
D) Malignant GIST.
E) Lymphoma.

87) Which of the following is correct regarding HNPCC?

A) Autosomal recessive.
B) Increase incidence of breast cancer.
C) Increase incidence of endometrial cancer.
D) Increase risk of left sided colon cancer.
E) APC gene mutation.
88) A patient undergoes elective right hemicolectomy with no stool spillage on
breach of sterile technique. This procedure is defined as:

A) Clean.
B) Dirty.
C) Contaminated.
D) Clean-Contaminated.
E) Infected.

89) 45 year old female arrives to clinic because of palpable mass in the tail of her
left breast since yesterday. Family history is positive for the mother with breast
cancer at age of 40. On exam there is a palpable mass 2cm with irregular borders
in the tail of the left breast with no axillary lymphadenopathy.
What is the next step in the management of this patient?

A) Excisional biopsy.
B) Breast US + mammography.
C) Observation and follow up in clinic in 3 months.
D) Breast MRI.
E) Genetic counseling.

90) 40 year old female admitted to the ER for fever 38.5c and RLQ pain. On exam
she has tenderness and peritonitis located to the RLQ. Labs are normal, except
WBC 18,000. Abdominal CT demonstrate right sided diverticulitis with no evidence
of an abscess, free air or contrast leak.
What is the most appropriate initial management?

A) Nothing per os, IV fluids and antibiotics.


B) Colonoscopy.
C) Right hemicolectomy with end ileostomy.
D) Right hemicolectomy and primary anastomosis.
E) Fiber enriched diet.
91) A 31 years old female arrives to the breast clinic for first examination after her
sister died of breast cancer at the age of 35, three months ago. A paternal aunt
had breast cancer at the age of 40. Breast examination - no palpable findings.
What would you recommend this patient?

a) Breast and axillar US


b) Follow up in clinic in 6 months
c) Plan for prophylactic bilateral mastectomy
d) Begin prophylactic therapy with tamoxifen
e) Genetic counselling

92) A 43 years old female is obese and usually healthy and has no prior history of
admissions or previous surgeries. She is brought to the ER complaining of right
upper abdominal pain that began following an especially large dinner. She
complains of nausea with no vomiting, no fever, no diarrhea and no urinary
complaints. On examination- soft abdomen with RUQ tenderness. Labs- mild WBC
elevation and all liver enzymes are normal.
What is the next step in establishing the diagnosis?

a) Abdominal and pelvic CT scan


b) Endoscopic US
c) No need for further tests and the patient can be discharged home
d) Upper abdominal US
e) HIDA (hepatobiliary imino-diacetic acid) scan
93) 91 years old male with dementia is nursed at an old age home. He has IHD
with CHF and EF of 25%, DM, HTN, PAF on Coumadin, and severe COPD. He is
referred to the ER for high fever, abdominal pain, vomiting, and low BP. on
examination he is deteriorated. Irregular HR of 130-150 bpm and borderline
systolic BP 90-100. On examination he has severe upper abdominal tenderness,
with a palpable very tender mass in the RUQ. Labs: WBC 29,600, 34% bands,
normal urine enzymes, glucose 430 mg/dL. Upper abdominal US shows a very
distended gall bladder up to 4.7cm with multiple gallstones, no bile duct dilation.
What is the most appropriate treatment for this patient?

a) Urgent laparoscopic cholecystectomy


b) Urgent open cholecystectomy
c) Percutaneous cholecystectomy tube
d) IB antibiotics and cholecystectomy in 4-6 weeks
e) Urgent ERCP

94) Which is the most common cause of incisional hernia:

a) Obesity
b) Diabetes
c) Immune deficiency medications
d) Advanced age
e) Surgical site infection

95) Which of the following electrolyte abnormalities is characteristic of massive


transfusion:

a) Hypocalcaemia
b) Hypokalemia
c) Hypercalcemia
d) Hypomagnesaemia
e) Hyperphosphatemia
96) A 60 years old male complains of epigastric pain for about 3 months. As part of
the evaluation a gastroscopy was performed which demonstrated a 3cm tumor at
the stomach fundus. A biopsy is positive for GIST (gastro-intestinal stromal tumor).
What is the appropriate treatment?

a) GLEEVEC
b) Chemotherapy
c) Resection of tumor with clear margins
d) Total gastrectomy with Lymph Node Dissection
e) Radiation therapy

97) A 60 year old male complains of epigastric pain for about 3 months. As part of
the evaluation a gastroscopy was performed which demonstrated a 3cm tutor at
the stomach fundus. A biopsy is positive for lymphoma.
What is the appropriate treatment?

a) GLEEVEC
b) Chemotherapy
c) Resection of tumor with clear margins
d) Total gastrectomy with Lymph Node Dissection
e) Radiation therapy

98) A 40 year old female that is usually healthy with no previous abdominal pain
arrived to the ER due to severe left flank pain. An abdominal CT scan
demonstrates left ureter kidney stone. There were gallstones noted also. She was
admitted to the urology department and was discharged after the kidney stone
has passed. Following her discharge she comes to the surgical clinic for further
management.
What would you recommend?

A) There is a need for cholecystectomy as soon as possible to prevent gallstones


complications
b) There is a need for cholecystectomy but is should be performed only in 6 weeks
c) There is a need to perform abdominal US to confirm gallstones
d) There is no need to cholecystectomy as this patient is asymptomatic
e) Patient should be referred for gallstone lithotripsy
99) A 35 years old female is found to have platelet level of 18,000 mm3 on routine
blood test. The patient is asymptomatic. She is admitted and diagnosed with ITP.
What will her treatment include?

a) 6 units of platelets
b) Initiate treatment with IVIG
c) Steroids
d) Laparoscopic splenectomy
e) Observation only

100) 61 years old women with a history of unstable angina complains of


hematemesis after retching and vomiting following a night of binge drinking.
Endoscopy reveals a longitudinal mucosal tear at the gastroesophageal junction,
which is not actively bleeding.
Which of the following is the next recommended step in the management of this
patient?

a) Angiography with embolization


b) Balloon temponade
c) Exploratory laparotomy, gastrostomy, and over sewing of the tears
d) Systemic vasopressin infusion
e) Expectant management

101) A 41 years old women, with regular menstrual cycles until a year ago,
complains of heavy vaginal bleeding. BP 116/73, pulse 86, Hb 11.5 gr%, b-HCG
negative, normal pelvic examination and endometrium is 15mm thick on US
examination. An endometrial biopsy is performed.
What is the preferred treatment under these circumstances?

a) Oral progesterone
b) No treatment should be initiated before biopsy result
c) Oral estrogen
d) IM injection of GnRH analogue
e) Ikaclomin
102) A 15 years old girl complains of vaginal bleeding. Menarche at the age of 13,
with no menstrual periods of the last 12 months. Physical examination is normal,
and the bleeding originates from the uterus. Abdominal US reveals a 1.5 cm
subserotic myoma and endometrial mucosa thickening of 10mm. b-HCG negative,
FSH 71 U/L , estradiol 1025 pMol/L.
Which of the following is the most appropriate next diagnostic test?

a) Repeat normal profile


b) Endometrial biopsy
c) Biopsy of the myoma
d) PAP smear
e) Hormonal therapy can be initiated with no further investigation

103) Which of the following is true regarding complication of IUD’s?

a) The risk of PID does not increase beyond the first month after insertion of the
device
b) The risk of PID is stable over the usage period
c) Infectious with Proteus Vaginalis is common
d) Mirena IUD include medroxy-progesterone and causes endometrial thickening
e) Increase the risk of ectopic pregnancy

104) Which of the following is caused by progesterone-only oral contraceptive


(minipill)?

a) Decrease viscosity and turbidity of cervical mucus


b) More efficient than combined contraceptives
c) Increase FSH but not LH
d) Increase libido and mood
e) Increase bone mass if initiated before the age of 30 years
105) Which of the following is true regarding the diagnosis of syphilis:

a) VDRL has increased sensitivity compared to FTA-abs


b) FTA-abs are a useful marker for disease activity
c) VDRL has high specificity and limited sensitivity
d) There is no need for diagnostic tests in the presence of typical lesions in tertiary
disease
e) FTA-abs are diagnostic even in the presence of SLE

106) Which is true regarding the treatment of bacterial vaginosis:


a) It is not a sexually transmitted disease and therefor there is no need to check
sexual partners
b) Vaginal ph is low, as apposing to trichmoniasis
c) Large quantities of lactobacillus bacterial on vaginal smear
d) Treatment during pregnancy reduces the incidence of preterm labor
e) Vaginal culture is essential to diagnosis

107) Which of the following is a sexually transmitted disease and therefor


warrants treatment of sexual partners
a) Recurrent vaginal trichomonos
b) First diagnosis of bacterial vaginosis
c) Recurrent secondary herpes genitalia
d) First diagnosis of vaginal candida glabrata
e) Premenopausal genital lichen sclerosis

108) A 25 year woman, amoxicillin sensitive, on the 10th weeks of her third
pregnancy is diagnosed with syphilis.
Which of the following most appropriate treatment:
a) Oral tetracycline
b) Intravenous erythromycin
c) Desensitation followed by penicillin treatment
d) Intravenous treatment with vancomycin
e) Oral doxycycline
109) A 45 year old woman complain of irregular vaginal bleeding. Abdominal
sonogram reveals a multi-myomal uterus appropriate for 15 week of gestation and
8mm regular endometrial thickness, PAP smear is normal, endometrial biopsy at
day 12 reveals proliferation. What is the best caurse of treatment:
a) GnRH Analogues
b) Combined oral contraceptive
c) Mirena IUD
d) IUD that secretes copper
e) Medroxy-progesterone every 3 month

110) What is characteristic of serous ovarian tumors compared to mucinous


ovarian tumors:
a) Lower malignancy rates
b) Smaller mean diameter
c) Older age of presentation
d) Later clinical appearance
e) Of non-epithelial origin
Part B:

1. A 38 Years old woman complains of severe pelvic pain and tensmus during her
menstrual period and Dyspareunia, on physical examination during her Menstrual
period there is marked sensitivity upon uteral mobilization and sensitive granules
on the sacrouteral ligament and vaginorectal septum with no additional findings.
Which of the following diagnostic tests will definitely verify the presumed
diagnosis:

a) No further testing needed


b) Abdominal and pelvic CT
c) Sonographic imaging
d) Laparoscopic investigation
e) Levels of CA125

2. In which of the following Dilation and Curettage will assist in Diagnosis and
treatment ?

a) Incomplete abortion, Missed abortion, Adenomyosis


b) Incomplete abortion, missed abortion, Placental polyps
c) Missed abortion, Endometriosis, Retention of Placenta
d) Retention of Placenta, Placental Polyps, submucosa fibroids
e) Retention of Placenta, Missed abortion, corneal pregnancy

3. Which of the following hormonal Profiles is characteristic for Kleinfilter


syndrome?

a) Low LH, Low FSH, Low Testosterone


b) High LH, Low FSH, High Testosterone
c) High LH, High FSH, High Testosterone
d) High LH, High FSH, Low Testosterone
e) High LH, Low FSH, High Testosterone (This answer was cancelled as it is the same
like B)
4. 25 Years old woman comes to the clinic for infertility workup، She has a regular
28-day menstrual cycle، in this woman، which of the following would indicate
ovulation?

a) Blood Progesterone of 12ng/ml on day 21


b) Gland proliferation on endometrial biopsy on day 21
c) Fern shaped dried cervical mucus on day 21
d) Decrease in Basal temperature on day 21
e) Increase in LH and decrease in progesterone on day 21

5. Which of the Following indicated complete absence of ovarian reserve?

a) High FSH, Low estradiol


b) Normal FSH, High estradiol
c) High FSH, High Estradiol
d) Low FSH, Low estradiol
e) High FSH, Slightly increased estradiol

6. 34 Years old couple has completed an infertility workup with the following
results
Female: 32 menstrual cycle, Progesterone 7 ng/ml, uterus imaging revealed
normal uterine cavity with contrast material spreading into the pelvis .
Male: Sperm count 19 million sperms/ml, 45% motility, 5%normal morphology
What is the most appropriate diagnosis for this couple ?

a) Infertility due to luteal phase failure


b) Infertility due to Male factors
c) Infertility due to uterine factors
d) Unexplained infertility
e) Infertility due to a combination of an ovulation problem and the woman’s age
7. Which of the following is a complete contraindication for Hysterosalpingogram?

a) Suspected uteral septum


b) Primary infertility
c) Presence of IUD
d) Suspected intrauterine mass
e) Suspected bicorneal uterus

8. Which of the following does not cause secondary amenorrhea?

a) Ovarian failure in a 35 years old woman


b) Pregnancy
c) Sheehan’s syndrome
d) Asherman’s syndrome
e) Kallman Syndrome

9. Which of the following injuries may lead to amenorrhea?

A. injury to posterior hypophysis.


B. injury to the olfactory nerve.
C. injury to the accurate nucleus.
D. injury to hypocampus.
E. injury to corpus callosum.

10. In which of the following in pregnancy (spontaneous or after treatment)


a possibility?

A. Deletion in the long arm of Y chromosome.


B. 47XXY karyotype.
C. A mutation in the AMH gene.
D. 45XO karyotype.
E. 48XXXy karyotype.
11. Which of the following does not cause hirsutism in a woman?

A. 5 alpha-reductase deficiency.
B. 21 hydroxylase deficiency.
C. Aromatase deficiency.
D. Sulphatase deficiency.
E. 11 hydroxylase deficiency.

12. 22 year old woman، completion of menstruation every 6 months and ask for
the treatment in order to establish a regular monthly pattern. She refuses to use
estrogen-containing agent.
Which of the following is not appropriate for treat this woman?

A. Cyproterome acetate.
B. Drospirinone.
C. 17 hydroxy progesterone.
D. Danazol.
E. Natural progesterone.

13. Which of the following is true regarding over-stimulation-syndrome?

A. There is reduced number of antral follicles early in the mens cycle.


B. Resolution is faster if there is implantation of 2 embryos.
C. Basal FSH levels are not are not increased on day 3 in patient at risk.
D. There is a better prognosis for rapid resolution if a pregnancy accomplished.
E. These women are at increased risk for bleeding.

14. Which of the following changes in the maternal gastrointestinal tract during
pregnancy?

A. Atrophy of gums.
B. Increase motility.
C. The appendix is shifted from its usual location.
D. Hypertonic hypotony of the gallbladder.
15. Which of the following is true regarding fetal-placental system?

A. the placental origin is partially maternal and partially fetal.


B. placental blood flow is protected from maternal hypotension.
C. placental Blood flow reaches 500-700 ml/min at term.
D .cholesterol is converted to HCG in the placenta.
E. The incidence of cord around the neck is 2% of all deliveries.

16. A 23 year old woman complain of abdominal pain 36 hours after a vaginal
delivery. On examination she has an oral temperature of 38.5.
Which of the following will contribute the most to her diagnosis?

A. BP, HR, rectal temp, breast examination.


B. Examination the lower back (location of epidural catheter insertion), suprapubic
tap, chest x-ray.
C. Gynecological sonogram, examination of the perineum, examination of the calves.
D. Complete blood count, palpation of uterus, urine culture.
E. Culture of uterine cavity

17. A woman on her 32rd week after first pregnancy presents to your clinic.
On examination the uterine height is appropriate with 36 weeks PMA.
Which of the following is the most correct regarding her condition?

A. Maternal obesity is the most likely explanation


B. Oral glucose tolerance test with 100 gr glucose will determine her diagnosis
C. A cesarean section should be recommended
D. A recalculation of gestational age is required

18. Which of the following tests is the most helpful in the evaluation of twin
pregnancy?

A. Weekly cardiotocography to determine early contractions from the 28th week


PMA onwards
B. Sonography to determine cervical length from the 16th week PMA onward
C. Early sonogram to determine the chorionicity of the fetuses
D. Genetic counseling on the 16th week PMA
E. Maternal CBC to rule out anemia.
19. Which of the following regarding the cardiovascular system during pregnancy
is false?

A.The maternal cardiac output peaks at 20-24 weeks PMA


B. The physiologic increase in maternal blood flow increase maternal cardiac output
C. The increase in cardiac output is mostly due to the increase in maternal heart rate
D. Cardiac output in a single heart beat is influenced by the maternal body position
E. The maternal cardiac output remains increased for the first few days following
delivery.

20. Which of the following presentations is consistent with vaginal delivery?

A. Brow and nose anterior presentation


B. Brow and nose posterior presentation
C. Face and chin anterior presentation
D. Face and chin posterior presentation

21. A 24-year-old G2P2 is now 8 months post a vaginal delivery. After the delivery
she developed massive bleeding and received blood transfusion.
She complains of amenorrhea and inability to breast feed her baby. She denies
headaches, visual disturbance, or any medications.
What is the most likely diagnosis ?

A. Sheehan’s syndrome (necrosis of the frontal hypophys)


B. Ovarian failure
C. Polycystic ovary syndrome
D. Asherman syndrome (uterine adhesion)

22. Which of the following is correct regarding cesarean section with a classical
incision ?

A.The incision is horizontal both to the uterus and to the woman’s body
B. The recovery is easier than when using the Pfannenstiel incision
C. It is required in placenta previa
D. It is the most common incision in modern obstetrics
E. It allows a vaginal delivery in the following pregnancy
23. Which of the following is the most important medication for
severe eclampsia?

A) Magnesium sulfate
B) Furosemide
C) Apresoline
D) Sodium nitroprusside
E) Phenytoin

24. A 20-year-old woman in her 33rd week of her first pregnancy presents with
severe headaches and visual blurring , her blood pressure is 180/110, urine
proteins is +++, AST 120, ALT 110 and platelet count is 60,000. Fetal monitor is
normal. Which of the following is the best management for this woman?

A. Magnesium sulfate, antihypertensive medication and induction of labor


B. Magnesium sulfate, antihypertensive medication, steroids for fetal lung
maturation and induction of labor
C. Magnesium sulfate, antihypertensive medication. If blood pressure decreases,
induction at 37 weeks PMA
D. Magnesium sulfate, antihypertensive medication and induction of labor only if
there are unfavorable changes in fetal monitor
E. Phenytoin, antihypertensive medication and immediate delivery

25. Which of the following test is the most important in ruling out birth defect
early in pregnancy in women suffering from gestational diabetes mellitus?

A. Optical fundus exam


B. Urine creatinine clearance and urine protein
C. ECG
D. Hemoglobin A1 C
E. Blood glucose levels
26. A 44-year old man complains of episodes of chest pressure during exercise.
He is healthy with no medication and has never undergone cardiac
catheterization. His ECG show normal sinus without evidence of ischemia.
What is the test of choice to rule out myocardial ischemia?

a. ECG recording during exercise


b. Cardiac scan during exercise
c. Cardiac scan following injection dipyridamole
d. Cardiac eco during exercise
e. Cardiac eco following injection of dobutamine

27. A 82-year old female is referred to the ER for general malaise, nausea and one
episode of vomiting. Below is her ER ECG.
What are the medical problem and the required treatment in this case?

A. Sinus bradycardia, admit for observation only


B. 2nd degree AV block, attempt treatment with Atropine
c. 2nd degree AV block, attempt adenosine treatment
D. 3rd degree AV block, there is indication for a pacemaker implantation
E. Atrial Flutter with pericardial conduction to the ventricles, attempt atropine
28. Which of the following is correct regarding the approach for medical
management of patient with congestive heart failure?

a. Initiating ACE inhibitor is recommended only for cases with NYHA functional class
less than III.
b. The is fluid retention the diuretic medication of choice is the aldosterone
antagonists.
c. In all cases with a QRS complex is wider than 120ms implantation of CRT type
pacemaker is recommended.
d. ACE inhibitor, is the medication of choice for every NYHA functional test.
e. ICD type pacemaker insertion is recommended for every NYHA functional class

29. A 42-year-old female had an echo because of heart murmur and demonstrated
mitral valve prolapse. Several month later she is due to have a dental procedure.
Which prophylactic antibiotic therapy is required?

a. Yes, because MVP is consider a valve defect.


b. No, because MVP is not valve defect that increase the risk for endocarditis.
c. Yes, only if she had previous episode endocarditis.
d. No, because dental procedure are not considered to increase the risk for
endocarditis.
e. Its depend in the procedure that is expected the degree of gingival injury.

30. A-30-year-old female is admitted with fever, chest pain and pathological ECG.
On echo mild pericardial effusion is diagnosis. Blood test show increase
inflammatory markers.
Which of the following statement is true regarding to this case?

a. The use of NSAID is not consider effective in pericarditis and steroid and
colchicine should be initiated.
b. In these cases there is a relative contraindication for anticoagulation as it increase
the risk for cardiac tamponade.
c. Prednisone at dose of 40-80 mg/day in treatment of choice in case of highly
suspicious of bacterial infection of the pericardial space.
d. use of NSAID, in contrast to colchicine reduce the risk of disease recurrence.
e. NSAIDs treatment should be use continuously at birth at high dose for at least 60
days to decreasing the dose.
31. Metabolic acidosis is one of the problems associated with chronic renal failure.
Which of the following is the physiological consequences and treatment?

a. Metabolic acidosis in patients with chronic renal failure is associated with normal
anion gap and therefor is not influenced by the patient's potassium
b. Metabolic acidosis in patients with chronic renal failure associated with wide
anion gap and therefore treatment is accompanied by correcting the patient's
potassium.
c. Metabolic acidosis in patients with chronic renal failure can be associated with
normal or wide anion gap and is not associated with patient's potassium.
d. Metabolic acidosis in patients with chronic renal failure can be associated with
normal or wide anion gap and requires the use of bicarbonate when its level is lower
than 12 mmol/l.
e. Metabolic acidosis in patients with chronic renal failure is often mild and
nevertheless is associated with injury to patient's proteins.

32. Which of the following medication, used for prevention of transplant of


kidney, causes paradoxically renal function injury and exacerbation of arterial
hypertension?
a. Glucocorticosteroids
b. Cyclosporine A
c. Azathioprine
d. Mycophenolate metetil
e. Takrolimus
33. An 85 female undergo cardiac catheterization with insertion of a stent to the
right coronary artery. A day following the procedure her body temperature
increase to 38.5 with new onset of renal failure, there is a blue and painful area on
the first toe of her right foot, and she become disoriented.
Which of the following is true?
a. It is the most probably a stent infection and it should be removed as early as
possible
b. It is most probably infective endocarditis and broad spectrum antibiotics should
be initiated as early as possible
c. It is most probably a cholesterol emboli and there is no effective treatment for
end organs injury
d. It is most probably a stroke due to atherosclerosis and patient should be referred
to angiography as early as possible
e. It is most probably a delayed reaction to the use of contrast drug catheterization
and patient should be referred to dialysis as early as possible

34. Which of the following the recommended urinalysis for patients with diabetes
type 1 or 2?
a. Patient with type 1diabetes microalbuminemia should be ruled out after 5 year of
disease as the renal injury does not begin with type 1 diabetes onset
b. Patient with type 2 diabetes microalbuminemia a should be ruled out after 5
years of disease as the renal injury does not begin with type 2 diabetes onset
c .In patient with type 1 diabetes microalbuminemia should be ruled out
immediately with disease onset as renal injury begin with type 1 diabetes onset
d. In patient with type 2 diabetes microalbuminemia should be ruled out
immediately with disease onset as renal injury begin with type 2 diabetes onset
e. In patient with type 1 diabetes microalbuminemia should be ruled out
immediately with disease onset as the renal type 2 injury dictates the medical
management of this disease.
35. A patient at menopause is consulting regarding the advantages and
disadvantages of hormonal replacement therapy.
What is recommended for this patient?
a. The use of HRT reduce the risk for overall mortality but increase the risk for
colorectal malignancy
b. The use of HRT reduce the risk for neck of femur fracture but does not reduce the
risk of death
c. The use of HRT increase the risk for pulmonary embolus but decrease risk of
breast cancer
d. The use of HRT increase the risk for strokes but decrease the risk for overall death

36. A 34 years old female describes pallor of the tip of her fingers with cold
exposure the pallor changes to cyanosis and then to redness once the hands are
warmed. An expert diagnosis her suffering from Raynaud's phenomenon.
What information can be given to this patient?
a. Avoid this phenomenon by avoiding exposure of the fingers to cold
b. Avoid smoking in most of the cases there is prophylactic therapy
c. Avoid smoking in most of the cases there will not be a new medication to avoid
this phenomenon
d. Medication of choice is an alpha receptor inhibitor
e. The first line of treatment is digital sympathectomy as most medication and
smoking cessation do not prevent the recurrence of this phenomenon
37. Which of the following is true regarding the chronic management of asthma
patient?
a. The accepted approach is step down- that is aggressive treatment with systemic
steroids is given at the beginning with mid disease.
b. The accepted approach is step up- that is treatment with systemic steroids and
gradual addition of beta-antagonist.
c. The initiation of (SABA) short acting beta antagonist is for disease graded as mild
persistent.
d. The initiation of (LABA) long acting beta antagonist is for disease graded as
moderate persistent.
e. The addition of omalizumab is accepted treatment for disease graded as
moderate persistent.

38. A 51 years old male arrives to the ER complaining of high fever and pus in his
cough. Physical examination and chest x-ray diagnosis right pneumonia.
The ER physician ask for gram stain and culture.
Which of the following is correct regarding efficacy of this test?
a. In real sputum there will be at least 25 neutrophils and no more than 10
squamous cells, but even in such cases the rate of positive sputum culture does not
exceed 50 %
b. Sputum culture have an efficacy of 80% (in diagnosis pathogen) in same samples
with less than 25 neutrophils and over 10 squamous cells.
c. For sputum culture there is a known equal valve for mild to moderate community
acquired pneumonia and severe community acquired pneumonia.
d. The only value of sputum gram stain is that it can be set to culture.
e. The yield of using sputum culture is equal when it is produced by cough or it is
aspirated through an endotracheal tube.
39. Which of the following statements is correct regarding different accept
treatments for patients with COBD?
a. The efficacy of theophylline is similar to beta agonist pr anti cholinergic
b. Adding antibiotics to treatment to COBD exacerbation is not an accepted without
any conclusive evidence of pneumonia
c. The most common side effect of systemic steroid treatment in COBD exacerbation
is marked increase in arterial blood pressure
d. Patient with COBD exacerbation should be supplied with high o2 in order to
increase the measured saturation to above 90% even in cases of hypertension
e. COBD exacerbation and hypercarbia that cause stupor are indication for
immediate uses of noninvasive positive pressure ventilation

40. A 73 year-old female weighting 80 KG is brought to emergency room by an


ambulance due to respiratory failure her blood pressure is high , low measured
saturation and lung auscultation and chest x-ray are compatible with acute
pulmonary edema , she does take any medication.
Which of the following is accurate regarding the use of furosemide?
a. it is given IV but the action begins only when increases urine output by kidneys an
adequate dose in this case is 4o mg
b. it is given IV and action begins even before it affects the kidneys, an adequate
dose in this case is at 40 mg
c. it is given IV and action begins even before it affects the kidneys, an adequate
dose in this case is at least 80
d. it is given IV and action begins only when increases urine output by the kidneys,
an adequate dose in this case is at least 80 mg
e. since this patient was not on previous permanent fusid, giving a single IV dose is
not expected to assist in treating pulmonary edema
41. Which of the following best differentiates between simple infectious cystitis
and pyelonephritis?
a. Costovertebral angel pain
b. Nausea and vomiting combined by increased body temperature
c .Presence of pus and blood on urinalysis
d. History of nephrolithiasis
e. Presence pf spiking fever
42. A 52 year old male complain of reflux bothering him for several weeks ,
the reflex is not sever but his appetite decreased and he has last 6 kg in the past
month.
Which of the following is most adequate for evaluation and for management of
this case?
a. breathe test to roll out helicobacter pylori infection
b. empiric course of treatment with H2 blocker
c. empiric course of treatment with PPI
d. empiric course of triple therapy for helicobacter pylori eradication
e. refer for endoscopy by an expert gastroenterologist

43. Which of the following statements regarding the efficacy of cancer screening
tests is correct?
a. chest x-ray for early detection of lung cancer were not shown to be effective as
far decreasing mortality
b. a single colonoscopy has equal efficacy to the combination of sigmoidoscopy and
fecal occult blood test for the early diagnosis of colon cancer
c. early detection of prostate cancer is effective in prolonging life expectancy in all
age groups
d. periodic testing of blood CA 125 was found to be effective for early detection of
ovarian cancer
e. self-examination of skin was proven on randomized trials to decrease mortality of
several skin cancer
44. A 42 year old male was admitted due to fever 9 days after chemotherapy for
lymphoma in the past month he was treated in the hematology clinic and not
hospitalized , his absolute neutrophil count is 230 , his blood pressure is normal
and he feels well.
Which of the following statements regarding the treatment of this patient is
correct?
a. empiric antibiotic treatment that cover gram positive bacteria should be given
b. there is no role for chest x-ray as the leucocyte count is too low to create an
infiltration
c. at least 2 blood cultures are needed from, for immediate antibiotic therapy
d. empiric broad spectrum antibiotic therapy should be initiated and continued until
the neutrophil count is above 500 cells /microliter
e. initiate antibiotic treatment according to clinical judgment and the patient status,
G-CSF be initiated with no delay

45. A 29 year old female complain of general malaise, dyspnea and intolerance to
perform minimal exercise, her blood tests are HG-7.7g/dl MCV-72 MCH-19 pg
ferritin 9mcg/l.
Which of the following diagnosis is the most probable?
a. Iron deficiency anemia
b. Thalassemia minor
c. Sideroblastic anemia
d. Chronic inflammatory diseases
e. Early stage myelodysplastic syndrome
46. A 81 old male is admitted to the hospital due to acute confusion during a
febrile disease. There is no nuclei rigidity or photophobia.
CBC demonstrates thrombocytopenia, there is also acute renal failure.
Which of the following testes is most crucial when he is admitted?

A. Blood cultures from two separate sites and with interval between them.
B. Lumbar puncture (LP)
C. Pheripral blood smear
D. Urine analysis & culture
E. Renal and ureters echo

47. A 30 year old female is suffering from multi-organ disease involving the
kidneys، joints، and malaria rash over the face.
Which of the following is correct due to clinical suspicion of SLE?

A. The probability of SLE is high even if ANA was negative several times
B. The positivity for anti-AS DNA correlates directly with kidney injury severity in SLE
C. Positive serology for anti RO is specific for SLE
D. History of rush improvement with sun exposure
E. Patient with positive serology for anti-phospholipid do not have recurrent
abortions

48. A 40 year old complains of joint pain and a rash witch of the following findings
on evolution supports the diagnosis of SLE:

A. X-RAY DEMONSTRATING erosive joints changes


B. History of epilepsy since age 4
C. CBC DEMOSTRATING LEUKOPNIA LESS THEN 200 WBC
D. History of rash important with sun exposure
E. Sacro iliac joints fusion on x-ray of peeves and lower back
49. A patient that is diagnosed with myositis is supposed to start steroids
treatment. Before the initiation of therapy the patient needs to be informed of the
drugs side effects.
Which of the following is a typical side effect of corticosteroids?

A. more sever hypoglycemia (in diabetic patients)


B. improvement in mood stabilization (in bipolar patients)
C. increased incidence of low blood pressure in the general population
D. hypokalemia on blood tests
E. bone structure improvement as per bone density test

50. A 35 year old male with a stroke at age 22 and 3 previous abortions is now
admitted with pulmonary embolus, LUPUS anticoagulant is positively high on
blood test.
Which of the following true regarding the patient population with this syndrome?

A. all the patients with positive serology for anti cardiolipin on high titter
B. high percentage of patients with the above patient’s disease will also have SLE
C. in the disease of the above patient there is also atypical rash based on
telangiectasia of the face.
D. in general the CBC of the patient with the above described disease will
demonstrate thrombocytosis
E. it is not acceptable to prescribe anticoagulation with cumin because this patient is
young with servile abortions

51. A 80 year old male is brought to the ER with fever of 40 ° c, low blood pressure
and blurred consciousness. He has permanent urinary chatter and 2 weeks ago
when he was hospitalized else where his urine catheter grew pseudomonas
aeruginosa. What is the treatment of choice in this setting?

A. Change urinary catheter, report urine cutlers and avoided antibiotics since this is
accouter from a permanent catheter
B. IV AUGMENTIN (AMOXICILLIN)
C. IV rocepin (ceftriaxone(
D. IV tazocin (piperacillin tazobactam)
E. IV vancomaycin
52. An HIV positive patient performed a tuberculin test.
What is the induration diameter that will justify prophylactic treatment for latent
tuberculosis?

A. at least 5 mm
B. at least 10 mm
C. at least 15 mm
D. at least 20 mm
E. above 25 mm

53. Below is a picture of the chest of 60 year old male, which complains of
stabbing chest pain. Which of the following statements is correct?

a. In any case there is contraindication for use of steroids as long as there is a rash
b. It is not possible to have dermatome pain without a rash
c. Rash can be contagious on contact
d. Gancyclovir is the treatment of choice in cases of immune deficiency
e. Recurrence of the rash at a later time is very common
54. 90- year-old male is admitted to internal medicine department for pneumonia.
During one of the nights he starts to attack the staff, claiming that they spread
parasites and rodents on department’s floor.
Which approach is most suitable in this case?

a. Use of benzodiazepines
b. Use of classic anti-psychotic drug (e.g. Halidol)
c. Use of atypical anti-psychotic drug (e.g. Seroquel)
d. Physical restraint to the bed to avoid physical harm to staff
e. Orientation to the department, bringing close family to the hospital and verbal
comforting

55. 19year old IDF soldier complains of fever up to 37.7degree accompanied by


headache when moving the neck, green nasal drip and general malaise. She has
been ill for a week and clinical suspicion of sinusitis.
Which of the following is recommended approach?

a. In the majority of cases the patient will recover without antibiotic therapy
b. The initial treatment should be broad spectrum antibiotics that also covers
resistant S.pneumoniae
c. If disease will continue more than 2 weeks it can be defined as chronic sinusitis
and she should be referred to an ENT expert
d. If antibiotic treatment is to be given then aspiration of the sinus by ENT should be
performed before initiating therapy
e. Once she recovers from this current episode the guidelines support prophylactic
antibiotic therapy to prevent future episodes.
56. In which of the following patient, suspect of having pulmonary embolism,
will D-dimer test have the highest yield?

a. 44 year old male with bilateral pneumonia, admitted to the ICU for last couple of
days. This morning developed swelling of the right calf and worsening shortness of
breath.
b. 22 year old woman, using oral contraceptives and a smoker, who was diagnosed
with DVT of the left leg a week ago. This morning developed new onset shortness of
breath
c. 51 year old man, heavy smoker, diagnosed with small cell lung cancer. Complains
of worsening of breath
d. 70 year old woman, who come to emergency room complaining of palpitations
and short of breath. ECG revealed atrial fibrillation with unknown onset
e. 41 year old woman on her first pregnancy after numerous infertility treatment
years is on her 3rd trimester. She complains of chest pain and short of breath over
the last couple of hours.

57. A 52 years old man, heavy smoker with background of coronary artery disease,
ischemic disease , cardiomyopathy, right and left heart failure and moderal renal
failure. Over the last 3 days developed worsening shortness of breath. Chest x-ray
reveals right pleural effusion.
Analysis of the pleural fluid (following pleurocentesis) reveals
proteinn 4.4 gr%(Blood 6.2gr %)
(Effusion: blood protein was > 50%) LDH 312 IU\L (blood LDH 336 IU\L).
Which of the following is the most likely diagnosis?

a The fluid has characteristic of transudate, most probably due to the patient’s heart
failure
b. The fluid has characteristic exudate, most probably due to a malignant tumor.
c. The fluid has characteristic of transudate, which is type of fluid collected in
pulmonary embolism
d. The fluid has characteristic transudate, most probably due to the patient’s renal
failure
e. The fluid has characteristic of exudate, most probably due to right sided
pneumonia
58. The IV drug user brought to the emergency room with spiking fevers,
low blood pressure, and stupor.
Physical examination reveals an inflamed/infected venipuncture site.
Assuming a working diagnosis of septic shock.
Which of the following antibiotics agent is choice?

a. Piperacil – tazobactam
b. Imipenem – Cilastatin
c. Ceftriaxone
d. Vancomycin
e. Cefataxime

59. 41year old man, who present to the clinic with shortness of breath lost
consciousness and fall on the ground. Doctor on the scene identify a pulse,
diagnose the cardiac arrest and put the defibrillator paddles on the man’s chest.
Which of the following monitor reading is the most likely?

a. Pulseless tachycardia
b. Pulseless electrical activity
c. Asystole
d. VF, ventricular fibrillation
e. The 3rd degree AV block

60. A 60 year old man with hepatic cirrhosis presents to the emergency room with
malaise nausea vomiting and abdominal pain. Physical examination reveals a
temperature of 38.2 ascites (positive shifting dullness) and flapping tremor of the
palms. Laboratory analysis reveals leukocytosis and thrombocytopenia of 74000.
Which of the following test are essential in the processing of this patient

A. Lumbar puncture
B. Emergency gastroscopy
C. Diagnostic peritoneocentesis
D. Bronchoscopy with peritoneal lavage
E. No invasive procedure should be performed on this patient due to
thrombocytopenia
61. Which of the following patients with primary hyperpathyroidism has the
strongest indication for surgical parathyroidectomy:

A. A 72year old man normal serum and urine calcium with evidence of significant
loss of bone density
B. A 80 year old man normal serum calcium renal failure with creatinine clearance of
65 ml/min normal bone density
C. A 44 old woman normal serum calcium increased calcium in 24 h urine collection
decreased bone density was reported
D. An 82 year old man slightly increased serum calcium increased calcium in 24h
urine collection and normal bone density
E. A 90 year old woman increased serum calcium serum PTH which high for years
has decreased and now low increased 24h urine collection

62. Which of the following is true regarding the complication of diabetes mellitus:

A. over the years and disease advancement the glomerular filtration rate (GFR)
consistently decreases
B. urine analysis for microalbumineuria should be performed yearly
C. prevention of arterial hypertension is effective in preventing diabetic retinopathy
D. autonomic neuropathy is not a part of diabetic neuropathy and should always
warrant further neurologic /metabolic investigation
E. cardiovascular morbidity is increased in type 2 diabetes but not in type 1

63. Angioedema is a rare but dangerous adverse effect of angiotensin converting


enzyme inhibitors (ACT-1).
Which of the following describes best the pathophysiology of this phenomena:

A. an allergic reaction mediated through Ig E


B. the reaction is mediated through bradykinin
C. the reaction is mediated through an increase of components of the compliment
system
D. the reaction is a result of direct integration of mast cells
E. the pathophysiology of angioedema is unknown
64. A 22 year old man attempted suicide by paracetamol ingestion he refuses to
say how many pills he took He ingested the pills a few hours ago Paracetamol
blood are 450 microgram/h.
Which of the following describes the best therapeutic approach

A. It is impossible to treat with N-acetylcysteine since the amount ingested and the
time that has passed from ingestion are unknown
B. The treatment should consist of gastric lavage and intravenous N-acetycysteine
C. There is no need for gastric lavage but intravenous N-acetycysteine treatment
should be initiated
D. The patient should receive intravenous N-acetycysteine and liver transplantation
should be considered according to the patient reaction
E. This patient should not receive N-acetycysteine treatment but should be
transferred to a liver transplantation center immediately

65. A 42 year old asymptomatic generally well woman underwent routine blood
analysis revealed increased TSH and normal T4.
Which of the following is the best approached in this case

A. There is no need for further investigation or treatment in an asymptomatic


patient
B. Anti thyroid peroxidase antibody levels should be checked and if increased T4
replacement therapy should be prescribed
C. Anti thyroid peroxidase antibody levels should be checked even if the levels are
increased T4replacement therapy should not be initiated
D. The hormone replacement therapy should be started without further
investigation
E. The TSR should be repeated in month to rule out a mistake

66. Which of the following lifestyle changes will have the greatest effect on
arterial hypertension control:

A. BMI under 25
B. Salt intake under 1gr per day
C. Complete avoidance of alcohol
D. Aerobic activity for a least 60 minute per day
E. Avoidance of dairy products
67. A 42 year old chrohn’s disease patient after partial small bowel resection
complains of hematuria.
Which of the following is the most likely etiology for hematuria in patient with
chrohn’s disease?

a) Malignant bladder epithelial tumors


b) Malignant renal tumors
c) Benign bladder tumors
d) Vascular malformations in the bladder wall
e) Calcium oxalate urolithiasis

68.A 72 year old patient with emphysema is admitted for worsening shortness of
breath، over the course of his admission he deteriorates and requires mechanical
ventilation. Chest x ray reveled a right pneumothorax and thoracostomy tube was
placed. The patient is sedated.
Which of the following ventilating patterns is most adequate for this patient?

a) Spontaneous pressure supported ventilation (psv)


b) Synchronized intermittent mandatory ventilation/volume controlled ventilation)
C) Synchronized intermittent mandatory ventilation/pressur controlled ventilation)
d) Assist control ventilation
e) Bi-level positive airway pressure (bipap)

69. During your work in various hospital departments it is very important to use the
quick hand disinfection agent available in every admission room or by patient beds.
Which of the following best describes the aim of this disinfection method?

a) Removal of the transient hand flora


b) Removal of transient and permanent hand flora
c) This disinfection does not remove hand flora does prevent transfer of bacteria from
the health care worker to the patient
d) Hosing disposable gloves eliminates the need to disinfect the hands
70. Which of the following is true regarding infection control is hospital wards?

a) Prevention of infection with influenza requires airborne isolation using of a an mask


b) Prevention of tuberculosis infection requires droplet isolation by using surgical
mask
c) Prevention of transfer of clostridium spores requires using agents containing
chlorhexidine
d) Contact with blood fluids and an obvious contamination of the hands require
washing with an antiseptic solution
e) Before preforming a sterile procedure there is no need to disinfect the hands if
gloves are worn.

71. Which of the following is true regarding blood transfusion guidelines in the
hospital?

a) In patients with a past malignancy blood should be filtered and irradiated


b) In the patients who have had an unusual reaction to blood transfusion blood should
be irradiated
c) In patients receiving ace inhibitor treatment there may be problems while using a
white cell filter when transfusing blood
d) The safest blood product is platelets since they don’t carry blood group antigens
and are infection free
e) Transfusing platelets requires cross match of the patient's blood (regarding RBC’s)
and matching platelet antigens
72. An 18 year old male is brought unconscious to the emergency. He has been
complaining of nausea and abdominal pain for the last few days. On examination
there is a pathological respiration pattern rapid and deep as described in the graph
attached.
Which of the following is the most appropriate description for this respiration
patterns?

a) This is Cheyne - Stokes respiration pattern typical for patients with brain stem
injuries
b) This respiration pattern is typical for patients with severe metabolic acidosis
(kussmal breathing)
c) The respiration pattern is-non-specific and represents agonal breathing in patients
that are about to die.
d) This respiration pattern is pathognomonic of hyperventilation and hysteria
e) This respiration pattern indicates a primary respiratory problem most likely a
severe obstructive disease.
73. 62 year old woman, heavy smoker, complains of worsening of chronic dyspnea
and cough, and episodes of hemoptysis .in addition, she is known have cirrhosis due
to a chronic HCV infection. In the picture below there is a distinct finding on her
physical examination.
Which of the following is true background diseases of the patient?

a) This is a common finding in COPD and has no association with liver cirrhosis
b) This finding may be associated with the patients liver disease but not with COPD
c) This finding is common both in COPD and in liver cirrhosis
d) This finding indicates high risk for hepyocellular carcinoma but is not associated
with lung diseases and their complications
e) This finding indicates high risk for lung malignancy and is not associated with liver
diseases or their complications.
74. A homeless 52-years old alcoholic male was admitted for investigation of
weight loss، heavy cough with purulent sputum، and fever.
Which of the following is true regarding the combination of the clinical
manifestations and the X-Ray?

A. The X-Ray reveals a right pulmonary abscess. The patient should be aided to emit
sputum that should be sent for gram stain and culture in most cases treatment is
conservative.
B. The X-Ray reveals a large pleural effusion. It is most likely a complication of the
patient's pneumonia.
C. The X-Ray reveals right middle lobe pneumonia, typical for patient's that are at
high risk of aspiration.
D. The X-Ray is highly suspicious for lung tuberculosis the patient should be
admitted in airborne isolation and an attempt to isolate acid fast stable bacteria
from the sputum should be per formed as fast as possible.
E. The X-Ray is highly suspicious of a space occupying or malignant lesion of the
lung. CT should be performed immediately to confirm this diagnosis.
75. An 82-years old man walks into the emergency and complains of sudden chest
pain and shortness of breath. On auscultation to the lung; bilateral wet crackles up
to half of the lung. The below cardiac monitor was performed immediately after
the patient was put in a bed, which of the following is the appropriate
management of this patient? Picture!

A. The patient has atrial fibrillation and slowing the ventricular response with beta-
blockers should be initiated .
B. The patient has ventricular fibrillation; he should receive unsynchronized
defibrillation immediately.
C. The patient has supraventricular tachycardia [SVT]; he should receive intravenous
adenosine 6-Mg to return to sinus rhythm.
D. The patient has atrial fibrillation, he should receive synchronized defibrillation
immediately.
E. The patient has supraventricular tachycardia [SVT]; he should receive intravenous
calcium channel blocker.

76. A 32-years old man turns the psychiatrist and complains that for years he has
been feeling compelled to check if the house door is locked dozens of times a day،
and that for the last couple of years he has been washing his hand over and over
to prevent an infection. The repetitive hand washing has caused behavior doesn't
make sense and feels like" he is losing his mind " and asks for professional help.
Which of the following is the best treatment option for this man ?

A. Combination of low dose atypical antipsychotic drug and a psychoanalysis .


B. Combination of low dose SSRI's and cognitive-behavioral therapy.
C. High dose atypical antipsychotic drug.
D. Combination of low dose SSRI's and psychoanalysis.
E. Combination of cognitive- behavioral therapy and high dose benzodiazepines.

77. Which of the following is the most common comorbidity in people with
substance abuse ?

A. Narcissistic personality disorder.


B. Abuse of an additional substance.
C. Depression.
D. Suicide.
E. Anxiety disorder.
78. Which of the following characterizes personality disorder clusters?

A. Cluster B is characterized by oddness.


B. Cluster A is characterized by impulsiveness.
C. Cluster B is characterized by suspiciousness.
D. Cluster C is characterized by instability.
E. Cluster A is characterized by emotionality.

79. All the following drugs are of the same group except?

A. Escitaloprame.
B. Paroxetine.
C. Venlafaxine.
D. Fluoxetine.
E. Flavoxetine.

80. Dan، a 35-years old male who has been abusing drugs and alcohol since he was
very young، is brought to the emergency room by his family who report that for
the last 3 days، since he stopped using drugs he is very depressed، sad، tearful،
and discouraged. He speaks of severe emotional pain, Expresses wishes of death.
Cannot see hope or a future for himself, and a lot eats, Dan approve the
information and adds he has been having vivid nightmares at nights.
Which of the following substance has Dan most likely ceased taking?

A. Methadone.
B. Amphetamine.
C. MDMA.
D. PCP,
E. Alcohol.

81. Which of the following drugs has been found the most efficient in the
management of treatment resistant schizophrenia?

A. Clozapine (leponex)
B. Perphenazine (perphenane)
C. Olanzapine ( byprexa)
D. Risperidone (risperidal)
E. Haloperidole (halidol)
82. Which of the flowing true regarding suicide?

a. children tend to commit suicide more than elderly


b. the tendency to commit suicide has genetic basis
c. of all the mental disorder completed suicide has the strongest associated with
psychosis
d. male gender has been found to be protective from completed suicide
e. old age has been found to be protective from completed suicide

83. 46 year old man brought to the psychiatric clinic due to lack of functioning for
the last 4 year .he had a few psychiatric hospitalization due to psychosis. he was
then convinced that his thought are transmitted and stolen in addition the family
report that for long time ha has been lying in bed , non-initiative, doesn't bath and
avoid company .he does however eat and sleep in bed and seems {shut down} but
not sad . He himself has no complaints and say his mood fine, and nothing
bothering him. On examination he speaks in laconic voice and monotonic tone.
There is no evidence of delusion .but thought content is poor and the thought are
covert and stereotypical. There was no depression in manic content.
The effect was limited and the understanding of his condition formal.
Which of the flowing most adequate monotherapy for this man?

a. Amisulpride 400 mg
b. Lithium 1200 mg
c. Lithium 800 mg
d. Venlafaxine 225 mg
e. Oxazepam 30 mg

84. Which of the flowing true regarding serotonin syndrome?

a. constipation and psychomotor slowdown is typical


b. may be caused by combination of paroxetine and lithium
c. warrant follow up and not treatment in the first stage
d. immediate ceasing of the drug with the serotonergic component is required
e. common in people taking drug the serotoninergic component
85. All of the following are adverse effect of clozapine except?

A. seizure
B. anticholinergic phenomena
C. xerostomia
D. metabolic syndrome
E. weight gain

86. A previously healthy 85 year old is brought by his family to the doctor and the
family reports that he has become demented of the last month. He doesn’t
remember anything, is inactive, and hardly speaks. During examination he sits
with his head down and replies with “I don’t know, I don’t remember”.
Which of the following is most likely to found on examination?

A. Confabulations
B. Very short, but not long term memory loss
C. Good cooperation
D. The patient will try to emphasize past successes
E. Depressive symptoms

87. Which of the following is most characteristic of a schizophrenic adult that has
been diagnosed at a very young age?

A. Prominent negative sign


B. Loss of basic functions such as reading, writing and understanding
C. Severe memory loss
D. Prominent positive sign
E. Severe depression

88. Which of the following agent is the cause of priapism?

A. Risperdone
B. Propranolol
C. Trazodone
D. Sildenafil
E. Fluvoxamine
89. All of the following are true regarding neurotransmitter except?
?????

E. An important role of acetylcholine has been demonstrated in the development of


dementia since anticholinergic drug can prevent cognitive deterioration
???

90. Which of the following reports of families of schizophrenic patients imply a


good prognosis for the patient ?

a) “He lives alone, I bring him food once a week, and he spends the rest of his time
in the mall “
b) “I remember it was the same when it began with his father. He thought we are
trying to poison him and surrender him to the hizballa
c) “It began suddenly , he claimed he is hearing voices , ran around restlessly and
warned us that very soon a bomb will destroy planet earth.
d) He started showing odd behavior as early as 14 years, kept to himself, at times
talked to himself, had problems concentration, and his grades deteriorated. I now
understand it was the beginning
e) This is his sixth admission in two years. Dr., what can I do? He refuses to take his
medication and keeps threatening us!

91. Which of the following characterizes adult (age 18-65) obsessive compulsive
disorder?

a) The obsessive symptoms relieve the emotional stress


b) The anxiety increases when the patient tries to resist compulsive rituals
c) The most common obsessive symptoms are intrusive thoughts
d) The most common compulsive symptom is counting
e) The obsessive disorder is usually ego-syntonic, but it may be ego dystonic
92. Which of the following is the least likely component in nacrocitic personality
disorder ?

a. Irresponsibility
b. Vanity
c. Lack of empathy
d. The tendency to use others
e. Feeling of specialness

93. A 26 year old woman complains of anxiety and loneliness. In school she used
to keep to herself and draw pictures of princesses and princes. She has served as a
quarter master. Since her discharge 4 years ago she lives in a rented apartment in
south Tel Aviv and works in the post office sorting letters. She doesn’t go to
parties, at work there is a young man who asks her out bus she is too embarrassed
to agree. She says she is not sure he really likes her.
Which of the following is the most likely diagnosis?

a) Schizoid personality disorder


b) Simple schizophrenia
c) Social phobia
d) Selective mutism
e) Avoidance personality disorder

94. Which of the following is truest regarding anorexia nervosa?

a) High suicide rate in patient with the restrictive type disorder


b) Compared to patient with bulimic episodes
c) 3% mortality rate mostly due to suicide
d) Admitting of hunger is positive prognostic sign
e) Psychodynamic treatment is the most efficient from psychotherapy
95. 20 year old woman complains of anti ty loneliness.
Which of the following is not true regarding the development and clinical course
of schizophrenia?
A. In most cases the critical course in the first 5 years
B. the positive signs moderate over the years
C. the negative sings develop over the years
D. 2 third of the patients experience a deterioatation in social and professional
function
E. age of onset is earlier in women than in men

96. When recommending bupropion (zyban) to quit smoking you should warn the
patient about:-
a. Orthostatic hypotension
b. Weight gain
c. Seizures
d. Neruonsness
e. Decrease in libido

97. a man is hospitalized for three days in an internal word due to fever over the
last hours he complains of increasing restlessness and the staff report a seizure the
previous day on examination the patient is lean obviously restless and has a
tremor . What is the most likely diagnoses?
A. The beginning of psychotic episode
B. Alcohol with drawl
C. Encephalitis
D. A result of abrupt- ceasing of lithium treat
E. Malignant neuroleptic syndrome
98. 55 year old women complains of malaria. Slowness, a fool mood, and difficulty
making simple decisions .the primary physician started treatment with
antidepressant without any relief.
Which of the following tests is the most important for making a diagnose?

A.Thyroid function test


B. Brain CT
C. Complete blood count
D. EEG
E. Vitamin D levels

99. Which of the following will be least likely cause seizers in either poisoning or
withdraw?

A. Clozapine
B. Paroxetine
C. Sustained release bupropion
D. Alcohol
E. Clonazepam

100. A 38 year old male is in the midst of depression episode that has begun a month
ago. In the past he had three hypomanic episodes, and no psychotic episode.
Which of the following is most likely diagnose?

A. Cyclothymic disorder
B. Schizoaffective disorder
C. Bipolar 1
D. Bipolar 2
E. Borderline personality disorder
101. A 10 year old girl is diagnosed with intellectual disability ،she has a flat nose،
slanted eyes، a protruding tongue ،her hands are broad and thick. She has a single
palmar transverse crease and quite friendly.
From which syndrome is she most likely suffering from?

A. Fragile x syndrome
B. Prader willi syndrome
C. Cats cry syndrome
D. Rett syndrome
E. Down syndrome

102. Stimulant medications may be beneficial in all of these disorders except?

A. ADHD
B. Narcolepsy
C. Withdrawal symptom from cocaine
D. A depression episode in medically ill
E. Primary hypersomnia

103) In the treatment of depressive episode occurring in the context of major


depressive disorder the initial treatment with an SSRI brought only a little
improvement . The next the therapeutic step could be all of the below except?

A) Augmenting the SSRI with bupropion


b) Switching the SSRI to duloxetine
c) Augmenting the SSRI with the thyroid hormone T3
d) Augment the SSRI with lithium
e) Switch the SSRI to alprazolam

104) In the treatment of panic disorder which treatment is not recommended?

a) A first generation antipsychotic


b) A benzodiazepine
c) A SSRI antidepressant
d) Cognitive behavioral therapy
e) A Combination of SSRI and benzodiazepine
105) A young man of 25 was let off from his work place he developed anxiety and
depressive symptoms that interfered with his daily life . His family doctor
diagnosed adjustment Disorder.
What is true regarding adjustment disorder?

a) There is no such diagnosis in the DSM -5


b) The disorder must developed within a year of the onset of the stressor
c) Clinical presentation can include delusions and Hallucination
d) Psychotherapy is the treatment of choice
e) The stressor severity is clearly correlated with the severity of response

106) For every disorder in the left column (A-E) MARK the most appropriate
statement in the right column (1-5)

Hoarding disorder Repetitive picking


Of the skin
Trichotillomania Preoccupation with
bodily symptoms and a
fear of having a serious
disease
EXCORIATION DISORDER Difficulty in throwing
away things that are not
needed
BODY dysmorphic disorder Preoccupation with
imagined defects in
appearance
Hypochondriasis Repetitive pulling of hair

a) A-3 , B-1 , C-4 , D-2 , E-5


b) A-3 , B-1 , C-5 , D-2 , E-4
c) A-4 , B- 5 , C-1, D-2 , E-3
d) A-3 , B-5 , C-1 , D-4 , E-2
e) A-3 , B-5 , C-1 , D-2 , E-4
107) When treating a bipolar I disorder patient suffering from his first acute
manic episode, there are several acceptable therapeutic options .
Which of the therapeutic option bellow is not appropriate for a first acute manic
episode?

a) The patient can start treatment with lithium alone


b) The patient can start treatment with lithium and SRRI
c) The patient can start treatment with lithium and second generation antipsychotic.
d) The patient can start treatment with valproic acid alone

108) Which of the treatment which is not indicated for children with autistic
spectrum disorder?

A) Parental support psycho-education and training


B) Intensive behavioral program for the child
C) Hospitalization in a psychiatry department for intensive pharmacotherapy
D) Placement in a special educational system for intensive intervention
E) Intensive communication, language and social skills training

109) A 68 years old women with no previous history of psychiatric disorder was
hospitalized for elective knee replacement , 3 days after the surgery she become
anxious , restless and slept poorly . She complained that nurses were stealing her
money. She tried to run away from the hospital and at times could see demons sent
to kill her. She was disoriented to time and place.
The most appropriate diagnosis in this case is?

A) Dementia
B) Delirium
C) Depression
D) Delusional
E) Post – Traumatic stress disorder
110) Catatonic symptoms can be found in several conditions and disorders which
condition or disorder would never have catatonic symptoms?

A) A severe medical condition


B) Schizophrenia
C) A depressive episode
D) A manic episode
E) Post traumatic stress disorder
State Exam 26.10.2014
By: Alhag Abu Anzeh Muhammad

Part A
1. 10 year old boy develop pancreatitis following blow abdomen, during 5 day
hospitalization he treated with IV fluid and analgesic and discharge home on
low fat diet, 4 week later come with abdominal pain and vomiting .DX?

A. functional abdominal pain


B. duodenal ulcer
C. chronic pancreatic
D. hemorrhagic
E. pancreatic pseudocyst

2. 3 month old infant present for well child evaluation which of the following
will be concern?

A. regurgitation 15-30 ml 3 time per day


B. one bowel movement per day
C. 3 bowel movement per day
D. liver span 5 cm
E. hemoccult +ve stool

3. 10 year boy has been having "bellyaches" for about 2 years , they occur
mostly on awakening and after meal.occasionally he vomits after the oncet
of pain , occult blood stool has been found in his stool . his father also get
freqwent stomachaches . which most likly DX?

A. peptic ulcer
B. appendicitis
C. meckelsdivertculum
D. functional abdominal pain
E. pinworminfrstation

1
4. Which of the following is regarded as know complication of TPN?

A. sepsis
B. renal faluir
C. chronic dirrea
D. vitA Deficincy

5. A 12 month old girl has been spitting up her meals since 1 month ago , her
growth at 95 precentil and other wise asymptomatic and without finding on
physical examination. DX?

A. pyloric stenosis
B. partial dudnal atresia
C. hypothyroidism
D. GER
E. trachoesophegal fistula

6. 4 week old infant was vomiting and hypochromin metabolic alkalosis next
study?

A. urin organic acid


B. urine 17 hydroxyprogestrone
C. stool
D. abdominal US
E. head US

7. 3 month infant present with persists stridor.what is the most frequent cause?

A. vascular ring
B. laryngomalaci
C. tracheomalacia
D. laryngealclwft
E. subglottic stenosis

2
8. 5year old boy present with acute severe chest pain EKG and enzyme studies
confirm acute MI cardiocathetarization reveals coronary artery aneurysm wit
thrombosis which constellation of symptoms in his past could explain this
finding?

A. conjunctivitis. Fever.cervicallymphodenopathy
B. meningitis. Conjunctivitis. Pallor
C. cervical lymphodenopathy. Hepatitis. Rash
D. fever irritability. Panceratitis
E. hepatosplenomegaly. Rash . Conjunctivitis

9. Patient with streptococcal pharyngitis develops tender red bumps along her
entire anterior tibia on both sides what most likely Diagnosis?

A. sarcoidosis
B. cellulitis
C. thrombophlebitis
D. insect bite
E. erythemanodosum

10. 2 years old child develops apnea cyanosis and loss of consciousness with
repeat generalized clonic jerk after being scolded by his mother , on
examination the child appears completely normal. What is the best treatment
options?

A. tagretol
B. valaporic acid
C. anti arrhythmia
D. cardiac pacemakers
E. parent consult

3
11. 16 years old male Brought to emergency room by his friends they relate they
were drinking alcohol and that their friends "passed out " about 2 hours ago
and he is increasingly difficult to arouse. Which of the following is the most
useful in your immediate management of this patient?

A. serum sodium
B. serum glucose
C. blood alcohol level
D. serum calcium
E. serum drug screening

12. you are evaluateing 10 month old infant for reccurent fracture following
relatively minor truma .you note deep blue sclera and bowing of lower
extremities .x rays examination reveals generalized osteopenia .which of the
following most likely diagnosis?

A. achondroplasia
B. histocytosis
C. osteogenesisimperfecta
D. osteoporosis
E. Rickets

13. A 3 yrd female was found with her mothers prenatal vitamins. The bottle
was empty and was estimated to contain 20-25 tablets. At 4 htspostingestion
her serum iron level is 550 mg/dl (toxic level) which of the following should
be administered?

A. Dimercaprol
B. deferoxamine
C. Edetate calcium disodium
D. Ipecac
E. Activated charcoal

4
14. An adolescent male presents with a urethraldischarge. Gram stain of the
exudate revealsintracellular gram-negative diplococci. The initialtreatment
of choice is which of the following?

A. azithromycin, 1 g orally once


B. doxycycline, 100 mg twice a day for 7 days
C. procaine penicillin G, 1.2 million unitsIM once
D. benzathine penicillin G, 2.4 million unitsIM once
E. ceftriaxone, 125 mg IM once

15. A 8 year old with uncontrolled asthma is treated with corticosteroids ,beta-
agonist and leukotriene antagonist , you disced to start her on theophylline .
You are worried about potential toxins , what side effect should you
monitor?

A. nystagmus
B. myopaty
C. polyuria
D. rash
E. vomiting

16. A 2 year old was found with an open bottle of acetaminophen and phill
fragments in her mouth. What is the major cause of morbidity and mortality
in acute acetaminophen poisoning is :

A. Hepatic injury
B. Gastric bleeding
C. Metabolic acidosis
D. Methemoglobinemia
E. Hypoglycemia

5
17. 8 year old boy returns from summer camp , he complains for 2 days of
abdomen pain. You suspected giardialamblia infection What additional
symptom would you see most commonly in a child with giardiasis ?

A. fever
B. watery diarrhea
C. Lymphadenophaty
D. faliur to thrive
E. Vomiting

18. You are consulting on a primagravid woman who has been found with
rubella nonimmune on pre-Nate labs evolution . She askes if the fetus is at
risk for malformation . When during pregnancy is rubella virus most
commonly associated with congenital deficencies ?

A. First 11 weeks of gestation


B. in 11-12 week of gestation
C. 4 th month
D. last trimester
E. Any time during pregnancy

19. 15 year old has exudative tonsillitis , cervical adenitis splenomegaly , a


positive monospot test. What is the most commun complication of this
infection ?

A. Pancreatitis
B. Hemorrhage
C. Pneumonia
D. Airwayobstraction
E. meningoencephalitis

6
20. 18 month old child ,presentes with a 2 day history of fever , not eating ,
examination shows 3 mm vesicles on erythematus base on soft palat And on
the tonsilitis , what is the most likely etiology ?

A. Group A strep
B. Enterovirus
C. Herpes simplex
D. herpes virus 6
E. candida

21. Newborn with delayed passage of mecanium Rx with barium


showesdilatated proximal colon and small obstraction of the distal colon ,
what is the next best diagnostic step?

A. abdominal CT
B. stool culture
C. rectal biopsies
D. sweat chloride test
E. Chromosome analysis

22. A newborn has been diagnosed with aniridia. Which of the following should
be performed on this patient?

A. chest radiograph
B. CT scan of head
C. echoradiography
D. testicular examination
E. renal ultrasound

7
23. 7 year old girl develops secondary nocturnal enuresis .what is the next best
study?

A. renal ultrasound
B. voiding cystourethrogram
C. abdominal plain radiograph
D. urine analysis
E. creatinine clearance

24. 2 year old boy from sudan has failure to thrive , chronic diarrhea and severe
candidiasis. you suspect HIV infection , which of the following organisms w
ould most likely be found on stool examination?

A. enterovirus
B. shigella
C. cryptosporidium
D. giardia
E. yersinia enterocolotica

25. A 5 year boy has severe pharyngitis and thoroat culture is positive forgroup
A streptococci of the following complication of group A
streptoccalpharygitis and skin infection which is associated only with
phoryngeal infections

A. scurlet fever
B. acutrhumatic fever
C. toxic shock syndrome
D. necrotizing fasciitis
E. glomeulonephritis

8
26. What is the most important test to obtain a diagnosis in a 14 year old girl
with primaryamenorrhea and short stature who has history of repaired
coarctation of aorta in infancy

A. sweet chloride
B. karyotyping
C. floorescent in situ hybridization (FISH)of chromosome 22q11
D. pelvic ultrasonography
E. lymphocyte subset analysis

27. A group of medical students of the newborn nursery rotation are discussing
the sensory and physical abilities of a newborn which of the abilities is
considered normal

A. inability to sense pain


B. preference for geometric shapes over mothers face
C. 6/6 vision
D. grasping a caregivers finger
E. ability to see objects 1 meter away

28. A 7 year old girl comes to her pediatrician because her teacher is concerned
that shemay have attention-dificit hyperactive disorder (ADHD)hallmarks of
ADHD include which of the following

A. often quiet demeanor


B. history of seizures at birth
C. history of recurrent ear infections
D. symptoms occur in more than one settings
E. the girl dose not appear to be easily distracted

9
29. Which of the following is the most typical featur of breast milk jaundice

A. it is a conjugated hyperbilirubinemia
B. it usually appwars within the first 12 hours of life
C. the elevated levels of bilirubin may persist from more than 60 day
D. kernikterus can occure in the patients
E. it is uncommon in caucasian infants

30. A 4 year old boy present with fever of 39c and cough on examination there
are crackles present over the left lower lobe region he has no wheezing he
has been previously healthy the chest radiograph shows a consolidative
infiltrate in the left lower lobe bhind the heart consistent with pneumonia
which of the following statement about this patient is most likely to be
correct

A. the most likely bacterial etiology is streptococcus pneumonia


B. the most likely infections etiology is respiratory syncytial virus RSV
C. this cannot be mycobacterium tuberculosis because the pneumonia is in a
lower lobe
D. this is characteristic of foreign body aspiration
E. the patient will likely develop a pleural effusion

31. A 1-month-old boy presents to your clinic with feeding intolerance, poor
weight gain and large tongue. On exam, you notice a large posterior
fontanelle and umbilical hernia. Your next step in making the diagnosis
would be:

A. abdominal radiograph
B. CBC and blood culture
C. Barium swallow
D. Follow-up on newborn screen results
E. Admit to the hospital for failure-to-thrive (FTT) work-up

10
32. A 8-year-old girl comes to your office because she has been having episodes
where she stops moving and talking, demonstrates a blank facial expression,
and rapidly blinks here eyelids. These episodes last about 5 to 15 seconds,
after which she returns to her normal behavior. A true statement about this
form of epilepsy is:

A. the episodes can often be induced by hyperventilation for 3 to 4 minutes


B. the typical EEG pattern is called hypsarrhythmia.
C. it usually starts in children less than 5 years old.
D. these seizures rarely occur more than once a day.
E. this type of seizure is commonly treated with dilantin.

33. you are seeing a 14-year-old boy with prune belly syndrome in your office
who has developed chronic renal failure over the past 5 years. Which of the
following metabolic abnormalities associated with untreated chronic renal
filure is a common finding?

A. microcystic anemia
B. delayed bone age
C. osteosclerosis
D. hypoparathyroidisim
E. hyperosmolar urine

34. A 17-year-old sexually active female presents to your office with vaginal
discharge. The discharge is white and cottage cheese-like and she complains
of pruritis. Vaginal PH is less than 4.5 and KOH prep reveals pseudohyphae.
Which of the following is the most appropriate therapy?

A. Sitz baths
B. Metronidazole vaginal gel for 5 days
C. Azithromycin 1-g single dose
D. Fluconazole 150-mg single oral dose
E. Metronidazole 2-g single oral dose

11
35. A 7-year-old female has been evaluated by her pediatrician for short stature
and chronic diarrhea. She has not been out of the country and there have
been no identified infectious causes. The lab calls her physician with
positive anti-transglutaminase antibody. Which of the following is true
regarding her disease?

A. there is an increase risk of DM type 1


B. there is a higher risk of malignancy with a gluten-free diet
C. polycythemia is likelyto be found on initial labs
D. often there is a recent exposure to carpet-cleaning solutions
E. the albumin level will be increased.

36. A 12 year old boy is admitted to the hospital after coughing up blood his Hb
and Hct are in safe range. During hospitalization he is noted to have
persistently high BP and urinalysis is shows hematuria and proteinuria.
Diagnosis is most likely:

A. hemolytic uremic syndrome


B. Goodpasture syndrome
C. nephrotic syndrome
D. poststreptococcal glomerulonephritis
E. Renal vein thrombosis

37. about 12 days after a mild upper respiratory infection , 12 years old boy
complains of a weakness in his lower extremities over several days , the
weakness is progresses to include the trunk . in the examination shows the
weakness described and absence of lower extremity deep tendon reflex.
spinal fluid studies sho elevated protein only . wich of the following is the
most likely diagnosis ?

A. bell palsy
B. muscular dystrophy
C. guillainbarre syndrome
D. charcotmarie tooth disease
E. werdnighoffman disease

35-A 36-B 37-C


12
38. 3 months old boy is brought to the hospital because of altered mental status
changes .examination show sleepy baby who is difficult to arouse,
funduscopic examination showed retinal hemorrhagic . Examination
otherwise is unremarkable . What the best next diagnosis?

A. spinal tap
B. hematology profile with smear review
C. CT scan of the head
D. bone marrow study
E. EEG

39. a newborn boy was diagnosed parentally with bilateral hydronephrosis ,


distended bladder and oligoamnioidrisi what will be the most likely
diagnosis ?

A. urethral stricture
B. ureteropelvic junction obstruction
C. prune belly sindrome
D. posterior urethral pelvic valve
E. meatal stenosis

40. A fetus which of the following would most likely require an intravascular
blood transfusion ?

A. erythroblastosisfetalis
B. sickle cell anemia
C. spherocytosis
D. fetal distress and bradycardia
E. congenital heart disease

13
41. Iron deficiency is the leading cause of anemia in infancy and childhood.
Which of the following infant is at greatest risk for iron deficiency anemia
early on?

A. a premature infant
B. an infant with ABO incompatibility
C. an infant with physiological hyperbilirubinemia
D. a postpartum infant
E. an infant with polycythemia

42. A previously healthy, term, 3 month old male is brought to the hospital with
tachypnea , poor feeding , and pallor over the last 24 hours. The patient is
tachycardic 240 beats per minutes with narrow QRS complex, vagal
maneuvers are attempted and prove to be unsuccessful. With of the
following drugs should be given?

A. lidocaine
B. procainamide
C. metoprolol
D. adenosine
E. magnesiosulfato

43. A child suffers a provoked bite from a stray dog that was captured by animal
(control) and appears healthy. The most appropriate action would be?

A. confine and observe the dog for 10 days for signs suggestive of rabies
B. submit the dog's head for examination for rabies
C. begin rabies vaccine
D. administer human rabies immune globulin (HRIG) and begin rabies
vaccination
E. None of the above, no postexposure prophylaxis is necessary because it
was a provoked attack

14
44. Wilms tumor (nephrobalstoma) is the most frequent tumor of the
genitourinary tract in childhood.The most common presenting sign of this
neoplasm is which of the following?

A. abdominal mass
B. abodominal pain
C. Edema
D. Hematuria
E. Hypertesion

45. Which of the following best describe atopic dermatitis icchildren ?

A. It tends to spare the face and arms


B. it is frequently associated with uvetis
C. it is rarely begins during the first 2 years of life
D. it is characterized by pruritus and lichnfication
E. it is associated with elevated serum levels of IgA and IgM and decrease
levels of IgE

46. The major concern regarding chronic otitis media with effusion is the
development of which of the following?

A. Meningitis
B. Mastoditis
C. permanent nerve deafness
D. perforation of the tympanic membrane
E. impaired speech and language development

15
47. Febrile seizure occur most frequently at what age?

A. in the first month of life


B. in the first 6 months of life
C. between 6 months and 5 years
D. between 5 and 10 years of age
E. around the time of puberty

48. Congintalhypothyrodism should be included in the deferential diagnosis of a


newborn with which of the following ?

A. coma
B. prolonged jaundice
C. pulmonary edema
D. renal failure
E. sever anemia

49. An 8 month old female presents with failure to thrive,constipation,fever and


polydipsia. On evaulation you find hypokalemia and hyperchoremia ,
metabolic acidosis and suspect Fanconi syndrome .Which of the following
would be the most likely inherited caues ?

A. cystinosis
B. cystic fibrosis
C. glycogen storage disease
D. Tay-sachs disease
E. tyrosinemai

16
50. A 50 month old male infant has been on three occasions with febrile illness
without localizing signs .the mother is concerned about some sort of
immune deficiency that is making her infant susceptible to infection .
Which of the following statements is true?

A. Low concentration of IgG compared to adult levels are a normal


occurrence at months
B. Infants are born with IgG .IgM and Ig Alevels near normal adult values
C. Boys with X- linked agmmaglobulinemia usually develop serious
infection before 6 months age
D. Infant with hypogmmaglobulinemia with do poorly ordinary viral
infection
E. Patient with hypogmmaglobulinemia can usually handle infection with
encapsulated organisms normally

51. A 63 year old man undergoes partial gastrostomy with Billroth II


reconstruction for intractable peptic ulcer disease which of the following
metabolic disturbance is not potential consequence of this procedure?

A. megaloblastic anemia
B. Irone deficiency anemia
C. Osteoparosis
D. Osteolitisfibroecystica
E. steatorrhee

17
52. A 52 year old woman undergoes sigmoid resection with primary
anastomosis for recurrent diverticulitis .she returns to the emergency room
10 days later with flank pain and increased urine output. Laboratory
examination is significant for a WBC 20000.she undergoes a CT scan that
demonstrates new left hydronephrosis but no evidence of intra-abdominal
abscess. Which of the following is the most appropriate next in
management?

A. IV pulgram
B. IV antibiotic and repeat CT scan in 1 week
C. Administration IV methylene blue
D. NO further management if urinalysis in negative for hematuria
E. Immediate reexploration

53. A 65 year old man has enterocutaneus fistula origination in the jejunum
secondary to inflammatory bowel disease . Which of the following would be
the most appropriate fluid for replacement of his enteric losses?

A. D5W
B. 3% NaCl
C. Ringer lactate solution
D. 0.9% NaCl
E. 6% sodium bicarbonate solution

54. 64 year old man with history of severe emphysema is admitted for
hematemiesis .the bleeding cease soon after adminission . But the patient
becomes confused and agitated .Arterial blood gases are as follows PH -
7.23 PO2 42 mmHg. PCO2 75 mmHg. Which of the following is the best
initial therapy for patient?

A. Correct hypoxemia with high flow nasal O2


B. Correct acidosis with sodium bicarbonate
C. Administer 10 mg IV Dexamethasone
D. Administer 2 mg IV Activan
E. Intubated the patient

18
55. A 43-year-old trauma patient develops acute respiratory distress syndrome
(ARDS) and has difficulty oxygenating despite increased concentrations of
inspired O2. After the positive end-expiratory pressure (PEEP) is increased,
the patient’s oxygenation improves. What is the mechanism by which this
occurs?

A. Decreasing dead-space ventilation


B. Decreasing the minute ventilation requirement
C. Increasing tidal volume
D. Increasing functional residual capacity
E. Redistribution of lung water from the interstitial to the alveolar space
.

56. An elderly nursing home resident arrives at the emergency room moribund
with little helpful history. She is profoundly alkalotic by blood gas
measurement, and her urine shows an acid pH. A possible explanation is

A. Release of inappropriate antidiuretic hormone


B. Severe crush injury
C. Acute tubular necrosis
D. Gastric outlet obstruction
E. An eosinophilic pituitary adenoma

57. A 72-year-old man has an altered sensorium after a high-speed motor


vehicle collision and is placed in the intensive care unit for monitoring
overnight. Which of the following strategies should be used in order to avoid
the development of an aspiration pneumonia?

A. Nasogastric decompression
B. Steroids
C. Prophylactic antibiotics
D. Antacid administration
E. High positive end-expiratory pressure

19
58. A 27-year-old man sustains a single gunshot wound to the left thigh. In the
ER, he is noted to have a large hematoma of his medial thigh. He complains
of paresthesias in his left foot. On examination, there are weak pulses
palpable distal to the injury and the patient is unable to move his foot. Which
of the following is the most appropriate initial management of this patient?

A. Angiography
B. Immediate exploration and repair in the operating room
C. Fasciotomy of the anterior compartment of the calf
D. Observation for resolution of spasm
E. Local wound exploration at the bedside

.
59. A 29-year-old man sustained a gunshot wound to the right upper quadrant.
He is taken to the operating room and, after management of a liver injury, is
found to have a complete transection of the common bile duct with
significant tissue loss. Which of the following is the optimal surgical
management of this patient’s injury?

A. Choledochoduodenostomy
B. Loop choledochojejunostomy
C. Primary end-to-end anastomosis of the transected bile duct
D. Roux-en-Y choledochojejunostomy
E. Bridging of the injury with a T tube

20
60. Following a 2-hour firefighting episode, a 36-year-old fireman begins
complaining of a throbbing headache, nausea, dizziness, and visual
disturbances. He is taken to the ER, where his carboxyhemoglobin (COHb)
level is found to be 31%. Which of the following is the most appropriate
next step in his treatment?

A. Begin an immediate exchange transfusion.


B. Transfer the patient to a hyperbaric oxygen chamber.
C. Begin bicarbonate infusion and give 250 mg acetazolamide (Diamox)
intravenously.
D. Administer 100% oxygen by mask.
E. Perform flexible bronchoscopy with further therapy determined by
findings.

61. A 49 y/o male,irritable, his face changed to become round,he is impotent


,with purplish lines on th flanks, hypertensive ,CT scan shows 3cm right
adrenal mass , what's true ?

A. a surgical intervention is indicated only for lesions more than 6cm


B. MRI and scintigraphic scan is not useful in distinguishing adenoma from
carcinoma preoperatively
C. operative exploration of both adrenals is indicated
D. preoperative CT guided biopsy of adrenal lesion should be performed
E. steroid therapy maybe required for up to 6-12 months postoperative

62. A 36 woman with palpitations, anxious, headache , diagnosed with


pheochromocytoma , which approach is to optimize pt postoperatively ?

A. fluid for 24 hours preop to prevent intraop congestive heart failure


B. alpha blocker for 24 hrs prior to surgery
C. alpha blockers for 1-3 weeks prior to surgery
D. beta blocker for 1-3 weeks
E. (sorry I couldn't copy it )

21
63. A 40 y/o woman with skin rash around the areola, erosive nipple, tenderness,
itching, inttermetent bleeding, what's true?

A. this usually preceedspaget disease of the bone


B. pt's with nipple areolar ectematous changes
C. negative lymph node because it's a manifestation of intraductal
carcinoma only
D. 10-15% of newly diagnosed breast carcinoma
E. treatment with wide excision when it is presents as a mass

64. A 51 y/o male pt with 2cm left thyroid nodule ,thyroid scan reveald cold
nodule ,FNA with follicular cells , about pt condition what's true ?

A. thyroid nodules in men is very rare to be malignant


B. increased age associated with improved prognosis with follicular
carcinoma
C. abnormal cytology ,is indicated for initial course of TSH suppression by
thyroid hormone is recommended
D. in the setting of possible follicular neoplasm ,prophylactic neck
dissection is indicated
E. thyroid lobectomy is accepted for this pt

65. A 41 y/o man complains of regurgitation of saliva and undigested food,


esophagogram reveals bird's beak ,what's true ?

A. chest pain is common in advanced disease


B. more pts will improve by forceful dilatation than surgical interventions
C. manometry with high resting pressue in LES
D. surgical tt consists primary of resection of the distal esophagus with
reanastomosis to stomach above the diaphragm
E. pt at no increase risk to develop cancer

22
66. A 56 y/o female pt, 3 months after colostomy to the sigmoid resection of ca ,
complains that stoma not functioning properly , the most common serious
complication af an end colostomy ?

A. bleeding
B. Skinbreakdown
C. parastomal hernia
D. colonic perforation during irrigation
E. stomal prolapsed

67. A 32 yr old alcoholic with end stage liver disease has been admited to the
hospital 3 times for bleeding esophageal varices. he has undergone banding
and sclerotherpay previously. he admits to currently drinking a six-pack of
beer per day, on his abdominal examination , he has a fluid wave. Which of
the following is the best option for long term managment of this patient's
esophageal varices?

A. orthotopic liver transplantation


B. transection and reanstomosis of the distal esophagous
C. distalsplenorenal shunt
D. end -to- side portocaval shunt
E. transjugular intrahepatic portosystemic shunt(TIPS)

23
68. A 48 yr old women develops pain in the right lower quadrent while playing
tennis. the pain progresses and the patient presents to the emergency room
later that day with a low-grade fever, a WBC count of 13000/mm3 and
complaints of anorexia and nausea as well as persistent pain of the right
lower quadrant. on examination she is tender in the right lower quadrant
with muscular atrophy, and there is a suggestion of a mass effect. an
ultrasound is orderd and shows an apparent mass in the abdominal wall.
which of the following is the most likely diagnosis?

A. acute appendicitis
B. cecal carcinoma
C. hematoma of the rectus sheath
D. torsion of an ovarian cyst
E. cholecystis

69. A 55 yr old man who is extremely obese reports weakness, sweating,


tachycardia, confusion and headche whenever he fasts for more than few
hours, he has prompt relief of symptoms when he eats. which of the
following statements is true regarding his diagnosis?

A. the tumor arises from pancreatic alpha-cells


B. simple excision is curative in the majority of the cases
C. symptoms arise from a rapidly rising glucose level
D. the majority of tumors are malignant
E. standard of treatment is chemotherpay and radiation

24
70. A 28 yr old women who is 15 weeks pregnant, has now onset of nausea,
vomiting, and right-sided abdominal pain. she has been free of nausea since
early in her first trimester.the pain has become worse over the past 6
hours.which of the following statements regarding appendicitis during
pregnancy is correct?

A. appendicitis is the most prevalent extrauterine indication for colliotomy


during pregnancy
B. appendicitisocurs more commonly in pregnant women than in non-
pregnant women of comparable age
C. suspected appendicitis in a pregnant woman should be managed with a
period of observation due to the risks of the laparotomy to the fetus
D. non-complicated appendicitis results in a 20% fetal mortality and
premature labor rate
E. the severity of appendicitis correlates with increased gestational age of
the fetus

71. A patient with a non-obstructing carcinoma of the sigmoid colon is being


prepared for elective surgery.which of the following reduces the risk of
postoperative infectious complications?

A. a single preoperative parenteral dose of antibiotics effective against


aerobes and anareobs
B. avoidence of oral antibiotics to prevent emergency of C.difficile
C. postoperative administration for 48 hour of parentersl antibiotic effective
against aerobs and snaerobs
D. postoperative administration of parenteral antibiotic effective against
aerobs and anaerobs until the patients intravenous times and all other
drains are removed
E. redosing of antibiotic in the operating room if the case lasts for more than
2 hour

25
72. A teenage boy falls for his bicycle and is run over by a truck.on examination
in the emergency room he is awake and alert and appears frightened but in
no distress. the chest radiograph suggest an air-fluid level in the left lower
lung field and the nasogastric tube seems to coil up and into the left
chest.which of the following is the next best step in the managment of this
patient?

A. placment of a left chest tube


B. thoracotomy
C. laparotomy
D. esophagogastroscopy
E. diagnostic peritoneal lavage

73. A 65 year old man who smokes cigarettes and has chronic obstructive
pulmonary disease falls and fractures the third , fourth and fifth ribs in the
left anterolateral chest . chest X-ray is otherwise normal. Which of the
following would be the most appropriate next step in his management?

A. Strapping the chest with adhesive tape


B. Admission to the hospital and treatment with oral analgesia
C. Tubethoracostomy
D. Placement of an epidural for pain management
E. Surgical fixation of the fractured ribs

74. A 32 year old man is in a high-speed motorcycle collision and presents with
an obvious pelvic fracture . on examination , he has a scrotal hematoma and
blood at his urethral meatus. Which of the following is the most appropriate
next step in his management ?

A. placement of a Foley catheter


B. cystoscopy
C. CT of the pelvis
D. retrograde urethrogram
E. nephrostomy tube placement

26
75. A 49 year old man was the restrained driver in a motor vehicle collision . he
decelerated rapidly in order to avoid hitting another car and swerved into a
ditch . he complains of chest pain . which of the following finding on chest
X-ray would be most suspicious for an aortic injury?

A. multiple right-sided rib fractures


B. a left pulmonary contusion
C. a left pneumothorax
D. widening of the mediastinum greater thean 8 cm
E. pneumomediastinum

76. A 62 year old woman presents withinvasive ductal carcinoma of the right
breast . Which of the following findings would still allow her to receive
breast conservation surgery (partial mastectomy)?

A. diffuse suspicious microcalcification throughout the brest


B. multifocal disease
C. previous treatment of a breast cancer with lumpectomy and radiation
D. large tumor relative to breast size
E. persistently positive margins after multiple reexcisions of the breast
cancer

77. Which of the following patients with primary hyperparathyroidism should


undergo parathyroidectomy?

A. A 62-year-old asymptomatic woman


B. A 54-year-old woman with fatigue and depression
C. A 42-year-old woman with a history of kidney stones
D. A 59-year-old woman mildly elevated 24-hour urinary calcium excretion
E. A 60-year-old woman with mildly decreased bone mineral density
measured at the hip of less than 2 standard deviation below peak bone
density

27
78. Incisional biopsy of a breast mass in a 35-year-old woman demonstrates
cystosarcomaphyllodes at the time of frozen section. Which of the following
is the most appropriate management strategy for this lesion?

A. wide local excision with a rim of normal tissue


B. lumpectomy and axillary lymphadenectomy
C. modified radical mastectomy
D. excision and postoperative radiotherapy
E. excision , postoperative radiotherapy , and systemic chemotherapy

79. A 69 y/o female notices a lump on both sides of her neck. a Fine Needle
Aspiration biopsy is non diagnostic, and she undergoes a total tyroidectomy.
Final pathology reveals a 2 cm Hurtle cell CC.What is the most appropriate
surgical treatment?

A. no further therapy
B. Chemotherapy
C. External beam radiotherapy
D. Radioiodine ablation
E. Chemotherapy and External radiotherapy and radioiodine ablation

80. A 41 y/o female notices bilateral thin serous discharge from her breasts.
There seems to be no mass associated with it. which statement would be
appropriate to tell the patient?

A. Intermittent thin milky discharge can be physiologic


B. expressible nipple discharge is an indication for an open biopsy
C. absence of a mass on a mammogram rules out malignancy
D. galactorrhea is indicative of underlying malignancy
E. pathological discharge is usually bilateral

28
81. A 39 y/o male, previously healthy is hospitalized for 2 weeks for epigastric
pain, radiating to the back with nausea and vomiting. Initial laboratory
values reveal increased amylase level, consistent with acute pancreatitis. %
weeks following discharge he complains of early satiety, epigastric pain and
fevers. On presentation his temp. is 38.9c and his heart rate is 120bpm. His
WBC count is 24000/mm3 and his amylase level is normal. He undergoes
CT scan demonstrating a 6*6 cm rim-enhanced fluid collection in the body
of the pancreas. Which of the following would be the most definitive
management of fluid collection?

A. Antibiotic therapy alone


B. CT guided aspiration with repeat imaging in 2-3 days.
C. Antibiotic therapy with CT guided aspiration with repeat imaging in 2-3
D. days.Antibiotic therapy with percutaneous catheter drainage.

82. 80 y/o male with a history of symptomatic cholelythiasis presents with signs
and symptoms of small bowel obstruction. which of following would
provide the most help in ascertaining the diagnosis?

A. coffee ground aspirate from stomach


B. pneumobilia
C. WBC count of 40000/ml
D. pH 7.5, pCO2 50 and paradoxically acid urine
E. palpable mass in the pelvis

29
83. A 22 y/o female is seen in surgery clinic for a bulge in the right groin. She
denies pain and able to make the bulge disappear by lying down and putting
steady pressure on the bulge. She never experienced nausea or vomiting. on
exam she had reducible hernia below the inguinal ligament. Which of the
following is the most appropriate management of the patient?

A. observation for now and follow up in surgical clinic in 6 months


B. observation for now and follow up in surgical clinic if she develops
further symptoms
C. elective surgical repair of the hernia
D. emergent surgical repair of the hernia
E. emergent surgical repair of the hernia and exploratory laparotomy to
evaluate small bowel.

84. 80 y/o male is admitted to the hospital complaining of nausea, abdominal


pain, distention and diarrhea. a cautiously preformed transnasal contrast
study reveals apple core configuration in the rectosigmoid area. Which of
the following is the most appropriate next step in his management?

A. colonoscopic decompression and rectal tube placement


B. saline enemas and digital disimpaction of fecal matter from the rectum
C. colon resection and proximal colostomy
D. oral administration of metronidazole and Clostridium Difficile titers
E. evaluation of an electrocardiogram and obtaining an angiogram to
evaluate for colonic mesenteric ischemia

85. A 30 y.o p who presents with diarrhea and abdom. Discomfort is found at
colonscopy to have colitis confiend to the transverse and descdnding colon.
A biopsy diagnostic is performed. Wich of the following is a finding
consistent with this patientdiagnostic?

A. The inflamatory process is confined to the mucoasa and submucoasa.


B. The inflamatory reaction is likely to be continuos
C. Superficial as aapposed to linear ulcerationcanpe expected.
D. Nonceatinggranulamos can be expected in up to 50% of patients
E. Microabcesses with crypts are comon.

30
86. 54 y.o man complains that his eyes are yellow his bilirubin level is elevated.
His physical exam. Is unremarkabel. a CT Scan of the abdomin shows a
small mass in the head of the pancreas ehcasing the superior mesenteric
arterycytology from ERCP is pozitiv for cancer. Wich of the following is the
most appropiate treat. For this patient?

A. pancreaticoduodenectomy
B. pancreaticoduodenectomy with reconstruction of the superior mesenteric
artery
C. distalpancreatectomy
D. distalpancreatectomy with reconstruction of the superior mesenteric
artery
E. chemoradiation therapy

87. A 28 Y.O woman ppresents with hematochezia she is admitted to the


hospital and undergoes upper endoscopy that is negativ. Colonscopy is
performed and no bleeding sources are identifed, although the
gastroenterelogist notes blood in the right colon and old blood coming from
above the ileocecal valve. Wich of the following the test of choice in this
patient?

A. angiography
B. small bowel enteroclysis
C. CT scan of the abdomen
D. Tecnetium99m(99mtc) pertechnetate scan
E. samll bowel endoscopy

31
88. A 35 y.o man presents with RUG pain,fever,jaundice and shaking chills. Usg
of the abdomin demonstrates gallstones, normal gallblader wall. Thickness
and comon bile duct of 2.1cm. The patient is admitted to the hospital and
given i.v fluids and antibiotic. He continues to be febrile with increasing
WBCs. Wich of the following is the most appropriate next step in this
patient managment?

A. ERCP
B. placement of cholocystostomy tube
C. Laparascopiccholecystostomy
D. opnen cholecystectomy
E. emerent operation and decompresion of the comon bile duct with T tube

89. An 88 y.o man with a history of end stage renal failure,severe coronary
artery disease and brain metastasis from lung cancer presents with
acutcholecystitis. His family wants evrething done. Wich is the best
managment option in this patient?

A. tubecholecystostomy
B. open cholecystectomy
C. laparascopiccholecystctomy.
D. I.V Antibiotic followed by elective cholecystectomy
E. lithotripsy followed by long term bile acid therapy

90. A 57 y.o woman presents with adenocarcinoma of the right colon.labaratory


evaluation demonstrates an elevation of carcinoembryonic antigen(CEA) to
123 ng/ml. Wich of the following is the most appropriate use of CEA
testing in patient with colorectal cancer?

A. As a screening test for colorectal cancer.


B. To determine wich patient should receive adjuvant therapy
C. To determine wichpatient should receive neadjuvant therapy.
D. To monitor for postoperative recurrence

32
91. The hallmark biochimico feature of the re feeding syndrome that associated
with increase mortality is:

A. Hypokalemia
B. Hypofosfatemia
C. Hypomagnesemia
D. Hypoglycemia
E. Hypocalcemia

92. Which of the following maneuver can correct hypoxemia in pazient that
receiving mechanical ventilation support

A. Increased peep
B. Decrease tidal volume
C. Decrease mean airway pressure
D. Increasei/e rate
E. Increase respiratory rate

93. After intubation the most accurate and expedition way to determine whether
the endotracheal is in the trachea.

A. Lung ascoltion
B. Stomacascoltion
C. tube misty
D. end tidal volume
E. chest wall raising

33
94. An 55 y old undergo an laparoscopic appendectomy for acute appendix,
final pathology reveals the presents of 2 cm carcinoide invading the
mesoappendix, whattx you would recommend at the clinic visit.

A. nonfurther intervention
B. Righthemicolectomy
C. Adjuvatinfalfa
D. Adjuvantocterotride
E. Cecectomy

95. An 25 y old woman is in 12 week of pregnancy and present to the


emergency department with right side abdominal pain for 24 h ,fever,
emesis,wbc 12,000 mm3 ,her exam reveals s gravid uterus and right sided
tenderness without peritoneal focal sign. Her urine analysis is
normal,ultrasound revealed a viable fetus and no cholelithasis,the appendix
is not seen on the ultrasound study, which of the most appropriate test.

A. Laparoscopic exploration
B. ct abdominal and pelvic
C. open appendectomy
D. mri abdominal and pelvic
E. Admit and observe

96. An 36 y man has additional ct scan after a motor vehicles crash, no injuries
are found but hid blood pressure is 160/100 ,the ct show 3 cm adrenal mass,
appropriate initial biochimico evaluation should include all except :

A. Plasmametapherins
B. Plasmaaldosteron
C. loedose overnightdesametason Suppression test
D. plasma rennin
E. serumacth

34
97. 52y.o woman with bmi 45 undergoes laparoscopic sleave gastrectomy.
postoperatively;she is at greatest risk for which of the following?

A. gastric stricture
B. surgical site infection
C. gastric leak
D. pulmonaru embolism
E. delayedgastring emptying

98. a 68 years old woman with insulin dependent diabetes mellitus and
myocardial infarction 3.5 weeks ago presents with right upper quadrant pain
;nausea ;and vomiting for 5 days.Her heart rate is 90 per minute ;her blood
pressure is 170/100 mmhg and her temperature is 38.0 with a tender right
upper quadrant . her bilirubin is 1.0 mg/dl her lipase is 48 and her wbc is
18.000/mm3 .ultrasound shows cholelithiasis ;a gallbladder wall of 6 mm;
after 24 hours of antibiotics therapy ;her abdominal pain does not improve.
Which of the following is the next step?

A. ercp
B. laparoscopic cholecystectomy
C. open cholecystectomy
D. percutaneouscholecystostomy tube
E. transhepatic biliary tube

99. A 65 years old woman in surgery clinic, it has been 6 week since recent
hospitalization, in which she received nonoperative treatment of presumed
perforated appendicitis (IV and oral antibiotics for 10 days ), she is now
feeling well and is eating regular diet. On physical exam she has normal
vital signs and an unremarkable abdominal exam, which is the next step?

A. interval appendectomy.
B. abdominal CT scam.
C. diagnostic laparoscopy .
D. colonscopy.
E. creactive protein measurement.

35
100. A 83 years old emaciated female resident of a nursing home who has
never undergone an abdominal operation presents with a distal small
intestinal obstruction and right thigh and knee pain that began 3 days earlier,
which of the following is true ??

A. celiotomy is unlikely to cure this patient.


B. abdominal/pelvic CT scan is useful in establishing the diagnosis.
C. hiposteoarthropathy with associated ileus in the most likely cause.
D. nasogastric suction should resolve the patients symptoms.
E. end of life care should be provided.

101. Fetal findings on an ultrasound scan during week 32 include skin


edema, pericardial fluid and ascites, the most likely caused these findings ?

A. CMV cytomegalovirus.
B. listeria.
C. parvovirus B19.
D. herpes simlex virus.
E. toxoplasma.

102. A fetal weigh evaluation on 34 weeks identifies a result of less than


10th percentile, what is the most common etiology for this finding ?

A. constitutional.
B. placenta insufficiency.
C. viral infection.
D. anaploidity.
E. smoking.

36
103. a woman that is diagnosed with severe eclampsia on 39th week of
gestation of her first pregnancy is treated in the delivery room with labetalol
and magnesium sulfate, her vital signs include HR/56, RR/10 , BP110/85,
which of the following medication should be immediately available ?

A. calcium gluconate.
B. bicarbonate.
C. hydralazine.
D. nitroglycerine.
E. ritordine.

104. a woman with twin pregnancy is admitted to the delivery room in


active labor on the 37th weeks of gestation, fetus A has a cephalic
presentation and fetus B has a breech presentation, there is no growth
retardation for both fetuses, what is the most appropriate management ?

A. only cesarean section.


B. the decision depends on the twins chorionic status, monochorionic twins
caesarean section, bichorionic twins vaginal delivery.
C. begin vaginal delivery if complicated cesarean section.
D. both deliveries are acceptable the recommendation should be given
according to the patients preferences.
E. the first fetus total breech extraction the second cephalic delivery.

105. Which of the following medication is the most important in


prevention of complication in imminent premature delivery ??

A. magnesium sulfate.
B. ritodrine.
C. betamethasone.
D. indomethacin.
E. progesterone caproate.

37
106. Herpes Simplex Infection during Pregnancy , what is correct?

A. Repeat flare up with HS during the 3.semester is associated with high


risk as compared to single Infection at the same time.
B. A woman with HS 1 infection schould not worry about genital Infection.
C. A negative result on viral Culture with negative result on PCR rules the
Diagnosis.
D. Acyclovir is a Class D medication -not to use in pregnancy.
E. Every active genital infection with HS2 during labor necessiate -
CesarienSection

107. A 22 years old female is on oral Contraceptive arrives to primary


Clinic and complaints about chronic pelvic pain,headache, and eating
disturbances . what is the Diagnosis?

A. Endometriosis
B. Sexual assault
C. Chronic PID
D. Dysmenorrhea
E. side effects of oral contraceptive

108. A 28 Years Old Female come to routine to genological examination.


On which a pap smear is suggested.Which of the following is correct?

A. the most common cause of cervical Carcinoma is HPV 31


B. performing Pap Smear is prefered over Cytology Examination immersed
in preversing Fluid
C. as many as 40% of the woman with cervical changes due to HPV will get
rid of the Virus with the Autoimmune System.
D. the Efficiency of HPV Immunization reach about to 70%.

38
109. What is the most common symptom of endometrial Carcinoma ?

A. the most common symptom is vaginal bleeding


B. the clinical symptom appear usually after the disease involves the
Adnexa
C. the first most common symptom is pelvic pain
D. the disease is usaullydiganosed late because it first their are no typical
symptoms

110. A 23 years old female come of dysmenorrhea and pain associated


with deep sexaulintercours .which of the following disease are this
symptoms most suggestive of ?

A. PID
B. recurrent corpus luteum Cyst
C. endometriosis
D. Gardinala Vaginitis
E. Pelvic Adhesion

39
part B

1. Which of the following medications most appropriate for the treatment of


urge incontinence?

A. Oxeybutynin chloride
B. Nifedipine
C. Estroderm
D. Probutondepo
E. Benoxybenzanin

2. Which of the following statements is the most accurate regarding gestational


diabetes mellitus (GDM)?

A. 50% of the females with GDM will develop type 2 diabetes in future.
B. There is damage to pancreatic insulin secretion.
C. There is increased rate of GDM in patients with polycystic ovary (PCOS)
D. All of the above are correct.
E. Answersa+c.

3. Which of the following is correct regarding physiologic cardiac changes in


pregnant women?

A. A holosystolic murmur can be heard in up to 90% of the pregnant


women.
B. There is a split S1
C. A prolonged murmur can be heard left sternal border that is caused by the
mammary artery.
D. Answersa+c
E. All answers are correct.

40
4. What is true regarding the human placenta?

A. The placenta arises from the endometrium.


B. Thecytotophoblast cells are the cells that invade the endometrium
C. PAPP- A originates from the placenta.
D. HCG has similar structure to FSH
E. HCG can be first detected on day 11 of a cycle of 28 days

5. A patient with SLE is 6 weeks pregnant. Which of the following is true?

A. She has increased risk of the developing gestational diabetes.


B. She should later on be referred to fetal echocardiography
C. There is no increase in fetal central nervous system defects.
D. Hydroxychloroquine is considered safe during pregnancy.
E. All answers are correct.

6. A 14 years old female comes to clinic accompanied by her mother and


complaints that she never had menses What should you do?

A. Check for secondary sex characteristics and if present follow up after a


year.
B. Evaluate sex hormones, thyroid function tests, and karyotype and
compete other work up at age 15
C. If she has headaches then immediate evaluation should be performed.
D. Answersa+c
E. Eating disorders is the most cause of amenorrhea of this age.

41
7. A 46 female describe her menses as following: menses every 35-40 day .
Lasting for 5 days , heavy bleeding during menses,light bleeding following
sexsual intercourse , painduring menses. And occasional bleeding during
midcycle. Which of the following amnestic detailes is most sospicious of
patology that requers evaluation?

A. Menses fracutures
B. Heavy bleeding
C. Midcycle bleeding
D. Bleeding following sexsual intercourse
E. Pain during menses

8. Which of the following myomas is the easiest to remove with hysteroscopy?

A. Peduncalsubserous
B. Subserous
C. Cervical
D. Submucus
E. Peduncalsubmucus

9. Which of the following statements regarding uterine myomas is correct?

A. About 60% of females with uterine myomas and unexplanable infertility


will conceive following the myomas removal
B. Increased pain due to myomas is common during pregnancy
C. Myomas are associated with malpresentation at birth
D. Oral contraceptives cause increase in myomas size
E. A+B+C

42
10. Which of the following statment is correct with regards to HPV infection?

A. HPV is involved in about 30% of CIN cervical lesion


B. In about 30% of the females with cervical HPV the virus will not be
present after 2 years
C. It appears that smoking has no sinergic effect with HPV infection
D. Incidence of CIN is 20-30%
E. HPV is the most virulent subtype

11. Which of the following is true regarding schiller test?

A. Synonym for HPV type testing


B. A routine part of colposcopy
C. Biopsy willbe performed in brown stained areas
D. Is based on glycogen staining with iodine
E. B+D

12. Which of yhe following is correct cervical adenomacarcinoma?

A. It is more charateristic in females over the age of 35


B. In contrast to squamous cell cancer, there is no association HPV
C. There is a type of cancer that is associated with exposure to
dietylsilbestrol(DES)
D. There is decreased incidence of this cancer with increased incidence of
epithelial cance

43
13. 26 year old female , mother of 1 child. Complaining of pain during menses
that began at age 16 and recently intensified. What is the first line treatment
for dysmenorrhea?

A. Oral contraceptive (combined estrogen and progesterone).


B. NSAIDs
C. Progesterone only oral contraceptive.
D. Calcium channel blocker.
E. SSRIs.

14. 55 year old female patient , mother of 3 children. Has irregular menses,
comes to clinic with cervical bleeding that occurs in the last 6 months and
between the menses as well. What would you recommend?

A. This is probably perimenstrual menses and therefore no concern at this time.


B. The complains are suspicious and therefore she should be referred to uterine
curettage ASAP.
C. Since she isn’t planning on another pregnancy she should be referred to
hysterectomy.
D. Pipelle test should be ordered ASAP.
E. She should be referred for a diagnostic hysteroscopy.

15. 18 year old female, complains of primary amenorrhea. On examination there


is uterus with no breast development. Karyotype result characteristic of?

A. GonadalDysgenesis.
B. Mullerian agenesis.
C. Lack of GnRH secretion.
D. Alpha-hydroxylase deficiency.
E. Aromatase deficiency.

44
16. 16 year old female presents with primary amenorrhea. On examination has
normal vagina and uterus with no breast development. Karyotype testing
demonstrates 46XX. What is the next step in the evaluation of this patient?

A. Pregnancy test.
B. Serum FSH level.
C. Serumfree testosterone level.
D. Urinary collection for cortisol.
E. Pelvic ultrasound to rule out PCO.

17. 15 year old female presents with primary amenorrhea. Her breast started to
develop 3 years ago. She denies having sexual intercourse and physical
examination id normal. What is the next step?

A. Reassure the girl that she is within the norm.


B. Pregnancy test.
C. Karyotype testing.
D. Serum FSH level.
E. Psychiatric evaluation to rule out bulimia.

18. Which of the following is characteristic of a female with kallmann


syndrome?

A. Female phenotype, no uterus, normal breasts, karyotype 46XX.


B. Female phenotype, uterus, no breasts development, karyotype 46XY.
C. Female phenotype, uterus, breasts, karyotype 46XY.
D. Female phenotype, no uterus, no breasts development, karyotype 46XY.
E. Female phenotype, uterus, no breasts development, karyotype 46XX.

45
19. Which of the following is not a cause of hirsutism:

A. danazol
B. combined contraceptive
C. hyperprolactenemia
D. cushing syndrome
E. idiopathic

20. Main reason that the spermatic fluid is removed from sperm cells prior to
uterine injection

A. to prevent urine system bacteria to penetrate the uterine cavit


B. to discard white blood cells that may contain pathologic virus of the
reproductive system
C. to prevent allergen from sensitizing the local immune system within the
uterus
D. to remove prostaglandins that may contract the uterus
E. todelute the sperm concentration so that sperm cells can spread on the
uterus & fallopian tubes

21. Mandatory to fertilization the ovum with ICSI during IVF cycle

A. when the ovum is from donor to decrease infection rate


B. ovum from a female of age more than 40y
C. sperm source from TESA
D. ovum underwent zona hatching
E. sperm tested by Comet prior to fertilization

46
22. Which of the following is a possible ovary stimulation protocol during IVF?

A. treat with GnRh agonist starting on cycle day 21 & add gonadotropines
with a beginning after next cycle
B. treat with gonadotropins until 13 mm follicle is achieved & than add
GnRh agonist
C. treat with antagonist & gonadotropins starting of the first cycle day
D. treat with gonadoropins start from the first cycle day for 5 days & than
add GnRh agonists

23. How will mature ovum look?

A. after first meiosis but no polar body can be seen


B. after first meiosis, polar body can be seen
C. after second meiosis one polar body can be seen
D. after second meiosis two bodies can be seen

24. 32 y.o. woman arrive to clinic and reports of a mentally retarded son due to
fragile x syndrome. On genetic testing the mother is found to carry a
premature with 80 repetition of CGG sequencing one allele while the other
allele is normal. What would you recommend?

A. spontaneous pregnancy with karyotype testing via amniotic fluid


B. IVF with ICSI fertilization and prenatal diagnosis
C. IVF and pre implantation gestational screening.
D. IVF and pre implantation gestational diagnosis

47
25. 30 y.o. couple that married for 4 years unsuccessful in trying to conceive.
The female evaluation is normal. Sperm counts demonstrate 2 ml, 2 million
cells/ml 50% motility, 5% normal cells. Repeat testing is similar. Which is
the appropriate treatment?

A. intrauterine sperm injection withouth ovulation encasement


B. ovulation encasement via clomifen& intrauterine sperm injection
C. ovulation encasement via gonadotropins & intrauterine sperm injection
D. IVF with drops containing about 100,000 sperm cells/ovum
E. IVF&intracytoplasmic sperm injections

26. 50 y.o. male admitted to the ER for front chest pain that begin 30 min ago &
is accompanied by dyspnea. On ECG a 3mm ST segment elevation is noted
in lead 2, 3, AVF. Blood troponin level normal. What is the most likely
diagnosis?

A. Acute pericarditis
B. Inferior wall MI
C. Acute PE
D. Subacute bacterial endocarditis

27. 75 y.o. female has CHF, NYHA 2 & treated with furosemide. On
echocardiogram a decrease in heart function is noted to EF 50%. She is
referred to ER for rapid palpitation of 5 days. ECG AF to 130 per min. what
the recommended treatment in this patient?

A. digoxin
B. angiotensin receptor blockers
C. beta-blockers
D. amiodorone

48
28. 40 y.o female house wife being referred for evaluation of dyspnea on short
exercise. On auscultation a diagnostic murmur can be heard at her apex an
accentuated S1. On echocardiogram the mitral valve area is measured 0.8
cm2. What would you recommend to this patient?

A. precautious valve dilatation.


B. repeated echocardiogram in 6 month.
C. treatment with calcium channel blockers
D. martial valve replacement

29. A young male with astma referred to spirometry. Which of the following
parameters will evaluate disease severity.

A. decreased FEV1/FVC
B. decreased FVC
C. decreased end expiratory flow
D. decrease FEV1

30. 65 y.o. male with known COPD is admitted due to exacerbation. He is


hemodinamicly stable and is treated with bronchodilators and steroids. On
blood gases there is acute respiratory failure with hypercapnia. What is the
next step management?

A. oxygen mask
B. intubation and ventilation
C. noninvasive ventilation
D. treatment with magnesium

49
31. 56 y.o. woman who is a smoker with a BMI of 32 being treated with
hormonal replacement therapy. She is admitted to the ER due to stabbing
chest pain and dyspnea that worsened with deep breath. On examination
regular HR 110 and heart lung are within the normal limits. Chest x-ray is
normal. Which imagining should be ordered for this patient?

A. Ventilation perfusion scan


B. US of deep vessels of the lower extremities
C. Pulmonary angiogram
D. CT angiogram of the pulmonary vessels

32. Which of the following is true regarding community acceured pneumonia?

A. Blood culture should be drown prior to initiation of antibiotic treatment.


B. empiric treatment should be initiated without need to identify the
organism
C. Sputum culture should be drown prior to initiation of antibiotic treatment
D. chest CT should be performed to diagnosis pneumonia

33. Which of the following patient qualified for definition of sepsis?

A. 60 y.o male with diagnosis of pneumonia, tem. 39, HR 100, BP 80/50,


without improvement after fluid administration.
B. 40 y.o. male with diagnosis of pneumonia, temp. 39, HR 95, BP 130/80
C. 30 y.o. male with diagnosis follicular tonsillitis and temp.40
D. 80 y.o male with diagnosis infected pressure sores, temp. 37.5 and WBC
1100 mm3

50
34. A 45 years old femal is complaining of general weakness.she is a mother of
3 childrens and has heavy bleeding during her mensis.On blood test-
hemoglobin level is 99/dl ,mcvis 73 and iron and ferritin level are low. What
is the most appropriate treatment for this patient?

A. unit of packed red blood cells


B. intavenous iron
C. oral iron
D. multivitamins

35. A 25 years old male is seen at clinic after he palpated a lymph node behind
the ear. He is usually healthy but in the last 2 months he reports of having
numerous night sweats. There is mediastinal narrowing on the chest x-
ray.What is the most likaly diagnosis ?

A. hodgkins lymphoma
B. diffuse B cell lymphoma
C. multiplemeloma
D. acutemyelocytic leukemia

36. Which of the following is more typical in ulcerative colitis than crohns dis?

A. abdominal mass
B. granulomas on intestinal biopsy
C. positiveASCa antibodies
D. substantial blood in stool

51
37. 35 year old male undergo gastroscopy and discovered to have peptic ulcer
and helicopacter pylori positive what is the most effective treatment ?

A. Clarithromycin + PPI for 1 week


B. Metronidazole + clarithromycin + PPI for 2 weeks
C. Metronidazole + PPI for 3 weeks
D. Clarithromycin + Amoxicillin + PPI

38. What is the viral hepatitis the increase risk of fluminent Hepatitis?

A. 25 year old female with Hepatitis E


B. 25 year old female with Hepatitis A
C. 80 year old male with Hepatitis B
D. 80 year old male with Hepatitis C

39. 46 year old male that have cirrhosis secondary to Hepatitis C is on


Propanolol, what is the effect of the medication?

A. Decrease hepatic encephalopathy


B. Decrease hematemesis
C. Decrease incidence of hepato-renal syndrome
D. Decrease incidence of spontaneous peritonitis

40. Previous healthy 18 year old male presented to the ER department


complaining from abdominal pain vomiting and diarrhea with decrease
alertness, tachycardia, tachypnea .lab values was : glucose :530 PH:7.15
HCO3:12meq what is the appropriate management ?

A. Insulin + bicarbonate
B. Subcutaneous insulin + normal saline
C. I.V insulin + normal saline
D. Subcutaneous insulin + bicarbonate

52
41. 65 year old male have lymphoma arrive to oncology department for the
treatment with chemotherapy his creatinine is 1.1mg/dl . 3 days later
presented complaing of general weakness + decrease urine output his new
creatinine is 3.5mg/dl what is your diagnosis

A. prerenal kidney disease


B. acute tubular necrosis
C. renal artery occlution
D. tumor lysis syndrome

42. A 16 year old male patient with tonsilitis started on treatment with penicillin
for 1 w then he came 2 wks after with headache and exacerbation edema
with blood pressure 160/100 and bloody urine :

A. after supportive and antibiotics treatment he will recover.


B. he is going to make renal biopsy after 2 wks beginning of disease.
C. he will have renal failure and will need dialysis.
D. he will have anemia and treatment with units of blood.

43. Which of the following findings is a criteria for the diagnosis of SLE

A. myocarditis
B. renal failure
C. pleuritis
D. liver failure

44. 60 y.o. female is seen in clinic for gradual weight gain, constipation and cold
intolerance. What blood test should be ordered?

A. cortisol level
B. TSH
C. PTH
D. albumin

53
45. 60 y.o. male with end stage renal failure and on hemodialysis is admitted to
the ER due to diffuse weakness. On ECG spiked T wave are identified in
anterior, lateral and frontal leads. What is the most probable course of
change?

A. hypokalemia
B. hypocalcaemia
C. hyperkalemia
D. Acute ischemia

46. 30 y.o male who is IV drug abuser is admitted due to a temp. 38, headaches
and general weakness. On physical examination he is with tachycardia,
tachypnea and normal heart sounds without murmurs. Chest x-ray
demonstrates multiple pulmonary nodules. What next step in management?

A. transesophagial echo
B. CT angio of the chest
C. lumbar puncture
D. bronchoscopy

47. A patient with RA has sever deformities of the wrist and fingers joints. What
treatment could have delayed irreversible changes?

A. ibuprofen (NSAID)
B. PREDNISONE
C. METHOTREXATE
D. penicillin

54
48. 60 y.o male with known diabetes, hypertension and hyperlipidemia is
referred to clinic due to increase swelling of the abdomen and extremity
edema. Laboratory test include sodium 129 mmol/l . ECG is normal. Chest
x-ray demonstrate moderate amount of right pleural effusion. What is the
most likely diagnosis?

A. pleural effusion secondary to hyponatrmia


B. pleural effusion secondary to nephrotic syndrome
C. pleural effusion secondary to pneumonia
D. pleural effusion secondary to excessive drinking

49. What will the plural tap of the above patient most likely to demonestrate ?

A. Exudate
B. Empyema
C. Chylothorax
D. Transudate

50. A 60 years old man is admitted to emergency room due to chest pain. On
ECG , ST elevation are noted on V1-V4 . What is the most appropriate
treatment ?

A. Fibrinolysis
B. Stenting
C. Beta Blocker
D. Heparine

51. What is true regarding colorectal cancer ?

A. There is family history in about 25% of patient


B. It is the most common cancer among men
C. Fiber rich diet is ver effective in its prevention
D. Chemotherapy is effective in about 50% of the patient

55
52. A 23 years old female is referred to hospital due to acute Renal failure that
developed about 2 weeks following antibiotic and NSAID treatment for
tonsils. What is the most likely promote the diagnosis ?

A. Renal biopsy
B. Abdominal MRI
C. Urine collection for protein quantification
D. Renal artery doppler

53. A 52 years old man who is a known drug abuser is brought to the ER by a
friend . The patient is somnolent and breathing slowly. On physical
examination he has hypothermia to 35c , bradycardia , and pinpoint pupils .
What is the most likely to overdose on ?

A. Amphetamines
B. Paracetamol
C. Benzodiazpine
D. Opiates

54. A 35 years old female is seen at clinic for weight gain mostly in the face and
abdomen . She also complain of general weakness and mood changes . Her
blood pressure at clinic is 160/105 and her fasting glucose level is 160 .
What is the most appropriate evaluation ?

A. T3 - T4 – TSH
B. PTH
C. Urinary free cortisol
D. Urinary catecholamines

56
55. An 80 years old disabled male was admitted 3 weeks ago to the Hospital
with urinary tract infection. He was treated with ceftriaxon for a week. He
now has high fever and watery diarrhea for three days. Which organism is
most likely to cause these symptoms??

A. Enterovirus
B. Clostridium
C. Shigella
D. Salmonella

56. Which of the following medication is the treatment of choice in diabetes and
was proven to decrease proteinuria?

A. Calcium channel blockers


B. ACE-Inhibitor
C. Beta-Blocker
D. Loop diuretics
E. alpha-Blocker

57. Which of the following tests can rule out pulmonary embolisim?

A. Normal chest x-ray


B. normal hemoglobin level
C. low D-Dimer level
D. Normal arterial pO2 level
E. Normal room air saturation

57
58. Which of the Following is characteristic of disease severity in a cirrhotic
patient due to hepatitis C???

A. Weight loss
B. palmar erythema
C. new onset of ascites
D. Decreased body hair, mainly in the lower extremities
E. Gynecomasty onset

59. A 54 years old man complains of right headaches for 2 months, fever up to
38 for two weeks, and some difficulty in chewing. On laboratory evaluation
he has anemia due to chronic disease and ERS of 110. What is the
appropriate Manegment?

A. Emergency CT-Scan ofthe Head


B. Lumbar puncture to rule out infection
C. Refer to Eeg to rule out Epilepsy
D. Refer to neurologist for evaluation an initiation of Anti-Parkinsn
treatment and follow up for improvement
E. initiation of high dose steroid therapy and refer the patient to temporal
artery biopsy

60. All of the following may be present in a patient with aortic stenosis except:

A. syncope
B. Dyspnea
C. supraventricular arrhythmia
D. chest pain
E. fatigue on exertion

58
61. All of the following are common etiology for acute pancreatitis except?

A. gallstone
B. acute and chronic alcoholism
C. hypertriglyceridemia
D. acute trauma or following ERCP
E. food Poisoning with fish that contain pancreatic toxin

62. A 62 old man is referred to a nephrologist for the evaluation for renal failure.
attributed to Diabetes that he has for 20 year. Laboratory evolution include
creatinin of about 4.0 and urea of about 230. The pat. Has anemia with hb
9,1 % and MCV 84, iron and transferrin are low, ferritin normal .the must
common cause for pat. Anemia ?

A. lack of folic acid that is characteristic in diabetic pat.


B. B12 deficiency due to increase loss in urin
C. iron deficiency to mal absorption in the gut
D. chronic disease anemia due to erythropoietin deficiency
E. direct bone marrow injury to diabetes

63. A 72 year old male is treated with oral Augmentin due to pneumonia. Pat.
Have fever and dyspnea improve but on treatment day 8. The pat. develop
watery diarrhea and stool culture are sent. What is the appropriate treatment?

A. switch the antibiotic treatment to cephalosporin


B. stop Augmentin treatment and begin flagyl treatment
C. stop Augmentin treatment and begin vancomycin
D. continue Augmentin with I.V fluid o avoid dehydration
E. stop Augmentin

59
64. A 62 year Adult with hypokalemia 2,9 meq/ml there is no increase in
potassium level. Follow correction attempts with i.v an oral potassium.
Which test should be ordered?

A. serum magnesium level


B. serum calcium level
C. serum phosphor level
D. serum glucose level
E. serum sodium level

65. A 52 year old admitted for rapid atrial fibrillation .laboratory demonstrated
thyreotoxicosis. TSH is undetected .what is the first diagnostic test?

A. thyroid ultrasound
B. thyroid scan
C. CT of the thyroid with the Parathyroid glands
D. thyroid biopsy
E. genetic evalution for familial hyperthyroidism

66. A 67 year male admitted to the ER complaining of dyspnea that started after
along flight. He is diagnosed Pulmonal embolism an admitted to the hospital
with initiation of clexanetreatment.o the following diagnostic the pat. is pale.
low blood pressure, melena, and coffe grand content Up on nasogastric tube
insertion. What is most appropriate management following
clexanecessation?

A. initiation heparin treatment for better Monitoringof coagulation function


B. initiation low dose Coumadin and carefullya Monitor coagulation
function
C. all Anticoagulation treatmen must be discontinued until the bleeding is
stopped
D. insert an V. cava filtter and discontinued clexane treatment.
E. new anticoagulation treatment may be initiated ( factor Xa inhibitor) as it
safe and effective

60
67. A 32 yr old alcoholic with end stage liver disease has been admited to the
hospital 3 times for bleeding esophageal varices. he has undergone banding
and sclerotherpay previously. he admits to currently drinking a six-pack of
beer per day, on his abdominal examination , he has a fluid wave. Which of
the following is the best option for long term managment of this patient's
esophageal varices?

A. orthotopic liver transplantation


B. transection and reanstomosis of the distal esophagous
C. distal splenorenal shunt
D. end -to- side portocaval shunt
E. transjugular intrahepatic portosystemic shunt(TIPS)

68. 32 years old man is admitted to the emergency room due to fever for 2 days
associated with headach, nausea,multiple vomit and photophobia .there is
clear nuchal regidity and leukocytosis of 12.000 next step in managment?

A. obstain blood and urine culture


B. obtain chest x ray for suspected atypical pneumonia
C. initiate antibiotics treatment with Rocephin, fundus examination and
lumber puncture.
D. initiate antibiotics treatment with augmentin and ENT consult to rule out
nasopharyngeal absess
E. it is most probably gastrointeritis and urgent fluid therapy should be
initiated to avoid delayed.

61
69. 72 years old man admitted to the department of internal medicine due to
palpitation with atrial fibrilation on ECG and ventricular response of 110
beats per minute . 1 hour later of his admission there is rapid deterioration
and he became tachycardic 180 irregular ,dyspnea,rales at the lung base,
desaturation and decrease blood presuure to 70/40most
apropriatemanagment?

A. decrease heart rate with calcium channel blocker infusion


B. decrease heart rate with beta blockers infusion
C. initiate digoxin loading
D. initiate treatment with adinosine in sequential dosing 6 to 12 mg
E. cardioversion

70. Which of the following is valve replacement indicator in infective


endocarditis?

A. largevagitation 10mm in size


B. involvment of 2 valves mostly aortic and mitral
C. presence of atrial fibrillation
D. abcess formation causing condution disorder
E. leukocytosis of over 25.000 that doesnt improve with antibiotics
treatment.

71. Most common tumor causing fever of unknown origin?

A. prostate cancer
B. breast cancer
C. lymphoma
D. non small cell lung cancer
E. sarcoma

62
73. which of the most common cause of hypercalcemia?

A. parathyroid adenoma
B. parathyroid carcinoma
C. bone metastasis
D. multiplemyloma
E. vitamin D intoxication

74. 50 years old man who has rheumatic arthritis and is treated with
methotrexate is admitted with macrocytic anemia ,hemoglobin level 9.1 and
MCV 102. most common cause?

A. B12 DEFICIENCY
B. hepatic disease secondary to RA
C. bone marrow supression by the medication
D. folic acid deficiency
E. iron deficiency due to duodenal mucosal injury

75. What is the most effective treatment to reduce acid in gastric peptic ulcer
disease?

A. H2 blocke
B. Proton pump inhibitor (H K ATPase)
C. Antacid
D. Bismuth containing preparation
E. Sucralfate

63
76. A 53-year-old man is admitted to ER due to compression chest pain for two
hours. Of EKG there are ST segment elevation in inferior AVF, II and III
led. Troponin is elevated 220. What is the most appropriate mangment.

A. Urgent PTCA
B. Urgent thrombolytic therapy (tPA)
C. Clexane administration at dose of 1mg/kg
D. Treatment with IV Isotek
E. Beta blocker loading by pluse monitoring with target rate of 6 beats per
minute

77. An 18 year old women was brought to the ER for dehydration. Her family
tell you she has been very thin for several years and she is a dancer so she is
very active. She love to cook for a family but not eat with them because she
is full from all the "testing" she did while cooking. Her height 1.63 meter
and her weight 40 kg. her skin is little yellow and she has enlarge masses on
both side of her lower jow. Her teeth in very poor condition Her blood test
show: metabolic alkalosis, Hypokalemia, Hypochloremia. The most
probable diagnosis is:

A. Bulimia nervosa purging type


B. Anorexia nervosa restrictive type
C. Eating disorder not otherwise specified
D. Anorexia nervosa binge eating/puring type

78. A 28 year old patient diagnosis with paranoid schizophrenia has been
receiving 4 mg/day of risperdrion for the past 6 months and is in full
remission. She has recently started to complain of reduced libido and
amenorrhea. Which hormone level is the most appropriate to assess?

A. LH and FSH
B. Estrogen and Progesterone
C. Testosterone
D. Prolactin
E. TSH

64
79. A 23 years old patient has been treated by his psychiatrist for the past 3
years, throughout this period he has remained psychotic and complains of
auditor, hallucinations, paranoid delusions, anhedonia, reduced motivation
and he is socially withdrawn. He has received several trials of treatment with
anti-Psychotic medication in adequate doses including: Olanzapine,
Perphenazine, Risperidone and Quatiepine, the most appropriate next step
would be?

A. Begin a course of ECT


B. Begin treatment with an Anti-depressant
C. Begin treatment with a mood stabilizer
D. Begin treatment with Clozapine
E. Begin treatment with Haloperidol

80. A 35 years old patient has started to come to his family doctor more
frequently over the past 2 months, "It all started 2 months ago after that car
accident when I got that whiplash injury". He complains of disturbed sleep,
disturbing dreams of the accident, pain in his neck, angry outbursts towards
his family and reduced concentration, he has not driven his car since the
accident for fear of another accident and when sitting in a car as a passenger
he is very nervous. He feels his life has totally changes since the accident
and does not believe he will ever 'be himself again". He is considering suing
the insurance company for compensation. The most appropriate diagnosis
and intervention would be?

A. Driving phobia- CBT with gradual exposure to driving


B. Major depressive disorder- begin treatment with an SSRI
C. Normal response to an accident- begin physiotherapy for neck pain
D. PTSD- CBT with prolonged exposure
E. Malingering- no treatment needed

65
81. A patient is complaining of difficulties falling asleep, awaken several times
during the night and finally waking up at 4AM after only 5 hours of sleep.
He was referred to a sleep laboratory and the pattern was: "reduced REM
latency, increased REM sleep, increased awakening and deficit in slow wave
sleep". He is most probably suffering from

A. manic episode
B. major depressive episode
C. generalised anxiety disorder
D. withdrawal from alcohol
E. PTSD

82. Which of the following statements regarding second generation


antipsychotics (SGA) versus first generation antipsychotics (FGA) is untrue

A. SGA are more likely to cause metabolic syndrome compared to FGA


B. FGA are associated with more EPS than SGA
C. The efficacy of SGA is greater than FGA
D. Since the development of SGA, the use of FGA has declined
E. Several SGA are approved for the treatment of bipolar disorder

83. What is the most appropriate treatment for 21-years old who performs 4
hours of daily compulsive hand-washing for fear his hands are contaminated

A. Exposure and response prevention together with an SSRI


B. Treatment with Risperidon
C. Weekly sessions of psychodynamic psychotherapy
D. Referral to dermatologist for an examination
E. Referral to a daycare center for multi-disciplinary intervention

66
84. Which of the statements regarding factitious disorder is wrong?

A. Strong counter-transference is common in treating physicians


B. Patients unconsciously create signs and symptoms
C. Patient create physical or psychological signs and symptoms
D. Factitious disorder may occur by proxy
E. Malingering is a differential diagnosis

85. Unexpected panic attacks are required for the diagnosis of

A. Specific phobia
B. PTSD
C. Social phobia
D. Panic disorder
E. All of the above

86. Which statement regarding generalised anxiety disorder is true

A. It is a mild condition
B. The female to male ratio is 1:2
C. Has a low prevalence in primary care settings
D. Is not likely to coexist with another psychiatric disorder
E. Has about 50% chance of reccurence after recovery

87. A 42 year old male was prescribed a medication by his psychiatrist. He


subsequently returned to her, complaining of reduced sexual desire and a
delayed orgasm. Which medication was he most likely prescribed?

A. a benzodiazepine
B. an anti-hypertensive
C. an anti-psychotic
D. a SSRI
E. an antihistamine

67
88. What would be the most useful sign to differentiate between schizophrenia
and amphetamine induced psychosis?

A. tactile or visual hallucinations


B. intact orientation
C. auditory hallucinations
D. clear consciousness
E. paranoid delusions

89. A 77 year old teacher who was in good health was admitted to the hospital
for having elective orthopedic surgery. He made no mention of drinking
alcohol. Ten hours after surgery the patient has a heart rate of 115 beats/min
and his blood pressure is 115/100 mmHg. He is sweating and anxious. When
you check his chart you see an MCV of 97.5 and gamma glutamyltransferase
value of 57 U/L. he has clear tremor in his hands. Upon examination the
patient was disoriented to time and place and very agitated and he spoke
incoherently. On a more thorough questioning his wife disclosed a history of
alcohol consumption: 5 beers at night. The patient was given diazepam on a
daily basis along with B1 (thiamine). His clinical state improved over the
next four days and he was discharged and sent to AA for further treatment.
Which condition is the best classification for this patient?

A. Korsakoff’s syndrome
B. Wernicke’s encephalopathy
C. alcohol induced anxiety
D. alcohol induced persisting dementia
E. alcohol withdrawal delirium

68
90. For the diagnosis of autistic disorder the diagnostic criteria include:

A. impairment in social interaction


B. impairment in communication
C. restricted, repetitive and stereotyped pattern of behavior, interests and
activities
D. persistent eating of nonnutritive substances such as hair, paper, clay etc.
E. 1, 2 and 3

91. Which statement regarding a child with ADHD is NOT true?

A. the risk of ADHD in this child’s siblings is increased


B. he is at lower risk of developing conduct disorder compared to the
general population
C. the symptom of ADHD most likely to remit is hyperactivity
D. he will probably respond to methylphenidate
E. all is true

92. Which statement regarding schizophrenia is true?

A. on MRI studies patients with schizophrenia demonstrate enlarged


ventricles (lateral & third)
B. the ventricular enlargement found is a pathognomonic finding in
schizophrenia
C. ventricular changes in schizophrenia are likely to be specific for the
pathophysiology underlying this disorder
D. all is true
E. non is true

69
93. What is true regarding the medication Clozapin?

A. It is used as a first line antipsychotic medication.


B. It is associated with a 10-20% incidence of agranulocytosis.
C. It is associated with many extrapyramidal side effects.
D. It can cause a significant increase in prloactin
E. none are true

94. Which statement regarding what appears to be negative symptoms in


Schizophrenia is NOT true:

A. may in fact be due to overmedication with antipsychotic medication.


B. may in fact be due to the presence of persecutory delusions.
C. may be due to circumstances that reduce stimulation and drive.
D. are more responsive to medication than positive symptoms.
E. all are true.

95. Which of the following are negative symptoms of Schizophrenia?

A. Anosmia
B. Anhedonia
C. Anosognosia
D. Alogia
E. 2 and 4

70
96. A 34 year old man was brought to the psychiatric center by his sister. He
was recently fired from his work because he harassed his co-worker
claiming he was stlaking him and trying to kill him in various ways. He does
complain of depressed mood and reduced interest of his surrounding.
Though he attributes these to: ''trying to deal with the knowledge that may
college has been trying to kill me for the past year and no one believes me. I
saw him out of my window sitting in his car, watching my home and waiting
for the right opportunity to come and get me''.. The sister adds that is
situation has been present for the past year and previously to

A. Schizophrenia. Paranoid type


B. Delusional disorder
C. Schizoaffective disorder
D. Due to the late onset of symptoms, he is probably suffering from
amrdical condition causing psychotic symptoms
E. Malingering

97. For the above pt, Psychiatrist decides to start treatment with first generation
antipsychotic medication. He appeared in the ER after 3 days of treatment ,
complaining of difficulty speaking, eating and drinking , very agitated and
anxious. He is drooling profusely, his tongue is twisted in his mouth and he
can’t move it voluntarily , the most likely diagnosis is :

A. Akathisia
B. Tardive dyskinisia
C. Parkinsonism
D. acute dystonia
E. malingering

71
98. Double depression occurred when:

A. a female pt with preexisting PMDD, develops recurrent depressive


disorder.
B. pt with recurrent depressive disorder responds to treatment but remains
with prominent symptoms.
C. pt with dysthymia develops superimposed recurrent depressive disorder
D. pt with general medical conditions developes superimposed reccurrent
depressive disorder.
E. there is no such psychiatric term.

99. Which of the following are not associated with atypical depression:

A. increase appetite with unintentional weight gain


B. increase desire to sleep, usually more than 10 hours a day
C. heavy, leaden feeling in your arms and legs that lasts an hour or more in a
day
D. sensitivity to rejection or criticism, with affects your relationships
E. depression that does not react or lifts when something good occurs

100. Which statement is false?

A. cyclothymic disorder is characterized by subsyndromal depression and


hypomania
B. dysthymic disorder is characterized by insidious onset of depression
C. bipolar 1 is characterized by full blown manic symptoms. But never
hypomanic symptoms.
D. bipolar 2 is characterized by manic like symptoms but never full blown
manic symptoms.
E. schizoaffective disorder is characterized by symptoms of both
schizophrenia and an affective disorder (Bipolar type or depressive type).

72
101. You saw a 38 yrs old depressed male and your diagnosis was a
moderate, major depressive episode of which you prescribed an SSRI .he
appeared in your office two weeks later, complaining of persistent suicidal
ideation, when you ask him if he has a plan he burst out crying and shows
you a gun he has been carrying around with him. The only thing stopping me
is the face that my wife is pregnant and she is due next week. The most
appropriate thing to do is:

A. call the wife and ask her to watch over her husband for the next week or
2 until the SSRI starts to work.
B. declare the antidepressant trial a failure and switch to another class of
antidepressant.
C. admit the pt to hospital immediately.
D. reassure the pt that suicidal thoughts are common at the beginning of
treatment with an antidepressant and that these thoughts should subside
withen a short time.
E. initiate psychotherapy to discuss the reasons behind his suicidal thoughts.

102. What is not true about anxiety disorder:

A. women have greater rates of almost all anxiety disorders.


B. gender ratios are nearly equal in OCD.
C. rates are lower among the ppl at lower socioeconomic levels.
D. increase rates of comorbidity with other disorders.
E. All are true.

103. Which of the following drugs is not considered a dual reuptake


inhibitors:

A. Venlafaxin
B. Duloxetine
C. Milnacipran
D. Escitalopram
E. Amitryptilin

73
104. Which of the statements about paraphilia’s is true

A. paraphilia’s with an early onset usually have a good prognosis


B. paraphilia’s are usually not distressing to the person with the disorder
C. paraphilia’s are found equal among female and male
D. paraphilia’s can not be treated with cbt
E. non are true

105. False statement:

A. social phobia is associated with blushing and palpitations


B. in panic and agoraphobia patients comforted by presence of others
C. high panic relapse rate if no therapy
D. CBT and Rx combination is superior to therapy of anxiety

106. Gender identity disorder treatment includes:

A. sex surgery
B. daily estrogen in women
C. Social transition before hormones
D. best in treatment gender dysphoria
E. All are correct

107. Borderline personality disorder

A. Increased risk of psychotic symptoms


B. easy to treat
C. Decreased risk of premature death
D. Decreased risk of PTSD
E. All are correct

74
108. ECT indication:

A. Bipolar
B. Major depression
C. Panic
D. PTSD
E. Borderline personality disorder

109. A family doctor wants to prescribe a selective serotonin reuptake


inhibitor (SSRI) to a mildly depressed patient with no significant medical
conditions, what is not true?

A. SSRIS Have a wide therapeutic index


B. SSRIS have different half lives.
C. SSRIS should not be used conjunction with benzodiazepines
D. SSRIS may cause sexual dysfunction.
E. SSRIS should not be used in conjunction oxidose inhibitors (MAOIS)

110. What is not true regarding lithium?

A. It is may cause thyroid dysfunction


B. It is may cause parathyroid dysfunction
C. It is may cause kidney dysfunction
D. It is may cause tremor
E. It is may cause Liver dysfunction.

75
State Exam 20.02.2014
By: Alhag Abu Anzeh Muhammad

PART A:

1.You have just prescribed phenytoin for 12 years old boy with new onset of epilepsy , which of the following side
effects is most common with the use this drug ?

A- lymphoma syndrome
B-Raynoudphenomen
C-Acute hepatic failure
D-Gingival hyperplasia
E-Optic atrophy

2.You examined a 17 years old boy student with history of fever , sore throat and fatigue , physical examination
reveals an exudative tonsillitis , and bilateral enlarged post cervical lymph nodes . The spleen is palpable in 3cm below
the rip case. Which agent is most likely responsible for patient's illness ?

A- Group A β-hemolytic streptococcus


B-Adenovirus
C- C.M.V
D- Epstein-Bar virus
E- Corynebacterdiphterea

3.A newborn infant with stigmata of Down syndrome has a heart murmur .Which of the following is most likely in
baby ?

A-Hypoplastic left Heart syndrome


B- Total anomalous pulmonary venous return
C- Coarctation of aorta
D- Anomalous coronary arteria
E-Atrioventricularseptal defect

4.It is recommended that young infant should sleep in the supine than in the prone position.This is based on data
suggesting that the prone position is associated with an increased incidence of which of the following ?

A- Delayed eruption of the first deciduous teeth


B- Gastroesophageal reflux and aspiration
C- Macrognathia
D-Strabismus
E- Sudden infant death

5. The feeding of honey in infant less than 6 months of age has been associated with which of the following ?

A-Infantile diabetes mellitus


B-Severe allergic reaction
C- Botulism
D- Jaundice
E-Hypernatremia
6. A 6 month old infant presents with poor growth and inadequate weight gain .There is no history of vomiting or
diarrhea , except for the appearance of malnutrition .Physical examination is normal , most likely cause of this child
failure to thrive is :

A - Celiac disease
B- Metabolic disorder
C -Tuberculosis
D- Endocrine disorder
E- Non-organic cause

7. A 2 years old is checked routinely by you .On physical examination you notice that the red reflex in right eye is
replaced by a yellow-white reflex. The child most likely has which of the following :

A- Retinitis pigmentosa
B- Retinoblastoma
C- Rhabdomyosarcoma
D- Severe myopia
E- Congenital glaucoma

8. A 2 years old child is admitted because of weakness proceeding to coma .According to the parents to the child has
been well until several hours prior to admission , when they noted diarrhea cough , wheezing and profuse sweating
.On examination you notice a comatose child with diffuse weakness and areflexia . Pupils are pinpoint and
unresponsive . Which of the following should administer at this time :

A- Adrenaline
B- Atropine
C- Cefotaxime
D- Methyl prednisolone
E- Adrophonium

9. A full term newborn infant is having episodes of cyanosis and apnea , which are worse when attempting to feed ,
but seems better when he is crying .Which of the following most important next step to quickly establish the
diagnosis?

A- Electrocardiogram
B- Ventilation perfusion scan
C- Passage of catheter into nose
D- Hemoglobin electropharysis
E- Bronchoscopic evaluation of palate and larynx

10. An 8 month old boy has vomiting and screaming episodes for 12 hours. Physical examination of the abdomen
sausage-shaped mass in the right upper quadrant which of the following most useful ?

A- Passage of nasogastric tube


B- extension of a stool specimen for ova and parasites
C- Blood culture
D-Abdominal CT
E-Contrast enema study
11. 4 years old boy has failed to grow and has evident of exocrine pancreatic deficiency most likely cause of this :

A) acute pancreatitis
B) biliary atresia
C) SCHWACHMAN diamond syndrome
D) congenital absent of pancreas
E) cystic fibrosis

12. a 5 years old diagnosed with PERIARTICULAR juvenile idiopathic arthritis has positive antinuclear anti body test ,
which of the following would most likely be found in the patient :

A) pericarditis
B) nephritis
C) uveitis
D) splenomegaly
E) lymphadenopathy

13. an18 month old toddler has MYCROCYTIC anemia which dietary history finding best explains this :

A) intake excessive amount of fat


B) lack of fresh vegetables in the diet
C) intake of inadequate amount of fruit juice
D) intake of an adequate amount of vitamin c
E) intake a large amount of unmodified cow's milk

14. 12 year old girl has had progressive proximal muscle weakness over the past week , she has also developed also
an erythematous scaly rash on the face arms , and thigh , and a lacy rash on her upper eyelids , what is the next best
laboratory study for diagnosis:

A) Rheumatoid factor
B) erythrocyte sediment rate
C) urine analysis
D) Creatine kinase
E) antinuclear antibody

15. the most important measure that can be taken to prevent the spread of RSV ( respiratory syncytial virus) disease
in hospitalized patient is :

A) wearing a mask in the room of an infected patient


B) wearing a paper gown when entering the room of an infected patient
C) keeping the patient in respiratory Isolation for 24 hours on appropriate antibiotics
D) testing children with respiratory illnesses for RSV during the winter
E) hand washing

16. A 5 YEAR BOY IS BROUGHT TO YOUR OFFICE WITH THE C/O OF PERSISTENT RHINORRHEA FOR THE PAST 8-
MONTHES OTHERWISE THE PATIET HAS BEEN ASYMPTOMATIC. ON PHYSICAL EXAM. YOU NOTE THAT THE PATIENT IS
MOUTH BREATHING AND HAS DARK CIRCLES UNDER THE EYES. EXAM. THE NOSE REVEALS A WATERY DISCHARGE
AND EDEMATOUS. BOGGY, BLUISH MUCOUS MEMBRANE WITH NO ERYTHEMA. DIAGNOSIS IS?

A- CHRONIC UPPER RESP. TRACT INFECTION


B- SINUSITIS
C- NASAL FB
D- ALLERGIC RHINITIS WITH CEREBRAL SPINAL FLUIDS LEAK
17. A 7 YEAR CAUCASIAN FEMALE IS SEEN IN YOUR OFFICE FOR HER ANNUAL PHYSICAL VISIT. HER PHYSICAL EXAM IS
REMARKABLE FOR PALPABLE GOITER. THE MOTHER HAS POSITIVE FAMILY HISTORY OF HYPOTHYROIDISM REQUIRING
THYROID HORMONE REPLACEMENT. THE PATIENT IS ON THE 90TH PERCENTILE FOR WT. AND ON THE 10TH
PERCENTILE FORE HIGHT. WHATE IS THE MOST LIKELY DIAGNOSIS?

A- HASHIMOTO THYROIDITIS
B- IODINE DEFICENCY
C- THYROID ADENOMA
D- BRAIN TUMOR CAUSING TSH DEFICENCY
E- THYROTROPIN RELEASING HORMONE DEFICENCY

18. 28- A woman without prenatal care is admitted at 38 weeks by dates for the delivery of her third child. The 8 lb 13
oz infant is born vaginally and APGAR scores are 6 and 9. The pediatrician is called several hours later by the nursery
staff to evaluate the left arm. The initial examination reveals an R-sided cephalohematoma. There is asymmetry of the
upper limb movements with minimal movement of the left arm. The grasp is preserved. The most likely cause for this
paralysis is:

A.Grade II intraventricular hemorrhage [IVH]


B.Hypoxic event during pregnancy secondary to drug abuse
C.Peripheral neuropathy associated with undiagnosed gestational diabetes
D.Shoulder dystocia
E.CNS manifestations of syphilis

19. 12 months old boy is brought to your clinic because he is refusing to walk , his mother states that he began
walking at 10 month of age and has doing well until this point , his temperature 39.2 c and
physical examination reveals a moderately ill infant with an extremely rotated left hip with limitation to passive and
active motion , the most likely causative organism is :

A) neisseriagonorhoea
B) adenovirus
C) staphylococcusaureus
D) group a streptococcus
E) group b streptococcus

20. 17 years old female present to your office with complaint of a missed period , menarche occurred of age 13 , and
her periods have been regular for the last 18 months she normally bleeds for 4 to 5 days with moderate flow , her last
menses was 2 month ago after
performing though history and physical examination what would be her next step in her evaluation :

A) complete blood count


B) urine beta human chorionic gonadotropin
C) serum quantitative human chorionic gonadotropin
D) luteinizing hormone level
E) serum testosterone level
21. 2 year old boy is seen in your office because of fever, ear pain and postauricular swelling, erythemaand
tenderness. The pinna protrudes out on the involved side. The tympanic membrane is red andbulging, with decreased
mobility seen on pneumatic otoscopy. the angle of the jaw is easily palpated andthe opening to stensen's duct appear
normal. Most likely Diagnosis :

A. Bacterial parotitis
B. Mumps
C .External otitis
D. Acute mastoiditis
E. Chronic mastoiditis

22. A 10yearold boy is examined because of recurrent headaches. The headaches started 6 months ago and occur
about once a month. He is asymptomatic between episodes. Each headache begins withblurry vision and abdominal
pain, followed by rightsided, throbbing pain. It lasts about 60 minutes,during which he feels better if he takes some
ibuprofen and rests in a darkened room. The most likely diagnosis is:

A. Brain abscess
B. Seizure disorder
C. Migraine
D. Todd's paralysis
E. heteropheria

23. becaus of splenic dysfunction children with sickle cell anemia should be placed on prophylactic penicillin vk by 4
weeks of age . children with sickle cell anemia are particularly succeptible to:

a. gram negative rods such as E.coli


b. encapsulated organism such as streptococcus pneumonia
c. fungal infections
d. viral infections
e. staphylococcus aureus

24.A newborn in the nursery is noted to have frothy secretions from mouth and nose and has coughing and cyanosis
when given a bottle .the nurse reports that she was able to insert a tube into the nose but was unable to pass a
nasogastric tube into stomach .a flat - plate x-ray of abdomen reveals gaseous dilatation of stomach .which most
likely diagnosis?

a - tracheoesophageal fistula
b - isolated esophageal atresia
c - choanal atresia
d -gastroesophegeal reflux disease
e - dudenal atresia

25. The family of a 2yearold child with TaySachs disease is interested in having another child. Theywould like to know
the risk of having a second child with TaySachs. What advice would you give?

A. No risk, since TaySachs is caused by a sporadic mutation


B. 100%, since TaySachs is an autosomaldominant disease
C. 25%, since TaySachs is an autosomal recessive disease
D. It depends on the gender of the fetus, since TaySachs is Xlinked
E. One can determine the risk only by genetic testing after conception
26. a 16 year old male who consumes a large quantity of alcohol on a daily basis presents with hemoglobin of 8.5 g/dl
a mean corpuscular volume of 110 fl and reticulocyte count of a 0.5% most likely diagnosis ?

A) Iron deficiency anemia


B) Thalassemia minor
C) Lead intoxication
D) vitamin FOLATE DEFICEINCY
E) anemia of chronic disease

27. you are asked to counsel a pregnant mother at 32 week gestations pregnancy . has been characterized by poor
fetal growth and oligohydramnios you are most concerned about ?

a. gastroschisis
b. neural tube defects
c. gestational diabetes
d. renal angesis and pulmonary hypoplasia
e. diaphragmatic hernia

28. A 5yearold boy presents with a history of grossly bloody urine, puffy eyes, and headache for oneday. He has been
a well developed child, but he did have a fever and sore throat about 10 days ago which resolved without treatment.
The most likely diagnosis is:

A. Acute cystitis
B. IgA nephropathy
C. Acute pyelonephritis
D. Post infectious glomerulonephritis
E. Benign hematuria

29. A mother presents for prenatal care with a complicated medical history as listed in the followingchoices. Which of
the conditions most predisposes her fetus to congenital heart disease?

(A) hypertension
(B) diabetes mellitus
(C) atherosclerotic coronary vascular disease
(D) anemia
(E) rheumatoid arthritis

30. a 3 days old infant is requiring phototherapy for hyper bilirubinemia . which of the following risk factor best
predicts the occurrence of ABO isoimune hemolytic disease in newborn ?

a - first pregnancy
b - more than four pregnancies
c - prior Rh disease
d - maternal blood type is O and infant is type A
e - preexisting maternal anemia .

31. Which of the following is most likely to be associated with a cataract in the newborn?

(A) maple syrup urine disease


(B) G6PD
(C) phenylketonuria
(D) galactosemia
(E) propionic academia
32. A newborn infant presents with cyanosis and mild tachypnea at about 6 hours of life. The infant is placed in 95%
oxyhood and saturations normalize. Which of the following is the most likely diagnosis in this infant?

(A) cyanotic congenital heart disease


(B) lung disease
(C) central nervous system disease
(D) liver disease
(E) methemoglobinemia

33. Hydrocephalus, chorioretinitis, and diffuse cerebral calcifications are present in a newborn male. Of The following
pets, which is most likely to be the source of this zoonotic congenital infection?

(A) dog
(B) cat
(C) horse
(D) rabbit
(E) sheep

34. Vision milestones are being discussed with a group of student nurses. One student asks when babies can begin to
visually follow people as they walk across the room. Fixation and tracking through the visual field are well developed
by which of the following ages?

(A) at 7 months gestation


(B) at birth
(C) at 2 months
(D) at 6 months
(E) at 1 year

35 . The pubertal growth spurt in males occurs at which of the following ages?

(A) precedes the growth spurt in females by 2 years


(B) has its onset at the same age as in females but lasts longer
(C) follows the growth spurt in females by 2 years
(D) coincides with attainment of the ability to ejaculate
(E) begins at Tanner IV stage of sexual maturity

36. An infant with failure to thrive has rectal prolapse. What test will most likely provide the diagnosis?

(A) abdominal CT scan


(B) rectal biopsies
(C) liver function testing
(D) barium enema study
(E) sweat chloride test

37. A 6 month old boy is found to have very low levels of IgG, IgM, and IgA. Which of the following organisms is most
likely to cause problems in this patient?

(A) enterovirus
(B) herpes virus
(C) Shigella
(D) Escherichia coli
(E) Mycobacterium tuberculosis
38. An infant is born to a mother who acquired Primary CMV infection during her third pregnancy trimester .what will
be the most likely finding in this infant at birth ?

(a) Hepatosplenomegaly and jaundice .


(b) subclinical infection
(c) microcephaly and intrauterine growth retardation.
(d) Sensory neural hearing loss
( e) Thrombocytopenia and purpura .

39. A 9 month old infant accidentally ingests unknown quantity of digitalis. Which is the most significant noncardiac
manifestation of toxicity in this child?

(A ) fever
( B) dizziness
(C ) vomiting
( D) visual disturbances
( E) urticarial

40. After being lifted up by one hand, a young toddler refuses to use that arm and holds it against her trunk flexed at
the elbow with the forearm midway between pronation and supination. The child most likely has which of the
following?

(A) a shoulder dislocation


(B) a radial head subluxation
(C) a fracture of a carpal bone
(D) avulsion of the ulnar nerve
(E) a fracture of the radius

41. what is the most likely clinical finding in an 8 year old with Wilson disease.

a. hepatitis like illness


b. acral cyanosis
c. polyuria
d. facial nerve palsy
e. poor visual acuity

42. among children short stature and infertitity are most commonly associated with which of the following:

a. klinefelter syndrome
b. beckwithwiedemann syndrome
c. turner syndrome
d. marfan syndrome
e. pierre Robin sequence

43. the presence of bilateral renal masses and a midline suprapubic mass in a newborn male infant is most suggestive
of :

a. bilateral wilms tumor


b. congenital neuroblastoma
c. congenital rubella infection
d. rhabdomyosarcoma of the bladder
e. congenital urethral and bladder neck obstruction
44. the fending of low complement C3 in serum of child with nephritic syndrome indicates which of :

a. a high likelihood of spontaneous remission


b. a high likelihood of good response to steroid therapy
c. the presence of focal segmental sclerosis
d. the presence of membranous glomerulonephritis
e. the presence of membranoproliferative glomerulonephritis

45. the most common roentgenographic abnormality in a child with asthma:

a. bronchiectasis
b. generalized hyperinflation
c. lower lobe infiltrates
d. pneumomediastinum
e. right middle lobe atelectasis

46. the most common neurology sequel associated with bacterial meningitis in children and usually present at the
initial presentation of infection :

a. mental retardation
b. chronic seizure disorder
c. impaired vision
d. impaired hearing
e. behavioral disturbance

47. a 2 month old boy is seen for nonbilious vomiting aftar feeding he had been breast feeding well until 2 weeks ago
when vomiting
worsened. On physical examention he has iaundice and gastric peristaltic wares are seen on the abdomen . Blood
tests reveal hypokalemic,hypochloremic alkalosis. most likely:

a. acute hepatitis
b. duodenal atresia
c. urinary tract infection
d. pyloric stenosis
e. cow’s milk protein allergy

48. a 4 month old child with vitamin D deficiency rickets would be expected to show all except :

a. carniotabes
b. bowleg
c. rosary
d. low serum phosphate levels
e. high alkaline phosphatase levels

49. a 10 yar old girl has a cold for 14 days in the last 2 days she developed a fever 39c,purulent nasal discharge , facial
paine and daytime cough . Examination of the nose shows puss in the middle meatuse , the most likely diagnosis:

a. brain abscess
b. maxillary sinusitis
c. streptococcal throat infction
d. adenoidal hypertrophy
e. middle ear infection
50. anobes 9 year old has a week long history of persistent headache and blurred vision. Past history is unremarkable
she has had some mild vomiting without fever on physical examination you find papilledema .visual field testing
reveals an inferior nasal blind spot. An MRI scan of the head is normal. On lumbar puncture , the opening pressure is
elevated but the CSF composition is normal the most likely giagnosis:

a. meningitis
b. migraine
c.pseudotumorcerebri
d. brain tumor
e. subdural hematoma

51. A 65-year-old man undergoes a technically difficult abdominoperineal resection for a rectal cancer during which
he receives three units of packed red blood cells. Four hours later in the intensive care unit he is bleeding heavily
from his perineal wound. Emergency coagulation studies reveal normal prothrombin, partial
thromboplastic, and bleeding times. The fibrin degradation products are not elevated but the serum fibrinogen
content is depressed and the platelet count is 70,000/Ul. Which of the following is the most likely cause of the
bleeding is :

a. Delayed blood transfusion reaction


b. Autoimmune fibrinolysis
c. A bleeding blood vessel in the surgical field
d. Factor VIII deficiency
e. Hypothermic coagulopathy

52. A 71 year old man develops dysphagia for both solids and liquids and weight loss of 60 lb over the past 6 months.
He undergoes endoscopy demonstrating a distal esophageal lesion, and biopsies are consistent with squamous cell
carcinoma. He is scheduled for neoadjuvantchemoradiation followed by an esophagectomy and preoperatively is
started on total parenteral nutrition, given his severe malnutrition reflected by an albumin of <1. Which of the
following is most likely to be a concern initially in starting total parenteral nutrition in this patient?

a. Hyperkalemia
b. Hypermagnesemia
c. Hypoglycemia
d. Hypophosphatemia
e. Hypochloremia

53. A cirrhotic patient with abnormal coagulation studies due to hepatic synthetic dysfunction requires an urgent
cholecystectomy. A transfusion of FFP is planned to minimize the risk of bleeding due to surgery. What is the optimal
timing of this transfusion?

a. The day before surgery


b. The night before surgery
c. On call to surgery
d. Intraoperatively
e. In the recovery room
54. A 19 year old man sustains severe lower extremity trauma, including a femur fracture and crush injury to his foot.
He is requires vascular reconstruction of the popliteal artery. On the day after surgery, he becomes dyspanic and
hypoxemia and requires intubation and mechanical ventilation. Which of the following is the most likely etiology of
his decompensation:

a. Aspiration
b. atelectasis
c. fat embolism
d. fluid overload
e. pneumonia

55. A 72 year old woman who is planning to undergo ventral hernia repair is on warfarin atrial fibrillation. she is
advised to cease her warfarin several day before her surgery and hospitalized preoperatively for heparization. During
her hospital stay she complains of severe abdominal and flank pain. Her PT is normal but her PTT is elevated. An
abdominal CT scan demonstrates a large retroperitoneal hematoma. Which of the following should be administered
to reverse the effect of heparin:

a. thrombin
b. Vit K
c. protamine sulfate
d. deferoxamine
e. platelet transfusion

56. A 42 year old man who was in house fire is transferred to your burn unit. he has singed nose nasal hairs and facial
burns. direct laryngoscopy in the emergency room demonstrates pharyngeal
edema and mucosal sloughing. He has 60 % total body surface are burns . Which of the following is the next step in
the management :

a. hyperbaric oxygen
b. intravenous steroids
c. inhaled steroids
d. bronchoscopy
e. intubation

57. The most common complication of chronic pancreatitis is:

a. hemorrhage
b. necrotizing pancreatitis
c. pseudocyst
d. duodenal
e. obstruction

58. A 30 year old man with history of crohn disease develops an enterocutaneous fistula and placed on TPN through a
right subclavian central venous catheter. After 5 days the patient develop fever and leukocytosis. CT scan of the
abdomen reveals no intra- abdominal abscess. The subclavian catheter insertion site is inspected and noted to be
erythematous and painful. Blood culture are positive . Which of the following organisms is the most likely fever:

a. coagulase- positive staph.


b. coagulase- negative staph .
c. gr. A streptococcus
d. enterococcus
e. Escherichia coli
59. A 27 year old man sustains a single gunshot wound to the left thigh. In the ER, he is noted to have a large
hematoma of his medial thigh. He complains of paresthesia in his foot on examination, there are weak pulses
palpable distal to the injury and the patient is unable to move his foot. The appropriate initial management of this
patient is :

a. Angiography
b. immediate exploration and repair in the operating room
c. fasciotomy of anterior compartment of the calf
d. observation of resolution of spasm
e. local wound exploration

60. A 31-year-old man is brought to the emergency room following an automobile accident in which his chest struck
the steering wheel . Examination reveals stable vital signs, but the patient exhibits multiple palpable rib fractures and
paradoxical movement of the right side of the chest. Chest x-ray shoes no evidence of pneumothorax or hemothorax.
Which of the following is the most management of this patient?

a. intubation, mechanical ventilation, and positive end-expiratory pressure


b. Stabilization of the chest wall with sandbags
c. Stabilization with towel clips
d. Immediate operative stabilization

61.A patient is brought to the ER after a motor vehicle accident. He is unconscious and has a deep scalp laceration
and one dilated pupil. His heart rate is 120 beats per minute blood pressure is 80/40 mm/Hg, and respiratory rate 35
per min. Despite rapid administration of 2 L normal saline, and patients vital signs do not change significantly. Which
of the following is the most appropriate next step in the
workup of his hypotension?

a. Neurosurgical consultation for emergent ventriculostomy to manage his intracranial pressure


b. Neurosurgical consulation for emergent craniotoymy for suspected subdural hematoma
c. Emergent burr hole draining at the bedside for suspected epidural hematoma
d. Administration of mannitol and hyperventilation to treat his elevated intracranial pressure
e. Abdominal ultrasound (focused assessment with sonography in trauma, FAST)

62.A 53yearold man presents with constipation and a 10kg weight loss over the course of 6 months. Colonoscopy
reveals a fungating mass in the sigmoid colon; biopsy is consistent with adenocarcinoma. His metastatic workup is
negative. A CEA level is obtained and is 4fold greater than normal. Which of the following is the appropriate use of
this test?

a. As an indication for neoadjuvant chemotherapy


b. As an indication for postoperative radiation therapy
c. As an indication for preoperative PET scanning
d. As an indication for a more aggressive sigmoid resection
e. As a baseline measurement prior to monitoring postoperatively for recurrence

63.A 49 year old obese man has become irritable, his face has changed to a round configuration, he has developed
purplish lines on his flanks, and he is hypertensive. A 24hour urine collection demonstrates elevated cortisol levels.
This is confirmed with bedtime cortisol measurements of 700 mg/mL. Which of the following findings is most
consistent with the diagnosis of Cushing disease?

a. Decreased ACTH levels


b. Glucocorticoid use for the treatment of inflammatory disorders
c. A 3cm adrenal mass on computed tomography (CT) scan
d. Suppression with high dose dexamethasone suppression testing
E- A 1CM bronchogenic mass on magnetic resonance imaging (MRI)
64.A 53 year old woman presents with complaints of weakness, anorexia, malaise, constipation, and back pain. While
being evaluated, she becomes somewhat lethargic. Laboratory studies include a normal chest x -ray, serum albumin
3.2 mg/dL, serum calcium 14 mg/dL, serum phosphorus 2.6 mg/dL, serum chloride 108 mg/dL, blood urea nitrogen
(BUN) 32 mg/dL, and creatinine 2.0 mg/dL. Which of the following is the most appropriate initial management?

a. Intravenous normal saline infusion


b. Administration of thiazide diuretics
c. Administration of intravenous phosphorus
d. Use of mithramycin
e. Neck exploration and parathyroidectomy

65.A 40yearold woman presents with a rash involving the nipple areolacomplex for the last month with associated
itching. On physical examination there is crusting and ulceration of the nipple with surrounding erythema involving
the areola and surrounding skin, no
palpable breast masses, and nocervical or axillary lymphadenopathy. Which of the following is the most appropriate
next step in the
management of this patient?

a. Reexamine the patient in 1 month


b. Corticosteroid cream to the affected area
c. Administration of oral antibiotics
d. Mammogram and biopsy of the affected area
e. Modified radical mastectomy

66.A 42yearoldman has bouts of intermittent crampy abdominal pain and rectal bleeding Colonoscopy is performed
and demonstrates multiple hamartomatous polyps. The patient is successfully treated by removing as many polyps as
possible with the aid of intraoperative endoscopy and polypectomy. Which of the following is the most likely
diagnosis?

a. Ulcerative colitis
b. Villous adenomas
c. Familial polyposis
d. PeutzJeghers syndrome
e. Crohn colitis

67.A 70yearold woman has nausea, vomiting, abdominal distention, and episodic crampymidabdominal pain. She has
no history of previous surgery but has a long history of cholelithiasis for which she has refused surgery. Her abdominal
radiograph reveals a spherical density in the right lower quadrant. Which of the following is the definitive treatment
for this patient’s bowel obstruction?

a. Ileocolectomy
b. Cholecystectomy
c. Ileotomy and extraction
d. Nasogastric (NG) tube decompression
e. Intravenous antibiotics
68.A 53yearold man presents to the emergency room with left lower quadrant pain, fever, and vomiting. CT scan of
the abdomen and pelvis reveals a thickened sigmoid colon with inflamed diverticula and a 7cm by 8cm rimenhancing
fluid collection in the pelvis. After percutaneous drainage and treatment with antibiotics, the pain and fluid collection
resolve. He returns as an outpatient to clinic 1month later. He undergoes a colonoscopy, which demonstrates only
diverticula in the sigmoid colon.
Which of the following is the most appropriate next step in this patient’s management?

a. Expectant management with sigmoid resection if symptoms return .


b. Cystoscopy to evaluate for a fistula
c. Sigmoid resection with end colostomy and rectal pouch (Hartmann procedure)
d. Sigmoid resection with primary anastomosis
e. Long term Antibiotic Therapy

69.A 47 year old asymptomatic woman is incidentally found to have a 5mm polyp and no stones in her gallbladder on
ultrasound examination. Which of the following is the best management option?

a. Aspiration of the gallbladder with cytologic examination of the bile


b. Observation with repeat ultrasound examinations to evaluate for increase in polyp size
c. Laparoscopic cholecystectomy
d. Open cholecystectomy with frozen section
e. En bloc resection of the gallbladder, wedge resection of the liver, and portal lymphadenectomy

70.A 55 year old man who is extremely obese reports weakness, sweating, tachycardia, confusion, and headache
whenever he fasts for more than a few hours. He has prompt relief of symptoms when he
eats. Laboratory examination reveals an inappropriately high level of serum insulin during the episodes of fasting.
Which of the following is the most appropriate treatment for this condition?

a. Diet modification to include frequent meals


b. Long acting somatostatin analogue octreotide
c. Simple excision of the tumor
d. Total pancreatectomy
e. Chemotherapy and radiation

71. During an appendectomy for acute appendicitis, a 4cm mass is found in the midportion of the appendix. Frozen
section reveals this lesion to be a carcinoid tumor. Which of the following is the most appropriate management of this
patient?

a. Appendectomy
b. Appendectomy followed by a colonoscopy
c. Appendectomy followed by a PET scan
d. Right hemicolectomy
e. appendectomy followed by chemoterapy

72. A 61 yearold woman with a history of unstable angina complains of hematemesis after retching and vomiting
following a night of binge drinking. Endoscopy reveals a longitudinal mucosal tear at the
gastroesophageal junction, which is not actively bleeding. Which of the following is the next recommended step in
the management of this patient?

a. Angiography with embolization


b. Balloon tamponade
c. Exploratory laparotomy, gastrotomy, and oversewing of the tear
d. Systemic vasopressin infusion
e. Expectant management
73.A previously healthy 20year old man is admitted to the hospital with acute onset of left sided chest pain.
Electrocardiographic findings are normal, but CXR shows a 40% left pneumothorax. Appropriate treatment consists of
which of the following procedures?

a. Observation
b. Barium swallow
c. Thoracotomy
d. Tube thoracostomy
e. Thoracostomy and intubation

74.A 4week old male infant presents with projectile, nonbilious emesis. Ultrasound of the abdomen reveals a pyloric
muscle thickness
of 8 mm (normal 3-4mm). Which of the following is the best initial management of this patient?

a. Urgent pyloromyotomy
b. Urgent pyloroplasty
c. Urgent gastroduodenostomy
d. Fluid hydration and correction of electrolyte abnormalities prior to operative management
e. Administration of sodium bicarbonate to correct aciduria prior to operative management

75. 29 year old woman present with 6 month history of erythema and edema on the right breast with palpable
lymphadenopathy. Punch biopsy skin reveals neoplastic cell in the........ lymphatic. Best next step in her management?

a. course of nafcilline to treat the overlying cell....and then neoadjuvantchemoterapy for breast cancer
b. modified radical mastectomy followed by adjuvant chemoterapy
c. modified radical mastectomyfollowed by hormonal therapy
d. combine modality chemoterapy and radiation therapy to the right breast and surgery reserved for residual disease
e. combine modality therapy with chemoterapy, surgery and radiation.

76. A 42yearold man had has bouts of intermittent crampy abdominal pain and diarrhea .hemoterapy, Colonoscopy is
performed. Which of the following colonic pathologies is though to have no malignant potential?

a. Ulcerative colitis
b. Villous adenomas
c. Familial polyposis
d. PeutzJeghers syndrome
e. Crohn colitis

77. A 72 years old man status post CABG 5 years ago present with hematochezia, abdominal pain and fever.
colonoscopy reveals patches of dusky appearing mucosa at splenic flexure without active bleeding. which is the most
appropriate management of this patient?

a. angiography with administration of intraarterialpapaverine


b. Emergent laprotomy with left hemicolectomy and tranverse colostomy
c. Aortomesenteric bypass
d. Exploratory laprotomy with Thrombectomy of the inferior mesenteric artery
e. Expectant management
78. A 52 yearold man with a family history of multiple endocrinoneoplasia type 1 (MEN1) has an elevated gastrin level
and is suspected to have a gastrinoma. Which of the following is the most likely location for his tumor?

a. Fundus of the stomach


b. Antrum of the stomach
c. Within the triangle formed by the junction of the second and third portions of the duodenum,the junction of the
neck and body of the pancreas, and the junction of the cystic and commonbile duct
d. Tail of the pancreas
e. Within the triangle formed by the inferior edge of the liver, the cystic duct, and the common hepatic duct

79. 46 year old woman ask about the need for surgery, since she was recently diagnosed with crohn's disease.
Indication for operation in Chron's include all but wich of the following:

a. Intestinal obstruction
b. Enterovesical fistula
c. recurrent bouts of abdominal pain
d. Enterovaginal fistula
e. Free perforation

80. A 45yearold man was an unhelmeted motorcyclist involved in a highspeed collision. He was ejected from the
motorcycle and was noted to be apneic at the scene. After being intubated, he was brought to the ER, where he is
noted to have a left dilated pupil that responds only sluggishly. What is the pathophysiology of his dilated pupil?

a. Infection within the cavernous sinus


b. Herniation of the uncal process of the temporal lobe
c. Laceration of the corpus callosum by the falx cerebri
d. Occult damage to the superior cervical ganglion
e. Cerebellar hypoxia

81. A 22-years-old man is examined following a motor vehicle accident, his right leg is injured, his elbow and clavicle
also injured, which of the following fractures dislocation is the most likely to result an associated vascular injury?

a. Knee dislocation
b. Closed posterior elbow dislocation
c. Midclavicular fracture
d. Supracondylar femur fracture
e. Tibial plateau fracture

82. Following a head on motor vehicle collision, a 22-year-old unrestrained passenger presents to the ER with
dyspnea and respiratory distress, she is intubated and physical examination reveals subcutaneous emphysema and
decreased breath sounds. Chest x ray reveals cervical emphysema, pneumomediastinum and right sided
pneumothorax. What is the most likely diagnosis?

a. Tension pneumothorax
b. Open pneumothorax
c. Tracheobronchial injury
d. Esophageal injury
e. Pulmonary contusions
83. A patient with solid malignancy is discussing chemotherapy with his oncologist, he is interested in the risks of
treatment. What is primary toxicity of doxorubicin (Adreamycin) ?

a. Cardiomyopathy
b. Pulmonary fibrosis
c. Peripheral neuropathy
d. Uric acid neuropathy
e. Hepatic dysfunction

84. A 26-year-old male is resuscitated with packed red blood cells following a motor vehicle collision complicated by a
fractured pelvis and resultant hemorrhage, a few hours later the patient becomes hypotension with a normal central
venous pressure CVP, oliguric and febrile, upon examination, the patient is noticed to have pressure oozing and blood
in his IV site. Which of the following is most diagnosis ?

a. Hypovolemic shock
b. Acute adrenal insufficiency
c. Gram negative bactermia
d. Transfusion reaction
e. Ureteral obstruction

85. A 23-year-old woman undergoes total thyroidectomy for carcinoma of the thyroid gland. On the first
postoperative day she begins to complain of tingling in her hands, she appears quite anxious and later complains of
muscle cramps. Which of the following diagnosis is the most apparitional treatment?

a. 10 mg of magnesium sulfate IV
b. Oral vitamin D
c. 100 mg oral synthroid
d. Contentious infusion of calcium gluconate
e. Oral calcium gluconate

86.A teenage boy falls from his bicycle and is run over by a truck. on arrival in the emergency room (ER), he is awake
and alert and appears frightened but in no distress, the chest radiograph suggests an air-fluid level in the left lower
lung field and nasogastric tube seems to coil upward into the left chest. Which of the following is the next best step
on his management?

a. Placement of left chest tube


b. Echocardiography
c. Laprotomy
d. Esophagogastroscopy
e. Diagnostic peritoneal lavage

87. A 10-years-old boy was the backseat belted passenger in a high speed motor vehicle collision, on presentation to
the ER he is awake, alert and hemodynamicaly stable. He is complaining of abdominal pain and has an ecchymosis on
his anterior abdominal wall where the seatbelt was located. Which of the following statements is true regarding
workup?

a. The boy can be safely discharged without any other workup since his abdominal pain is probably secondary to
ecchymosis.
b. The boy can be safely discharged if amylase level is normal
c. The boy can be discharged if abdominal films are negative for the presence of free air
d. The boy can be discharged if CT scan is negative
e. The boy should be observed regardless of test results
88. Which of the following may be appropriate initial treatment therapy for a 4 cm anal cancer squamous carcinoma
involving the internal sphincter?

a. Abdominoperineal resection
b. Wide local excision
c. Radiotherapy and chemotherapy
d. Interstitial radiotherapy
e. Wide local excision and bilateral groin dissection

89. Which of the following is the most important determinant of survival after treatment of colorectal cancer?

a. Lymph node involvement


b. Transmural extension
c. Tumor size
d. Histological differentiation
e. DNA content

90. Treatment of toxic megacolon includes all of the following except:

a. Steroid adenine
b. Antibiotics
c. Nasogastric intubation
d. Early colecotomy
e. Opiates

91. All of the following are indicative of poor prognosis in acute pancreatitis except?

A. Serum calcium less than 8.0 mg/dl


B. Hyperglycemia
C. Serum amylase more than five times the normal on admission
D. Atrial oxygen tension less than 60mmhg
E. Serum lactic dehydration more than 3 times normal or greater than 360 iu/l

92. Which of the following procedures is the best treatment for a 1.5 cm insulinoma located on the mid pancreas?

A. Enucleation
B. Wedge resection
C. Distal pancreatectomy
D. Duodenal sparing total pancreatectomy

93. A patient with hypertension is diagnosed with hyper-aldosteronism. A CT scan shows bilaterally enlarged adrenals
without a mass.
The most appropriate next intervention is?

A. Unilateral adrenolectomy
B. Bilateral adrenolectomy
C. Selective venous catheterization
D. Medical management
94. The most common adrenal mass incidentally found on CT scan ( adrenalincidentaloma ) is?

A. Adrenal cyst
B. Adrenal hemorrhage
C. Cortical adenoma
D. Myelolipoma

95. Which of the following should be the first treatment administered to a patient with a potassium level of 6.3 and
flattened P waves on their ECG?

A. kayexalate
B. Insulin and glucose
C. Calcium gluconate
D. Inhaled albutrtol

96. All of the following are false except?

A. Ghrelin is secreted from adipocytes


B. Ghrelin is high in the obese
C. Leptin comes from the stomach
D. Leptin is high in the obese
E. PYY and GLP-1 decease after gastric bypass

97. Patient undergoing Roux-en-Y gastric bypass are known to be at increased risk for developing gallstone. What is
the current recommended management for patients with a negative ultrasound ( nocholelithiasis ) prior to surgery?

A. Prophylactic cholecystectomy at time of surgery


B. Serial ultrasound every 3 months after surgery for 2 years
C. Ultrasound after surgery only if the patient develops symptoms
D. Fat free diet

98. Which of the following procedures has the highest rate on nutritional complication?

A. Laparoscopic adjustable banding


B. Roux-en-Y gastric bypass
C. Sleeve gastrectomy
D. Duodenal switch

99. What is the appropriate treatment for a patient with type 3 ( mixed ) hiatal hernia and iron deficiency anemia ?

A. Observation
B. Acid reducing agent
C. Repairing of the hiatal hernia alone
D. Repairing of the hiatal hernia and fundoplication

100. What is the initial pathologic change that leads to the clinical findings of achalasia ?

A. Hypertension of the LES


B. relaxation of the LES
C. Hypertension of the body of pancreas
D. Diffuse relaxation of the body of the pancreas
E. Reflux
101. a 14 year old female menarche one year ago present with vaginal bleeding, BP 116/73 , HR 86, hemoglubin 10.5 ,
BhcG negative , pelvic ultrasound normal, uterine mucosa thickness 5 mm, the most appropriate management ?

A-urgent D&C.
B- observation in ER and repated vital sign.
c-oral estrogen administration
d-GnRH agonist injection.
e- ikaclomin administration.

102. a 55 year old woman has a menopause for a year and is being evaluated for a vaginal bleeding, FSH 34, normal
pelvic ultrasound demonstrated a 1,5 cm subseroticmyoma and a uterine mucosa of 10mm, on examination vaginal
atrophy, what is the most important step in her evaluation ?

a-repeat hormonal profilax.


b-samply endometrial mucosa.
c-estrogen treatment.
d- PAP.
e-hesterectomy.

103. which of the following is true regarding the treatment with progesterone only oral contraceptives ?

a-causes decrease in cervical mucus viscosity.


b-more efficient than combined oral contraceptives .
c-LH surges but no FSH surges.
d-FSH surges but no LH surges.
e-increase in mood and libido.

104. which of the following is no absolute contraindication for combined oral contraceptives ?

a- smoking over age 35.


b- active lupus.
c-diabetes without end organ disfunction.
d-previous idiopathic thrombotic event.
e-vaginal bleeding that was not yet evaluated .

105. which one of the following is true regarding syphilis ?

a-VDRL test is more sensitive than FTA-ABS test.


b-specific antibodies are not an accurate measure for disease activity.
c-VDRL very high specificity and limited sensitivity.
d-tepical lesion in tertiary disease enable disease diagnosis.
e-in the presence of lupus specific antibodies are not diagnostic.

106.which disease the partner needs to be treated ?

a-bacterialvaginosis.
b-genital herpes.
c-candida.
d-trichomonas.
e-lichen sclerosis.
107. which of the following is the best treatment for syphilis during pregnancy in a woman with allergy to penicillin ?

a-tetraciclin 500 mg/gid.


b-erythromycin.
c-desensitization with the use oral phenoxymethyl penicillin.
d-vancomycin.
e-doxycycline bid 2 weeks.

108. which treatment decreasing uterine myoma size ?

a-GnRH agonist.
b-combined oral contraceptives.
c-mirena IUD.
d-copper IUD.
e-medroxyprogesterone every three months .

109. wich of the following is not treatment of uterine myomas ?

a-uterine artery embolization.


b-laparoscopic resection.
c-focused ultrasound.
d-cryolisis.
e-hysteroscopic resection.

110.which of the following is correct regading mucinous ovarian tumors ?

a-usually very large.


b-usually malignant.
c-usually younger age (2 or 3).
d-usually early clinical symptoms.
e-germ cell origin.
PART B:

1.A 60 year old woman is being evaluated for vaginal bleeding. Past medical history is significant for controlled
hypertension and diabetes. On examination normal cervix , bleeding trough cervical canal. Ultrasound demonstrate
irregular endometrium mucosa, 10mm thick. On Pipelle biopsy grade 2 endometrium carcinoma. What is the most
appropriate management?

a) Vaginal hysterectomy
b) Diagnostic hysteroscopy prior to surgery
c) Hysteroscopy and oophorectomy and lymph node sampling
d) External radiation and brachytherapy
e) High dose systemic progesterone therapy

2. A 44 years old woman with 3 children, undergoes cone biopsy of cervix. Malignancy is indentified with 2mm
invasion and lymphatic vessels involvement. Which of the following is recommended management?

a) Simple hysterectomy
b) Radical hysterectomy
c) Observation with Pap smears
d) Chemotherapy alone
e) Observation with colposcopies

3. Which of the following is not routinely excided during radical hysteroscopy for cervical squamous cell carcinoma?

a) Parametrium
b) Sacro-uterine ligament
c) Ovaries
d) Pelvic lymphatic nodes
e) Vaginal alone

4. Which of the following is the most likely to be found in a male with non-obstructive azoospermia, small testicles,
lack of pubic and facial hair and gynecomastia?

a. High LH, high FSH, low testosterone


b. Low LH, low FSH, low testosterone
c . High LH, low FSH, high testosterone
d. High LH, high FSH, high testosterone
e. High LH, low FSH, high testosterone

5. What is the treatment for sperm collection for fertilization in cases of retrograde ejaculation?

a) Evaluation for diabetes (DM)


b) Use of alfasympathomimetics
c) Use of beta sympathomimetics
d) Alkalization and collection from urine
e) Instruction to urinate prior intercourse
6. A healthy young couple arrives to fertility clinic after failing to conceive for past one year and a half. The female
manses every 24days for 6 days. For religious reasons, they have intercourse only on 14th day of cycle. What is the
most likely etiology for their infertility?

a) Female hyperprolactinemia
b) Female hypergonadotropichypogonadism
c) Male impotence
d) Prolactinoma
e) Discoordination between estimated ovulation and intercourse

7. A 25 year old female is being evaluated for infertility. She has regular menses every 28days. Which of the following
is positively confirms ovulation?
a Progesterone blood level >5 mg/ml during luteal phase

b) Proliterative histology on endometrial biopsy during luteal phase


c) Fernlike pattern of cervical mucosa during luteal phase
d) Increased in body temperature immediately after the menses termination
e) Increased in LH blood levels and decreased in progesterone blood levels during luteal phase

8. A couple at 34 year old is being evaluated for infertility. Female manses every 32days, progesterone level 7ng/ml
on 22nd day and hysterosalpingogram demonstrates normal uterine cavity and normal passage of contrast to the
pelvis. Man sperm 22millions, 55% motility, 30% normal morphology. What is the cause of couple infertility?

a) Due to luteal phase deficiency


b) Due to male etiology
c) Due to uterine etiology
d) Idiopathic etiology
e) Combined ovulation disorder and female age

9. Which of the following is an absolute contraindication for hysterosapingram(????) ?

a) Adnexal abscess
b) Previous PID
c) Secondary infertility
d) Asherman’s syndrome
e) Salpingitisisthmicanodosa

10. A 30 year old woman is being evaluated for secondary amenorrhea and interested in conceiving. FSH level 35.
Most likely assist in conceiving?

a) Induce ovulation with hCG product


b) Induce ovulation with clomiphene citrate
c) Evaluate TRH and prolactin levels
d) Ovum donation or adoption
e) Induced ovulation by wedge resection

11. Which of the following train injuries may cause amenorrhea :

A-injury in the arcuatenucleus .


B-injury in the olfactory nerve .
C- injury in the posterior pituitary .
D- injury in the medio-lateral hypothalamus .
E- prolactin deficiency .
12. which of the following does not cause hirsutism ?

A- 5 alpha reductase deficiency


B- 21 hydroxylase deficiency
C- aromatase deficiency
D- sulfatase deficiency
E- 11 hydroxylase deficiency

13. which of the following is correct regarding ovarian hyper stimulation syndrome :

A- there is allow number of antral follicles of the beginning of cycle .


B- in case of implantation there is relief of symptoms during the pregnancy beginning
C- basal FSH levels are high on day 3
D- there is better prognosis for rapid Resolution of the syndrome if pregnancy achieved
E- thesepatient's are at increased risk for venous thrombus formation

14. A 52 year old female is being evaluated for heat waves sensation during the past 6 month , she has amenorrhea
for the past 6 month which of the following is most likely to be found ?

A- vaginal mucosal atrophy ,thin endometrial mucosa on U/S , elevated gonadotropin levels and elevated estrogen
level due to anovulatory cycles and lack of progesterone
B-normal vaginal mucosa , thick endometrial mucosa on U/S , elevated gonadotropin and elevated estrogen level due
to decreased ovarian inhibin levels
C-vaginal mucosal atrophy , thin endometrial mucosa on U/S , decreased gonadotropin levels due to decreased
estrogen level.
D-the examination and hormonal profile will usually be normal due to a-relatively short duration of complaints.
E- vaginal mucosal atrophy , thick endometrial mucosa on U/S , normal gonadotropin and normal estrogen levels

15. which statement is correct regarding calculating expected delivery ?

A- pregnancy is dated from conception


B-the last menstrual period is reliable even if cycles are irregular
C-the average length of pregnancy is 280 day
D-LMP defined dates are more accurate than those calculated from U/S
E-head circumference may be used to date pregnancy until 25 weeks .

16. A woman is found to have oligohydranmios at 28 weeks of gestation ,which of the following is the most likely
causes?

A- duodenal atresia
B-esophageal atresia
C-DM
D-renal agenesis

17. the obstetric doctor on call is asked to review a cardiotocoghraph performed prior to induction of labor in a 39
week multipara, which of the following features should help to reassure the doctor that is a normal trace ?

A-the baseline heart rate is 100 beats Per min


B- the baseline variability is less than 5 beats per min
C- an acceleration of 15 beats per min for 15 second ,is present on the trace
D- there is no significant acceleration of the fetal heart on 30 min recording
18. on routine antenatal blood test , a 30 year old woman is found to be RH (-) which advice regarding the
management of have pregnancy is correct ?

A-her fetus will also be RH (-)


B- she should have a routine dose of anti D at 28 weeks
C-once she has had two doses of anti D , further administration will not be required
D-if this pregnancy is not affected by RH disease , there should be no problem in subsequent pregnancies

19. a pregnancy woman with gestational diabetes asks you about the increased risks of her fetus , you described all
the following except ?

A-polycythemia
B-hypermagnesemia
C- traumatic delivery
D-neonatal jaundice
E-hypoglycemia

20. chose the best option with regard to a non-immune pregnant woman with an exposure to chicken box ?

A- should be given the varicella-zoster vaccine as soon as possible after exposure


B- should be given the varicella-zoster immunoglobulin as soon as possible after exposure.
C-dose not need intervention unless symptoms of chicken box accur.
D- should not be treated With acyclovir in the 3ed trimester
E- has a 15% risk of having a baby with fetal varicella syndrome

21.consuptive coagulpathy is most likely to be seen in wich of the following obstetric conditions?

a-placental abruption
b-placentaprevia
c-gestational diabetes
d-multiple gestation
e-gestationaltrophoplastic disease

22. a 24 years old g2p2 woman deliverdvaginaly 8 months ago ,her delivery was complicated by massive post
partumhemmoragy .requiring blood transfusion ,she complains of amenoreahsince her delivery,she was not able to
breast feed her baby ,she denise taking medicationsor having head aches or visual abnormalities ,her pregnancy test
is negative.what is the most likely diagnosis?

a-sherman syndrome (anterior pituitary necrtosis)


b-ovarian failure
c-polycystic ovarian syndrome pcos
d-ashermans syndrome(intrauterine adhesions)

23. which of the following is consistent with intrahepatic colelethiasis pregnancy

a-hepatic trans amenase level in the 2000 U/L range


b-papular rash on the abdomen
c-may be associated with increased perinatal morbidity
d-associated with thrombocytopenia
24. a 26 years old g2p1 woman has been "pushing" during her labor for 3 hours without progress .throught this time
her vaginal examination had remained completely delatedcervix,and the head was at 0 station (s-0) in occipet-
posterior position. Wich of the following accurately decribes the situation ?

a-arrest of dilatation
b-arrest of descent
c-protracted descent
d-normal labor progress for a patient with epidural analgesia

25. in a woman with favorable cervix and estimated fetal weight 3850 gram .what is the most common management
at 42 weeks gestation
a-labor induction
b-cesarian section schedule
c-fetalsurverliance started
d-ameniocentesis performed before lung maturation

26. a 40 years old female complains of 7 weeks of pain and swelling in both wrists and knees .the patient complain of
fatigue and lethargy.
Several weeks before noticing the joint pain the patient notice that after a period of rest resistance to movement is
more striking .on examination the metacarpofalangeal joints and wrists are warm and tender .there are no other joint
abnormalities there is no alopecia , photosensitivity,kidney disease or rash.Witch of the following is correct ?

a-the clinical picture suggest early rheumatoid arthritis,and rheumatoid factor should be obtained .
b-the prodrome of lethargy suggest chronic fatigue syndrome
c-lack of systemic symptoms suggest osteoarthritis
d-x-rays of the hand are likely to show joint space narrowing and errusion .

27. a 60 years old female complain of dry mouth and a gritty sensation in her eyes . she states it is sometimes difficult
to speak for more than afewminutes,there is no history of diabetes mellitus or neurologic disease .the patient in on
no medication .on exam the buccal mucosa appears dry and the salivary glands are enlarged bilaterally?What is the
next step in the evaluation of this patient?

a-lip biopsy
b-schrimmer test and misurament of antibodies
c-IgG antibodies to mumps virus
d-use of corticosteroids

28. a 40 years old male complain of exiquisite pain and tenderness over left ankle .there is no history of trauma .the
patient is taking a mild duritic for hypertension .on exam the ankle is very swollen and tender there are no other
physical exams abnormalities .what is the next step management?

a-begin colichicine and broad-spectrum antibodies


b-obtain uric –acid level and perform a arethrocentices
c-begin allopurinol if uric acid is elevated
d-obtain ankle x-ray to rule out fracture
29. a22 years old male develops the insidious onset of law back pain improved with exercise and worsening by rest
.there is on history of diareha ,conconjunctivitis , urethrites,eyeproblems,or nail changes. on exam the patient has
loss of mobility with respect to lombar flection and extension.he has a kyphotic posture ,aplain film of the spine show
widening an sclerosis of the sacroiliac joints ,some calcifications are noted in the anterior spinal legaments.wich of the
following best characterizes this patient disease process?

a-he is most likely to have a cut lombosacral back strain and reuires bed rest
b-the patient has a spondilo-arethropathy,most likely ankylosingspondilitis.
c-the patient is likely to die from polmunaryfebrosis and extrathoracic restrictive lung disease
d-a rheumatoid factor is likely to be positive.
e-a xolonoscopy is likely to show crohn disease

30. a 75 years old mail complains of unilateral headache in one occasion he transiently lost his vision .he also
complains of aching in the shoulders and neck .there are no focal neurologic findings ,carotid pulses are normal
without bruits
,there is some tenderness over the left temple .laboratory data shows a mild anemia . witch of the following tests is
most likely to be abnormal?

a-carotid ultra sound


b-CT scan
c-erythrocytes sedimentation rates
d-x-ray of the left shoulder
e-skull films

31. A 50yearold patient with longstanding chronic obstructive lung disease develops the insidious onset of aching in
the distal extremities, particularly the wrists bilaterally. There is a 10lb weight loss.
The skin over the wrists is warm and erythematous. There is bilateral clubbing. Plain film is read as periosteal
thickening, possible osteomyelitis. What should be the next step in management?

a. Start ciprofloxacin
b. Obtain chest xray
c. Aspirate both wrists
d. Begin gold therapy

32. A patient with low grade fever and weight loss has poor excursion on the right side of the chest with decreased
fremitus flatness to percussion, and decreased breath sounds all on the rights. Trachea is deviated to left. The most
likely diagnosis is

a. pneumothorax
b. pleural effusion
c. consolidated pneumonia
d. atelectasis

33. a 40 year old alcoholic develops cough and fever. Chest x ray shows air fluid level in the superior segment of the
right lower lobe. what is the most etiologic agent?

a. streptococcus pneumonia
b. haemophilus influenza
c. legionella
d. anaerobes
34. A 70-year-old patient with chronic obstructive lung disease requires 2 L of nasal O2 to treat his hypoxia, which is
sometimes associated with angina. While receiving nasal O2, the patient develops pleuritic chest pain, fever, and
purulent sputum. He becomes stuporous and develops a respiratory acidosis with CO2 retention and worsening
hypoxia. The treatment of choice is

A) Stop oxygen
B) Begin medroxyprogesterone
C) Intubate the trachea and begin mechanical ventilation
D) Observe patient 24 hours before changing therapy
E) Begin sodium bicarbonate

35. 2 weeks after hospitalization discharge for documented MI, a 65 years old returns to your office very concerned
about low grade fever and pleuritic chest pain. There is no associated shortness of breath, lungs are clear and heart is
free of sign murmurs, gallop or rubs. ECG from last one in the hospital. What is the most affective therapy?

a. antibiotics
b. anticoagulation with warfarin (Coumadin)
c. an anti inflammatory agent
d. an increase in antianginal medication
e. an antianxiety agent

36. A 52 years old man complains of impotence. On physical examination, he has an elevated jugular venous pulse, S3
gallop and hepatomegaly. He also appears tanned, with pigmentation along joint folds. His left knee is swollen and
tender. The plasma glucose is 250 mg/dL and liver enzymes are elevated. Your next study to establish the diagnosis
should be ?

a. detection of nocturnal penile tumescence


b. Determination of iron saturation
c. Determination of serum copper
d. detection hepatitis b surface antigen
e. echocardiography

37.A 70yearold male presents with a complaint of fatigue. There is no history of alcohol abuse or liver disease; the
patient is on no medication. Scleral icterus is noted on physical exam. There is no evidence for chronic liver disease on
physical exam, and the liver and spleen are non palpable. The patient is noted to have a normocytic, normochromic
anemia. The first step in evaluation of this patient is:

A.CT scan of the abdomen


B. Hepatitis profile
C. Liver function tests, including direct versus indirect bilirubin and urine bilirubin
D. Abdominal ultrasound
38.A 40yearold male with longstanding alcohol abuse complains of abdominal swelling, which has been progressive
over several months. He has a history of gastrointestinal bleeding. On physical exam, there are spider angiomas and
palmar erythema. Abdominal collateral vessels are seen around the umbilicus. There is shifting dullness, and bulging
flanks are noted. An important first step in the patient’s evaluation is :

A. Diagnostic paracentesis
B. UGI series
C. Ethanol level
D. CT scan

39. A 34 year old white woman is treated for UTI with amoxicillin. Initially she improves, but 5 days after beginning
treatment, she develops recurrent fever, abdominal bloating and diarrhea with six to eight loose stools per day. You
suspect antibiotic-associated colitis. The best diagnostic test is?

a. Identification of Clostridium difficile toxin in the stool


b. Isolation of Clostridium difficile in a stool culture
c. Stool positive for WBC
d. Detection of IgG antibodies against Clostridium difficile in the serum

40. A 59 years old man presents to the emergency department (ED) complaining of new onset chest pain that radiates
to his left arm. He has a history of hypertension, hypercholesterolemia and a 20 pack year smoking history. His ECG is
remarkable for T wave inversions in the lateral leads. Which of the following is the most appropriate next step in
management?

a. give the patient two nitroglycerin tablets sublingually and observe if his chest pain resolves
b. place the patient on a cardiac monitor, administer oxygen and give aspirin
c. call the cardiac catheterization lab for immediate percutaneous coronary intervention (PCI)
d. order a chest x -ray, administer aspirin, clopidogrel and heparin
e. start a beta blocker immediately

41. A 38 years old man with a history of testicular cancer presents with fatigue, dyspnea on exertion, and generalized
weakness, he underwent chemotherapy 10 years ago. Subsequent serial evaluations have not revealed any remaining
testicular cancer. On physical examination, the patient is found to have pale
conjunctivae and nail beds, as well as scattered lower-extremity petechiae. What possible long-term side effects of
chemotherapy could be responsible for this patient’s symptoms?

a. Cardiovascular disease.
b. Hypercholesterolemia.
c. Leukemia.
d. Cerebrovascular disease.
e. Retroperitoneal fibrosis

42. A 55 year old male with urosepsis started on IV fluid and pipercillin-tazobactam. His symptoms gradually improved
until hospital day-5, when he develops a rash and low grade fever. Lab tests reveal an increased creatinine and a
urinalysis with blood cells and eosinophils. What is the most likely diagnosis?

a. prerenal disease
b. Nephrotic syndrome
c. acute tubular necrosis.
d. acute interstitial renal disease
43. A 72 year old woman complains of fatigue, easy bruising, weight loss, and abdominal distention. Her pulse is
slightly fast, and she has notable pallor and splenomegaly. Blood counts reveal anemia and thrombocytosis, you are
concerned that she may have myelofibrosis .what would you expect to see on this patient's peripheral blood smear if
she did have myelofibrosis?

a. Teardrop cells
b. Schistocytes
c. Burr cells
d. Acanthocytes

44. A 67-year-old woman is brought to the ED by paramedics complaining of dyspnea, fatigue, and palpitations. Her
blood pressure is 80/50 mm Hg, heart is 139 beats per minutes, and her respiratory rate is 20 breaths per minute. Her
skin is cool and she is diaphoretic. Her lung exam reveals bilateral crackles and she is beginning to have chest pain.
ECG shows a narrow complex irregular rhythm with a rate in the 140s. Which of the following is the most appropriate
immediate treatment for this patient?

a. Diltiazem
b. Metoprolol
c. Digoxin
d. Coumadin
e. Synchronized cardioversion

45. A 45 years old woman with history of ovarian cancer presents to the emergency department with acute onest of
right-sided chest pain , shortness of breath and dyspnea , her BP is 131/75 mmHg , HR is 101 beats per minute , PR is
18 breaths per minute . and oxygen saturation is 97% on room air . you suspect that this has a pulmonary embolism
(PE) . which of the following tests is most likely to be abnormal ?

A- Arterial blood gas


B- Oxygen saturation
C- ECG
D- Chest radiograph
E- D-dimer

46. Which of the following cardiac markers begins to rise within 3–6 hours of chest pain onset, peaks at 12–24 hours,
and returns to baseline in 7–10 days?

a.Myoglobin
b.Creatinine kinase (CK)
c.Creatinine kinase-MB (CK-MB)
d.Troponin
e.Lactic dehydrogenase (LDH)

47.A 27 years old man complains of chest pain palpitations and light headedness for the past hour .he has no past
medical history and is not taking medications . he drinks beer occasionally on the weekend and does not smoke
cigarette . his HR is 180 per minute . BP is 110/65mmHg , and oxygen saturation is 99% on the room air . An ECG
reveals a HR of 180 beats /min with QRS complex of 90 Msec with a regular rhythm . there are no discernible P wave .
which of the following is the most appropriate medication to treat this dysrhythemia ?

A- Digoxin
B- Lidocaine
C- Amiodarone
D- Adenosine
E- Bretylium
48. A 55-year-old man presents to the ED at 2:00 a.m. with left-sided chest pain that radiates down his left arm. He
takes a b-blocker for hypertension, a proton-pump inhibitor for reflux disease, and an antilipid agent for high
cholesterol. He also took sildenafil last night for erectile dysfunction. His blood pressure is 130/70 mm Hg and heart
rate is 77. Which of the following medication is contraindicated in this patient?

a. Aspirin
b. Unfractionated heparin
c. Nitroglycerin
d. Metoprolol
e. Morphine sulfate

49. 71 years old man is playing card with some friends when he starts to feel pain in the left side of his chest . his
fingers in the left hand become numb and he feels short of breath . his wife calls the ambulance and he is brought to
the hospital . in the ED , ECG is Performed . which of the following is the best describe the order of ECG changes in an
MI ?

A- Hyper acute T wave , ST – segment elevation , Q Wave .


B- Q Wave , ST – segment elevation , Hyper acute T wave .
C- Hyper acute T wave , Q wave , ST – segment elevation.
D- ST-segment elevation , Q wave , hyper acute T wave .
E- ST –segment elevation , hyper acute T wave , Q wave .

50. Which of the following patients has the lowest clinical probability for the diagnosis of a PE?

a.A 21-year-old woman two days after a cesarean delivery


b.A 55-year-old woman on estrogen replacement therapy who underwent a total hip replacement procedure 3 days
c.A 39-year-old man who smokes cigarettes occasionally who underwent annul complicated appendectomy 1 month
d.A 62-year-old man with pancreatic cancer
e.A 45-year-old man with factor V Leiden deficiency

51. a 75 years old man goes out to shovel snow from drive way .after 5 minutes of shoveling ,he feels short of breath
,chest pain ,and then passes out.he awakens minutes later to his wife shaking him.
In the emergency department he denise chest pain or dyspnea.his BP is 160\85 and his HR is 71 per minute ,and
oxygen saturation is 97% in air room.on physical examination you hear a harsh systolic ejection murmur and ECG
reveals sinus rythem with left ventricular hypertrophy ,wich of the following is the most likely diagnosis?

a-asystolic cardiac arrest


b-brugada syndrome
c-subclavian steal syndrome
d-PE
e-aortic stenosis

52. a 55 years old man presents to the emergency department with chest pain and shortness of breath.his BP is
170\80 and his HR is 89 per minute ,his oxygen saturation is 90% in room air .his physical examination reveals crackles
midway up both lung fields and a new holosystolic murmur that is the loudest at the apex and radiate to the left
axilla. ECG reveals ST elevation in inferior leads .chest Rx shows PE(polmunary edema) with normal sized cardiac
silhouette. Which of the following is the most likely cause of the cardiac murmur?

a-critical aortic stenosis


b-papillary muscle rupture
c-pericardial effusion
d-CHF
e-aortic dissection
53. a 22 years old presents in the emergency department with history consists with acute MI.The ECG reveals a ST
elevation and his cardiac markers are positive.he smokes a half pack of cigarets per day for the last 3 months .he
drinks alcohol when hang out with friends .his grandfather dead for heart attack at 80 years old.no hypertension and
no diabetes mellitus and no medications use.recent check revealed a normal total cholesterol ,LDL,LDH.wich is the
most likely expectation for his presentation?

a-cigarete smoke
b-family history of hear attack at age 80
c-incorrectly placed ECG leads
d-undisclosedcocain use
e-alcohol use

54.a 43 years old homeless had been brought to the emergency after finding him intoxicated in the street ,he is
currently arousable and expresses a request to be alone.initials vitals include HR 92 per minute,BP 125\80 and an RR
of 14 per minute with oxygen saturation of 93% on air room.rectal temperature is 101.2 F.chest Rx shows infiltrate
involving the right lower lobe .given this clinical presentation ,what initial coverage of antibiotics is most appropriate
for this patient ?

a-gram negative coverage only


b-gram positive coverage only
c-broad spectrum with anaerobic coverage
d-pneumocystistcarinii pneumonia coverage
e-anti fungal therapy

55.a 62 years old man presents to the emergency department with gradual dyspnea over the last few weeks .he
reports that he is a daily smoker and have not seen a physician for years .upon physical examinations there are
decreased breath sounds on the right as compared to the left .chest Rx indicates blunting of the right costophrenic
angle with a fluid line .a thoracocentisis is performed.given this mentioned history,wich of the following most likely
describes his effusion?

a-transudative effusion
b-exudative effusion
c-transudative and exudative effusion
d-lactate dehydrogenase < 200 U
e-fluid to blood protein ratio <0.5

56.a 49 years old woman presents to the emergency department with difficulty breathing after a morning jog.her
initial vitals are HR 60 per minute,BP 120\55 ,RR 20 per minute,with saturation 94% on air room. Upon physical
examination the patient appears to be in mild distress with audible wheezing ,she is able to speak in partial sentences
and states that she occasionally uses an inhaler .given this patient history and physical examinations.wichmisures
should be taken next?

a-peak respiratory flow


b-chest Rx
c- beta-natriuretic peptid level
d-rectal temperature
e-ABG
57.a 43 years old man who currently uses drug intravenously (IV), present to the emergency department with 2 weeks
of fever,back pain and progressive weakness in his arms bilateraly ,he reports having cough with witish sputum .he
denise any recent trauma,his BP is 130\75,HR 106 per minute,temperature 103 F,RR 16 breath per minute .on
physical examination he has tenderness to palpation in thoracic spine and decreased strength in the upper
extremities bilaterally with normal range of motion.laboratory results reveals WBC 15000\microliter , hematocrit
40%,platlets 225micro\liter.Wich of the following is the most likely diagnosis

a-lung abscess
b-ankylosing spondylitis
c-spinal epidural abscess
d-vertebral compression fracture
e-spinal metastatic lesion

58-a 70 years old non sexually active male complains of fever and pain in his left knee.several days previously the
patient skinned his knee while working in his garage ,the knee is red ,worm ,swollen,anarthocentesis is performed
wich shows 200000 leukocytes\microliter and glucose of 20mg\dL.no crystals are noted .wich of the following is the
most important next step?

a-gram stain and culture of joint fluid


b-urethral culture
c- uric acid level
d-antinuclear antibody

59-a 55 years old man with a history of alcoholism and osteoarthritis developed in the left knee pain for several days
after a fall from a standard height.the patient was brought to the emergency department by ambulance after being
found in a park bench stating,he was unable to walk because of the pain.On Physical examination his left knee is
warm,diffusely tender and swollen with a large effusion,he has pain on passive range of motion.his BP 150\85,his HR
is 105 per minute,temperature is 102.7 F,RR 16 breath per minute, and fingerstick glucose shows 89 mg\dL .wich of
the following is the most appropriate diagnostic test?

a-knee Rx
b-MRI
c-ESR and c-reactive protein
d-arthocentesis
e-bone scan

60-a 35 years old woman with SLE is brought to the emergency department by her brother after he found her febrile
and confused .physical examination reveales ,fever,tachycardia ,awaxing and waning mental status ,petechiae over
the oral mucosa,pallor,and mildly heme positive stool,her urinalysis is positive for blood,red cells
casts,andproteinuria.laboratory results reveals BUN 40mg\dL ,and creatinine 2mg\dL.her bilirubin is elevated
(unconjucated>conjucated),INR 0,98,her CBC reveals WBC 12000\microL,hematocrit 29%,and platlets count
17000\microL with schistocytes on the pereferal smear ,witch of the following is the most appropriate next step in
management?

a-admit to the ICU forplasmapherisis and close monitoring for acute bleeds
b-admit to the ICU for platletstransfusion and monitor for acute bleeding
c-begin corticosteroids,transfuseplatlets and call surgery for immediate splenoctomia
d-admit to the ICU for dyalisis and close monitor for acute bleeds
e-perform non contrast head CT followed by lumbar puncture (LP) for analysis of the CSF.
61. A 67 years old patient present to ER with painful rash that has been worsening over the past 2 days in physical
examination there’re is deeply shiny erythemoutous area warm and tender skin in the left face with sharply
demarcated and indurated border,
There are a minimal edema
BP:125/70
RR:16
TEM:101 F

Next step is :

A. IV antibiotic and hospital admission


B. oral cephalosporin and out patient follow up
c. IV acyclovir and Tzanck smear
d. systemic steroids laparotomy and test for rheumatoid factor
e. dermatology consult And biopsy of rash

62.A 69 women have history of HTN ,DM ,type 1 ,hypercholesterolemia


And alcohole abuse present to ER by her daughter mother was act funny in at home he dosnt know that he present at
home or not and dosnt recognize her family Speaking incomrehensivly BP:150/80 HR 90 in examination she is
diaphoteric ,agitated , tremnlousECG was sinus rhythm ,Normal ST and T wave, Which action you make :

A. administration of benzodiapine to treat ethanol withdrawal


B. activate stroke team and bring pt to ER
C .get STAT administration dextrose if blood sugar is low
D .psychitristconsult for sundowning
E.administrate haloperidol for sedation

63. 74 years old lethargic women present to ER progressive somnolent and last week couldn’t woken today ,takes
medication for DM ,HTN ,hyperthyroid and recent ankle sprain treated with hydrocodone/acetaminophen
she is profoundly lethargic and response only to pain , periorbital edema ,delayed tendon deep reflexes .the most
likely diagnosis is :
BP:145/84 HR 56 TEM 94 F RR 12 .

A. hypoglycemia
B. opioid overdose
C. stroke
D. myxedema coma
E. depression

64. A 20 year old man presents with fever and severe right lower quadrant pain for 1 day. Prior to this episode he has
a 2 month history of generalized malaise, 10 Lb weight loss and occasional bloody diarrhea. Past medical history:
perirectal abscesses. Physical examination: tender mas in RLQ with no guarding/rebound. Rectal examination: traces
of blood. CT: no periappendiceal fat stranding, inflammation of distal ileum and several areas of the colon, no recctal
inflammatory changes. What is the most likely treatment:

A. Corticosteroids IV
B. Anti-TNF
C. Immediate surgery
D. Metronidazole
65. A 78 year old man presents with left arm weakness started 10 minutes ago. He has history of hypertension and
diabetes. Temperature 98.8 'F, normal neurologic exam.Some time after the patient has no more weakness. BP:
157/85 HR 87 TEM 98 F RR 14 ,What is the most likely diagnosis:

A. Thrombotic stroke
B. Conversion disorder
C. Migraine with local neurologic defect
D. TIA
E. Todd's paralysis

66. A 57 year old male with history of hypertension and migraines presents to ER with headaches that started 2 days
ago. The headache began suddenly with peak on defecating, the headache is storngest in the occipital region with
mild nuchal rigidity and mild photophobia. He has no fever. CT is normal. What is the most likely next step in the
management of this patient:

A. admission Metoclopramide for nausea and ketorolac for pain


B. Lumbar puncture
C. Empiric meningitis treatment with IV antibiotics
D. IV mannitol to lower intracranial pressure
E. Angiography to evaluate for aneurism

67. A 24 year old female presents with dizzines, numbness and tingling in her fingertips with decreased range of
motion. HR 100 bpm, RR 20. She is anxious and has clawed appearance of both hands. Arterial blood gas: pH 7.55,
decreased HCO3. Which is the most likely metabolic disorder:

A. Metabolic acidosis
B. Metabolic alkalosis
C. Respiratory acidosis
D. Respiratory alkalosis
E. Hyperthyroidism

68. A 40 year old alcoholic man presents with cough and fever. Chest X ray: air fluid level in superior segment of right
lower lobe. Which is the most likely etiologic agent:

A. Streptococcus pneumonia
B. H. Influenza
C. Legionella
D. Anaerobes

69. A 35 year old alcoholic male with nausea, vomiting and abdominal pain that radiates to the back. A Laboratory
value that suggests prognosis in this is:

A. elevated serum lipase


B. elevated serum amylase
C. Leuckocytosis> 20000
D. Diastolic blood pressure > 90
70. A 60 year old man with leukocytosis, no physical abnormalities, spleen normal, leucocytes 40000, peripheral blood
smear: 97% of small lymphocytes. HB:9 mg/dl .
The most likely diagnosis is:

A. Acute monocytic leukemia (AML)


B. Chronic myelogenousleukemia(CML)
C. Chronic lymphocytic leukemia(CCL)
D. Tubercolosis

71. Which of the following statements regarding nutritional support is true?

a) enteral nutrition is less likely to cause infection than parenteral nutrition


b) parenteral nutrition has consistently been shown to result in a decrease in mortality, compared with standard
cares
c) the use of oral supplements in hospitalized elderly patients has been shown to be harmful
d) parenteral nutrition is the preferred mode of nutrition in cancer patients because of its lower incidence of infection

72. A 72 years old men comes to the office with intermittent symptoms of dyspnea on exertion, palpitations and
cough occasionally productive of blood. On cardiac auscultation, a low-pitched diastolic rumbling murmur is faintly
heard toward the apex. The origin of the patients problem probably relates to:

a) rheumatic fever as a youth


b) long standing hypertension
c) Silent MI within the past year
d) congenital origin

73. A 70 years old men complains of the sudden onset of syncope. It occurs without warning and with no sweating,
dizziness or light-headedness. He believes episodes tend to occur when he turns his head too quickly or sometimes
when he is shaving. The best way to make a definitive diagnosis in this patient is:

a) ECG
b) Carotid massage with ECG monitoring
c) Holter monitor
d) electrophysiologic studies to evaluate the AV node

74. A 30 years old women presents to your office complaining of parasthesias, weakness, lack of coordination and
difficulty with gait. Her symptoms are worse after a hot shower. Examination of the cerebrospinal fluid shows
oligoclonal bands of IgG. The most likely diagnosis is:

a) multiple sclerosis
b) Huntington’s disease
c) Parkinson’s disease
d) Neurofibromatosis
e) Amyotrophic lateral sclerosis (ALS)
75. Which of the following is the most likely factor contributing to chronic myeloid leukemia?

a) exposure to radiation
b) Family history of cancer
c) tobacco use
d) Philadelphia chromosome

76. A 41 years old business executive presents to your office and complains of palpitations and shortness of breath.
After further questioning he admits to heavy alcohol consumption the previous evening. On exam he is found to have
an irregular HB of 130 b/min. The most likely diagnosis is:

a) Ventricular tachycardia
b) Ventricular fibrillation
c) Premature ventricular contractions
d) Atrial fibrillation
e) Wolff-Parkinson-White syndrome

77. A 24 years old salesperson is referred for an evaluation of difficulty dealing with customers because she gets “so
nervous and anxious that my mind goes blank”. A careful evaluation reveals that she has social phobia. She is
motivated to obtain treatment and continue doing her job. All of the following are effective interventions except:

a) SSRI
b) Flooding
c) modeling
d) systematic desensitization
e) electroconvulsive therapy

78. The term double depression is used to describe:

a) major depressive disorder superimposed on grief reaction


b) major depressive disorder superimposed on dysthymia
c) major depressive, patient not responding to treatment
d) major depressive disorder with psychosis
e) major depressive disorder with anxiety

79. A 36 years old Caucasian male is stable on lithium for bipolar disorder for the past 4 years. During his routine visit
with his family physician, he complains of fatigue and lack of motivation. He is also feeling very cold during this winter
and has gained weight. He wonders if he is getting depressed again. That is the next most appropriate step to take in
this case?

a) diagnose depression and start the patient on antidepressants


b) obtain the patients lithium levels
c) check the patients TSH levels
d) discontinue lithium
e) consider ECT
80. A 46 years old female who is 6 week postpartum complains of feeling low in her mood and having crying spells,
lack of motivation and feeling of hopelessness for the past 4 weeks. She is diagnosis to have postpartum depression.
Which of the fallowing is true about postpartum depression?

a) risk of infanticide is minimal


b) SSRI are contraindicated
c) risk of depression during subsequent pregnancies is increased
d) suicide risk is low
e) hospital admission is rarely required

81. A good test for recent memory is to ask patients :

a. to subtract 7 from 100


b. their date of birth
C. how many siblings they have
d. what they had to eat for their last meal
e. who is the president of the United States

82. Mouth ulceration is associated with which of the following types of withdrawal ?

a. Alcohol
b. Benzodiazepines
c. Cocaine
d. Nicotine
e. Opioids

83. Depression and mania share which of the following symptoms?

A. Psychomotor acceleration
B. Low-self esteem
C. Grandiosity
D. Anger
E. Pessimism

84. The most accurate statement regarding pain disorder is

a. It is diagnosed equally among men and women.


b. Peak ages of onset are in the second and third decades.
C. It is least common in persons with blue-collar occupations.
d. First-degree relatives of patients have an increased likelihood of having the same
disorder.
E. Depressive disorders are no more common in patients with pain disorder than in
the general public.

85. the prevalence of schizophrenia is :

a.10%
b.1%
c.5%
d.0.1%
e.2%
86. A 56 year old anxious executive calls at 5:00 am because he woke up with chest tightness and shortness of breath.
He was scared that he was having a heart attack and he also reported feeling dizzy, trembling, and his heart was
pounding, he thought he was going to die. The symptoms subsided in 20 minute and by the time the emergency
medical services arrived he was sitting in his living room and in no distress. The patient's electrocardiogram reading
was normal and his cardiac enzymes were also found to be normal. What is the most likely diagnosis?

a. Nocturnal panic attack


b. PTSD
c. Psychophysiological insomnia
d. Somatization disorder
e. Myocardial infraction

87. A neurologist is consulted by psychiatrist to assess a 42 years old depressed woman with recent onset of “muscle
twitches”. She was taking Citalopram 80mg. Duloxetine 40mg was recently added because of suboptimal response to
Citalopram alone .She denies any alcohol or substance abuse. On examination, the neurologist notes that the patient
is slightly disoriented and also has mild tremors and hyperreflexia. What is the most likely cause of the “muscle
twitches” in this patient?

a. Serotonin syndrome
b. Alcohol withdrawal syndrome
c. Neuroleptic malignant syndrome
d. Malingering
e. Encephalitis

88. Which of the following is not a negative psychotic symptom?

a. Flat affect
b. Apathy
c. Disorganized thinking
d. Avolition
e. Poverty of thoughts

89. which of the following electrolyte abnormality is associated with polemic patent ?

a.metabolic acidosis
b.respiratory acidosis
c.metabolic alkalosis
d.respiratory alkalosis
e.normal electrolytes

90. A 16-year-old male suffers from irritable mood, increased energy, decreased need for sleep, and pressured
speech. He was recently started on medication by his psychiatrist to control these symptoms. He comes into your
office complaining of asignificant worsening of his acne since starting this new medication. What drug was he started
on?

A. Oxcarbazepine
B. Lorazepam
C. Risperidone
D. Lithium
E. Lamotrigine
90. A 16 years old patient with irritability, increased energy, decreased need for sleep, and increased speach.
Psychiatrist prescribe drugs to control these symptoms. Complains to a significant worsening of acne since then. What
medicine was prescribed?

A. Oskarbazepin
B. Lorazepam
C. Risperidone
D. Lithium
E. Lamotrigine

91. What is true of suicide:

A. Committing suicide is most often associated with bipolar disorder


B. Teenagers often successfully commit suicide by hanging
C. In recent years, the suicide rate among middle-aged adults has increased significantly
D. Attempted suicide in the past is the best prediction of the risk of suicide in the future
E. Women successfully commit suicide more often than men

92. 45 years old patient with bipolar disorder complains of amenorrhea, galactorrhea, decreased libido, and
anorgasmia. She was admitted to the emergency room with elevated prolactin in the blood, has been appointed
risperidone 4 mg per day for bipolar disorder. Neurological examination revealed decreased vision in both lateral
visual fields. What is the most likely diagnosis:

A. acute right-sided parietal stroke


B. bleeding in the thalamus
C. pituitary macroadenoma
D. acute left-sided parietal stroke
E. midbrain infarction

93. With of the following personality disorder patients most often harm themself:

A. border
B. narcissistic
C. hysterical
D. dependent
E. schizoid

94. Which of the following is not characteristic of narcissistic personality disorder:

A conviction in his own grandiosity


B. the need for the admiration of others
C. exponential self dramatization, theatricality, excessive expression of emotion
D. concern fantasies in an unbounded success, power and brilliance
E. Using of others

95. Which of the following is not a criteria of post-traumatic stress disorder:

A. re-experiencing the event


B. increased возбеждение
C. avoidance of stimuli associated with the trauma
D. Duration disorder more than 2 months
E. the patient's response to trauma include acute fear nightmare
96. A female patient is presented in the office complaining of intense fear that causes the ability to assume a social
role in your child's school. The patient is afraid to act in a way that is humiliating or embarrassing. Becomes anxious
when she meets unfamiliar people. What is the most likely diagnosis?

a) borderline personality disorder


b) obsessive compulsive personality disorder
c) Narcissistic personality disorder
d) avoidant personality disorder
e) dependent personality disorder

97. Substance that would cause dopaminergic status when used chronically?

a) Heroin
b) PCP
c) Alcohol
d) Amphetamine
e) Cocaine

98. That can cause Valproic Acid in a pregnant woman?

a) Spina bifida
b) Macrocephaly
c) Congenital Hyperthyroidism
d) Oliogohydramnios
e) Fetal growth retardation

99. A woman is presented to the emergency room, that her husband is convinced that she deceives him and that is
not true. The husband follows her everywhere, smells her clothes, checks her wallet and accuses regularly. The
husband does not meet the criteria for a mood disorder. What is the diagnosis?

a) Schizophrenia
b) Major depression with psychotic features
c) Dilutional Disorder
d) Delirium
e) Shared Psychotic Disorder

100. Disorder that presents in a patient concerned about suffer a disease based on misinterpreting body sensations.

a) Somatoform disorder
b) fictitious disorder
c) Conversion disorder
d) Pain Disorder
e) Hypochondriac disorder

101. 20 year old comes to the ER with hypokalemic alkalosis , enlargement parotid glands ,hypotention ,Russll's sign
,what is the diagnosis?

A. psychosis
B.major depression disorder
C.bulimia
D.inhalant induced euphoria
E.HIV
102. which of the following ...... children with depression ?

A.urinary incontinence
B.violence
C.irritability
D.hallucination
E.delusions

103. which of the following sign that best differentiate between delirium and dementia?

A.sleep disturbance
B.hallucination
C.disorientation to place
D.violent behavior
E.alteration of consciousness

104. which of the following is NOT constintent with a major depressive episode?

A.anhedonia
B.withdrawal from social situations
C.low frustration tolerance
D.weight loss
E.increased libido

105.A patient inter your office , she is agitated ,acts seductive ,wears colorful clothes that are bizarre in appearance ,
excessive amount of makeup
and vacillates between being entertaining hypo excited and threatening , based on this information ,her most likely
diagnosis is?

A.major depression disorder


B.breathe psychotic disorder
C.body dystrophic disorder
D.bipolar disorder
E.delusional disorder

106.which of the following would be listed under thought content in the mental status ex..?

A.obsession
B.word salad
C.flight of ideas
D.circumstantiality
E.tangentiality

107.which of the following symptoms NOT part of dysthymic disorder ?

A.poor appetite
B.low self esteem
C.feeling of hopelessness
D.hallucination
E.fatigue
108.which of the following symptoms NOT involved in heroin withdrawal ?

A.pinpoint pupils
B.abdominal pain
C.piloerection
D.muscle twitching
E.dysphoria

109.which of the following is NOT an example of secondary gain ?

A.getting money
B.getting medical help
C.getting out of having to work
D.getting out of family responsibility
E.getting drugs of abuse

110.the highest suicide ,in which of the following age groups?

A.younger than 15 year old


B.between 15-24 year old
C.between 25-44 year old
D.between 45-64 year old
E.older than 65 year old
State Exam 07.10.2013
By: Alhag Abu Anzeh Muhammad

Part A:

01.A 52-year-old man with gastric outlet obstruction secondary to a duodenal ulcer presents with
hypochloremic, hypokalemic metabolic alkalosis.
Which of the following is the most appropriate therapy for this patient?

a.Infusion of 0.9% NaCl with supplemental KCl until clinical signs of volume depletion are eliminated
b. Infusion of isotonic (0.15 N) HCl via a central venous catheter
c. Clamping the nasogastric tube to prevent further acid losses
d. Administration of acetazolamide to promote renal excretion of bicarbonate
e. Intubation and controlled hypoventilation on a volume-cycled ventilator to further increase PCO2

02.A 65-year-old man undergoes a low anterior resection for rectal cancer. On the fifth day in hospital,
his physical examination shows a temperature of 39°C (102°F), blood pressure of 150/90 mm Hg,
pulse of 110 beats per minute and regular, and respiratory rate of 28 breaths per minute. A computed
tomography (CT) scan of the abdomen reveals an abscess in the pelvis.
Which of the following most accurately describes his present condition?

a.Systemic inflammatory response syndrome (SIRS)


b.Sepsis
c.Severe sepsis
d.Septic shock
e.Severe septic shock

03.A 71-year-old man develops dysphagia for both solids and liquids and weight loss of 60 lb over the past 6 months. He
undergoes endoscopy, demonstrating a distal esophageal lesion, and biopsies are consistent with squamous cell carcinoma.
He is scheduled for neoadjuvant chemoradiation followed by an esophagectomy. Preoperatively he is started on total
parenteral nutrition, given his severe malnutrition reflected by an albumin of less than 16r/dl.
Which of the following is most likely to be a concern initially in starting total parenteral nutrition in this patient?

a. Hyperkalemia
b. Hypermagnesemia
c. Hypoglycemia
d. Hypophosphatemia
e. Hypochloremia

04.A 23-year-old woman undergoes total thyroidectomy for carcinoma of the thyroid gland. On the
second postoperative day, she begins to complain of a tingling sensation in her hands. She appears quite
anxious and later complains of muscle cramps.
Which of the following is the most appropriate initial management strategy?

a.10 mL of 10% magnesium sulfate intravenously


b.Oral vitamin D
c.100 μg oral Synthroid
d.Continuous infusion of calcium gluconate
e.Oral calcium gluconate

05.A 65-year-old man has an enterocutaneous fistula originating in the jejunum secondary to
inflammatory bowel disease.
Which of the following would be the most appropriate fluid for replacement of his enteric losses?

a.5% glucose
b.3% normal saline
c.Ringer lactate solution
d.0.9% sodium chloride
e.6% sodium bicarbonate solution
06.A 45-year-old woman undergoes an uneventful laparoscopic cholecystectomy for which she
Receives 1 dose of cephalosporin. One week later, she returns to the emergency room with fever, nausea, and copious
diarrhea and is suspected of having pseudomembranous colitis. She is afebrile and has no
peritoneal signs on abdominal examination. She has a mild leukocytosis with a left shift.
Which of the following is the appropriate initial management strategy?

a.Administration of an antidiarrheal agent


b.Exploratory laparotomy with left hemi-colectomy and colostomy
c.Exploratory laparotomy with subtotal abdominal colectomy and ileostomy
d.Administration of intravenous vancomycin
e.Administration of oral metronidazole

07.A 42-year-old man sustains a gunshot wound to the abdomen and is in shock. Multiple units
of packed red blood cells are transfused in an effort to resuscitate him. He complains of numbness around
his mouth and displays carpopedal spasm. An electrocardiogram demonstrates a prolonged QT interval.
Which of the following is the most appropriate treatment?

a.Intravenous bicarbonate
b.Intravenous potassium
c.Intravenous calcium
d.Intravenous digoxin
e.Intravenous parathyroid hormone

08.A 36-year-old man who was hit by a car presents to the ER with hypotension. On examination,
he has tenderness and bruising over his left lateral chest below the nipple. An ultrasound examination is
performed and reveals free fluid in the abdomen.
What is the most likely organ to have been injured in this patient?

a.Liver
b.Kidney
c.Spleen
d.Intestine
e.Pancreas

09.A 25-year-old woman arrives in the ER following an automobile accident. She is acutely
dyspneic with a respiratory rate of 60 breaths per minute. Breath sounds are markedly diminished on the
right side. Which of the following is the best first step in the management of this patient?

a.Take a chest x-ray.


b.Draw arterial blood for blood-gas determination.
c.Decompress the right pleural space.
d.Perform pericardiocentesis.
e.Administer intravenous fluids

10. Following blunt abdominal trauma, a 12-year-old girl develops upper abdominal pain, nausea, and vomiting. Total
obstruction of the duodenum with coild spring appearance in the second and third portions .
in the absence of the suspected injuries which of the following is the most appropriate management:

a. Gastrojejunostomy
b. Nasogastric suction and observation
c. Duodenal resection
d. TPN (total parental nutrition) to increase the size of the retroperitoneal fat pad
e. Duodenojejunostomy
11.28. A 27-year-old man presents to the ER after a high-speed motor vehicle collision with chest pain
and marked respiratory distress. On physical examination, he is hypotensive with distended neck veins
and absence of breath sounds in the left chest.
Which of the following is the proper initial treatment?

a. Intubation
b. Chest x-ray
c. Pericardiocentesis
d. Chest decompression with a needle
e. Emergent thoracotomy

12.19-year-old college student presents with a testicular mass, and after treatment he
returns for regular follow-up visits. Which of the following is the most useful serum marker for detecting
recurrent disease after treatment of nonseminomatous testicular cancer?

a. Carcinoembryonic antigen (CEA)


b. Human chorionic gonadotropin (hCG)
c. Prostate-specific antigen (PSA)
d. CA125
e. p53 oncogene

13.An 11-year-old girl presents to your office because of a family history of medullary carcinoma
of the thyroid. Physical examination is normal. Which of the following tests should you perform?

a. Urine vanillylmandelic acid (VMA) level


b. Serum insulin level
c. Serum gastrin level
d. Serum glucagon level
e. Serum somatostatin level

14.A 50 years old man, with low grade MALT lymphoma, on a chest, abdomen and a pelvic CT you see enlarged lymph nodes
or metastases. What the therapy:

A) Total gastrectomy and oesophagojejunostomy


B) Total gastrectomy, oesophagojejunostomy + adjuvant therapy
C) Chemotherapy
D) Steroids
E) Antibiotics

15. a 50 years old woman, on an examination you find fresh blood per rectum, raised abdominal distention, increased
weight loss, an 8 cm mass is found next to the anal verge, you found an invasion of the tumour to the perirectal fat +
multiple enlarged lymph nodes without distant metastasis, she wishes to preserve the sphincter if possible.
what is the best treatment:

A) Abdominoperineal resection
B) Neoadjuvant therapy and anterior resection
C) Neoadjuvant therapy and abdominoperineal resection
D) Transanal resection and adjuvant therapy
E) Neoadjuvant therapy and transanal resection

16.57 years old woman with bone metastasis 1 year after mastectomy for a breast tumour, estrogen receptor – negative ,
progesterone receptors – negative,Her2-Neu positive.
what is the best therapy:

A) Antiestrogen (tamoxifen)
B) Raloxifen
C) Monoclonal antibodies (Tranzumab)
D) Anastozole
E) 5-fluoruracil
17. 35 year old man with history of right thyreoidectomy for benign thyroids nodule , now after a scheduled TTE for
suspicious thyroid mass. Later, noted a swelling under incision, difficulty breathing, intubation was successful.
What should be the next course of action?

A) Fiberoptic laryngoscopy
B) I.V. calcium
C) Broad spectrum ATB and debridement
D) Wound exploration
E) High dose steroid and antihistamine

18.A 35 year woman, found a lump in the left breast, with no family history of breast cancer, the mass is rubbery, mobile,
non tender on palpation, no skin changes and axilla is negative for lymfadenopathy. USG reveals a 1 cm cyst.
What is the proper management:

A) Reassure and re-examine in a yearly interval


B) immediate excision for biopsy
C) Cyst aspiration and cytology
D) Fluoroscent needle localisation biopsy
E)Mammography and reevaluation for new information

19.55 year woman with slow growing painless mass on the Rt. Side of her neck. FNA reveals a well differentiated papillary
carcinoma. u/s neck – 1cm nodule in the Rt. Thyroid without masses in the other lobe, no LN metastases in the central or
lat. Neck compartments . with regard to this patientbwhich of the following is associated with poor prognosis?

a.Age
b.Sex
c.Grade of tumor
d.Size of tumor
e.LN status

20.45y woman found to have suspicious appearing calcification in the Rt. Breast on a screening mammogram. Stereotactic
biopsy of the calcification show lobular carcinoma in situ (LCIS). On exam. Both breasts are dense without palpable masses.
Neck & bilateral axilla are negative for lymphadenopathy.
Which of the following is the most appropriate management of this patient.

a.Frequent self breast exam. & yearly screening mammogram.


b.Chemotherapy.
c.Radiation.
d.Rt. Total mastectomy with sentinel LN biopsy.
e.Bilateral modified radical mastectomy.

21.A 45 year woman with HTN, development of facial hair & a 7cm suprarenal mass.
Which of the following is the most likely diagnosis:

a.Myelolipoma
b.Cushing disease
c.Adrenocortical carcinoma
d.Pheochromocytoma
e.Carcinoid
22.A 36 year woman presents with palpitations, anxiety, & HTN. Work-up reveals a pheochromocytoma.
Which of the following is the best approach to optimizing the patient preoperatively?

a.Fluid restriction 24 h preoperatively to prevent intraoperative congestive heart failure.


b.Initiation of an alpha-blocker 24 hours prior to surgery
c.Initiation of an alpha-blocker at least 1-3 weeks prior to surgery
d.Initiation of B-blocker 1-3 weeks prior to surgery
e.Escalating antihypertensive drug therapy with b-blocker followed by alpha-blocker starting at least 1 week prior to surgery

23. 35 YEARS OLD WOMAN PRESENT WITH MULTIPLE AND FREQUENT AREAS OF CUATANEOS ECCHYMOSIS ,PLATLETS
COUNT 10000 ,BONE MARROW STUDY SHOWED NORMAL MEGAKARYOCYTES ABDOMINAL ULTRA SOUND SHOWED
NORMAL SIZED SPLEEN,BY EXCLUSION OF THE CAUSE OF HER THROMBOCYTOPENIA ,SHE DIAGNOSED TO HAVE
IMMUNOTROMBOCYTOPENIC PURPURA.
WHICH IS THE MOST BEST TREATMENT FOR HER:

A-EXPECTANT MANAGEMENT WITH CLOSE FOLLOW UP PLATLETS COUNT


B-IMMIDIATE PLATLETS TRANSFUSION TO INCREASE PLATLETS COUNTS MORE THAN 50000
C-CORTICOSTEROIDS
D-IVIG
E-REFERAL TO SURGERY FOR LAPROSCOPIC SPLEENECTOMY

24.A 59 YEARS OLD WOMAN COMPLAINS OF RIGHT UPPER QUADRENT PAIN ,NAUSEA,VOMITING,SHE GONE
UNCOMPLICATED ,POSTOPERATIVELY PATHOLOGY REVEALS 2.5 CM MUCINOUS ADENOCARCINOMA WITH L.N INVASION
STAGING INCLUDING COLONSCOPY,CHEST X-RAY,ABDOMINOPELIVIC CT SCAN ALL ARE NEGATIVE.
NEXT STEP FOR BEST MANEGMENT:

A-NO FURTHER MANEGMENT AT THIS TIME, FOLLOW UP EACH SIX MONTH FOR TOW YEARS
B-CHEMOTHERAPY
C-NEOADJUVENT CHEMOTHERAPY THEN RIGHT HEMICOLECTOMY
D-ILEOCOLECTOMY
E-RIGHT HEMICOLECTOMY

25.A 41-year-old man complains of regurgitation of saliva and of undigested food. An esophagram
reveals a dilated esophagus and a bird’s-beak deformity. Manometry shows a hypertensive lower
esophageal sphincter with failure to relax with deglutition.
Which of the following is the safest and most effective treatment of this condition?

a. Medical treatment with sublingual nitroglycerin, nitrates, or calcium-channel blockers


b. Repeated bougie dilations
c. Injections of botulinum toxin directly into the lower esophageal sphincter
d. Dilation with a Gruntzig-type (volume-limited, pressure-control) balloon
e. Surgical esophagomyotomy

26.A 56-year-old previously healthy physician notices that his eyes are yellow and he has been losing weight. On physical
examination the patient has jaundice and scleral icterus with a benign abdomen. Transcutaneous ultrasound of the
abdomen demonstrates biliary ductal dilation without gallstones.
Which of the following is the most appropriate next step in the workup of this patient?

a. Esophagogastroduodenoscopy (EGD)
b. Endoscopic retrograde cholangiopancreatography (ERCP)
c. Acute abdominal series
d. Computed tomography (CT) scan
e. Positron emission tomography (PET) scan
27.A 22 Y/O COLLAGE STUDENT NOTICES ABULGE IN HIS RIGHT GROIN INCREASES WITH COUGHING BUT EASILY REDUCABLE
WHICH OF THE FOLLOING HERNIAS ENETER THE SPERMATIC CORD TO CREMASTERIC MUSCLE:

a.FEMORAL
b.DIRECT
c.INDIRECT
d.SPIGELIAN
e.INTRAVESIRAL

28. A 22-YEAR OLD WOMAN PRESENTS WITH A PAINFUL FLUCTUANT MASS IN THE MIDLINE BETWEEN THE GLUTEAL
FOLDS.SHE DENIES PAIN ON RECTAL EXAMINATION. WHICH OF THE FOLLOWING IS THE MOST LIKELY DIAGNOSIS?

A.PILONIDAL ABSCESS
B.PERIANAL ABSCESS
C.PERIRECTAL ABSCESS
D.FISTULA-IN-ANO
E.ANAL FISSURE

29.A 62-YEAR-OLD MAN HAS BEEN NOTICING PROGRESSIVE DIFFICULTY SWALLOWING,FIRST SOLID food and now liquids as
well. A barium study shows a ragged narrowing just below the cranial level. Endoscopic biopsy confirms squamous cell
carcinoma.
Which of the following provides the most accurate information regarding the T stage of an esophageal carcinoma?

a.computer tomography
b.positron emission tomography
c.magnetic resonance imaging
d.endoscopic ultrasound
e.bronchoscopy

30.a 73-year-old woman presents to the emergency room complaining of severe epigastric pain radiating to her back,
nausea , and vomiting. Ct scan of the abdomen demonstrates inflammation and edema of the pancreas. A right upper
quadrant ultrasound demonstrates the presence of gallstones in the gallbladder.
Which of the following is an important prognostic sign in acute pancreatitis according to ransons criteria?

a.amylase level
b.age
c.total bilirubin level
d.albumin level
e.lipase level

31.cytokinase are which type of hormone?

a.polypeptide
b.amino acid
c.fatty acid
d.carbohydrate

32.The primary physiologic effect of nitric oxide [NO] is:

a.icreased platelet adhesion


b.increased leukocyte-endothelial adhesion
c. increased microthrombosis
d. increased smooth muscle relaxation
33.which of the following is the primary fuel source after acute injury:

a.fat
b.muscle[protein]
c.glycogen
d.ketone bodies

34.which of the following drugs irreversibly inhibits platelet COX [cyclo-oxygenase]?

a.ibuprofen
b.clopidegrel
c.aspirin
d.celebrex

35. the diagnosis of heparin-induced thrombocytopenia is made by?

a.>20% fall in platelet count


b.positiveserotonin release assay
c.plateletes <25000 with clinical bleeding
d. prolonged a PTT

36.a patient with breast cancer is considered to be cancer free { no further risk of primary recurrence or metastatic tumor}
after:

a.3 years
b. years
c.10 yea
d.never

37. BRCA2 mutations are associated with all of the following EXCEPT:

A. gastric cancer
B. lung cancer
c. ovarian cancer
d. prostate cancer

38. for average risk patients routine cancer screening is recommended for all but the following disease?

A. breast cancer
B. colorectal cancer
c. cervical cancer
d. pancreatic cancer

39. pneumoperitonium results in which of the following?

a. decrease plasma renin


b. decrease ADH
c. decrease glomerular filtration rate
d. decrease free water absorption in the distal tubules

40. which of the following does not require an electric current to coagulate tissue ?

a. monopolar cautery
b. bipolar cautery
c. mixed cautery
d. ultrasonic shear device
41. traction divertecula in the esophagus are the results of ?

a. a congenital defect
b. infection or inflammation
c. motility disorders
d trauma, usually iatrogentic

42.which of the following makes curative resection of the esophageal cancer unlikely ?

A. Tumor length >6cm


B. Weight loss > 10%
C. recurrent laryngeal nerve Palsy
D. >2 abnormal lymph nodes on CT scan

43. Which of the following is consistently the largest artery to the stomach?

A. left gastric artery


B Right gastric artery
C left gastroepiploic artery
D Right gastroepiploic artery

44. Which of the following is the best test to confirm eradication of H. pylori:

A. Negative histology after biopsy during EGD


B. Negative fecal antigen
C. Negative urea breath test
D. Negative urea blood test

45. Which of the following gastric polyps is considered pre-malignant?

A. Hamartomatous
B. Heterotopic
C. Inflammatory
D. Adenomatous

46. Which of the following patients would be considered candidate for bariatric surgery?

A. 70 year old, BMI 48, with well-controlled diabetes


B. 22 year old, BMI 34, with brittle (uncontrollable) diabetes
C. 35year old, BMI 38, with no comorbidities
D. 56 year old, BMI 42, with no comorbidities

47. extensive perianal condyloma acuminata best treatment is:

A. Topical steroids
B. Topical podopyllin
C. Topical imiquimod
D. Surgical resection and fulguration

48.A 2-cm invasive cancer of the toproximal transverse colon carcinoma should be treated with which of the following
procedures ?

A. Ileocecetomy
B. Ascending colectomy
C. Left hemicolectomy
D. Extended right hemicolectomy
49. Which of the following is an indication for cholecystectomy in an asymptomatic patient with an incidental finding of
gallstones?

A. Any history of abdominal pain


B. Family history of complications of cholelithiasis
C. Porcelain gallbladder
D. Frequent travel out of the country

50. In addition to regional lymphadenectomy, appropriate surgical treatment for T2 carcinoma of the gallbladder is?

a. Cholecystectomy only
b. Cholecystectomy with resection of liver segments IVB AND V
c. Cholecystectomy with limited right hepatectomy
d. Cholecystectomy with extended right hepatectomy

51.9 years old boy is seen with skin rash, he has had for the past 2 days. The rash is mostly on the trunk and seems to come
and go. The rash is red and slightly raised, appears to migrate and is non pruritic. He gave a history of having a sore throat
about 2 weeks ago, and has had some low grade fevers and joints pains. An ASO titer is 1625.
Which one group of findings would confirm the diagnosis of acute rheumatic fever?

A. arthralgia, no fever, rash resembling erythema multiform.


B. subcutaneous nodules, fever, arthralgia
C. erythema multiform, arthralgia, prolonged PR interval on ECG
D. arthralgia, fever, no rash, ESR 120
E. arthritis, no fever, ESR 10

52. A 2 year old boy is seen because of pallor, he drinks 600 cc of cow’s milk per day ,a CBC reveals hemoglobin 8.2 g/l , MCV
65, which of the following indicates is compatible with the patient’s diagnosis?

A. decrease red blood cell distribution width


B. increase serum ferritin
C. increase total iron biding capacity
D. increase reticulocyte count
E. increase serum iron level

53.1 month old baby is seen for regular checkup. Examination is completely normal, but the mother is concerned that her
baby's feet are crooked. The forefoot is deviated medial and there is a prominence at the base of the fifth metatarsal. The
forefoot can be easily manipulated into the normal position. In discussion the child condition with the mother,
which is the most accurate information to discuss.

A. The condition is usually unilateral


B. radiological evaluation should be performed early
C. Most cases will resolve spontaneously without treatment
D. Even if the foot is flexible casting is needed if the foot is not straight by 2-3 months
E. Surgery is required in about half of cases

54. An 18 year old female, u have followed for 6 years, has recently married , she & husband are planning to
start a pregnancy .u advise her to start taken folic acid , this is important to prevent ?

a.prematurity
b.skull defects
c.chromosomal defects
d.neural tube defects
e.osteopenia
55. you are seeing a 2 year old boy for evaluation of speech delay. the mother says he has 5 words and answers all
commands. after detailed history you are concerned about hearing deficit.
what would be the best next step in the evaluation?

A. audiogram
b. head MRI
c. EEG
d. speech therapy referral
e. head ct with special attention to inner ear

56. a 4 year old boy presents to you with fever of 39C and cough. on physical examination you hear crackles over his left
lower lobe region. there is no wheezing and he has been previously healthy. you order chest x ray in which you find a
consolidative infiltrate in the left lower lobe behind the heart consistent with the diagnosis of pneumonia.
what is the most likely pathogen to be found in this case?

a. staphylococcus aureus
b. streptococcus pneumonia
c. mycobacterium tuberculosis
d. neisseria meningitis
e. respiratory syncytial virus

57. full term newborn infant is having episodes of cyanosis and apnea which are worse with attempting to feed but seem
better with crying.
which of the following is the most important next step to quickly establish the diagnosis ?

a. echocardiogram
b. ventilation perfusion scan
c. passage of catheter into nose
d. hemoglobin electrophoresis e. bronchoscopic evaluation of palate and larynx

58 : As you are about to step out of a newly delivered mother’s room, she mentions that she wants to breast-feed her
healthy infant, but that her obstetrician was concerned about one of the medicines she was taking.
Which of the woman’s medicines, listed below, is clearly contraindicated in breast-feeding?

a.Ibuprofen as needed for pain or fever


b.Labetalol for her chronic hypertension
c.lithium for her bipolar disorder
d.Carbamazepine for her seizure disorder
e.Acyclovir for her HSV outbreak

59. A 2-year-old boy has been doing well despite his diagnosis of tetralogy of Fallot. He presented to an outside ER a few
days ago with a complaint of an acute febrile illness for which he was started on a “pink antibiotic.” His mother reports that
for the past 12 hours or so he has had a headache and is more lethargic than normal. On your examination he seems to
have a severe headache, nystagmus, and ataxia.
Which of the following would be the most appropriate first test to order?

a.Urine drug screen


b.Blood culture
c.Lumbar puncture
d.CT or MRI of the brain
e.Stat echocardiogram
60. A 6-year-old boy had been in his normal state of good health until a few hours prior to presentation to the ER room. His
mother reports that he began to have difficulty walking, and she noticed that he was falling and unable to maintain his
balance. Which of the following is the most likely cause for his condition?

a.Drug intoxication
b.Agenesis of the corpus callosum
c.Ataxia telangiectasia
d.Muscular dystrophy
e.Friedreich ataxia

61.A previously healthy 8-year-old boy has a 3-week history of low-grade fever of unknown source, fatigue, weight loss,
myalgia, and headaches. On repeated examinations during this time, he is found to have developed a heart murmur,
petechiae, and mild splenomegaly. Which of the following is the most likely diagnosis?

a.Rheumatic fever
b.Kawasaki disease
c.Scarlet fever
d.Endocarditis
e.Tuberculosis

62.Regarding resuscitative efforts in child, the most important goal is:

a.Restoration of age-appropriate heart rate.


b.Appropriate movement of the chest wall.
c.Auscultation of equal breath sounds in both lungs field.
d.Adequate oxygen delivery and utilization for the body tissues.
e.Palpation of equal pulses in all four extremities.

63.A 9 month old boy is brought to the emergency room in limp and unresponsive state. Initial examination shows a pulse
rate of 35 /min and occasional irregular breaths. After initiation of cpr (including tracheal intubation) delivery of oxygen via
positive - pressure breaths and chest compression, multiple attempts to insert an intra - venous line fail.
The most appropriate next step in management should be:

a.obtain an arterial blood gas sample


b.place in intraosseous needle and administer fluids and intropic agents
c.obtain a "state" head ct study to evaluate reasons for unresponsiveness
d.place transthoracic cardiac pacemaker
e.place a thoracotomy tube to evacuate a possible pneumothorax

64.The recommended treatment for severe combined immunedeficiency is:

a.gene therapy
b.monthly IVIG
c.monthly IVIG and IFN-g
d.monthly IVIG and IL-8-monoclonal antibody
e. stem cell transplantation

65.A 12-yr old white girl present with arthralgia of the knees and elbow and swollen hands of 6 months duration. She has
intermittent fever and has lost 5 kg in weight. Other than swollen joints, findings on physical examination are normal. 3
years earlier, she was found to have thrombocytopenia and was diagnosed with idiopatic thrombocytopenia purpura . In
addition, one summer, she had severe sunburn, and 2 years ago she had mouth sores. Today she has a positive combs test,
and the urinalysis shows multiple Red Blood Cells. The most likely diagnosis is:

a.JRA
b.ITP
c.evans syndrome
d.periarteritis
e.SLE
66. A newborn is found to have congenital heart block. Which of the following is the most likely etiology?

a.Group B streptococcal infection.


b.Neonatal echovirus infection.
c.Gestational diabetes.
d.Maternal lupus .
e.Gray baby syndrome

67. A 12-years old boy develops progressive symptoms of fever, lassitude, arthralgias, headache and abdominal pain.
Physical examination shows hepatosplenomegaly. Further questioning discovers that he and his family live in a rural area
and include unpasteurized dairy products in their diet.
The the most likely etiologic agent of this illness is:

a.actinomyces
b.bartonella hensallae
c.brucella
d.francisella tularensis
e.Yersinia enterocolitica

68.A 5 years old is noted by the parents to snore at night. The child also has had problems staying awake in preschool and
has had behavioral problems. The father also snores. Physical examination of the child reveals large, pink, nonexudative
tonsils. The most appropriate next step is:

a.Laryngoscopy
b.Polysomnography
c.Ambulatory apnea monitoring
d.Telemetry
e.Arterial blood gas analysis

69.The appropriate therapy for severe obstructive sleep apnea syndrome in a 4 years old boy is :

a.Adenotonsillectomy
b.Tracheostomy
c.Parapharyngeal muscle surgery
d.Ventolin
e.Bilevel positive airway pressure

70. A 7 months-old girl present with temperature 40 C, BP 70/30mmHg, diffuse petechiae first noted 4 hours before
presentation. Platelets count 88,000/mm3, and white blood cell count of 43,000/mm3 with 23% neutrophils and 42%
bands. The infant has received all recommended vaccination.
Which of the following is the most likely bacterial etiology of this presentation?

a.staphylococcus aerus
b.streptococcus pneumonia
c.neiserria meningitides
d.haemophyllus influenza type b
e.Escherichia coli o 157:H7

71. A 10 year old boy with 4 day fever, temperature 40, with watery diarrhea. Presented with a generalized seizure.
Which of the following is the causative agent?

a. salmonella gastroenteritis
b. aeromonas gastroenteritis
c. shigella gastroenteritis
d. rotavirus
e. drug ingestion
72.Which of the following reflexes is normally absent in a newborn?

a. Startle (Moro)
b. hand grasp
c. crossed adductor
d. Asymmetric tonic neck
e. parachute

73.A 2 year old boy is admitted to the hospital with high fever for 6 days, swelling of the hands and fee ,
scarlentin form changes of the tongue, generalized red macular rash, high sedimentation rate and thrombocytosis.
Which of the following is the best initial management?

a. cardiac catheterization
b. IVIG & aspirin orally
c. low dose aspirin orally
d. I.V pulse corticosteroids
e. methotrexate orally

74 .10-month-old infant has poor weight gain, a persistent cough, and a history of several bouts of pneumonitis.
The mother describes the child as having very large, foul-smelling stools for months.
Which of the following diagnostic maneuvers is likely to result in the correct diagnosis of this child?

a. CT of the chest
b. Serum immunoglobulins
c. TB skin test
d. Inspiratory and expiratory chest x-ray
e. Sweat chloride test

75.A 7-month old with short hut syndrome receives total parenteral nutrition.
Which of the following is most typical complication of this therapy?

a. sepsis
b. renal failure
c. chronic diarrhea
d. vitamin A deficiency
e. irreversible atrophy of the mucosa of the small intestine

76.The signs and symptoms of meningitis can be different than in adult.


Which of the following signs and symptoms of meningitis is more helpful in adult patient than in a 3-month old?

a. lethargy
b. jaundice
c. vomiting
d. Brudzinsky sign
e. hypothermia

77.You are called to delivery of a women with no parenteral care. Her examination reveal oligohydroamnion.
When you get to the newborn at the nursery you should carefully evaluate him for which of the following:

a. anencephaly
b. trisomy 18
c. renal agenesis
d. duodenal atresia
e. tracheoesophageal fistula
78 .A newborn infant become markedly jaundiced on the second day of life and faint petechia eruption first noted at birth.
He has now a generalized purporic rash. Hematology study for hemolytic disease are negative.
Acute management should include which of the following?

a. liver ultrasound
b. isolation of infant from pregnant hospital personnel
c. urine drug screen of the infant
d. discharge with early follow-up visit in 2 days
e. thyroid hormone assessment

79. 18 month boy was seen by his 15 year old brother that he is holding in his mouth the contents of a bottle with drain
cleaner. Best treatment:

a.Induce vomiting
b.Endoscopy within 12-24hrs
c.Activated chorcal
d.Neutralize with special liquid for balanced ph.
e.Administer big amounts of water or milk.

80. you see a 12 months boy because he is refusing to walk. His mother tells you that he started walking at 10 month and
was developing normally until this moment.
On examination: t 39.0C, baby looks mildly ill with medially rotated left hip and limitation of active and passive motions.

a.Neisseria gonorrhea
b.Adenovirus
c.S. aureus
d.Group A streptococcus
e.Group B streptococcus

81. Because of abnormal spleen functions and high risk of bacterial infections, infants, who have sickle cell anemia get
penicillin prophylaxis at age of 4 weeks.
To which organisms infants with sickle cell anemia are mostly sensitive?

a.Gram negative
b.Encapsulated organisms
c.Fungal infections
d.Viruses
e.Staphylococcus

82. A 9 year-old patient asthmatic patient uses albuterol 3 times per week, for the last 10 days she had wheezing day and
night ,increased the inhaler to 3-4 times a day.
on examination there is a diffuse wheezing with moderate subcostal retraction, next step in management:

a. order chest XR to assess for pneumonia


b. Systemic steroids
c. Lekutriens
d. Low dose inhalation steroids
e. Start 5 days course of systemic corticosteroid
83. 12 months old girl is development delay noted with watery diarrhea with odor 2 times a day. Serological analysis reveals
increased levels of antibodies to transglutaminasa in serum.
Which product does not cause exacerbation or symptoms of this disorder?

A. rice
B. wheat
C. oat
D. barley
E. rye

84. 4 year old boy with impaired verbal and nonverbal communication and lack of empathy.
Doesn’t have any friends in the kindergarden.
Diagnosis:

a.Attention deficit hyperactivity disorder


b. Deaf mutism
c.Autism
d.Cerebral palsy

85. 1 month old boy with food intolarence, slow weight gain and a large tongue.
On examination you note a large rear fontanelle and umbilical hernia.
The next step in diagnosis:

a.Abdominal x-ray
b.CBC and blood culture
cX-ray with barium contrast
d.Serum TBG
e.Hispitalization for further examination.

86. patient with Down syndome refers to you as during the week he had bleeding gums.
Chils less energetic than usual.
Examination: t 37.8C per os, pallor, splenomegaly, bleeding gums and bruising on the lower extremities.
Diagnosis:

a.Aplastic anemia
b.ITP
c.Leukemia
d.Leukemoid reaction
e.Megaloblastic anemia

87. A recommended diet for a 3 years old child with cystic fibrosis ?

a. Folate
b. Sodium
c. Vitamin C
d. Vitamin D
e. Vitamin B12

88. A 6 month healthy girl having fever 38.9c , no accompanying symptoms, urinalysis shows 1+ leukocytes 10WBC per
high - powered field and moderate bacteria. Most reasonable step to take?

A. Obtain relabel US
B. begin broad spectrum intravenous antibiotic
C. Obtain DMSA renal scan
D. Obtain a urine culture by catheter and start antibiotic
89. 10 years old girl has a cold for 14 days, in the 2 days prior to the visit she has developed a fever of 39c, purulent nasal
discharge, facial pain and dry cough. Examination of the nose after topical decongestant shows pus in the middle meatus,
which is most likely diagnosis ?

A. Brain abscess
B. maxillary sinusitis
C. Streptococcal throat infection
D. Retro pharyngeal abscesses

90. 2 months old child is seen in your clinic for the first time, the child was born at home and this is the first well child visit.
Risk factors for infant botulism that should be communicated to the parents include?

A. Gardening
B. Homs construction
C. Frozen vegetables
D. Honey
E. Eggs

91. Which of the following cancers occur primarily during childhood ?

A. Breast cancer
B. renal cel cancer
C. Wilm's tumor
D. Prostate cancer
E. Colon cancer

92. 10 years old girl has had diplopia and ptosis and weakness of her neck flexors for 2 months, symptoms are worse in the
evening and are usually less severe at awakening in the morning, she has no fasciculation or myalgias and her deep tendon
reflexes are normal.
The most likely diagnosis?

A. Hysterical weakness
B. muscular dystrophy
C. Spinal muscular atrophy
D. Botulism
E. Myasthenia gravis

93. 7 days old boy is admitted to the hospital for evaluation of vomiting and dehydration. Physical exam is normal except
for minimal hyper pigmentation of the nipples. Serum sodium and potassium concentrations are 120meq/L ( low) and
9meq/ L ( high ).
Respectively which of the following is the most likely diagnosis ?

A. Pyloric stenosis
B. congenital adrenal hyperplasia
C. Secondary hypothyroidism
D. panhypopituitarism
E. Hyper aldosteronism

94. 12 year old girl experience acute monocular blindness of 2 days duration, past medical history reveals that she has had
headache for the past 3 years that she cannot characterize on the brief episode of diplopia and one episode of parethesiasis
of the feet, these episodes were not related in time did not occur in immediate proximity to the headache and resolved
spontaneously. Findings on physical examination including funduscopic examination are unremarkable other than reduced
visual acuity.
The most important diagnostic step is to perform ?

A. CT
B. MRI
C. An ECG
D. Peripheral nerve conduction test
E. A sural nerve biopsy
95. Trisonomy 21 is most commonly associated with:

a.malrotacion
b.atrioventricular canal
c.cleft palate
d.renal failure
e.sensory neural hearing loss

96. infant sits with minimal support who attempts often a toy beyond renal and who roles our prone supine to the prone
position didn’t have pincer grasp which is level development:

a.2 month
b.4 month
c.6 month
d.1 year

97.a previously healthy full term 15 month old present to the emergency room with a history of sudden onset of rectal
bleeding . his parents deny fever , diarrhea abdominal pain 3 times bright red blood per rectum stable hemodynamic , hb
11 mg dl ,
Which of the following test most likely to the correct diagnosis:

a.shunt x-ray to the abdomen


b.abdominal ultrasound
c.ct of the abdomen
d.mickel scan
e.upper GI series

98. A 16-year-old girl presents with lower abdominal pain and fever. On physical examination, a tender adnexal mass is felt.
Further questioning in private reveals the following: she has a new sexual partner; her periods are irregular; she has a
vaginal discharge.
Which of the following is the most likely diagnosis?

a. Appendiceal abscess
b. Tubo-ovarian abscess
c. Ovarian cyst
d. Renal cyst
e. Ectopic pregnancy

99.7 y.o boy has crampy abdominal pain and rash in the back and his legs and buttocks .
Laboratory test proteinuria and microhematouria wich of the following is the most likely diagnosis:

a.SLE
b.Henoch-Schonlein purpura
c.PSGN
d.Takayasu arthritis
e.Dermatomiosites

100.which of the following causes of conginital infection is associated with cats:

a-cytomegaloviros
b-rubella
c-toxoplasma gondo
d-syphilis
e-parvoviros b 19
101.Most state-of-the-art serum pregnancy tests have a sensitivity for detection of β-human chorionic gonadotropin
(β-hCG) of 25 mLU/ mL.
Such tests would diagnose pregnancy as early as which of the following?

a.5 days after fertilization


b.24 hours after implantation
c.day of the expected (missed) menses
d.5 weeks’ gestation age by menstrual dating
e.6 weeks’ gestation age by menstrual dating

102.A 21-year-old G1 now P1 just delivered after a prolonged induction of labor due to being postdates. After the placental
delivery she continues to bleed excessively. Your initial intervention to address this bleeding is to activate the normal
physiologic mechanisms.
Which of the following is the most important hemostatic mechanism in combating postpartum hemorrhage?

a.contraction of interlacing uterine muscle bundles


b.fibrinolysis inhibition
c.increased blood-clotting factors in pregnancy
d.intramyometrial vascular coagulation due to vasoconstriction
e.markedly decreased blood pressure in the uterine venules

103 . The physiologic changes of pregnancy can alter many of the common laboratory tests. During the evaluation of a
patient with tachycardia, hypertension, and headache you are considering both hyperthyroidism versus an atypical
preeclampsia and draw the following laboratory tests. To correctly interpret the results, it is necessary to distinguish
between normal versus abnormal changes during pregnancy.
Which of the following would normally be expected to increase during pregnancy?

a.alanine aminotransferase (ALT)


b.aspartate aminotransferase (AST)
c.hematocrit
d.plasma creatinine
e.thyroxine-binding globulin (TBG)

104. Worldwide, which of the following is the most common problem during pregnancy?

a.diabetes
b.preeclampsia
c.iron-deficiency anemia
d.heart disease
e.urinary tract infection (UTI)

105. A pregnant woman not previously known to be diabetic, who is at 26 weeks’ gestation, had a routine 50-g (GTT) with a
1-hour blood glucose value of 144 mg/dL. A follow-up 100-g, 3-hour oral GTT revealed plasma values of fasting blood sugar
of 102; 1 hour, 180; 2 hours, 162; and 3 hours, 144.
You should do which of the following?

a.begin American Diabetes Association (ADA) diet and daily glucose monitoring
b.repeat the OGTT in early or mid-third trimester
c.start oral hypoglycemic agents in the diet
d.treat the patient as one with normal gestation.

106. There is good evidence that a woman who gave birth to an infant with a neural tube defect (NTD) can substantially
reduce the risk of recurrence by taking periconceptional folic acid supplementation.
What is the recommended dose?

a.0.4 mg
b.0.8 mg
c.1.0 mg
d.4 mg
e.8 mg
107. A 28-year-old G2P0 at 39 weeks is in early labor. She is 2 cm dilated and 90% effaced, with contractions every 4 to 5
minutes. The fetal heart tones are reassuring. Her nurse steps out for a moment and returns to find her having a seizure.
The nurse administers a 4-g magnesium bolus. The seizure stops. The fetal heart tone variability is flat, but there are no
decelerations. What would your next therapies be aimed at?

a.reducing edema with diuretics


b.giving hypotensive agents until the blood pressure is 110/70 mm Hg
c.prepare for immediate delivery by cesarean section
d.Continuing Magnesium Sulfate ,Stabilizing the Patients condition, and continue for prompt delivery

108. Which of the following will decreased in pregnant woman with severe preeclampsia, as compared with pregnant
women without preeclampsia?

a.response to pressor amines


b.plasma volume
c.total body sodium
d.uric acid
e.serum liver functions

109. 16. A patient at 34 weeks’ gestation develops marked pruritus especially on her palms and soles, and mildly elevated
liver function tests and elevated bile acids.
Which of the following diagnostic possibilities is most consistent with the clinical presentation?

a.pancreatitis
b. hyperthyroidism
c.diabetes insipidus
d. cholestasis of pregnancy
e. progesterone allergy

110. You estimate that the pelvic outlet is adequate, but there may be a problem in the midpelvis.
The interspinous diameter of a normal pelvis should be at least how many centimeters?

a. 5
b. 6–8
c. 9–11
d. 12
e. The interspinous diameter is not a clinically important assessment
Part B:

1. The relationship of which fetal part to the mother's pelvis determines the cephalic presentation?

A.mentum
B.sacrum
C.acromion
D.occiput
E.sinciput

2.A fetus presents in breech position and is delivered without assistance as far as the umbilicus.
The remainder of the body is manually assisted by the obstetrician.
What is this situation called?

(A) version and extraction


(B) spontaneous breech delivery
C) partial breech extraction
(D) total breech extraction
(E) Pipers to the aftercoming head

3.The clitoris is a major sensory sexual organ. Where does it get its major nerve supply from?

a.lumbar spinal nerve


b.pudendal nerve
c.femoral nerve
d.ilioinguinal nerve
e.anterior gluteal nerve

4.How do nabothian cysts occur?

(A) Wolffian duct remnants


(B) blockage of crypts in the uterine cervix
(C) squamous cell debris that causes cervical irritation
(D) carcinoma
(E) paramesonephric remnants

5. Pelvic inflammatory disease (PID) occurs in women because of which of the following characteristics
of the fallopian tube?

(A) It is a conduit from the peritoneal space to the uterine cavity.


(B) It is found in the utero-ovarian ligament.
(C) It has five separate parts.
(D) It is attached to the ipsilateral ovary by the mesosalpinx.
(E) It is entirely extraperitoneal.

6.Anterior vulvar cancer is most likely to spread primarily to which of the following lymph nodes?

(A) inguinal
(B) para-aortic
(C) obturator
(D) femoral
(E) ovarian

7.An 18-year-old woman presents for care because of condom broke during sexual intercourse. Coitus occurred 1 day ago
when she was at midcycle. She does not wish to be pregnant and will terminate the pregnancy if menses does not occur.
Regarding her fear of pregnancy.
which is the most appropriate next step in her management?

(A) advise her that unprotected midcycle coitus has 5% risk of pregnancy
(B) prescribe intravaginal misoprostol (Cytotec)
(C) advise immediate douching
(D) prescribe a brief course of levonorgestrel
(E) advise her to await her next menses before taking any action
8. Over the years the estrogen component of the oral contraceptive pill has been dramatically decreased.
This has in turn minimized certain side effects.
Reducing the estrogen content of OCs has resulted in an increase in the rate of which of the following?

(A) pregnancy
(B) breakthrough bleeding (BTB)
(C) thromboembolic complications
(D) insulin resistance
(E) premenstrual symptoms.

9. Conization of the cervix would be inappropriate in which of the following instances?

(A) when there is disparity between Pap smear and biopsy results
(B) when colposcopy is inadequate
(C) when microinvasion is diagnosed by biopsy
(D) when deeply invasive cancer is shown on a biopsy
(E) for treatment of biopsy-proven CIN III

10. If a nonhealing ulcer is seen on the cervix, it is best evaluated by which of the following?

(A) repeat examination


(B) Pap smear
(C) punch biopsy
(D) cone biopsy
(E) vaginal steroid cream

11. A patient with two previable pregnancy losses has been told that she likely has an incompetent cervix. She asks you to
tell her about this entity.
Which of the following can you correctly tell her?

(A) It is associated with first-trimester spontaneous abortions.


(B) It is easily diagnosed by precise measurement of cervical resistance to dilatation.
(C) It is characterized by painless dilatation of the cervix after the first trimester of pregnancy.
(D) It is inherited as an autosomal recessive disease.
(E) It is primarily treated by medical therapy

12. A patient has just been diagnosed with endometrial cancer by endometrial biopsy. During her
counseling regarding the disease, staging, management, and prognosis the patient is told that most
endometrial cancers are diagnosed as which of the following stages:

(A) I
(B) II
(C) III
(D) IV
(E) recurrent

13.Ovarian neoplasms most commonly arise from which of the following cell lines?

(A) ovarian epithelium


(B) ovarian stroma
(C) ovarian germ cells
(D) ovarian sex cords
(E) metastatic disease
14.Which of the following pubertal events is not mediated by gonadal estrogen production and therefore would occur even
in the absence of estrogen production?

(A) breast development


(B) menstruation
(C) pubic hair growth
(D) accelerated skeletal growth
(E) vaginal cornification

15.Over the years the estrogen component of the oral contraceptive pill has been dramatically decreased.
This has in turn minimized certain side effects.
Reducing the estrogen content of OCs has resulted in an increase in the rate of which of the following?

(A) pregnancy
(B) breakthrough bleeding (BTB)
(C) thromboembolic complications
(D) insulin resistance
(E) premenstrual symptoms

16.A patient with two previable pregnancy losses has been told that she likely has an incompetent cervix.
She asks you to tell her about this entity.
Which of the following can you correctly tell her?

(A) It is associated with first-trimester spontaneous abortions.


(B) It is easily diagnosed by precise measurement of cervical resistance to dilatation.
(C) It is characterized by painless dilatation of the cervix after the first trimester of pregnancy.
(D) It is inherited as an autosomal recessive disease.
(E) It is primarily treated by medical therapy.

17. A 31-year-old infertility patient with regular ovulatory menstrual cycles has begun therapy with clomiphene citrate.
Before she starts therapy.
what information should you provide her regarding the medication?

(A) Typically, the timing of ovulation is increased by a week.


(B) Approximately 40% of patients will respond to clomiphene citrate with increased endometrial thickness.
(C) The risk of multiple gestation is 25%.
(D) Clomiphene citrate improves the fecundity rate principally through its effect on the endometrial lining.
(E) Risk and side effects of clomiphene citrate include nausea, hot flushes, weight gain, and mood swings.

18. An 18-year-old woman presents for care because a condom broke during sexual intercourse. Coitus occurred 1 day ago
when she was at midcycle. She does not wish to be pregnant and will terminate the pregnancy if menses does not occur.
Regarding her fear of pregnancy.
which is the most appropriate next step in her management?

(A) advise her that unprotected midcycle coitus has a 5% risk of pregnancy
(B) prescribe intravaginal misoprostol (Cytotec)
(C) advise immediate douching
(D) prescribe a brief course of levonorgestrel
(E) advise her to await her next menses before taking any action

19 .A patient with hypogonadotropic hypogonadism desires ovulation. What is the initial treatment of choice?

(A) low-dose estrogen therapy


(B) hMG therapy
(C) bromocriptine mesylate
(D) cyclic progesterone
(E) clomiphene citrate
20 .Different sex hormones have different effects on the cervical mucus.
Which of the following statements accurately describes the effect of estrogen?

(A) It decreases the water content of cervical mucus.


(B) It decreases the palm-leaf crystallization pattern of mucus upon drying (ferning).
(C) It decreases formation of glycoprotein channels, which favor sperm penetration.
(D) It increases cervical mucus stretchability (spinnbarkeit).
(E) It increases the amount of potassium chloride in the cervical mucus.

21.A 31-year-old patient is preparing to start in vitro fertilization (IVF) because of obstructed fallopian tubes.
On hysterosalpingogram (HSG), it is noted that she has large dilated hydrosalpinges present bilaterally.
What should be your next step?

(A) The patient should begin her IVF treatment cycle.


(B) The patient should repeat the HSG to confirm the result.
(C) The patient should not be offered the opportunity to have IVF.
(D) Bilateral salpingectomies should be done prior to starting IVF.
(E) Her hydrosalpinges should be drained via transvaginal aspiration prior to starting IVF.

22. A 27-year-old azoospermic male undergoes a testicular biopsy revealing normal seminiferous tubules. He is diagnosed
with hypogonadotropic hypogonadism and receives FSH and human chorionic gonadotropin (hCG) injections.
What is the minimal time required before repeating the semen analysis for spermatogenesis response?

(A) 15 days
(B) 30 days
(C) 60 days
(D) 90 days
(E) 120 days

23. A 43-year-old woman accompanied by her husband reports to you a history of pelvic adhesions and bilateral distal
occlusion of both fallopian tubes with large hydrosalpinges. Both ovaries are buried in thick vascular adhesions.
Adoption is not a consideration for the couple.
What is the most appropriate recommended therapy for this couple?

(A) gamete intrafallopian transfer (GIFT)


(B) IVF using her own eggs
(C) lysis of adhesions and surgical mobilization of the ovaries
(D) ovulation induction using gonadotropins with intrauterine inseminations
(E) IVF using donor eggs

24. During an ultrasound examination to harvest oocytes, the reproductive endocrinologist must be able to identify the
position of the patient’s ovaries.
Which landmark is most helpful in locating the ovaries?

(A) bladder
(B) cul-de-sac
(C) iliac vessels
(D) uterus
(E) rectum

25. A 33-year-old patient has incapacitating midline dysmenorrhea. Cyclic oral contraceptive pills previously had been
unsuccessful and she gets only mild relief from analgesics. She wishes to retain her uterus in hope of becoming pregnant in
the future.
Which of the following current treatment options may be helpful for dysmenorrhea but should be avoided because of
reducing her chances of successful future pregnancy?

(A) diagnostic laparoscopy


(B) continuous oral contraceptive pills (OCPs)
(C) levonorgestrel-releasing intrauterine system (Mirena)
(D) endometrial ablation
(E) depo-lupron
26.What nonpharmacologic treatments are available for the prevention of sudden cardiac death in
patients with ischemic cardiomyopathy?

A. Ventricular assist device (VAD)


B. Implantable cardioverter defibrillator (ICD)
C. Biventricular pacemaker
D. Intra-aortic balloon pump (IABP)

27.Which of the following pharmacologic agents used in the management of heart failure lacks trial
data indicating a mortality benefit and does not prevent maladaptive ventricular remodeling?

A. ACE inhibitors or angiotensin receptor blockers (ARBs)


B. Spironolactone
C. Beta blockers
D. Digoxin

28.Which of the following statements incorrectly characterizes attributes of the medications


to be considered for this patient?

A. Hydrochlorothiazide may exacerbate was hypoglycemia


B. Without a loading dose, the blood level of digoxin will plateau in 7 days
C. Oral bioavailability of loop diuretics varies little from drug to drug
D. Spironolactone has been associated with gynecomastia
E.Nonsteroidal anti-inflammatory drugs (NSAIDs) may cause diuretic Unresponsiveness

29.Which of the following statements regarding the clinical course of CHF and the prognosis of patients
with this condition is true ?

A. Arrhythmias cause the majority of deaths in patients with CHF


B. Signs of chronic right-sided heart failure portend a poorer prognosis
C. A persistent fourth heart sound portends a poorer prognosis
D. Annual mortality increases by 5% to 7% for each NYHA class (i.e., from class I to IV)
E. Once heart failure has developed, sex has no prognostic significance

30. A 55-year-old patient of yours presents for routine follow-up of CHF. An echocardiogram done 6 months ago showed a
left ventricular ejection fraction of 20%. He feels quite well, has unlimited exercise tolerance while exercising on flat
ground, and has dyspnea only with climbing more than two flights of stairs. His current regimen is enalapril, 40 mg/day, and
furosemide, 40 mg b.i.d. On physical examination, his blood pressure is 135/80 mm Hg, his pulse is 88 beats/min, and his
respirations are normal. The rest of his examination is remarkable only for moderate obesity and 1+ pretibial edema.
He has heard that medications called beta blockers could be helpful for people with heart trouble and wonders if he should
be taking them.
Which of the following statements regarding beta-blocker use is true?

A. Beta blockers are contraindicated in patients with CHF


B. Beta blockers are contraindicated in this patient because his ejection fraction is so severely diminished
C. Beta blockers are not indicated in this patient because his blood pressure is already well controlled
D. Beta blockers are not indicated in this patient because his CHF is well controlled by his current therapy
E. Beta blockers are indicated in this patient

31.Which of the following statements regarding renovascular hypertension is true?

A.Renovascular hypertension is an exceptionally rare cause of hypertension in patients with treatment resistant
hypertension
B.Stenosing lesions of the renal circulation cause hypertension throughischemia-mediated activation of the
renin-angiotensin-aldosterone system
C.Fibromuscular disease is an uncommon cause of renovascular hyperttension in patients of this age
D. Atheromatous disease and fibromuscular disease are equally frequent causes of renovascular hypertension
32.Your nurse alerts you that a patient in your clinic has severely elevated blood pressure. The patient is a
45-year-old man without other significant medical history. The patient’s blood pressure is 220/125 mm
Hg; blood pressure measurements are essentially the same in both arms. The patient says that he is
feeling fine. He has had no symptoms of flushing, sweating, or headache, nor has he had visual changes,
focal weakness, numbness, chest pain, dyspnea, or decreased urine output. On examination, neurologic
status is normal. An S4 gallop and trace pretibial edema are noted. The lungs are clear to auscultation.
An ECG shows sinus rhythm with LVH. There is no evidence of ischemia or infarction.
How should you manage this patient?

A. Administer sublingual nifedipine, 10 mg


B. Administer atenolol, 50 mg, and follow up in 24 hours
C. Prescribe atenolol, 50 mg, and follow up in 1 week
D. Admit him to the intensive care unit for cardiac and blood pressure
monitoring and intravenous nitroprusside therapy

33.A 45-year-old man presents with acute onset of flank pain and hematuria. He gives a history of several
months of increasing peripheral edema in his lower extremities.
His urinary protein-to-creatinine ratio is 3.9.
What would be the best step to take next in this patient's management?

A. Renal ultrasound and duplex scan


B. Gram-negative antimicrobial therapy
C. Intravenous furosemide
D. ASO titer

34.Which of the following is true regarding establishment and maintenance of normal sinus rhythm,
as compared with pharmacologic rate control?

A. Establishment and maintenance of sinus rhythm provides no survival advantage


B. Establishment and maintenance of sinus rhythm reduces thromboembolic risk
C. Establishment and maintenance of sinus rhythm improves the degree of symptomatic impairment
D. Conversion to normal sinus rhythm is rarely needed for patients with unstable angina, acute myocardial infarction, heart
failure, or pulmonary edema

35.Which of the following is NOT a risk factor for cardioversion failure in this patient?

A. Duration of AF of longer than 1 year


B. Older age
C. Left atrial enlargement
D. Normal-sized heart

36.Which of the following is NOT an indication for implantation of a cardiac pacemaker?

A. Temporary pacing in the setting of acute myocardial infarction complicated by conduction abnormalities and hemodynam
ic instability
B. Resynchronization in the treatment of heart failure
C. Type I second-degree atrioventricular (AV) block in an asymptomatic athlete
D. Complete AV block
E. Neurocardiogenic syncope with significant bradycardia
37.A 56-year-old man presents to the emergency department with complaint of chest pain of 20 minutes’
duration. The pain is severe, crushing, substernal, and without radiation. He has associated nausea and
diaphoresis without vomiting. He has had no previous episodes of chest discomfort. He has not seen a
doctor for over 20 years and takes no medications. He has smoked two packs of cigarettes a day for the
past 35 years and has lived a sedentary lifestyle. His family history is remarkable for an MI in his father
at 49 years of age. Physical examination reveals a thin man, sitting upright, breathing rapidly on 2 L of
oxygen. His vital signs include the following: temperature, 98.8° F (37.1° C); pulse, 98 beats/min; respir
atory rate, 22 breaths/min; blood pressure, 150/95 mm Hg. Cardiac examination reveals normal rate and
rhythm without murmur, and neck veins are not elevated. Lungs are clear to auscultation. ECG shows
normal sinus rhythm with occasional premature ventricular contractions and ST segment elevations of
0.2 mV in leads II, III, and aVF.
Which of the following interventions is NOT indicated for this patient at this time?

a-mophine
b-lidocaine
c-aspirine
d-metoprolol
e-streptokinase

38.Which of the following findings would be most highly suggestive of significant ischemic heart disease
(IHD) on exercise ECG testing?

A. Chest discomfort before completion of the test


B. Hypertension during the test
C. An S3 heart sound during the test
D. A 0.5 mm ST segment depression during the test

39.A 67-year-old man with type 2 diabetes and a long history of cigarette smoking develops severe exer-
tional chest pain. Cardiac catheterization reveals three-vessel disease. Twenty-four hours later, he devel-
ops abdominal pain, painful toes, and a rash. On examination, he has purple discoloration of the sec-
ond and fourth toes on his right foot and a lacy rash on both legs. Laboratory results are as follows: Hb,
13; HCT, 39; WBC, 9.0; BUN, 26; and Cr, 1.8.
What is the most likely diagnosis for this patient?

A. Contrast nephropathy
B. Aortic dissection
C. Atheromatous emboli syndrome
D. Abdominal aortic aneurismal rupture

40.Which of the following statements regarding DVT is true?

A. Thrombi confined to the calf are large and typically result in pulmonary venous thromboembolism (VTE)
B. The postthrombotic syndrome is a rare sequela of DVT and is associated with low morbidity
C. Most patients presenting with a new DVT have an underlying inherited thrombophilia
D. The most common cause of inherited thrombophilia associated with
this illness is activated protein C resistance (factor V Leiden)

41.Which of the following statements regarding anticoagulation and thrombolysis for thromboembolism is true?

A. When using unfractionated heparin, the therapeutic range for the aPTT (activated partial thromboplastin time) is 2.5 to
3.5 times the normal value
B. Low-molecular-weight heparin (LMWH) is safe and effective for the treatment of pulmonary thromboembolism
C. Because of a delay in achieving a therapeutic INR with lower doses, a starting dose of warfarin should
be no less than 10 mg
D. In contrast to other thrombolytic agents, rt-PA (recombinant tissue plasminogen activator) stimulates antibody
production and can induce allergic reactions
42.Which of the following statements regarding adrenal insufficiency is true?

A. The most common cause of adrenal insufficiency in the United States is tuberculosis
B. The critical test for the diagnosis of chronic adrenal insufficiency is the cosyntropin test
C. Chronic secondary adrenal insufficiency is treated with hydrocortisone and fludrocortisone, whereas chronic primary
adrenal insufficiency is treated with hydrocortisone alone
D. In idiopathic or autoimmune adrenal insufficiency, CT of the abdomen shows enlarged adrenal glands

43.Which of the following statements regarding pheochromocytomas is true?

A. 90% of pheochromocytomas are malignant


B. Almost all patients with pheochromocytomas have episodic hypertension, making diagnosis difficult in a single clinic visit
C. The incidence of pheochromocytoma is increased in patients with multiple endocrine neoplasia type 1
D. Treatment of pheochromocytoma is surgical, alpha blockade should be induced with phenoxybenzamine, beginning 7
days before surgery

44.Which of the following statements regarding hyperaldosteronism is true?

A. The most common causes of secondary hyperaldosteronism are congestive heart failure and cirrhosis with ascites
B. The main treatment of primary adrenal hyperaldosteronism is spironolactone
C. Patients with primary adrenal hyperaldosteronism usually present with hypertension, hypokalemia, and metabolic
acidosis
D. Diagnosis of primary adrenal hyperaldosteronism is confirmed by elevation in the levels of both renin and aldosterone

45. A 54-year-old woman with mild asthma and arterial hypertension has a strong family history of osteoporosis. DEXA scan
results a T score of –2.6. Her creatinine and albumin levels and liver function tests are normal except for a slightly elevated
alkaline phosphatase level; calcium level is 11 mg/dl.
What is the most appropriate next step in the treatment of this patient?

A. Start bisphosphonate, calcium, and vitamin D, and reassess in 6 Months


B. Measure the parathyroid hormone (PTH) level and assess 24-hour urinary calcium output
C. Order CT scans of the chest and abdomen to look for an occult malignancy
D. Start hormone replacement therapy with estrogens and progestins

46.Which of the following descriptions is characteristic of irritable bowel syndrome?

A. Painless diarrhea that occurs during the day or night


B. Abdominal pain with defecation and an altered bowel habit
C. Painless, chronic watery diarrhea of moderate severity
D. Diarrhea associated with postprandial flushing and a drop in blood pressure

47. A 32-year-old woman presents as a walk-in patient in the emergency department. She complains of
nausea and diarrhea that began early that evening. She reports that she ate a sandwich at a fast-food
establishment for lunch, and she began experiencing symptoms several hours later. She denies seeing blood
in the stool. She reports no similar experiences in the past; she has no recent travel history, nor has she
had any contacts with sick persons. She was treated with a 3-day course of antibiotics for an upper
urinary tract infection 2 months ago and is otherwise healthy.
Which organism is the most likely cause of this patient’s acute diarrheal illness?

a. Campylobacter jejuni
b.Salmonella enteritidis
c.Staphylococcus aureus
d.difficile
48.A 26-year-old man presents with intermittent crampy abdominal pain, diarrhea without noticeable
blood, and weight loss of 15 lb over 10 months. The bowel symptoms, including the diarrhea, wake him
from sleep. On a few occasions, he has had fevers, nausea, and vomiting. The patient is an architect, and
he describes his work as being stressful; he resumed smoking cigarettes a year ago. His older brother has
had similar symptoms but has not yet been evaluated. On examination, the patient is a slender man with
normal vital signs. He has an oral aphthous ulcer and poorly localized lower abdominal to midabdominal tenderness
without peritoneal signs. Anal and rectal examinations are normal, and a stool guaiac test
is negative. Stool leukocytes are present. The hematocrit is 34%. Results of examination with flexible
sigmoidoscopy are normal.
Which of the following is the most likely diagnosis for this patient?

a. Irritable bowel syndrome


b.Acute appendicitis
c. Crohn disease
d. Ulcerative colitis
e. Colon cancer

49.Which of the following statements regarding nutritional support is true?

a. Enteral nutrition is less likely to cause infection than parenteral nutrition


b. Parenteral nutrition has consistently been shown to result in a decrease in mortality, compared with standard care
c. The use of oral supplements in hospitalized elderly patients has been shown to be harmful
d. Parenteral nutrition is the preferred mode of nutrition in cancer patients because of its lower incidence of infections

50.Which of the following laboratory findings is the most easily monitored immediate effect of erythropoietin therapy?

a. An increase in the reticulocyte count


b. An increase in the mean corpuscular volume
c. An increase in the hemoglobin level
d. An increase in the mean corpuscular hemoglobin level

51.Which of the following statements about megaloblastic anemia is false?

a.Absorption of cobalamin in the small intestine is dependent on proteins produced in the mouth and stomach
b.Megaloblastic erythropoiesis is characterized by defective DNA synthesis and arrest at the G2 phase, with impaired matur
ation and a buildup of cells that do not synthesize DNA and that contain anomalous DNA
c. In most patients with severe cobalamin deficiency, the neurologic examination is normal
d. Cobalamin deficiency is treated with parenteral cobalamin therapy

52.Which of the following statements regarding autoimmune hemolytic anemia is true?

a. Autoimmune hemolytic anemia typically results in intravascular hemolysis


b. Autoimmune hemolytic anemia may be idiopathic or secondary to disorders such as systemic lupus erythematosuchronic
lymphocytic leukemia (CLL), HIV infection, or hepatitis C infection
c. Most patients with autoimmune hemolytic anemia are cured with steroid therapy
d. Splenectomy is curative for those patients who do not respond to simple steroid therapy

53.A 60-year-old man presents with complaints of headache, lightness, blurry vision, and fatigue; these symptoms have
been increasing over the past month. He reports that he has felt weak and has nothad much energy. He also reports
generalized itching, which usually occurs after he takes a hot shower. Physical examination reveals facial plethora. His
spleen is palpable 2 cm below the left costophrenicangle. Laboratory results reveal the following: Hb, 18; Hct, 61; platelets,
500,000; leukocytes, 17,000.
Which of the following is the most appropriate diagnosis for this patient?

A. Gaisböck syndrome (relative polycythemia)


B. Pickwickian syndrome
C. Polycythemia vera
D. Acute myeloid leukemia
E. Chronic myeloid leukemia
54. 49 years man goes to hospital with severe anemia. Hb 7 mg/dl ,blood transfusion is ordered.
Which of the following statements about blood components is true?

A. Whole blood transfusion would be preferable to red cell transfusion in this patient
B. Leukocyte transfusion reduces febrile transfusion reactions
C. Cryoprecipitate consists of albumin and platelets
D. Single-donor platelet transfusions carry a higher risk of blood-borne infection than platelet concentrates

55. 33 years old man con Crohn disease is transfused with red cells for anemia, 30 minutes later he “doesn’t feel well”.
Which of the following statements regarding transfusion complications is true?

A. Immediate hemolytic reactions are the result of an anamnestic response to an antigen to which the recipient is already
sensitized
B. Delayed hemolytic reactions occur during primary sensitization and can be as severe as immediate hemolytic reactions
C. Until the cause of the hemolytic transfusion reaction is identified, the patient may only receive type O red cells or AB
plasma
D. Fever without signs of hemolysis can be managed with acetaminohen; no further laboratory workup is necessary

56. 30. A 63-year-old multiparous woman is receiving packed red cells to treat symptomatic anemia following surgery.
Fifteen minutes into the transfusion, she has rigors. On physical examination, she appears anxious and diaphoretic; her
temperature is 102.2° F (39° C); the rest of her examination is normal.
What is the first step in the diagnosis and management of this transfusion reaction?

A. Administer acetaminophen or meperidine for symptomatic relief


B. Draw blood for culturing
C. Stop the transfusion
D. Send the untransfused blood back to the blood bank for analysis

57. 31. Which of the following statements about ITP is false?

A. ITP is associated with splenomegaly


B. ITP is associated with HIV infection, acute viral illnesses, and some autoimmune diseases
C. Patients with platelet counts over 50,000/µl do not routinely require treatment
D. Treatment is indicated for patients with platelet counts below 20,000 to 30,000/µl or for those whose platelet count is
less than 50,000/µl with risk factors of bleeding including hypertension, peptic ulcer disease or vigorous life style
E-the disorder is relatively benign –the cause of death in adult pz with this disorder is intracranial hemorrhage

58. 24 years old woman with 1 day of pain and swelling in right leg,had DVT,she receiving oral contraceptives. US
demonstrate DVT in her thigh. helpful test in acute setting to determinate the cause of acute setting to determine the
cause of her suspected hypercoagulability state ?

A. Protein S level
B. Protein C level
C. Activated protein C resistance (Factor V Leiden)
D. AT-III level

59.A 68-year-old man presents with new onset of right-sided DVT without apparent risk factors. Therapy is initiated, and the
possibility of underlying cancer is raised. You are consulted regarding appropriate evaluation for occult malignancy.
What would you recommend for this patient?

A. Careful history, physical examination, routine blood counts and chemistries, chest x-ray (CXR), fecal occult blood testing
(FOBT), and prostate-specific antigen (PSA); if these are not revealing, no further evaluation is necessary
B. Careful history, physical examination, routine blood counts and chemistries, CXR, FOBT, and PSA; if these are not
revealing, proceed with colonoscopy
C. Careful history, physical examination, routine blood counts and chemistries, CXR, FOBT, and PSA; if these are not
revealing, proceed with CT scan of the chest, abdomen, and pelvis
D. Careful history, physical examination, routine blood counts and chemistries, CXR, FOBT, and PSA; if these are not
revealing, proceed with bone scan
60. A 68-year-old man is evaluated for symptoms of fever, weight loss, and dyspnea on exertion. Physical
examination reveals a new diastolic murmur and stigmata of peripheral emboli. He is admitted to the
hospital for further evaluation and management of endocarditis. A cardiac echocardiogram shows a 1 cm aortic valve
vegetation, and two of two blood cultures subsequently grow Streptococcus bovis that is susceptible to penicillin.
After appropriate management of this patient's endocarditis, the patient should undergo which of the following?

A. Glucose tolerance test


B. Chest radiograph to screen for lung cancer
C. Upper endoscopy
D. Colonoscopy
E. No specific further evaluation is warranted

61. A 65-year-old man with poorly controlled diabetes underwent transurethral resection of the prostate
2 days ago. Today he presents with a complaint of scrotal pain. On physical examination, the patient is
somnolent but arousable. His temperature is 101.7° F (38.7° C), his blood pressure is 100/70 mm Hg,
and his pulse is 120 beats/min. His scrotum is markedly swollen, erythematous, and exquisitely tender.
What is the best step to take next in the treatment of this patient?

A. Immediate institution of broad-spectrum antibiotics


B. Immediate institution of broad-spectrum antibiotics and hyperbaric oxygen therapy
C. Immediate surgical exploration and resection without regard to reconstruction
D. Immediate surgical exploration and resection with caution with regard to future reconstruction

62. Which of the following statements about septic (bacterial) arthritis is true?

A. Local inoculation of organisms into the joint space is the most common route of acquisition
B. HIV-infected patients are at increased risk
C. Patients with underlying joint disease (e.g., rheumatoid arthritis) are at increased risk
D. The finger joints are the most commonly involved site
E. Most cases are polyarticular

63. A 57-year-old man presents for evaluation of a left lower extremity ulcer. He has a history of hypertension and poorly
controlled type 2 diabetes mellitus. The lesion began approximately 4 weeks ago in the absence of any known trauma. The
patient reports experiencing subjective fevers, chills, and malaise over the past few days. On physical examination, the
patient’s temperature is 100.4° F (38° C). A nontender stage 3 ulceration of the plantar surface is noted on the patient’s left
first metatarsal, with surrounding erythema and mild discharge.
Which of the following statements regarding osteomyelitis in this patient is true?

A. The most likely reason for osteomyelitis in this patient is hematogenous seeding
B. Prolonged antibiotic therapy alone cures the majority of these patients
C. Infections are rarely polymicrobial
D. Vascular insufficiency impairs wound healing and allows bacterial proliferation
64. A 72-year-old man with a history of hypertension, diabetes mellitus, and aortic stenosis returns to your
clinic 3 weeks after being diagnosed with a viral upper respiratory infection. Today he complains of
continued fever, myalgias, malaise, and night sweats. At the time of initial presentation, the patient was
treated conservatively with acetaminophen and encouraged to maintain oral fluid intake. He denies
having rigors, chest pain, dyspnea, cough, diarrhea, or dysuria. Results of physical examination are as
follows: temperature, 100.5° F (38° C); blood pressure, 156/87 mm Hg; heart rate, 92 beats/min;
respiratory rate, 14 breaths/min; and O2 saturation, 99% on room air. The patient is in no acute distress.
The lungs are clear to auscultation bilaterally. Heart examination reveals a regular heart rate with
a 3/6 systolic ejection murmur that radiates to the carotid arteries bilaterally. There is no skin rash.
The neurologic examination is nonfocal. You are concerned about the possibility of occult infection or
malignancy and admit the patient for workup for fever of undetermined origin (FUO).
A transthoracic echocardiogram demonstrates a 6 mm vegetation on the aortic valve.
Blood cultures from three sites are obtained.
For this patient, which of the following statements about subacute bacterial endocarditis (SBE) is true?

A. Vegetations of more than 5 mm in size are associated with an increased risk of embolization
B. In most patients with SBE, blood cultures will be positive in the absence of previous antibiotic use
C. Over 50% of patients with SBE are afebrile
D. Physical examination findings frequently include clubbing and splenomegaly

65. which of the followings about subacute bacterial endocarditis is true?

a.Vegetation for more than 5 mm are associated with increased risk of emboli.
b.In most patient with SBE blood culture will be positive in the absence of previous antibiotic use.
c.Over 50% of patient with SBE are afebrile.
d.Physical examination finding frequently include clubbimg and splenomegaly.

66.34 year old air condition repairman admitted because of respiratory distress. He report fever chills mildly, productive
cough, myalgia, head ache, nausea. Physical examination show toxic appearing in moderate respiratory distress, T: 39 cₒ HR:
120 beats/m, BP: 124/78, Rr:24. Crackles at both lung base with diffuse wheezes. Laboratory show profound leukocytosis
with left shift, Na: 126.
The treatment of choice ?

a.Penicillin
b.Clindamycin
c.Gentamycin
d.Azithromycin or Levofluxacin

67. 47 year old man with cirrhosis brought due to mental status changes, physical examination he is afebrile, somnolent,
with shifting dullness, with no abdominal tenderness, peripheral WBC 9400, peritional fluid revels WBC 200 with 80% PMN.
Gram stain of the ascetic fluid revels no organisms.
which makes the diagnoses of spontaneous bacterial peritonitis unlikely?

a.Absence of fever
b.Absence of elevated peripheral WBC
c.Absence of abdominal pain or tenderness on examination
d.Gram of ascitis fluide revaling no organisms
e.PMN count in ascetic fluide < 250 cell/mm
68. which of the followings CSF profile is most compatible with acute strep pneumonia meningitis?

a.Normal glucose, normal total protein, normal cell count


b.Decrease glucose, increase total protein, increase cell count with neutrophilic predominance.
c.Normal glucose, increase total protein, increase cell count with lymphatic predominance.
d.Decrease glucose, increase total protein, increase cell count with lymphatic predominance.
e.Normal glucose, increase total protein, increase cell count with red cell.

69.68 year old woman is now in the intensive care unit after taking excess of propranolol, her pulse 35 beats/m, BP: 65/35
she is unresponsive, her skin is mottled, therapeutic option include which of the followings?

a.Dopamine drip
b.Intravenous glucagon
c.Isoproterenol drip
d.Epinephrine drip
e.Intravenous glucagon and epinephrine drip

70. which of the following statements regarding delirium in the elderly is false?

a.In medically ill patient delirium is most commonly associated with acute infection, hypoxemia, hypotension and
psychoactive medication.
b.By definition delirium can be acute or chronic disorder
c.Medication frequently associated with delirium include antiarrhythmic agents, tricyclic antideprassent, neuroleptics
gastrointestinal medication and antihistamines.
d.Patient with delirium can have perceptual disturbances such as hallucination and can have fluctuating level of alertness

71. A 49 year old man was discovered to have blood glucose level of 246 mg/dl during evaluation of an acute
gastrointestinal syndrome subsequently, a diagnosis of diabetes was confirmed by fasting blood glucose level, overall this
patient's clinical picture is consistent with type 2 diabetes.
which of the following is the most reasonable approach for evaluating this patient?

a. Perform a 24 hour urine collection of his creatinine clearance and measure total protein clearance
b. Measure albumin-creatinine ratio on apot urine sample
c. Defer specific assessment because he has just been diagnosed and diabetic nephropathy is unlikely ti develop
d. Measure serum BUN and creatinine concentration
e. Perform renal ultrasound

72. A 52 year old women presenting after experiencing a 4 days of worsening mental status. Her medical history is
unremarkable she takes no medications physical examination shows a somnolent obese women with dry mucus membrane,
neurological examination is non focal, laboratory studies as follows: serum sodium is 128meq/L ,serum potassium is 4.5
meq/L, serum chloride 94 meq/L, serum glucose is 810 mg/dl, renal function is normal.
which is the most appropriate intervention in the care of this patient?

a. Fluid restriction and loop diuretics


b. Administration of hypertonic saline
c. Administration of normal saline and insulin
d. Demeclocycline
73. A 60 years old man has a history of hypothyroidism and mild asthma his medications include levothyroxine and
albuterol. The patient has been smoking 2 packs of cigarets per day for 30 years, physical examination is unremarkable,
laboratory studies: serum sodium is 126 meq/L, serum potassium 3.8 meq/L, serum chloride 96 meq/L, bicarbonate 24
meq/L BUN 6 mg/dl, glucose and creatinine were normal, serum osmolarity 258 moms/L, urine sodium 56 meq/L, urine
osmolarity 360 moms/kg, thyroid function tests were normal.
Which of the following interventions is most likely to normalize this patient's sodium level?

a. Increasing the dose of levothyroxine


b. Restricting fluid intake
c. Administration of intravenous normal saline
d. Starting a thiazide

74. A 45 years old man with a history of alcohol abuse was found lying on floor. On examination je is unresponsive and
dehydrated, blood work reveals hematocrit of 41% BUN 60 mg/dl ,creatinine 3.2 mg/dl, creatine kinase and LDH levels
elevated at 12,000 U/L and 475 U/L respectively . urine analysis shows reddish urine with specific gravity of 1.020:dipstick
assay shows 3+ blood and 1+ protein, microscopic examination of urine sediment demonstrates 0-2 RBCs and 0-5 white cells
per hight power field, hyaline casts was also observed, urinary findings was suggestive of which of the following ?

a. Acute glomereulonephritis
b. Hemoglobinuria
c. Allergic interstitial nephritis
d. Myoglobinuria

75. A 45 years old man, history remarkable for bleeding peptic ulcer at age 30 that required transfusion of packed RBCs he
has been without peptic symptoms since then he consumes no alcohol , physical examination is unremarkable, laboratory
studies is remarkable for ALT and AST that were 2.5 times normal and dipstick that was positive for blood and protein.
Which of the following is the most appropriate next step in the management of this patient?

a. Referral for lever biopsy


b. Renal arteriogram
c. Helicobacter pylori antibody testing
d. Serum renin measurement
e. Serum antibody testing for hepatitis C

76. A 21 years old women presents with hypertension, fatigue and microscopic hematuria, a renal biopsy demonstrates
glomereulonephritis secondary to FSGN.
Which of the following is the most appropriate next step in the management of this patient?

a. Renal diagnosis
b. Cyclosporin
c. Prednisone
d. Cyclophosphamide
e. Captopril

77. Mouth ulceration is associated with which of the following types of withdrawal ?

a.alcohol
b.Benzodiazepins
c.Cocaine
d.Nicotin
e.Opoids

79. Postpartum psychosis:

A. Occurs more commonly in multigravida women


B. Rarely correlates with perinatal complications
C. delivery almost always begins within 8 weeks of
D. Usually occurs abruptly, with no prodromal psychotic symptoms
E. Is essentially an episode of a psychotic disorder
80.Depression and mania share which of the following symptoms:

A. Psychomotor acceleration
B. low self esteem
C. Grandiosity
D. anger
E. Pessimism

81. A hypomanic episode differs from a manic episode in that a hypomanic episode:

A. Lasts at least 1 week


B. lack psychotic features
C. Is sever
D. Causes greater social impairment
E. All of the above

82. The highest suicide rates are in which of the following age group:

A. Under age 15
B. 15 to 24
C . 25 to 44
D. 45 to64
E. Older than age 65

83. A 36 y o pregnant woman is admitted to the medical center for possible infection associated with i.v heroin injection.
She admits to heroin dependency. She is 5 month into her pregnancy and would like to quit the heroin for good.
What is the best course of action in this patient?

A. Allow her to continue


B. methadone maintenance
C. Naltrexone
D. Advice her to stop heroin and start supportive therapy
E. Buprenophine

84.The following may precipitate mania except:

A. Carbamezopine
B.disulfirm
C. Isoniazide
D. Propranolol

85.A 35-year-old woman has just been diagnosed with major depressive disorder. For the past 8months, she has had a
depressed mood, decreased energy and concentration, and loss of interest in previously enjoyed activities. Although she
never attempted suicide, she acknowledges that she thought she would probably jump off a local bridge if she ever had the
chance. She denies any history of excessively elevated moods. You decide to start her on antidepressant therapy.
Two weeks later, this patient is at greatest risk for:

a.extrapyramidal symptoms
b.hypomanic episode
c.manic episode
d.medication noncompliance
e.suicide completion
86. Tourett’s disorder has been shown to possibly have a familial and genetic relationship with:

a.panic disorder
b.OCD
c.generalized anxiety disorder
d.social phobia
e.none of the above

87. early prognosis of anorexia nervosa?

a.early onset

88. A patient with somatization disorder:

A.has had physical symptoms for only 3 months


B.usually experiences minimal impairment in social or occupational functioning
C. may have a false belief of being pregnant with objective signs of pregnancy, such as decreased menstrual flow or
amenorrhea
D. presents the initial physical complaints after age 30 years
E. has complained of symptoms not explained by a known medical condition

89. Most indicative of Violence in bipolar/mania patient?

c.history of violence

90.What is the “gold standard” to diagnose munchausen by proxy?

A.the child will confess


B.you find disease producing subjects in the care giver belonging
C. By witness the caregiver causing harm to the child
D. the child improve when removed from caregiver

91.Anorexia nervosa has mortality rate of up to approximately:


A. 1%
B. 18%
C. 30%
D. 42%
E. 50%

92.Most common case of delirium within 3 day of postoperative in a 40 year old man with history of alcohol
dependence?

a.Delirium tremens
b.Infection
c.Pain medication postoperative
d.Pain stress of surgery

93. A 14 years old girls present to her pediatrician complaining that she had been “freaking out" the girl dexribes episodes
of shaking gasping for air. And feeling like she in going to die. The feelings in tensiy for a few minutes and resolve
spontaneously. These episodes have occurred various times in various
Situation and the gil is worried she is going crazy . A complete history and physical does not reveal any farther relevant
symptoms or signs .
Which of the following is the most likely diagnosis of this patient:

A. dysthymic disorder
B. social phobia
C. separation anxiety disorder
D. panic disorder
E. acute schizophrenia like psychotic disorder
94.A pervasive pattern of grandiosity lack of empathy and need for administration suggest the diagnosis of which of the
following personality disorder:

A. borderline
B. narcissistic
C. paranoid
D. passive aggressive
E. schizotypel

95. Antidepressants should be used cautiously in cardiac patients because of increased risk of which of the following?

A. Conduction side effects


B. Hypertension
C. Noncompliance
D. Suicide

96 .Suicide among schizophrenic patients:

a. is most frequently secondary to command hallucinations


b. occurs most often in the later years of the illness
c. occurs most often in older female patients
d. is approximately 10 percent
e. is low

97.A36 year old female patient with schizophrenia present to the emergency department after experiencing auditory
hallucinations commanding her to kill herself. she reports that the hallucinations have become more frequent over the
course for the past week .the patient has previously attempted to take her life an tow occasions when she overdosed on
sleeping pills .she is also an alcoholic and recently divorced .her husband of 4 years.
of the followings factors , which poses the greatest risk factor for suicide in this patient?

A .alcohols
B .divorce
C .age
D .gender
E .previous suicide attempts.

98.Which of the following disorders is most closely associated with thyroid disease???

A .schizophrenia
B .alcoholism
C .depression
D . phobia
E .borderline personality disorder

99. Which of the following factors is common in victims of elder abuse?

A . alcohol abuse
B . social isolation
C . mental illness
D . hostility
E . prescription drug abuse
100.A45 year old women with history of dysthymic disorder recently started taking sertraline 75 mg/day she now complains
of sudden onset of myclonus ,diaphoresis ,diarrhea, shivering and worsened anxiety, on physical examination she has a low
grade fever and exhibitis heper reflexia ,further history reveals she has been taking st.johns wort valerian and multivitamins
over the counter for the past 9 month.
what is the most likely diagnosis ?

A . Nms
B . malignant heperthermia
C . lehhal catatonia
D . serotoin syndrome
E . anticholinergic toxicity

101. Which of the following is not associated with an increased intensity of benzodiazepine withdrawal symptoms?

A . abrupt dicontinuation of benzodiazepines


B . use of short half life benzodiazepines
C . prolonged treatment with benzodiapines
D . higher doses of enzodiapines
E . gradual taper of benzodiapines

102. An 18 y.o arrived to ED after assaulting a police officer in the street…generalized muscle weakness ataxia, nystagmus,
slurred speech ...…. what did he take?

A. Inhalant
B. Benzodiazepines
C. Opioids
D. Hallucinogens
E. Amphetamines

103.positive factor in anorexia nervosa:

a.younger age
b.chronicty
c.self vomiting
d.binge eat

104. 43 years old a man with bipolar, enter the ER with agitation which the grievous risk factors for violence:

a. Diagnose of bipolar
b. cocaine
c. acute mania
d. persecutory delusion

105. which disease has the highest rate of mortality from natural or unnatural causes:

a. schizophrenia
b. Anorexia nervosa
c. major depressive disorder
d. bipolar disorder
e. panic disorder
106. after week of treatment with intramuscular of haloperidol a 26 old women with schizophrenia develop confusion ,
fever, high wbc levels ,muscle rigidity and dark urine .
what is the less effective procedure ?

a. Prescribe antibiotic for the presumed infection at the ingest site


b.consider a trial of dantrolene
c.medial admission fluid
d. closely monitor vital sign and renal function

107.a 24 year salesperson is referred for an evaluation of difficulty dealing with costumers because she gets " so nervous
and anxious that mind gets blank ". A careful evaluation revels that she has social phobia. she is motivated to obtain
treatment and continue doing her job. All of the following are effective except:

a.selective serotonin reuptake inhibition – SSRI


b.flooding
c.modeling
d.systemic desensitization
e.electrical convulsion therapy- ECT

108. the term double depression is used to describe:

a.a major depressive disorder superimposed on grief reaction


b. major depressive disorder superimposed on dysthymia
c. major depressive patient not responding to treatment
d.major depressive disorder psychosis
e.major depressive disorder with anxiety

109. A 56 year old anxious executive calls at 5:00 because he woke up with chest tightness and shortness of breath , he was
scared than e was having a heart attack and he also reported feeling dizzy , trembling and his heart was pounding , he
thought he was going to die . The symptoms subsided in 20 minute and the time the emergency medical services arrived he
was sitting in his living room and in no distress. The patient's ECG reading was normal and his cardiac enzymes were also
found to be normal.
What is the most likely diagnosis?

a.nocturnal panic attack


b.PTSD
c. psychophyisological insomnia
d.somatization disorder
e. myocardial infraction

110. Which best predictor for suicide likelihood?

A .alcohol
B .gender
c. previous suicidal behavior
d. divorce
e. unemployment
State Exam 18.02.2013
By: Alhag Abu Anzeh Muhammad

Part A:

1. The breast fed infant of a mother who is a strict vegetarian may experience deficiency of which of the following vitamins if the
mother is not receiving supplements of the vitamin?

A. Vit B1
B. Vit B4
C. Vit B12
D. Vit C
E. Vit D

2. A 6-year-old girl who was previously healthy presents with a 1 week history of nocturnal perianal itching. There are no other
symptoms and findings on physical examinations are normal. The most appropriate therapy is:

A. Bacitracin ointment to the perianal area


B. Diphenhydramine orally as needed for itching
C. Single oral dose of mebendazole repeated in 2 weeks
D. A 2 week course of amoxicillin/clavulanate
E. Ketoconazole in a single dose

3. A child suffers a provoked bite from a stray dog that was captured by animal control and appears healthy. The most appropriate
action would be to:

A. Confine and observe the dog for 10 days for signs suggestive of rabies
B. Submit the dog's head for examination for rabies
C. Begin rabies vaccination
D. Administer human rabies immune globulin (HRIG) and begin rabies vaccination
E. None of the above because it was a provoked attack

4. A mentally retarded 14-year-old boy has long face, large ears, micropenis and large testes. Chromosomal analysis is likely to
demonstrates which of the following?

A. Trisomy 21
B. Trisomy 18
C. Trisomy 13
D. Fragile X syndrome
E. Williams syndrome

5. A 10-year-old boy is examined because of recurrent headaches. The headaches started 6 months ago and occur about once a month.
He is asymptomatic between episodes. Each headache begins with blurry vision and abdominal pain, followed by right-sided, throbbing
pain. It lasts about 60 minutes, during which he feels better if he takes some ibuprofen and rests in a darkened room. The most likely
diagnosis is:

A. Brain abscess
B. Seizure disorder
C. Migraine
D. Todd's paralysis
E. Maxillary sinusitis
6. A 2-year-old boy is seen in your office because of fever, ear pain, and postauricular swelling, erythema, and tenderness. The pinna
protrudes out on the involved side. The tympanic membrane is red and bulging, with decreased mobility seen on pneumatic otoscopy.
The angle of the jaw is easily palpated and the opening to Stensen’s duct appears normal. The patient has never had an MMR vaccine.
The most likely diagnosis is:

A. Bacterial parotitis
B. Mumps
C. External otitis
D. Acute mastoiditis
E. Chronic mastoiditis

7. New parents ask you how to reduce the chance of their baby suffering from sudden infant death syndrome (SIDS). You tell them to
place the child in which of the following for sleep?

a. Supine position
b. Prone position
c. Seated position
d. Trendelenburg position
e. A hammock

8. An 8-year-old girl is brought to the hospital while actively seizing. She has been hospitalized many times before for status epilepticus.
She is receiving valproic acid at home to control the seizures. The first step in the management of this patient is to:

A. Administer 20 mL/kg 0.9% normal saline


B. Establish secure intravenous access and administer an anticonvulsant
C. Administer activated charcoal via NG tube
D. Stabilize airway and provide 100% oxygen
E. Perform gastric lavage

9. A 5-year-old boy presents with a history of grossly bloody urine, puffy eyes, and headache for one day. He has been a well developed
child, but he did have a fever and sore throat about 10 days ago which resolved without treatment. The most likely diagnosis is:

A. Acute cystitis
B. IgA nephropathy
C. Acute pyelonephritis
D. Postinfectious glomerulonephritis
E. Benign hematuria

10. A 3-yr-old boy presents to an urgent care clinic with a 3-day history of abdominal pain and difficulty walking. Abnormal findings
include blood pressure of 120/80 mm Hg, diffuse abdominal tenderness, purpuric rash of the hands and ankles, and diffuse periarticular
tenderness and swelling of the ankles. The most likely diagnosis is:

A. Systemic lupus erythematosus


B. Kawasaki's disease
C. Juvenile rheumatoid arthritis
D. Henoch-Sch nlein purpura
E. Stevens-Johnson syndrome
11. What is the most significant serious complication arising from Kawasaki disease?

A. Coronary aneurism
B. Kidney failure
c. Stroke
d. Pulmonary embolism
e. Acute leukemia

12. A 12 months old male infant Mediterranean origin noted to have pallor and has been fed similac with iron since birth ,his exam
otherwise normal except for palpable spleen .HB:9.9 / MCV:67/MCHC:32/RDW:12/Reticulocyte 1.5% /PLT:240000 which one is true?

a. Hb level of 9.9 gm/dl is the lower limit of normal for this 12 months old
b. The dietary history is probably not true since the patient is iron deficient
c. The reticulocyte is high suggest hemolytic process
d. The infant must he losing blood and stool should be checked for occult blood
e. Hg electrophoresis should make the correct diagnosis

13.15 year old complaining of fever, abdominal pain, sexually active with 2 partner complaint of dyspareunia on examination yellow
thick vaginal discharge but no visible genital lesion most probably infected with:

a. H.influenza
b. Candida albicans
c. Chlaymidea palledum
d. Trepenoma
e. Human papilloma virus

14. Parents awakened at night by 2 years old son develop noisy breathing inspiration ,marked retraction of chest ,flaring nostril ,barking
cough has mild upper respiratory tract infection 2 days ago. Which the most likely diagnosis:

a. Asthma
b. Epiglottitis
c. Bronchilitis
d. Viral croup
e. Foreign body in RT bronchus

15. The most important extra medullary site for relapse in childhood acute lymphatic leukemia (ALL):

a. Adrenal gland
b. Kidney
c. Lung
d. Heart
e. Central nervous system

16. Newborn develops sepsis and shock which pathogens most likely can system or focal infection of newborn:

a. Staph aureaus
b. Group A strept
c. Group B strep
d. E.coli
e. Herpes simplex virus
17. A 4 years old girl seen 10 days following upper respiratory infection ,her knees and ankle swollen and painful and temp 39 C ,cardiac
examination 4/6 systolic murmur at the apex, antistreptolysin titer high. These findings consist with:

a. Acute rheumatic fever


b. Septic arthritis
c. Juvenile idiopathic arthritis
d. Viral pericarditis
e. Acute leukemia

18.12 years old boy presented with 24 h history of sharp pleuritic chest pain worsen in supine position, temperature 38.5 C, pericardial
rub is heard. The most likely diagnosis is:

a. Muscloskletal chest pain


b. Pericarditis
c. Bacterial endocarditis
d. Mycoplasma pneumonia
e. Pulmonary embolism

19. The most common cause of syncope in childhood:

a. Tachycardia ass with wolf parkinsonian white syndrome


b. Long Q-T syndrome
c. Breath holding spell
d. Hypertrophic cardiomyopathy
e. Neurocardiogenic syncope

20. A 6 weeks boy known congestive heart failure due to cardiomyopathy Wg 4 Kg taken 360 ml formule daily ,which of the following
most likely consequence of decrase intake in this infant:

a. Hypocalcemia
b. Hypoglycemia
c. Hypokalemia
d. Poor growth in length
e. Poor weight gain

21. The stinging insect most likely to cause an anaphylactic reaction in a child is:

A. Mosquito
B. Sand fly
C. Honey bee
D. Ant
E. Head lice

22. The chest x-ray of a 3-day infant with congenital heart disease demonstrates an abnormally shaped heart and no thymic shadow.
What immunodeficiency should you suspect?

A. Ataxia-telangiectasia
B. X-linked hyper-IgM syndrome
C. Wiskott-Aldrich syndrome
D. DiGeorge syndrome
E. Leukocyte adhesion deficiency
23. A 12 month-old infant was diagnosed with X-linked agamaglobulinemia, a recessive B-cell deficiency after having had multiple
sinopulmonary tract infections such as otitis media, sinusitis, and pneumonia. What is an appropriate treatment for him?

A. Plasmaphoresis
B. Intravenouse immunoglobulin
C. Chemotherapy
D. High dose steroids
E. Enzyme replacement therapy

24. A mother of a 6-year-old girl states that her daughter's academic performance has decreased during the last year. Her teacher
notices her staring frequently throughout the day. Sometimes she seems "off in her own world" and does not respond to questions. An
EEG examination reveals a 3-Hertz generalized spike and wave pattern. What seizure type is this patient most likely to have?

A. Absence
B. Tonic–clonic
C. Myoclonic
D. Salaam attacks
E. Complex partial

25. A 2.5 year old boy is seen because of a second febrile seizure. The two episodes occured 4 months apart and described as
generalized tightening of the body followed by jerking involving all four extremities and lasting 3 to 5 minutes each time. On both
occasions the temprature was 38.8°C. At this time, your recommendation would be:

A. Further tests including EEG and MRI scan


B. Treatment with phenobarbital
C. Treatment with valproate
D. Admit to hospital for cardiac respiratory monitoring
E. Education and counseling for the family

26. The skills of kicking a ball and jumping in place are gross motor milestones that occur at which age?

A. 15 months
B. 18 months
C. 24 months
D. 30 months
E. 36 months

27. Based on orientation to child development, when would you tell parents the highest risk of accidental poisoning in children is
present?

A. 6 months
B. 1 year
C. 2 years
D. 4 years
E. 6 years

28. Which of the following is a frequent manifestation of enterovirus infection?

A. Ataxia
B. Intussusception
C. Parotitis
D. Herpangina
E. Renal failure
29. A 6-year-old girl is hospitalized after a convulsion. On examination she is alert and without distress. On her skin you notice the
presence of cafe-au-lait spots, a Shagreen patch, and subungual fibromas. What is the most likely diagnosis?

A. Neurofibromatosis
B. Sturge–Weber syndrome
C. McCune-Albright syndrome
D. Addisson's disease
E. Tuberous sclerosis

30. A 15-month old male is brought to the pediatrician because he seems much smaller than his two older brothers were at that age. He
has been generally healthy except for 2 episodes of otitis media and occasional "cold". He began walking at 11 months and can now say
"mama" "dada" and names of his brothers. What is the most likely appropriate next step?

A. Perform a Danver Development screening test


B. Ask the mother to complete a 3 day diary of all the food that the child has eaten
C. Send blood for quantitive immunoglobulins
D. Plot his height and weight on growth chart and compare to previous charts
E. obtain a sweat chloride test

31. A 1-month-old infant is seen because of prolonged jaundice. He was born at home after a normal pregnancy. On examination you
notice a jaundice of the skin. His liver is not enlarged. There is a white pupillary reflex in both eyes, and the urine examination is positive
for reducing substances. What is the most likely diagnosis?

a. Sepsis
b. Glucose-6-phosphate dehydrogenase deficiency
c. Phenylketonuria
d. Viral hepatitis
e. Galactosemia

32. Which of the following statements about neuroblastoma is true?

a. Neuroblastoma is a benign tumor of the neural crest cells that form the adrenal cortex and the paraspinal parasympathetic ganglia
b. The majority of neuroblastoma tumors occur in the thoracic cavity
c. Neuroblastoma is the most common solid malignant tumor in infancy
d. In neuroblastoma of the abdomen, displacement of the kidney and distortion of the calyceal system often occurs
e. Most patients are treated with surgery alone because distant metastases are rare

33. No red reflex is seen on fundoscopic examination of a newborn. Which is the most likely diagnosis?

a. Retinoblastoma
b. Congenital cataract
c. Pigmentary keratitis
d. Congenital glaucoma
e. Toxocariasis
34. A neonate born at 28 weeks' gestation is now 2 weeks of age. Nasogastric feeds are started. Forty-eight hours after starting feeds,
the neonate develops a distended abdomen, bloody stool, pneumatosis intestinalis, and free air on abdominal radiograph. Laboratory
studies reveal thrombocytopenia. The child becomes persistently hypotensive despite maximal medical therapy. The most likely
diagnosis is:

a. Necrotizing enterocolitis
b. Sepsis
c. Aspiration pneumonia
d. Malrotation
e. Jejunal atresia

35. A 12-year-old male adolescent presents with a 1-month history of fever, weight loss, fatigue, night sweats, and pain andlocalized
swelling of the midproximal femur. Which of the following is the most likely diagnosis?

a. Ewing's sarcoma
b. Osteosarcoma
c. Chronic osteomyelitis
d. Benign bone tumor
e. Eosinophilic granuloma

36. An afibrile 5-year-old girl presents with tachycardia at 220 beats per minute. On ECG a regular narrow-complex tachycardia is seen.
The rhythm converts with one dose of adenosine intravenously to normal sinus rhythm with pre-exitation (delta waves) noted
throughout the precordial leads. There is no cardiomegaly on chest X-ray. The tachycardia is most likely consistent with:

a. Long Q-T syndrome


b. Wolf-Parkinson-White syndrome
c. Sinus tachycardia
d. Atrial flutter
e. Atrial fibrillation

37. A 5-year-old boy presents with prolonged fever and a new 1/6 systolic ejection murmur heard best at the right upper sternal border.
On extremity examination, splinter hemorrhages and petechia are noted. Which of the following is the most likely diagnosis based on
clinical description?

a. Endocarditis
b. Rheumatic heart disease
c. Kawasaki disease
d. Pericardial effusion
e. Dilated cardiomyopathy

38. A 6-week-old breast fed infant is seen appearing quite well. His mother states that for the last week the infant has had numerous
periods of inconsolable crying lasting few hours each. Nothing seems to help. Most of the spells occur in the late afternoon and evening
and between the episodes the infant looks and feeds well. What is the most likely diagnosis?

a. Otitis media
b. Intussusceptions
c. Milk protein intolerance
d. Colic
e. Celiac disease
39. A previously healthy 2-wk-old now has progressive lethargy. Physical examination reveals muscle rigidity, opisthotonos posture,
periods of hypertonicity, and flaccidity. Laboratory data reveal Hypoglycemia, metabolic acidosis, and cerebral edema. Plasma levels of
leucine, isoleucine, and valine are elevated. The most likely diagnosis is:

a. Hartnup disease
b. Maple syrup urine disease
c. Phenylketonuria
d. Homocystinuria
e. Galactosemia

40. Short stature and growth failure may be the presenting complaints for which of the following conditions?

a. Juvenile idiopathic arthritis


b. Insulin-dependent diabetes mellitus
c. Crohn's disease
d. Acute leukemia
e. Familial Mediterranean Fever

41. A 16 year old female patient present with short stature and no secondary sexual characteristics. Which diagnosis should be mostly?

a. Turner syndrome
b. Isolated growth hormone deficiency
c. Cushing disease
d. Familial short stature
e. Addison disease

42. A 3-year old girl is diagnosed with new onset insulin dependent diabetes mellitus. Which of the following laboratory findings is
consistent with diabetic ketoacidosis?

a. Hypoglycemia
b. Hypercarbia
c. Ketones in urine
d. Increased venous blood pH
e. Decreased BUN

43. An infant who was born at home presents to your office at 3 days for check up. The teenaged mother did not receive prenatal care.
You notice bilateral purulent discharge from the eyes of the baby. There is marked eyelid edema and conjunctival swelling. What is the
most likely pathogenic agent?
a. Chlamydia trachomatis
b. Neisseria gonorrhea
c. Group B streptococcus
d. Toxoplasma gondii
e. Treponema pallidum

44. Which of the following clinical presentation is most consistent with an infant with pyloric stenosis?

a. Projectile non bilious emesis


b. Bilious emesis
c. Bloody diarrhea
d. Violent episodes of intermittent colicky pain and emesis
e. Right lower quadrant abdominal tenderness
45. A 4-year old boy presented with a 5 day history of generalized edema. On examination you notice puffy eyes, scrotal edema and
ascites. Urinalysis reveals a specific gravity of 1.020,PH-7.0, 4+ proteinuria and is otherwise unremarkable. Serum cholesterol is 648
mg/dl (elevated, albumin is 2.3 g/dl(low), C3 is 83 mg/dl (normal) and serum creatinine 0.3 mg/dl. What is the next step in managing
this child?

a. Perform kidney biopsy


b. Begin therapy with amoxicillin
c. Begin therapy with oral prednisone
d. Admit the patient for intravenous fluids
e. Start peritoneal dyalisis

46. A 19-year old female you have followed for 10 years in your practice has recently married, she and her 22-year old husband are
planning to start a pregnancy. You advice her to start taking folic acid. This is important to prevent:

a. Prematurirty
b. Skull defects
c. Chromosomal defects
d. Neural tube defects
e. Osteopenia

47. A-6 day old infant is reported to have an abnormal screening test for congenital hypothyroidism, the most likely etiology for this
result is:

a. Maternal graves' disease treated with propylthiouracil


b .Maternal antithyropin antibodies
c. Iodine deficiency
d. dysgenetic thyroid gland
e. The newborn screen was performed at 12 hours of age

48. A 3-year old boy is brought to the emergency room with complaint of persistent rhinorrhea for the past 6-weeks. Otherwise the
patient has been asymptomatic on examination, you note that the patient has mouth breathing and has dark circles under his eyes. In
the nose you find watery discharge and edematous, swollen, bluish mucous membrane without erythema. The most likely diagnosis is:

a. Chronic upper respiratory infection


b. Sinusitis
c. Nasal foreign body
d. Allergic rhinitis
e. CSF leak

49. A previously healthy 18-month-old has been playing in a separate room from his family. The family notes the sudden onset of
coughing which resolved in a few minutes, subsequently the patient appears to be normal except for increased amounts of drooling and
refused to take foods orally. Which of the following is the most likely explanation for this toddler's condition?

a. Severe gastro esophageal reflux


b. Foreign body in the air-way
c. Croup
d. Epiglottitis
e. Foreign body in the esophagus
50. A previously healthy and fully immunized 13 year-old boy presents with 2-week history of nonproductive cough and low-grade fever.
On examination you note normal respiratory rales are deleted at the bilateral lung bases. Which of the following is the most likely cause
of pneumonia in this adolescent?

a. Pneumocyctis carnii
b .Staphylococcus aureus
c. Group B streptococcus
d. Haemophilus influenza type B
e. Mycoplasma pneumonia

51. A 27-year-old man presents to the emergency department after receiving blows to the head. He opens his eyes with painful stimuli,
is confused, and localizes to pain. What is his glasgow coma score?

A. 13
B. 11
C. 9
D. 7

52. A 45-year-old otherwise healthy woman presents after a motor vehicle accident. She is hemodynamically stable and only minimal
tenderness in her right upper quadrant. A FAST exam (focused abdominal sonography for trauma) is positive with fluid seen in the
hepatorenal fossa and the pelvis. Which of the following is the next best step in her management?

A. Observation only
B. CT scan
C. Laparoscopy
D. Exploratory laparotomy

53. The ideal time to administer prophylactic antibiotics to a patient undergoing colon resection is:

A. 8 hour before surgery with a dose repeated at the time of incision


B. 2 hour before surgery with a dose repeated at the time of incision
C. 1 hour before surgery
D. At the time of incision

54. Tumor staging for most epithelial cancers includes all of the following except:

A. Tumor size
B. Tumor mutations
C. Nodal involvement
D. Distant spread

55. Which of the following conditions increases a woman's risk of breast cancer?

A. Sclerosing adenosis
B. Fibroadenoma
C. Atypical lobular hyperplasia
D. Intraductal papilloma

56. Which of the following is an indication for cholecystectomy in an asymptomatic patient with an incidental finding of gallstones?

A. Any history of abdominal pain


B. Family history of complications of cholelithiasis
C. Porcelain gallbladder
D. Frequent travel out of the country
57. A 48-year-old patient presents with sudden onset of bilateral lower abdominal pain after spasmodic coughing. On examination,
there is a 8-cm, tender mass in the mid lower abdomen that remains unchanged with contraction of the rectum muscles. Which of the
following is the most likely diagnosis?

A. Ruptured aortic aneurysm


B. Obturator hernia
C. Spigelian hernia
D. Rectus sheath hemathoma

58. Following documentation of a firm mass in the testes by ultrasound in a 32-year-old male, tissue should be obtained for diagnosis
by:

A. Fine-needle aspiration
B. Core-needle biopsy
C. Open biopsy
D. Orchiectomy

59. A 75 year-old man with a history of myocardial infarction 2 years ago, peripheral vascular disease with symptoms of claudication
after half block, hypertension, and diabetes presents with a large ventral hernia. He wishes to have the hernia repaired. Which of the
following is the most appropriate next step in his preoperative workup?

A. A normal electrocardiogram (ECG) precludes the need for further cardiac testing
B. He should undergo an exercise stress test
C. He should undergo coronary artery bypass prior to operative repair of his ventral hernia
D. He should undergo a persantine thallium stress test and echocardiography
E. His history of myocardial infarction within 3 years is prohibitive for elective surgery. No further testing is necessary.

60. A 40-year-old man undergoes an appendectomy for acute appendicitis. Final pathology reveals a 2.5-cm carcinoid at the tip of the
appendix. Lymph nodes are negative. Which of the following is true about this condition?

A. No further treatment is necessary


B. There is a significant chance that carcinoid syndrome will develop in the patient
C. The patient should receive chemotherapy
D. The patient should undergo re-exploration and a right hemicolectomy
E. Most appendiceal carcinoids are 2.5 cm or larger when discovered

61. A 45 year-old woman undergoes excisional biopsy for a mammographic abnormality. Final pathology reveals benign pathology
without atypia however, several foci of lobular carcinoma in situ (LCIS) are detected at the excision margins. Which of the following
choices would be the best management option?

a. Excisional biopsy to achieve negative margins


b. Mastectomy with sentinel lymph node biopsy
c. Radiation therapy
d. Bilateral total mastectomies with immediate reconstraction
e. Close clinical and mammographic follow-up
62. A 28-year old lactating woman presents with a 2-day history of breast pain that is progressively worsening. On examination, a 4-cm
area of skin adjacent to the nipple is red and tender with some edema and no detectable fluctuance.
Management would consist of:

a. Fine-needle aspiration
b. Skin biopsy
c. Incision and drainage
d. Oral antibiotics
e. Mammography

63. A 60year-old man presents with obstructive jaundice, acholic stools, and weight loss. An ultrasound scan demonstrates a dilated
biliary tree and no gallstones. A dynamic contrast-enhanced CT scan demonstrates a mass localized to the head of pancreas without
evidence of distant metastasis, adenopathy, or vascular invasion. The patient is otherwise in good health. Which of the following best
describes the role of preoperative percutaneous biopsy of the mass in this setting?

a. It is highly sensitive
b. It is nonspecific
c. It is associated with a high complication rate
d. It should be routinely performed
e. It is unnecessary

64. A 4-week old infant presents to the emergency department with increasingly projectile vomiting that is nonbilious and a palpable
small right upper quadrant mass. The infant has severe hypochloremic-hypokalemic metabolic alkalosis. Which of the following is true?

a. The condition is more common in females


b. The urine pH will likely be acidic
c. Emergency surgery is indicated
d. Ultrasonography is always necessary to confirm the diagnosos
e. Surgery will likely require a gastrointestinal bypass

65. Two months after splenectomy for ITP, the patient is noted to have petechia and a decrease platelet count. A peripheral blood
smear is noteworthy for the absence of Howell-Jolly bodies. Which of the following is the best recommendation?

a. CT scan of the abdomen


b. Bone marrow biopsy
c. No work-up needed; administer steroids
d. Radiolabeled red blood cell (RBC) scan
e. No work-up needed; administer immunoglobulin

66. The most accurate mean of determining T and N staging of gastric adenocarcinoma is:

a. Triple-phase helical computed tomography (CT) scan


b. Diagnostic laparoscopy
c. Endoscopic ultrasonography
d. Magnetic resonance imaging with gadolinium
e. Positron emission tomography scan
67. A 28-year old woman undergoes a segmental ileal resection during the course of an adhesiolysis for an acute small bowell
obstruction. On postoperative day 6, she is noted to have thick bile-colored fluid emanating from the the midline wound. After IV
hydration, the next step in management should be:

a. CT scan of the abdomen


b. Upper gastrointestinal small bowell follow-through with water-should contrast.
c. Fistulogram
d. Operative re-exploration
e. Octreotide

68. After a total thyroidectomy, the right vocal cord is noted to be fixed in a paramedian position. The most likely represents:

a. Injury to the RLN (Recurrent laryngeal nerve)


b. Injury to the external branch of the superior laryngeal nerve
c. Injury to the internal branch of the superior laryngeal nerve
d. Trauma from endotracheal intubation
e. Compression from hematoma

69. A 23-year old man sustains blunt chest trauma following a high-speed motor vehicle accident. He is hemodynimacally stable. Which
of the following conditions is an indication for thoractomy?

a. Undrained hemothorax despite a tube thoracostomy


b. Continuous chest tube drainage of more than 200mL/hr of blood for 4 hours.
c. Intial chest tube outputs of 1100ML
d. Wide mediastinum
e. Flail chest

70. The three findings in achalasia are:

a. Increased LES resting pressure, decreased LES relaxation, increased esophageal peristaltic activity
b. Decreased LES resting pressure, increased LES relaxation, decreased esophageal peristaltic activity
c. Increased LES resting pressure, decreased LES relaxation, decreased esophageal peristaltic activity
d. Decreased LES resting pressure, increased LES relaxation, increased esophageal peristaltic activity

71. The most common etiology of esophageal perforation is:

a. Spontaneous rupture (Boerhaave's syndrome)


b. Instrumental perforation
c. Foreign body in esophagus
d. Barrett's esophagus

72. Which of the following is true regarding GIST (gastrointestinal strumal tumors)?

a. Surgery is the mainstay of treatment


b. Most GISTS are found in the stomach
c. GISTS respond well to imatinib mesylate (Gleevec)
d. GISTS usually present with upper gastrointestinal bleeding
e. All the above
73. Which of the following complications of crohn's disease is least common?

a. Enteroenteral fistulas
b. Enterocutaneous fistulas
c. Free perforation
d. strictures

74. Which is the most likely cause of intussuception in a 20-year-old man?

a. Idiopathic
b. Lymphoma of the small bowel
c. Carcinoid
d. Adhesions

75. Which of the following is true regarding adenomatous colonic polyps?

a. Polyps less than 1 cm in diameter are unlikely to be malignant


b. Villous adenomas most commonly occur in the cecum
c. FAP (familial adenomatous polyposis) is an autosomal recessive condition
d. Hamartomatous polyps have a high incidcncc of malignant transformation
e. Fecal occult blood testing has not changed the mortality from colon cancer

76. A sixty-old patient with two bouts of Hinchey Stage II diverticular disease should be treated with:

a. Sigmoid colectomy and anastomosis


b. Transverse colostomy
c. Sigmoid resection, end colostomy and mucous fistula
d. Bowel rest, IV antibiotics
e. Total abdominal colectomy

77. All of the following are indicative of poor prognosis in acute pancreatitis except:

a. Serum calcium level less than 8.0 mg/dL


b. Hyperglycemia
c. Serum amylase level more than five times normal on admission
d. Arterial oxygen tension less than 60 mmHg
e. Serum lactic dehydrogenase (LDH) more than three times normal

78. A victim of blunt abdominal trauma undergoes a partial hepatectomy. During surgery, he receives 12 units of packed red blood cells.
In the recovery room, he is noted to be bleeding from intravenous puncture sites and the surgical incision. Which of the following
statements regarding the coagulopathy is most likely true?

a. The patient has an unknown primary bleeding disorder


b. The coagulopathy is secondary to the partial hepatectomy
c. The coagulopathy is secondary to dilutional thrombocytopenia and deficiency of clotting factors from the massive blood transfusion
d. The treatment is oral vitamin K.
e. The treatment is intravenous vitamin K
79. Which of the following is true of mammography?

a. Is the most effective means of screening for breast carcinoma


b. Is more effective in detecting breast carcinomas in postmenopausal women
c. When normal, should not exclude biopsy of a palpable suspicious breast mass
d. All of the above

80. Statistically, the most powerful predictor of prognosis for breast cancer is:

a. The presence of intramammary lymphatic involvement.


b. The grade of differentiation of the tumor
c. The presence of marked intraductal carcinoma around the primary tumor
d. The size of the tumor
e. The number of axillary lymph nodes involved with metastatic tumor

81. A 33-years old woman pregnant for the third time present at 3 months with a 2-cm mass, in the inner aspect of the left breast. A
needle aspiration reveals no fluid, you would:

a. Arrange for mammogram because multicentric lesions are common during pregnancy
b. Consider termination of pregnancy because chemotherapy has been shown to be useful in node-negative premenopausal patients
c. Expeditiously obtain a histologic diagnosis of the mass
d. Wait until the third trimester because surgery is safer at that time

82. Fine needle aspiration biopsies are often useful for which of the following entities?

a. Thyroid nodules
b. Breast mass
c. Liver mass
d. Lymphoadenopathy
e. Lung mass

83. Acceptable biopsy methods for a pigmentation lesion include all for the following except:

a. Excision biopsy
b. Puncher biopsy
c. Shave biopsy
d. Incisional biopsy

84. A 65 years old man has an enterocutaneus fistula originating in the jejunum secondary to inflammatory-bowel disease. Which of the
following would be the most appropriate fluid replacement of the enteric loss?

a. Dextrose 5%
b. 3% normal saline
c. Ringer lactate solution
d. 0.9% sodium chloride
e. 6% sodium bicarbonate solution
85. A 72 years old man status post-coronary artery bypass graft (CABG) 5 years ago presents with hematochezia, abdominal pain, and
fever. Colonoscopy reveals patches of dusty-appearing mucose at the splenic flexure without active bleeding. Which of the following is
the most appropriate management of the patient?

a. Angiography with administration of intra-arterial paraverine


b. Emergent laparotomy with left hemicolectomy and transverse colostomy
c. Aortomesenteric bypass
d. Exploratory laparotomy with thrombectomy of the inferior mesenteric artery
e. Expectant management

86. A 2-weeks old infant in found to have a hydrocele. Which is the best course of treatment?

a. Surgery of the age of 6 months


b. Surgery of the age of 2 years
c. Aspiration of the hydrocele
d. Transscrotal hydrocelectomy of the age of 2 years
e. None of the above

87. A 48 years old man is stabbed in the right upper quadrant, he arrives in the emergency department hypotensive and with abdominal
tenderness. He is taken emergently to the operation room for laparotomy. At exploration he is found to have a 5-cm laceration to the
dome of the liver that is no longer bleeding. No other injuries are found, the next step in management this injury would be:

a. Pringle maneuver
b. Pringle maneuver and finger fracture exploration of the injury
c. Exploration of the injury
d. Drain injury area and close the abdomen
e. Close the abdomen and get an angiogram

88. The first priority during evaluation of the multiple injury patient who is hypotensive is to:

a. Establish intravenous access


b. Obtain blood for crossmatch
c. Perform a mini-neurologic examination
d. Access the airway
e. Search for occult bleeding

89. The initial treatment for patient with multiple bilateral ribs fractures and flail chest with CO2 retention is?

a. Endotracheal intubation and positive-pressure ventilation


b. Surgical fixation of the ribs
c. Chest wall immobilization with sand bags
d. Intercostal nerve blocks

90. A 41 years old man complains of regurgitation of saliva and of undigested food, an esophagogram reveals a birds-beak deformity,
which of the following statements is true about this condition?

a. Chest pain is common in the advanced stages of this disease


b. More patients are improved by forceful dilatation than by surgical intervention
c. Manometry can be expected to show high resting pressure of the lower esophageal sphincter (LES)
d. Initial surgical treatment consists primarily of resection of the distal esophagus with re-anastamosis to the stomach above diaphragm
e. Patients with this disease are at no increased risk for the development of carcinoma
91. A 43- year old woman with irritable bowel syndrome was in her usual state of health until approximately 2 months ago. She
complained of intermittent abdominal pain with nausea no fever. Last night her pain became more severe and she presented to the
emergency department with fever 39.7 with rigors. In the emergency department she had rigors and sclera icterus, direct bilirubin 2.4 ,
alkaline phosphatase 400 , WBC 18.500 , AST 20 , and ALT 21 . Ultrasound revealed a common bile duct stone. What is the next step?

a. Antibiotics and urgent surgical biliary decompression (ERCP)


b. Antibiotics and endoscopic retrograde cholangiopancreatography
c. Antibiotics and trans hepatic biliary decompression
d. Antibiotics, surgical decompression and cholecystectomy

92. Regarding urinary retention after ambulatory surgery, which of the following statements is not true ?

a. Urinary retention is most frequently associated with herniorrhaphy and anorectal procedures
b. Spinal anesthesia but not general anesthesia is a predisposing factor for postoperative urinary retention
c. Postoperative urinary retention can frequently be asymptomatic
d. Ambulatory surgery patients must void as criterion for discharge

93. Select the correct statement regarding flail chest:

a. It occurs when three or more adjacent ribs are fractured in one place
b. Work of breathing is increased secondary to paradoxical chest wall motion
c. Patients with flail chest should be aggressively resuscitated because of the probable development of a pulmonary contusion
d. Patients with this condition should be prophylactically intubated secondary to high likehood of respiratory failure
e. If a patient does require mechanical ventilation, it is important to avoid the use of positive end-expiratory pressure

94. A 20 years old Woman is involved in a high-speed motor vehicle accident with signification damage to the front of the car. She
arrives in the emergency department with GCS score of 15. Heart rate of 120 beats/min, respirator rate of 18 breaths/min, and a
systolic blood pressure of mmHg. A chest radiograph is obtained and demonstrates a 10-cm mediastinum a deviation of the left stem
bronchus. What the most appropriate next step in management?

a. Observation
b. Left-sided chest tube
c. Repeated chest radiograph in 6 hours
d. CT angiogram of the chest
e. Transesophageal echocardiogram

95. Which of the following is true of rhabdomyolysis?

a. Acute renal failure occur secondary to the release of myoglobin


b. An alkalotic environment promotes the formation of myoglobin casts in the renal tubules , thereby worsening the kidney damage
c. The renal failure from rhabdomyolysis typically resolves within 3-5 days
d. Severe hyponatremia is a frequent complication
e. Alkalinization to PH between 8 and 9 is an important treatment goal

96. With regard to breast carcinoma in men, which statement is true?

a. it is detected most commonly in men 60-70 years old


b. Gynecomastia is a risk
c. It is commonly associated with a mutation in BRCA1 gene
d. The prognosis is worse stage for stage that for woman
e. Sentinel lymph node biopsy is contraindicated
97. A 33 years old asymptomatic women is referred to the clinician with abnormal findings on a mammogram. No masses are palpable
in either breast. The mammogram shows a tight cluster or microcalcifications at 2 o'clock position in her left breast. Magnification
compression views show at least 20 tiny irregular calcifications in a 2-cm area that vary in shape and density with no associated mass
lesion. There are no other calcifications present in either breast. Which of the following is the most likely diagnosis?

a. LCIS
b. Fibroadenoma
c. Infiltrating ductal carcinoma
d. DCIS (ductal carcinoma in situ)
e. Fibrocystic changes

98. A 72 years old man undergoes an aortobifemoral graft for symptomatic aortoiliac occlusive disease. The in inferior mesenteric artery
(IMA) is ligated in its aortic attachment. 24 hours after surgery the patient has abdominal distension, fever and bloody diarrhea. Which
of the following is the most appropriate diagnostic study for this patient?

a. Aortogram
b. Magnetic resonance imaging (MRI)
c. Computed tomography (CT) scan
d. Sigmoidoscopy
e. Barium enema

99. The most common cause of gastric outlet obstruction in adults is?

a. Peptic ulcer disease


b. Extrinsic neoplastic compression
c. Gastric cancer
d. Primary lymphoma of the stomach
e. Duodenal Crohn's disease

100. Which of the following is true regarding the initial treatment of patients with acute complete small bowel obstruction?

A. Immediate surgery is warranted as soon as the diagnosis is made


B. Nasogastric decompression for 24 hours allows spontaneous resolution of complete bowel obstruction on all patients
c. The present of fever, tachycardia, localized pain or leukocytosis suggests strangulation and warrants prompt surgery
d. All patients with complete small bowel obstruction require blood plasma for resuscitation
e. If a small bowel resection must be performed, a stoma and mucous fistula are necessary because an anastomosis is subject to non
healing in the face of obstruction

101. Which of the following serve as landmarks when assessing descent of the fetal head?

A. Ischial spines
B. Symphysis pubis
C. Ischial tuberosities
D. Sacral promonotory
102. The diagonal conjugate is calculated by measuring the distance between which of the following anatomical structures?

A. Coccyx and inner margin of symphysis pubis


B. Tip of scarum and inner margin of symphysis pubis
C. Ischial spine and external margin of symphsis pubis
D. Sacral promontory and inferior point of symphysis pubis

103. At what gestational age does the uterus become too large to lie totally within the pelvis?

A. 6 weeks
B. 12 weeks
C. 16 weeks
D. 20 weeks

104. What is pigmentation of the midline, anterior abdominal skin during pregnancy called?

A. Striae gravidarum
B. Linea nigra
C. Chloasma
D. Melasma

105. At what gestational age should laboratory testing for gestational diabetes (50 Gr glucose challange test) be performed?

A. 10-16 weeks
B. 20-24 weeks
C. 24-28 weeks
D. 33-36 weeks

106. What nutrient during pregnancy is NOT adequately provided by diet alone?

A. Calcium
B. Magnesium
C. Iron
D. Vitamin A

107. Your patient, a Para 1, is considering a second pregnancy. Her first child was anencephalic and died soon after birth. You
recommend what daily dose of folic acid periconceptually to reduce the risk of recurrence?

A. 0.4 mg
B. 4 mg
C. 40 mg
D. 400 mg

108. Which of the following characterizes true labor?

A. Painless contractions with rupture of membranes


B. Engagement of the fetal head
C. Progressive cervical dilatation and effacement
D. Rhythmic lower abdominal pain
109. What is a laceration involving the skin, mucous membrane, perineal body, and anal sphincter called?

A. First degree
B. Second degree
C. Third degree
D. Fourth degree

110. A gradual, smooth deceleration of the fetal heart rate that follows the peak of a contraction describes which of the following
deceleration types?

A. Late
B. Early
C. Variable
D. Prolonged
Part B:

1. Which of the following ranges reflects the typical lenghth of contractions during labor?

a. 20-30 secunds
b. 30-40 secunds
c. 50-60 secunds
d. 100-120 secunds

2. Which of the following is not risk factor for gestational diabetes?

a. Age <25 year


b. Prior macrosomia infant
c. Prior still born infant
d. Sister with diabetes

3. A woman with severe preeclampsia was given MgSO4. How is the 'agnesium' excreted from the body?

A. By diffusion through the lungs


B. By conjugation in the liver
C. In the urine through the kidneys
D. By secretion into the gastrointestinal tract

4. What is the fetal and neonatal risk of maternal ITP?

a. Increased absorbtion rate


b. Thrombocytopenia
c. Necrotizing enterocolitis
d. No risk

5. A 41 year old gravida 3 woman presents with heavy prolonged menstrual bleeding. Up until 8 months ago she had regular monthly
menses which lasted 4-5 days .her physical examination and pap smear are normal. On pelvic ultrasound, the endometrium is 14 mm
and there is a 2cmx1.2cm lesion in the uterine cavity . her B-hcG, TSH, and endometrial biopsy are normal. Which of the following is the
most likely diagnosis?

a. Endometrial polyp
b. Endometrial hyperplasia
c. Fibroid uterus
d. Adenomyosis
e. Dysfunctional uterine bleeding

6. 47-year-old gravida 3 para 3 female has had fibroids all of her life and has always regulary timed heavy menses. Recently the
frequency and quantity of her bleeding increased. Her TSH and endometrial biopsy were normal. Her hematocrit is 31 %, pelvic US
shows a multifibroid uterus of 18 cm. 3 year ago her uterus was 15 cm and 4 years before that it was 12 cm. The patient symptoms
would best treated by:

a. Oral contraceptive pills


b. Total abdominal hysterectomy
c. Sequential hormone replacement therapy
d. Hysteroscopic myomectomy
e. Abdominal myomectomy
7. The mainstay of treatment of epithelial ovarian cancer at stage IIIA is:

a. Radiation therapy alone


b. Surgery therapy alone
c. Surgery followed by chemotherapy
d. Surgery followed by radiation therapy
e. Chemoradiation

8. The absolute contraindication for the use of estrogen-containing oral contraceptive includes:

a. A history of migraine headaches


b. A history of pulmonary embolism
c. A history of biliary colic
d. Symptomatic fibroid uterus
e. Syptomatic controlled hypertension

9. A 25 year old woman presents to her gynecologist with a principal complaint of primary infertility lasting one year. She has regular
periods that she consider somewhat heavy associated with cramping pain during the first 2-3 days of her menses . she undergoes a
hysterosalpingogram which confirmed tubal patency but demonstrated an irregular uterine cavity surface consistent with sub-mucosal
myoma. Considering this patients desire for a pregnancy, what is the most appropriate treatment for her fibroid?

a. Uterine artery embolization


b. Combined oral contraceptive
c. Depot medroxyprogesterone
d. Hysteroscopic myomectomy
e. Total hysterectomy

10. A colleague asks for your opinion with a patient she is currently examining. The patient is a 37-year-old G2 P2 who is presenting with
postcoital spotting. Her menstrual periods are regular in amount and timing. She has never had this symptom before and has had the
same sexual partner, her husband, for 8 years. On exam she has normal external female genitalia and a normal vagina. Her cervix shows
no discharge or cervical motion tenderness. The cervical os is noted to contain a 3-mm fleshy pedunculated mass. Her uterus is normal
in size, mobile and non tender. Her last Pap smear was 6 months ago and was negative for malignancy. How would this patient be best
managed?

a. Observation and menstrual calendar


b. Repeat Pap smear
c. Pelvic ultrasound
d. Remove the mass in the office at that time
e. Abdominal hysterectomy

11. A 27year old woman present to the emergency department complaining of vaginal discharge and abdominal pine have fever is
38.1"c . and on abdominal examination she has tenderness is the right upper guardant .and lower abdominal with minimal peritoneal
sings .she was found to have a mucous yellow discharge on bimanual examination she has cervical motion tenderness and bilateral
adnexal tenderness her whit blood count is 14.3 and pelvic US shows a normal uterus and normal ovaries bilaterally .the most likely
diagnosis for this patient is which of the following?

a. cervicitis
b. Endomyometritis
c. PID
d. TOA
e. UTI
Canceled 12. A 49 year old woman present compiling of vulvar pin that increases with ambulation and intercostal .she also notes a lump
on her right labia that has increased in size over the past 48 hours and is quit . painful .the patient has had these same symptoms in the
past .on exam she has a 5cm .tender cyst on the medical aspect of the right labia that extends 1 to 2cm away .what would be year first
step in management of the patient?

a. Sits baths
b. Antihistamine agent
c. Local antibiotics
d. Simple vulvectomy
e. Marsupialization

Canceled 13. A 65 year old post-menopausal female present for her annual exam without complains. she has a without patchy area
between the posterior forchette of the vaginaWithout pruritus or irritation. you obtain a biopsy of the area .and the pathology report
diagnosis lichen sclerosis what is the first line treatment for this lesion?

a. Wide local excision


b. Clobetasol
c. Laser raporization
d. Tropical antifungal
e. Tropical estrogen cream

14. A 28 year paint underwent a dilatation and evacuation of the uterus for complete molar pregnancy .she was followed withe weekly
B-hcg levels .initially her B- hcg levels declined . unfortunately the level then plateaued and 8-weeks after evacuation .began to rise for
two week the patient has been reliably taking oral contraceptive pills .the evaluation of metastatic disease in negative .how do you
mange her disease of this stage?

a. Continue expectant management


b. Repeat D&C
c. Total abdominal hysteroctomay
d. Single –agent chemotherapy
e. Multi-agent chemotherapy

15. Which of the following is not regarding sperm preparation and intrauterine injection (iui)?

a. IT is a mean to treat sperm disorder defined as mild to moderate


b. The purpose of sperm enhancement is to choose sperm with advanced movement
c. The purpose of sperm enhance is to improve sperm capacitation
d. The purpose of sperm enhance is to concentrate the semen sample
e. The purpose of sperm enhance is to discard the seminal fluid

16. Which of the following is correct regarding varicocele in men?

a. It involves substantial stenosis of the testicular venous pampiniform plexus


b. Clinical varicocele is defind as palpale dilated veins on physical examination of the testes
c. Varicocele.ligation or embolization always improve sperm quality
d. Male infertility is always an indication for repair of subclinical(seen by ultra sound) varicocele
e. It is widely accepted that varicocele reduces sperm quality by decreasing testes temperature
17. Which of the following is correct regarding testosterone?

a. 50% of the testosterone in produced by the ovary


b. 80% is binded of SHBG and 19% is binded to albumin
c. The testosterone that is binded to SHBG is the active testosterone
d. 25% of the testosterone is peripheral formed
e. Circulating testosterone level in mature woman are about 100ng/dl

18. In which of the following cases pregnancy my be achieved only via invitro fertilization?

a. A fallopian tube that was damaged and obstructed following PID and a normal contralateral on
b. An obstructed fallopian tube following (PID) and a contra lateral one after pregnancy and salpingostomy
c. Unilateral salpingectomy and male sperm cunt of 10^5 cells/ml
d. Male sperm cunt of 100*10^6 cells/ml and an obstructed fallopian tube following (PID)
e. Fallopian tube with hydrosalpinx and passage of color to the pelvis and normal contralateral on

19. Which of the following is a test for ovarian reserve?

a. FSH level one cycle day3


b. Estradiol level on cycle day 6
c. Progesterone level following ovulation
d. LH level before ovulation
e. Inhibin A level

20. Which of the following is not an IVF complication?

a. Ovarian over stimulation


b. Fallopin tube pregnancies
c. Multifetal pregnancies
d. Heterotopic pregnancies
e. Ovarian cancer

21. What is correct for oseoperosis?

a. Bone mass increase till age 40


b. women loose on average 10% bone mass after menopause
c.25% of women above age 50 will have osteoporosis
d. T score < -1
e. Estrogen in not the first line therapy

22. What are the indications for estrogen treatment in menopause?

a. Osteoporosis prevention, coronary heart disease prevention, management of vaginal and vulvar atrophy
b. Osteoporosis prevention, treatment of vasomotor symptoms, management of vaginal and vulvar atrophy
c. Coronary heart disease prevention, mood improvement, prevention of Alzheimer`s disease
d. Mood improvement, treatment of vasomotor symptoms, prevention of Alzheimer`s disease
e. Coronary heart disease prevention, management of vaginal and vulvar atrophy, prevention of Alzheimer`s disease
23. What is not true about ovarian stimulation?

a. Can be triggered by recombinant or urinary gonadotropins


b. There is indication for serum estradiol test and US for follicular monitoring
c. HCG give at least one follicle at size 14mm
d. GnRH agonist and antagonist given to prevent
e. Ovulation occur 36hr after HCG administration

24. What is true regarding a woman with Müllerian agenesis?

A. They lack the upper part of the vagina


B. Fallopian tubes exist
C. 46XY or 45X0 karyotype
D. Entire vagina and uterus are missing
E. Bleeding will appear after estrogen and progesterone administration

25. Which of the following placental implantation most like predisposed to inverted uterine in 3rd stage of labor?

a. Fundal
b. Anterior
c. Posterior
d. Lateral
e. Lumbar stage of labor

26. A 70-year-old man complains of fever and pain in his left knee. Several
days previously, he suffered an abrasion of his knee while working in his garage.
The knee is red, warm, and swollen. An arthocentesis is performed, which shows 200,000 leukocytes/μL and a glucose of 20 mg/dL. No
crystals are noted. Which of the following is the most important next step?

a. Gram stain and culture of joint fluid


b. Urethral culture
c. Uric acid level
d. Antinuclear antibody
e. Antineutrophil cytoplasmic antibody

27. Septic arthritis ???

a. Strep pneumonia
b. Nessiriha
c. E.coli
d. Stap arus

28. 65 year, man, sudden onset of sever knee pain. Knee is red, swollen and tender. DM in the history and cardiomyopathy. In X ray
linear calcification. Knee diagnosis made by:

a. Serum uric acid


b. Serum calcium
c. Arthcentesis and positive birefrigniat
d. Crystals
e. Rehumatid factor
29. 70-year-old female with mild dementia complains of hip pain. There is some limitation of motion in the right hip. The first step in
evaluation is:

a. CBC and erythrocytes sedimentation rate


b. Rheumatoid factor
c. X-ray of right hip
d. Bone scan

30. A 40-year-old alcoholic develops cough and fever. Chest X-ray shows an air-fluid level in the superior segment of the right lower
lobe. The most likely etiologic agent is:

a. Streptococcus pneumonia
b. Haemophilus influenza
c. Legionale
d. Anaerobes

31. A 60 year-old male on aspirin, nitrates and beta blocker, being followed for chronic stable angina, present to E.R with history of 2-3
episodes of more severe and long lasting angina chest pain each day over the past 3 days, ECG and cardiac enzymes are normal. Best
course of action

a. Admit the patient and begin I.V digoxin


b. Admit the patient and begin I.V heparin
c. Admit the patient and give prophylactic thrombolytic therapy
d. Admit the patient for observation with no change in medications
e. Discharge the patient from E.R with increase nitrates and beta blocker

32. Alcoholic 35 year-old male is admitted for nausea, vomitting and abdominal pain that radiates to back, Lab value that suggests a
poor prognosis in the patient is:

a. Elevated serum Lipase


b. Elevated serum Amylase
c. Leukocytosis of 20.000/µL
d. Diastolic blood pressure higher than 90 mmHg

33. 40 year-old cigarette smoker complais of epigastric pain, well localized, nonradiating and described a burning. The pain partically
relieved by eating. There is no weight loss he has not used NSAID, the pain has gradually worsened over several months. Most sensitive
way to make a specific diagnosis:

a. Barium X-ray
b. Endoscopy
c. Serologic test for H. Pylori
d. Serum Gastrin

34. The best way to eradicate H. Pylori is?

a. Omeprazole 20 mg PO for 6 weeks, Ranitidine 300 mg PO qhs 6 weeks


b. Omperazole 20 mg BID, Amoxicillin 1000 mg BID, Clarithromycin 500 mg BID for 14 days
c. Bito-Bismol and Metronidazole BID for 7 days
d. Sucralfate 200 mg QID for 6 weeks
35. 40 year-old male with long standing alcohol abuse complains of abdominal swelling which has been progressive over several
months, History of GI bleeding, on physical examination spider angigrams and palmar eythema , abdominal collateral vessels are seen
around umbilicus, shiftting dullness and dullness flank are noted. Most important 1st step in patient evaluation :

a. Diagntostic pancreatitis
b. Upper GI series
c. Ethanol level
d. C.T scan

36. While Hospitalized a cirrhotic patient mental status deteriorates, He has been having guaiac-positive stools and low grade fever, He
also received sedation for agitation, on physical examination patient is confused, no meningeal signs and no focal neurological findings,
there is hyperreflexia and nonrhythmic fapping tremors of the wrist, most likely explanation for the mental status change:

a. Tuberculosis meningitis.
b. Subdural hematoma.
c. Alcohol withdrawl seizure.
d. Hepatic encephalopathy

37. A 40 year-old white male complains of weakness, weight loss and abdominal pain, on physical examination the patient has diffuse
hyperpigmentation and palable liver edge, polyarthritis of the wrists and hips is also noted, fasting blood sugar 185 mg/dl most likely
diagnosis :

a. Insulin-dependent DM
b. Pancreatic carcinoma
c. Addisons Disease
d. Hemochromatosis

38. A 34 year-old white woman is treated for urinary tract infection with amoxicillin, Initially she improves but 5 days after beginning
treatment she develops recurrent fever, abdominal bloating and diarrhea with 6-8 loose stools per day, you suspect Antibiotic-
associated colitis, The best diagnosis test is:

a. Identification of Clostridium difficile toxin in the stool


b. Isolation of Clostridium difficile in a stool culture
c. Stool positive for WBC
d. Detection of IgG antibodies against Clostridium difficile in the serum

39. A 55 year-old male is being evaluated for constipation there is no history of prior G.I. Symptoms, Hb is 10 g/dl, MCV is 72 fl, Serum
iron is 4 ug/dl [Normal 50-150], Saturation is 1% [Normal 20-45 %], ferritin is 10 ug/l [Normal 15-400 ug/l] next step in the evaluation of
this patient anemia:

a. RBC folate
b. Iron absorption studies
c. Colonoscopy
d. Bone Marrow examination
40. A 60 year-old asymptomatic man is found to have leukocytosis when a routine CBC is obtained, Physical exam shows no
abnormalities, spleein is notmal size, Labratory data:
Hgb: 9 g/dl [Normal : 14-18]
Leukocytes: 40,000 /ul [Normal: 4300-10800]
Peripheral blood smear shows a differential that include 97% small lymphocytes.
The most likely diagnosis?

a. Acute monocytic leukemia


b. Chronic myelogenous leukemia
c. Chronic lymphocytic leukemia
d. Tuberculosis

41. A patient diagnosed to suffer from chronic lympocytic leucemia will require chemotherapy:

a. If the white blood cell count rises


b. If lymphadenopathy develops
c. To control anemia or thrombocytopenia
d. Only when acute lymphocytic leukemia develops

42. A 64 year old male is hospitalized with a transient ischemic attack and is evaluated for carotid disease. Physical exam is normal. CBC
on admission is normal. the patient is started on heparin. A repeat CBC 1 week later shows an Hg of 14, WBC 9000 and PLT 10000.
You should?

a. Obtain a bone marrow study


b. Obtain a liver spleen scan
c. Suspected drug induced thrombocytopenia purpura
d. Begin corticosteroids for ITP

43. The patient described develops thrombosis of the brachial artery. The next step in management is:

a. Lupus anticoagulant
b. Antinuclear antibody
c. Lepirudin or danaparoid
d. Increase heparin dose

44. A 70 year old intensive care unit patient complains of fever and shaking chills. The patient develops hypotension, and blood cultures
are positive for gram negative bacilli. The patient begins bleeding from venipuncture sites and around his foley catheter. laboratory
studies are as follows: HCT 38%, WBC 15000, PLT 40000 (normal 130000to 40000), periferal blood smear: fragmented RBC's. PT
elevated, PTT elevated, plasma fibrinogen 70 (normal 200-400). The best course of therapy in this patient is:

a. Begin heparin
b. Treat underlying disease
c. Begin plasmapheresis
d. Give vitamin K
e. Begin red blood cell transfusion
45. A 30 year old female with graves' disease has been started on prophylthiouracil. She complains of low grade fever, chills and sore
throat. The most important initial step in evaluating this patients fever is?

a. Serum TSH
b. Serum T3
c. CBC
d. Chest X-ray
e. Blood culture

46. A 29 year old woman presents to the emergency department with a complaint of sudden onset of left facial weakness that was
noticed by her coworker. She denies fever, rash or any other symptoms. On physical examination she has no other neurologic deficits
other than what is shown described. When asked to shut her left eye, she cannot. Which of the following is the most likely diagnosis?

a. Bell palsy
b. Malingering
c. Ramsay hunt syndrome
d. Brain tumor
e. Cerebrovascular event

47. A 48 year old man with a past medical history of hepatitis C and cirrhosis presents to the emergency department with a complaint of
acute onset of abdominal pain and chills. You decide to perform a paracentesis. You retrieve 1 L of cloudy fluid. Laboratory analysis of
the fluid shows a neutrophil count of 550 cell/mm3. Which of the following is the most appropriate choice of tratment?

a. Metronidazole
b. Vancomycine
c. Amphotericine
d. Neomycine and Lactulose
e. Cefotaxim

48. A 72 year old woman with a history of hypertension and tobacco use presents to your clinic complaining of chest pain for the last 2
weeks. She describes the pain as pressure like, with radiation to the left arm. It is exacerbated by exertion and relieved by rest. There
are no associated symptoms. She denies any history of chest pain. Her vital signs are stable and the physical examination is
unremarkable. An ECG shows signs of left ventricular hypertrophy. Laboratory tests including cardiac biomarkers are within normal
limits. which of the following is the most likely diagnosis?

a. Non STEMI
b. Pericarditis.
c. Unstable angina
d. Stable angina

49. A 52 year old man with a history of hypertension, diabetes and hyperlipidemia presents to the emergency department with chest
pain of 3 hours duration. an ECG shows 1 mm ST depression in the inferior leads. Troponin is elevated and the patient is diagnosed with
a non STEMI. He is treated with aspirin, metoprolol, heparin and clopidogrel and admited to the coronary care unit. a left heart
catheterization reveals 99% occlutes right coronary artery. PCI is performed with adrug eluting stent. How long should this patient be
treated with clopidogrel?

a. One week
b. 1 month
c. At least 3 months
d. At least one year
50. A 59 year old man is diagnosed with a non STEMI. She receives a drug eluted stent to her middle left anterior descending coronary
artery. Transthoracic echocardiography reveals an EF of 35%. Her medications at discharge include aspirin, clopidogral, atorvastatin,
metoprolol and ramipril. According to the guidelines, which of the following medication should be added to this patient's regimen?

a. Warfarin
b. Furosemide
c. Ezetimibe
d. Spironolactone

51. A 45-year-old female presents to your office. She was see 10 days ago by a colleague with complaints of progressive fatigue,
malaise, and abdominal discomfort occurring over the last 10 months. She has also complained of polyarthralgias. She is not obese and
does not drink alcohol. Her recent laboratory tests reveal normal serum chemistries, normal total bilirubin, normal alkaline
phosphatase, aspartate aminotransferase (AST) 92 IU/L, alanine aminotransferase (ALT) 86 IU/L, serum albumin 3.2 g/dL, total protein
6.8 g/dL and increased y-globulin level. You suspect autoimmune hepatitis (AIH). In addition to obtaining a liver biopsy, which of the
following laboratory tests helps establishing a diagnosis of AIH in this patient?

A. Antinuclear antibody (ANA)


B. Antimitochondrial antibody (AMA)
C. Anti-Saccharomyces cerevisiae antibody (ASCA)
D. Anti-smooth muscle antibody (ASMA)

52. A 62-year-old female presents to your office prior to starting CHOP chemotherapy (cyclophosphamide, Adriamycin [doxorubicin],
vincristine, and prednisone) for newly diagnosed non-Hodgkin lymphoma. She has a history of treated hepatitis B 10 years previously
and has had no sequela from the disease or treatment, confirmed by a seroconversion of HBeAg positive status to anti-HBe and HBsAg
to anti-HBs status. She is not on any medications currently. Her physical examination is significant only for bulky adenopathy. Prior to
initiating chemotherapy, which of the following diagnostic tests should be checked in this patient?

A. Serum cryoglobulin levels


B. Abdominal ultrasonography to evaluate for biliary obstruction
C. Hepatitis B viral load by PCR amplification, HBsAg, and anti-HBc
D. Repeat liver biopsy to ensure true eradication of hepatitis B

53. A 43-year-old man presents with palpitations and shortness of breath. On examination, his blood pressure is 124/80 mm Hg, heart
rate is 128 beats / min and irregular, and temperature is 36.6°C. He has moderate bilateral exophthalmus, a palpable goiter, and mild
weakness, in his bilateral lower extremities. An ECG confirms atrial fibrillation with rapid ventricular response and TSH at 0.003 μU/mL
(0.5 to 4.7 μU/mL). Which of the following is the most appropriate treatment for the atrial fibrillation in this patient?

A. Propylthiouracil
B. Iodide
C. Propranolol
D. Prednisone
54. A 66-year-old man is brought to the emergency department by his wife. She reports that he accidentally took an extra dose of each
of his medications about 3 hours ago. His medications are aspirin, metoprolol, and fluvastatin. Physical examination reveals a somnolent
man with a heart rate of 33 bpm and a blood pressure of 75/30 mm Hg. He is treated with intravenous fluids (IV), activated charcoal,
and a cathartic agent. Which of the following is the best treatment for hypotension and bradycardia caused by acute beta-blocker
toxicity?

A. Dopamine
B. Isoproterenol
C. Glucagon
D. Digoxin

55. You are asked to evaluate a 55-year-old man with G6PD deficiency who developed acute onset of shortness of breath after receiving
topical benzocaine prior to an endoscopy. Physical examination reveals a mildly cyanotic man who is tachypneic and tachycardic. Pulse
oximetry reveals an oxygen saturation of 82%. Which of the following laboratory tests is most likely to confirm this patient's diagnosis?

A. Methemoglobin level
B. Carboxyhemoglobin level
C. Acetaminophen level
D. Venous oxygen saturation

56. A 72-year-old woman with rheumatoid arthritis is brought to the emergency department by her daughter. The patient's daughter
reports that her mother has been having more joint pain for the last 2 days, and this morning, she took an extra dose of oxycodone. She
then became confused, prompting her daughter to bring her to the emergency department. Physical examination reveals an elderly
woman who is somnolent and difficult to arouse. Her heart rate and blood pressure are stable, but she is only
breathing 8 times per minute. Her pupils are 1 mm in diameter and symmetric. Of the following, what is the best treatment for this
patient at this time?

A. Intravenous flumazenil
B. Intravenous fomepizole
C. Transvenous cardiac pacemaker
D. Intravenous naloxone

57. A 75-year-old patient presents to the ER after a sudden syncopal episode. He is again alert and in retrospect describes occasional
substernal chest pressure and shortness of breath on exertion. His lungs have a few bibasilar rales, and his blood pressure is 110/80. On
cardiac auscultation, the classic finding you expect to hear is:

A. A harsh systolic crescendo-decrescendo murmur heard best at the upper right sternal border
B. A diastolic decrescendo murmur heard at the mid-left sternal border
C. A holosystolic murmur heard best at the apex
D. A midsystolic click

58. A 72-year-old male comes to the office with intermittent symptoms of dyspnea on exertion, palpitations, and cough occasionally
productive of blood. On cardiac auscultation, a low-pitched diastolic rumbling murmur is faintly heard toward the apex. The origin of
the patient’s problem probably relates to:

A. Rheumatic fever as a youth


B. Long-standing hypertension
C. Silent MI within the past year
D. Congenital origin
59. A 70-year-old male complains of the sudden onset of syncope. It occurs without warning and with no sweating, dizziness, or light-
headedness. He believes episodes tend to occur when he turns his head too quickly or sometimes when he is shaving.
The best way to make a definitive diagnosis in this patient is:

A. ECG
B. Carotid massage with ECG monitoring
C. Holter monitor
D. Electrophysiologic studies to evaluate the AV node

60. A 32-year-old diabetic female who takes an estrogen-containing oral contraceptive and drinks three beers per day is found to have a
triglyceride level greater than 1000 mg/dL. She is at risk for which of the following complications?

A. Acute pancreatitis
B. Sudden cardiac death
C. Acute peripheral arterial occlusion
D. Acute renal insufficiency
E. Myositis

61. A healthy 45 year old women with no medication comes to your office with 10 days of nasal congestion, sore throat, dry cough and
initial low temperature, all that nearly resolved. Over the past 24-48 hours she develops a sharp chest pain by deep inspiration or cough
but no dyspnea. Due the severity of the pain the nurse makes ECG that shows a diffuse ST-elevation.
On the physical examination is most likely to find:

a) Loud pulmonic component of S2


b) A S3 gallop
c) A pericardial friction rub
d) A bilateral basilar rales
e) A high blood pressure of 160/100 mmHg

62. A 20 years old nursing student complains of asthma on surgical rotation. She develops dermatitis of the hands and the symptoms
worse when she in the operation room.
Which of the following is correct?

a) This is an allergic reaction that is always begins


b) The patient should evaluate for latex allergy with skin test or specific IgE antibody
c) This syndrome is less common now than 10 years ago
d) An oral corticosteroids is indicated

63. A 30 years old man develops skin rash, purities and mild wheezing 20 minutes after an I.V Pyelogram preformed for evaluation of a
renal stone symptomatic.
A best approach to diagnosis includes:

a) Perform a 24 hours urine-histamine measurement


b) Measure of IgE to radio contrast media
c) Diagnosis of radio contrast media sensitivity history
d) Recommend intra dermal skin testing
64. A 27 years old undergoes a follow up evaluation 5 months after the diagnosis of idiopatic pulmonary embolism for which she was
prescribed at 6 month course anticoagulant therapy. Family history, a maternal grandmother took warfarin many years for unknown
reasons. An older brother with history of DVT at age 32. Patient takes no oral contraceptive pills or other medication. She is healthy.
The CBC is normal, INR is 3.0 Which of the following is most appropriate next step of evaluation of the patient:

a) Immediate thrombophilie-screening
b) JAK2 mutation analysis
c) No further evaluation is needed
d) Thrombophilie-screening at least two weeks after therapy session

65. A 82 years old man complains of two hours severe chest pain that occurred while playing tennis. Blood pressure on admission is
140/70, pulse 110. No signs of congestive heart failure. Pulses all palpable, abdominal exams are normal, neurologic exams are normal
and stool-Guaniac is negative, no history of gastro intestinal bleeding, previous stroke head trauma or major surgery, no history of
vascular disease or liver disease. ECG shows ST segment elevation of 3 mm in leads V1-V3 and 3 premature ventricular beats/min. Initial
treatment of choice is:

a) Prophylactic Lidocaine
b) Thrombolytic therapy with aspirin
c) Heparin
d) Aspirin alone

66. A frail 80 years old nursing home resident had several episodes of syncope, all which have occurred while she was returning to her
room after breakfast. She complains of light headedness and stats she feels cold and weak. She takes nitroglycerin in the morning for
history of chest pain but denies recent chest pain or shortness of breath. The most likely method of diagnosis is:

a) Cardiac catheter
b) Postprandial blood pressure monitoring.
c) Holter-monitor
d) CT scan

67. A 30 years old women present with complains of paresthesias, weakness, leak of coordination and difficulty with gait. Her symptoms
are worse after hot shower. Examination of cerebrospinal fluid shows oligoclonal immunoglobulin antibodies IgG. The most likely
diagnosis is:

a) Multiple sclerosis
b) Huntington disease
c) Parkinson disease
d) Neurofibromatosis disease
e) Amyotrophic lateral sclerosis (ALS)

68. A 56 years old man present to your office with complains of chronic diarrhea. He states that he has loose stool for the last 2 days. He
denies blood in stool and fever. He has no weight loss and no recent travel. Appropriate management at this time includes:

a) Observation
b) Check stool culture
c) Colonoscopy
d) Stool fat study
69. A 23 years old man climbing Mount Kilimanjaro has no medical problems and takes no medication. Shortly after the climbs he
develops sever shortness of breath, physical examination shows diffuse bilateral crackles. Which is the most likely etiology?

a) Acute interstitial pneumonitis


b) Acute respiratory distress syndrome
c) Cardiogenic shock
d) Community acquired pneumonia
e) High altitude pulmonary edema

70. A 65 years old man admitted to the hospital with complains of shortness of breath. He has no prior medical care. Laboratory notifies
you of B- natriuretic peptide value of 1500 pg/dl, you suspect the following diagnosis:

a) Asthma exacerbation
b) COPD exacerbation
c) Pulmonary embolism
d) DVT
e) Congestive heart failure

71. A hospitalized patient is receiving a blood transfusion. The floor nurse reports that the patient is flushed, is complaining of
abdominal discomfort, and has a temperature of 38.4°C. The most appropriate management is to:

A. Give the patient acetaminophen and continue the transfusion at a slower rate
B. Administer diphenhydramine and continue the transfusion
C. Administer 100 mg of hydrocortisone intravenously and reduce the rate of the transfusion
D. Stop the transfusion and increase intravenous fluids
E. Administer intravenous ranitidine and order an abdominal series x-ray

72. A 58-year-old woman with a previous history of infectious endocarditis presents for advice regarding antibiotic prophylaxis prior to a
dental procedure. The procedure will involve significant gingival manipulation. She has no known allergies. Which of the following is
recommended prophylaxis for this patient?

A. Levofloxacin
B. Clarithromycin
C. No antibiotic prophylaxis is needed
D. Amoxicillin

73. Which of the following is the most likely factor contributing to chronic myeloid leukemia?

A. Exposure to radiation
B. Family history of cancer
C. Tobacco abuse
D. Philadelphia chromosome

74. A 41-year-old business executive presents to your office and complains of palpitations and shortness of breath. After further
questioning, he admits to heavy alcohol consumption the previous evening. On examination, he is found to have an irregular heartbeat
of 130 bpm. The most likely diagnosis is:

A. Ventricular tachycardia
B. Ventricular fibrillation
C. Premature ventricular contractions (PVCs)
D. Atrial fibrillation
E. Wolff-Parkinson-White syndrome
75. A previously healthy 26-year-old man presents with abdominal cramping and Fever that has lasted for 2 days. He has had 10 stools
in the last 24 hours. A stool specimen reveals the presence of blood and WBCs. The most likely diagnosis:

A. Staphylococcal food poisoning


B. Rotavirus
C. Crohn disease
D. Shigellosis
E. Irritable bowel syndrome

76. All of the following drugs can cause psychotic symptoms similar to schizophrenia EXCEPT :

A. Ecstasy
B. Opioids
C. Phencyclidine
D. Ketamine
E. Lysergic acid diethylamide (LSD)

77. The prevalence of schizophrenia is:

A. 10%
B. 1%
C. 5%
D. 0.1%
E. 2%

78. A 32-year-old male with chronic paranoid schizophrenia, in full remission is married and living a productive life. He and his wife
decide to have children, but would like to know the probability of children developing schizophrenia. Which of the following statements
is correct:

A. The risk is similar to that of general population (1%).


B. The risk is about 50%
C. The risk is much lower than the general population because he will be able to recognize the early signs of schizophrenia and seek
prompt treatment for the child
D. The risk is about 10%
E. The risk is about 100%

79. A 36-year-old pregnant women is admitted to the medical center for possible infection associated with intravenous heroin injection.
She admit to heroin dependency. She is 5 month into her pregnancy and would like to "quit the heroin for good" What is the best
course of action in this patient?

A. Allow her to continue


B. Methadone maintenance
C. Naltrexone
D. Advise her to stop heroin and start supportive therapy
E. Buprenophinea

80. Which of the following has the most reinforcing effects, that is, the greatest potential to become addicted?
A. Cocaine
B. Alcohol
C. Marijuana
D. Heroin
E. Nicotine
81. The relationship between mitral valve prolapse (MVP) and panic disorder can be described as?

a. Important to recognize and treat mitral valve prolapse in panic disorder


b. High clinical relevance.
c. Patient with panic disorder are predisposed to develop MVP
d. Higher incidence of MVP in patients with panic disorder
e. Panic disorder is very well controlled if MVP is corrected.

82. A 56 year old anxious executive calls at 5:00 because he woke up with chest tightness and shortness of breath. He was scared that
he was having a heart attack and he also reported feeling dizzy, trembling, and his heart was pounding, he thought he was going to die.
The symptoms subsided in 20 minute and by the time the emergency medical services arrived he was sitting in his living room and in no
distress. The patient's electrocardiogram reading was normal and his cardiac enzymes were also found to be normal. What is the most
likely diagnosis?

a. Nocturnal panic attack


b. PTSD
c. Psychophysiological insomnia
d. Somatization disorder
e. Myocardial infraction

83. A 22 year old female is self – referred for an evaluation to rule out bipolar disorder. She complains of rapid mood swings with a
controllable anger and irritability. During the assessment it is noted that she has not had any sustained relationships for many years and
feels empty she fear abandonment by her fries and blames other of making her feel angry . What is the most important differential
diagnosis in this patient?

a. Major depressive disorder


b. Borderline personality disorder
c. Histrionic personality disorder
d. Depressive personality
e. Generalized anxiety disorder

84. 56 years – old female is admitted to an acute psychiatric inpatient unit for severe depression the patient was ruminating about
suicide and guilt feelings that were distressing her. She is "convinced" that she has committed sin and deserved to be punished.
although there was nothing in her history to justify this the resident doctor thinks that the patient is obsessed with thoughts of guilt
and feel that the patient has a primary obsessional disorder a trial of antidepressant medication was not helpful despite trying a high
dose the attending physician explains that the patient is not obsessional but severely depressed because ?

a. She has suicide ideation


b. She has indeed committed sin and is now depressed
c. She is distressed by the guilt feeling and is "convinced" of the sin
d. The attending physician thinks so
e. Antidepressants alone were not effective

85. The manifestation of depression symptoms are slightly different age groups and also in men and women. Agitated depression is a
term more commonly used to describe depression in which group of patients?

a. Young children
b. Adolescents
c. Adults
d. Women
e. Elderly
86. The term double depression is used to describe?

a. Major depressive disorder superimposed on grief reaction


b. Major depressive disorder superimposed on dysthymia
c. Major depressive disorder, patient not responding to treatment
d. Major depressive disorder with psychosis
e. Major depressive disorder with anxiety

87. A 36 – year old Caucasian male is stable on lithium for bipolar disorder for the past 4 years .during his routine visit with his family
physician. He complains of fatigue and lack of motivation. He is also feeling very cold during this winter and has gained weight. He
wonders if he is getting depressed again. What is the next most appropriate step to take in this case?

a. Diagnose depression and start the patient on antidepressants


b. Obtain the patients lithium levels
c. Check the patient's thyroid – stimulating hormone
d. Discontinue lithium
e. Consider ECT

88. A 46 old female who is 6 week postpartum complains of feeling low in her mood and having crying spells, lack of motivation and
feeling of hopelessness for the past 4 week she is diagnosed with postpartum depression. Which of the following is true about
postpartum depression?

a. Risk of infanticide is minimal


b. Selective serotonin reuptake inhibitors (SSRIs) are contraindicated
c. Risk of depression during subsequent pregnancies is increased
d. Suicide risk is low
e. Hospital admission is rarely required

89. Which of the following SSRIs is least likely to cause serotonin discontinuation syndrome?

a. Paroxetine
b. Citalopram
c. Escitalopram
d. Fluoxetine
e. Sertraline

90. Of all of the tricyclic antidepressants which of the following has the most potent serotonin reuptake inhibition activity?

a. Clomipramine
b. Imipramine
c. Amitriptyline
d. Nortriptyline
e. Desipramine
91. A neurologist is consulted by psyciatrist to assess a 42 years old depressed woman with recent onset of “muscle twitches”. She was
taking Citalopram 80mg. Duloxetine 40mg was recently added because of suboptimal response to Citalopram alone. She denies any
alcohol or substance abuse. On examination, the neurologist notes that the patient is slightly disoriented and also has mild tremors and
hyperreflexia. What is the most likely cause of the “muscle twitches” in this patient?

a. Serotonin syndrome
b. Alcohol withdrawal syndrome
c. Neuroleptic malignant syndrome
d. Malingering
e. Encephalitis

92. Which of the following antidepressant blocks reuptake of dopamine?

a. Venlafaxine
b. Bupropion
c. Buspirone
d. Mirtazapine
e. Fluoxetine

93. The advancement in Central Nervous System (CNS) pharmacology is attributed to the better understanding of neurotransmitters
and neuroreceptors. Which of the following is true about neuroreceptors ?

a. 5-HT2A antagonists enhance Rapid Eye Movement (REM) sleep


b. 5-HT1A antagonists are anxiolytics
c. Most typical antipsycotics are D2 agonists
d. D2 receptors are found in the limbic system
e. α2-adrenergic agonists cause increase of norepinephrine release

94. A 56 years old male with history of cronic benzodiazepine use for insomnia is brought to Emergency Department (ED) by his
daughter with complaints of sedation, incoherence and problems with coordination. His daughter states that he had been using
diazepam 10-20mg every night for many years. She also recalls that a few days ago her father saw his family physician and got a new
medication for some other problem. The ED physician checks the current medication list and believes that the patient has
benzodiazepine overdose. Which of the following can increase the levels of benzodiazepine?

a. Topiramate
b. Cimetidine
c. Phenytoin
d. Carbamazepine
e. Rifampicin

95. Selective mutism is defined as:

a. Refusal to speak in learning situations


b. Inability to speak in specific social situations
c. Occasional refusal to speak in school
d. Consistent refusal to speak in specific social situations
e. None of the above
96. First symptom of Attention Deficit Hyperactivity Disorder (ADHD) to remit is usually:

a. Hyperactivity
b. Distractibility
c. Careless mistakes in school work
d. Impulsivity
e. Learning difficulties

97. In Tourette’s disorder, the initial tics are in the:

a. Face and neck


b. Arms and hands
c. Body and lower extremities
d. Respiratory system
e. Alimentary system

98. Before issuing diagnosis of enuresis, a urinalysis must be done to rule out which common condition?

a. Bladder infection
b. Structural malformation
c. The common cold
d. A urinary tract infection
e. Urethra stenosis

99. Major depression disorder in school aged children?

a. May present as irritable mood rather than depressed mood


b. Usually includes pervasive anhedonia
c. Includes mood-congruent auditory hallucinations less commonly than in adults with the same disorder
d. Is never diagnosed in the context of bereavement
e. None of the above

100. Which of the following is not a negative psychotic symptom?

a. Flat affect
b. Apathy
c. Disorganized thinking
d. Avolition
e. Poverty of thoughts

101. Which of the following psychotic symptoms commonly co-occure with adolescent substance use disorder?

a. Suicidal ideation and suicide attempts


b. Panic attacks
c. Re experiencing numbing and avoidance
d. Binging and puring
e. All of the above
102. Risk factors for the development of adolescent substance abuse includes all of the following except:

a. Early onset of cigarette smoking


b. Diminished parental supervision
c. Pervasive developmental disorder
d. Parental substance abuse
e. Conduct disorder

103. Dementia of the Alzheimer`s type account for what percentage of old age dementia?

a. 10%
b. 20%
c. 40%
d. 60%
e. 80%

104. Elderly persons taking antipsychotic are especially susceptible to the following side effects except?

a. Akathesia
b. Paresthesias
c. Dry mouth
d. Tradive dyskinesia
e. A toxic confusal state

105. Which of the following factor is common in victims of elder abuse?

a. Alcohol abuse
b. Social isolation
c. Mental illness
d. Hostility
e. Prescription drug abuse

106. Which of the following is the greater risk factor for suicide?

a. Unemployment
b. Single status
c. Low income status
d. Resident of urban area
e. History of admission to a psychiatry hospital

107. Which of the following electrolyte abnormalities is associated with bulimic patients?

a. Metabolic acidosis
b. Respiratory acisdosis
c. Metabolic alkalosis
d. Respiratory alkalosis
e. Normal electrolytes
108. A 42-year old man is seen in your office. He has recently seen a psychiatrist and is being treated for severe depression. The patient
cannot recall the medication that he is taking, but remembers that his psychiatrist told him not to eat cheeses or aged meats. Which of
the following agent is this patient most likely taking?

a. Tricyclic antidepressant
b. Selective serotonin release inhibitor
c. Monoamine oxidase (MAO) inhibitor
d. Neuroleptic
e. Anxiolytic

109. High fever, tachycardia, tachypnea, diaphoresis, hypertension and seizures develop in a psychiatric patient who is receiving
haloperidol. The most likely diagnosis?

a. Malignant hypertension
b. Rhabdomyolysis
c. Neuroleptic malignant syndrome
d. Sepsis
e. Serotonin syndrome

110. Children who exhibit symptoms of school avoidance with nausea, vomiting and abdominal pain may benefit from?

a. Antidepressant medication
b. Methylphenidate
c. Psychotherapy
d. Benzodiazepine
State Exam 20.09.2012
By: Alhag Abu Anzeh Muhammad

Part A

1. 6 years old boy has stool in his underwear daily. He was toilet trained at 2 without difficulty. Over the
last2 years he had developed chronic constipation. The fecal soiling developed over the last 3 months. He
is otherwise normal. Examination reveals stool in rectal vault. Initial management should include which of the
following?

a. barium enema followed by rectal biopsy


b. time out from school when he has stool in his underwear
c. family consulting
d. clear fecal impaction and short term stool softener use
e. daily enemas for 4 weeks

2. A 2 year old child has multiple episodes of brief shrill cry followed by a prolonged expiration and apnea.
He was born after normal pregnancy and delivery. Growth and development are normal. The first episode
occurred immediately after the mother refused to give the child some juice; the child became cyanotic,
unconscious, and had generalized clonic jerks. A few moments later the child awakened and had no
residual effects. Physical examination is normal. Which of the following is most likely diagnosis?

a. Seizure disorder
b. Drug ingestion
c. Hyperactivity with attention deficit
d. Pervasive development disorder
e. Breath-holding spell

3. A 5-month-old child with poor growth presents to the ER with generalized tonic-clonic seizure activity of about 30-
minute duration that stops upon the administration of lorazepam. Which of the following information gathered from
the mother will be most helpful?

a. The child has had rhinorrhea


b. The child is developmentally normal, as are his siblings
c. The mother has been diluting the infant’s formula to make it last longer
d. number and type of pets
e. The mother is single and unemployed

4. Full term newborn infant is having episodes of cyanosis and apnea which are worse with attempting to feed but
seems better with crying. Which of the following is most important next step to quickly establish the diagnosis?

a. Echocardiogram
b. Ventilation perfusion scan
c. Passage of catheter into nose
d. Hemoglobin electrophoresis
e. Bronchoscopic evaluation of palate and larynx

5. 3 month old child is been crying at the same time each day, you suspect colic. What is true about colic?

a. onset at 3 month and peak at 6 months


b. almost all cases are due to lactose intolerance
c. typical at afternoon and evening
e. with any degree of abdominal distension should have CT abdomen
6. 3 year old has persistent rhinorrhea for the last past 6 weeks. Otherwise the patient has been
asymptomatic. On physical examination you notice mouth breathing and dark circles under the eyes. In the nose,
watery discharge and edematous boggy bluish mucous membrane with no erythema. Most likely diagnosis:

a. chronic bronchitis
b. sinusitis
c. nasal foreign body
d. allergic rhinitis
e. CSF leak

7. 5 year old immunized girl is seen in the emergency department because of fever, sore throat and
respiratory distress that has developed in the last 3 hours. She is drooling, holding her neck in a
hyperextended position, she has mild stridor but does not have barky cough. The patient most likely to have:

a. viral croup
b. spasmodic cough
c. epiglottitis
d. bacterial tracheitis
e. asthma

8. A 9 mounts boy, previously healthy and normally gaining weight was recently discharge from the
hospital. The hospitalization was due to a severe rotavirus diarrhea that requires rehydration. 4 days later the baby
still has loss stools. He is now drinking his regular cow's milk formula well. On physical examination he is happy and
well hydrated. Repeat rotavirus test is negative. What is the reason for the baby's continued diarrhea?

a. cow milk protein allergy


b. starvation diarrhea
c. secondary lactose intolerance
d. viral gastroenteritis
e. cystic fibrosis

9. a mother brings her 2 year old baby for regular checkup. She is confused about which position to put her baby to
sleep. With regard to infant sleeping position, how should you consult the mother?

a. place the baby on the belly


b. place the baby on the side
c. elevate head of the baby's crib
d. elevate the foot of infant crib
e. place on the back

10. The most common neurological sequela associated with bacterial meningitis, and usually presents at the time of
initial infection is?

a. mental retardation
b. chronic seizure disorder
c. impaired vision
d. impaired hearing
e. behavioral disturbances

11-THE MOST COMON CAUSE OF VIRAL MENINGOCEPHALITIS IN CHILDREN IS:

A-ENTEROVIROS
B-HERPES SIMPLEX VIROS
C-ARBOVIROS
D-MUMPS VIROS
E-RESPIRATORY VIROS
12-A 12 YR OLD BOY PREZENTS WITH A 1 YR HISTORY OF WORENING POLIYURIA AND A 2 TO 3
WEEKS HISTORY OF NAUSEA ,FATIGUE AND MALAISE . SERUM CREATENIN IS 4.0 MG .BICARBONAT 15
MG, URINALISIS SHOWS SPECIFIC GRAVITY OF 1.OO4 , TRACE LEUKOCYTES , TRACE BLOOD , AND NO PROTEIN , WITH
3-5 WBS PER HIGH POWER FIELED , 3-5 BBC PER HIGH POWER FIELED , AND NO RBC CASTS. WHICH OF THE
FOLLOWING IS THE MOST LIKELY DIAGNOSIS:

A-ACUTE POST STREPTOCOCAL NEPHRITIS


B-CHRONIC INTERSTITIAL NEPHRITIS
C-MINIMAL CHANGE NEPHROTIC SYNDROM
D-ACUTE INTERSTITIAL NEPHRITIS
E-CHRONIC GLOMERONEPHRITIS

13-TRISOMY 21 IS ASSOCIATED WITH:

A-MALROTATION
B-HYPOTONIA
C-CLEFT PALATE
D-RENAL DISEASE
E-PER CAVUS

14-PECAUSE OF SPLENIC DYSFUNCTION AND AN INCREASED RISC OF BACTERIAL


INFECTION , CHILDREN WITH SIKLE CELL ANEMIA SHOUD PLACED ON PROPHYLACTIC PENICILIN
VK BY 4 WEEKS . AGE CHELDREN WITH SIKLE CELL ANEMIA ARE PARTICULARY SUSCEPTIBLE TO :

A-GRAM NEGATIVE RODS


B-ENCAPSULATED ORGANISMS
C-FUNGAL INFECTION
D-VIRAL INFECTION
E-STAPHLOCOCUS AUREUA

15-YOU HAVE JUST GIVEN A 10 YR OLD AN INJECTION OF POLLEN EXSTRACT AS


PRESCRIPED BY HIS ALLERGIST IN HIS CHEST. YOU NOTE THAT HIS FACE IS FLASHED AND HIS
VOICE SOUNDS MUFFLED AND STRAINED. WHICH OF THE FOLLOWING IS THE FIRST PRIORITY IN
MANAGIG THIS EPISODE OF ANAPHYLAXIS:

A-ENDOTRACHEAL INTUBATION
B-INTRAMUSCULAR INJECTION OF DIPHENHYDRAMINE
C-ADMINISTRATION OF INHALED BRONCHODILATOR
D-INTRAMUSCULAR INJECTION OF EPINEPHRINE
E-ADMINISTRATION OF INTRAVINOS CORTICOSTEROIDS

16-A 15 YR OLD GIRL WITH SHORT STATURE , NECK WEBBING AND SEXUAL INFANTILISM IS FOUND
TO HAVE COARCATION OF AORTA. A CHROMOSOMAL ANALYSIS IS LIKELY TO DEMONSTRATE
WHICH OF THE FOLLOWING:

A-MUTATION AT CHROMOSOME 15 Q 21.1


B-TRISOMY 21
C-45 XO KARYOTYPE
D-DEFECT AT CHROMOSOME ANALAYSIS
E-NORMAL CHROMOSOME ANALYSIS:
17-A 2 MONTH OLD BOY IS SEEN FOR NONBILOUS VOMITING AFTER HE HAD BEEN
BREASTFEEDING WELL UNTIL 2 WEEKS AGO, WHEN VOMITING WORSENED. ON PHYSICAL EX HE HAS
JAUDANCE AND GASTRIC PERISTALTIC WAVES ARE SEEN ON THE ABDOMEN.
BLOOD TEST REVEAL HYPOKALEMIC , HYPOCHLOREMIC ALKALOSIS. THE MOST LIKLEY
DIAGNISIS IS:

A-ACUTE HEPATITIS
B-DUODENAL ATRESIA
C-UTI
D-PYLORIC STENOSIS
E-MILK ALLERGY

18-YOU ARE SEEING A 10 YR OLD BOY WITH COMPLAINT OF DOUBLE VISION ESPECIALLY IN THE
AFTERNOON. HE TIRES FASTER THAN HIS CLASSMATS IN GYM CLASS DUE TO
MUSCLE FATIGUE WITHAWT RESPIRATORY SYMPTOMS. HIS PUPILS AND VISUAL ACUITY ARE
NORMAL BUT YOY NOTICE A DISTINCT PTOSIS. THE MOST LIKELY DIG IS:

A-PITUITARY TUMOR
B-AMBLYOPIA
C-BOTULISM
D-MYASTHENIA GRAVIS
E-TICK PARALAYSIS

19-A 4 MONTH OLD CHILD WITH VITAMIN D DEFICIENTY RICKETS WOULD BE EXPECTED TO SHOW
ALL OF THE FOLLOWING EXCEPT:

A-CRANIOTABES
B-BOWLEG
C-ROSARY
D-LOW SERUM PHOSOHATE LEVELS
E-HIGH ALKALINE OHOPHATASE LEVELS

20-WHICH OF THE FOLLOWING CAUSES OF CONGINITAL INFECTION IS ASSOCIATED WITH CATS:

A-CYTOMEGALOVIROS
B-RUBELLA
C-TOXOPLASMA GONDO
D-SYPHILIS
E-PARVOVIROS B 19

21. Obese, 9 years old girl, history of persistent headache and blurred vision. Mild vomiting . No fever. In physical
examination papilledema. Visual field testing inferior nasal blind spot. MRI normal. LP pressur IS elevats, CSF
composition IS normal. Lab. Normal. What IS the most likely diagnosis?

Pseudotumor cerebri

22. 4 day old infant, noted to be sucking on a honey- filled pacifier. You caution the mother against its isr because its
contents can cause:

Infantile botulism

23. Infant 8 day old, high TSH and low T4. If this condition IS Left untreated:

Prolonged jaundice
24. Complication of hypernatremic dehydration IS:

Cerebral edema.

25. you are evaluating an Obese, 10 years old boy for diabetes. He has strong Family history of type2
diabetes mellitus. His mother is concern about recent symptoms of polyuria and polydipsia. What of the following is a
diagnostic for diabetes mellitus:

a. Non fasting plasma glucose of 210 mg/dl


b. fasting plasma glucose of 110 md/dl
c.2-hour glucose during glucose tolerance test of 165mg/dl
d. acanthosis nigricans on neck
e. symptomes alone are enough to make the diagnosis.

26. Hb 8.5 mcv110fl, reticulocyt count 0.5. Most likely diagnosis is:

Vitamin B12 deficiency

27. Women O positive give birth at term infant A positive. Ht 55% , bilirubin 12 mg/dl. Wich of the following
lab findings ABO hemilytic disease:

a positive coombs test

28. 1 years old, failur to thrive, frequent large voids of diluted urine, excessive thirst and episods of
dehydration without vomiting and diarea. Over the years family members reports similar history . The most likely
diagnosis is:

Diabete insipidus.

29.10 years old , Is having "cold" for 14 days, purulent nasal discharge, facial pain. Most likely diagnosis:

Maxillary sinusitis

30. A 6 month infant with large foul smelling stools Is not gaining weight.

Chloride 68 ????

31. A 5 month old child regularily regurgitates large portions of her feeds. PH probe study showed significant
periods of low esophageal PH. The child has normal growth and no other significant past medical history. Which
is the best management at this point:

a. barium swallow and upper (GL) gastrointestinal series


b. oral reflux medications
c. esophageal manometry
d. close observation only
e. surgical correction

32. a 6 week old is admitted with jaundice, total Bilirubin is 12mg/dl with direct Bilirubin 5mg/dl. What is most likely
the disorder?

a. ABO incompability
b. biliary atresia
c. Rh incompability
d. Gilbert disease
e. breast-milk jaundice
34. a 2 year old boy has Teratology of fallot. Today he arrives with acute febrile illness. On examination,
he complains of headaches, lethargy, has nystagmus and ataxia. What will be the most appropriate first test
to order?

a. Urine drug screen


b. Blood culture
c. lumbar puncture
d. CT of brain
e. echocardiogram

41- 2 Year old boy is seen because of pallor. His mother sees e drinks 4 glasses of cow's milk a day. A
CBS reveals a hemoglobin of 8.2 g/DL an a MCV of 65 fl. Which of the following is compatible with this patients?

A-decreased red blood cell distribuition


B- decreased serum feritin
C- increased reticolocyt count
D- increased serum iron level
E- increased white blood cell count

42- Which of the following reflexes is normally absent in a newborn ?

A- STARTLE (moro)
B- hand grasp
C- crossed adductor
D- Asymmetric tonic neck
E- parachute

43- A 2 year old boy is admitted to the hospital with high fever four 6 days, swelling of the ands and feets,
scarlentiniform changes of the tongue, generalized reed macular rash, dray sedmentation reet &
thrompocytosis. Which of the following is the best initial mangmanet?

A- cardiac catheterization
B- IVIG & aspirin orally
C- naproxen orally
D- I.V pulse corticosteroids
E- methotrexate orally

44- A 9 month old girl present to the ER with respiratory distress, grunting & cyaiosis . u start O2 by face mask, but
the child is still in a significant respiratory distress & become apnoic. The best next step is?

A- start chest compretion


B- performe Heimlich maneuver, maybe the child aspirate something C- obtain
blood gas
D- send the child for RX-chest
E- start bag - valve mask ventilation with 100/100 o2

45- A 5 year old boy present with a history of grossly blood urine, puffy eyes & headache for 1 day. He
has been will with fever & sore throat about 10 days which resolved with out treatment;
most likely diagnosis:

A- acute cystitis
B- IgA nephropathy
C- acute pyelonefritis
D- postinfaction glomerulonephrtis
E- benign hematuria
46- 10 years old female with fever, pain , swelling in her left elbow for 2 days & right knee for 3 days,
today her right knee us OK . The arthritis was preceded by a 3 days history of fever & sore throat 2
weeks ago. Laboratory results reveal un elevated antistreptolwsin. The most likely diagnosis us ?

A- S.L.E
B- idiopathic R.A
C- acute rheumatic fever
D- gonococal arthritis
E- psoriatic arthritis

47- Which of the following cancer occurs primarily during childhood?

A- Breast cancer
B- Renal cell cancer
C- Wilms tumor
D- Thyroid cancer
E- Colon cancer

48- The most important extramedullary site of relapse in childhood acute lymphoblastic leukemia is?

A- Adrenal glands
B- kidney
C- lung
D- heart
E-Central Nervous System

49- Very-low-birth -weight ( < 1500 g ) infant are best describes as?

A- predominantly growth restricted


B- predominantly PREMATURE
C- predominantly postdate
D- the result of maternal illness
E- the result of placental infraction

50- An 18 year old female, u have followed for 6 years, has recently married , she & husband are planning to
start a pregnancy .u advise her to start taken folic acid , this is important to prevent ?

A- prematurity
B- skull defects
C- chromosomal defects
D- neural tube defects
E- osteopenia

51) A 75 year-old man is complaining that for several months he has been experiencing increasing
fatigue. He has a history of hypertension, gastroesophageal reflux disease, and large diaphragmatic hernia. He has not
experienced any weight loss or other general symptoms.On physical examination, conjunctival pallor is noted. A fecal
occult blood test(FOBT) is positive. Laboratory testing reveals iron deficiency anemia. Which of the following is most
likely cause of this patient’s anemia?

a. Gastric cancer.
b. Colorectal cancer.
c. Angiodysplasia.
d. Peptic ulcer disease.
e. Diaphragmatic hernia and esophagitis.
52) A 67 year-old man with a medical history of hypertension presents is complaining of pain of
acute onset in his right knee. He was well at the time he went to bed last night. He denies undergoing any trauma. On
examination of the right knee: range of motion is limited, surrounding erythema and an effusion are noted. All other
joints are normal. For this patient, which of the following measures can be used to confirm the diagnosis of acute
gout?

a. Serum uric acid measurement.


b. Arthrocentesis.
c. Trial of steroid injection for pain relief.
d. Trial of colchicines for pain relief.
e. X-ray of the knee.

53) A 36 year-old man who contracted hepatitis B 15 years ago is checked for viral serologies to
better understand his hepatitis B status. Test results are negative for hepatitis B surface antigen
(HBsAg), hepatitis B surface antibody (HBsAb), and IgM antibody to core protein (IgM anti HBc), but the patient test
positive for total anti-HBc. What is the best explanation for this patient’s serologic profile?

a. He has acute hepatitis B infection.


b. He has chronic hepatitis B infection.
c. He has remote hepatitis B infection.
d. He has no reliable evidence for ever having been infected with virus hepatitis B.

54) a 43 year old woman was recently diagnosed with diffuse scleroderma . for the past two years she has
been experiencing skin thickening with pain and itching. She also reports increasing difficulty with movement
of her knees because of thick skin. She denies having any shortness of breath, on examination the patient’s
blood pressure is 145/92 mmHg, her pulse is 84 beats/min, and her oxygen saturation is 97%. On room air,
the remainder of the physical examination is consistent with her diagnosis of diffuse scleroderma . laboratory
data show blood urea nitrogen (BUN) value of 24 mg/dl and a creatinine chemistries urinalysis shows 1+ and
2+ protein. Six months ago, her blood pressure was 125/78 mg/tg and her BUN and creatinine were 12 mg/dl
and 0.9 mg/dl, respectively which of the following interventions is most appropriate for this patient of this
time?

a. Initiation of angiotensin - converting enzyme (ACE) inhibition.


b.initiation of calcium channel blocker.
c. Initiation of a thiazide diuretic.
d. Renal biopsy.
e. Close monitoring for further elevation of blood pressure.

55) A 64 year old man is found to have a markedly elevated absolute lymphocyte count during a preoperative
evaluation for a total knee arthroplasty. He denies experiencing any weight lose, night sweets, or ” lumps and
bumps”, which of the following is required to make the diagnosis of CLL in this patient?

a. An ALC (absolute lymphocyte count) greater that 10,000/ml


b. Identification of a monoclonal B cell population with characteristic phenotype by flow cytometny .
c. A bone marrow biopsy with characteristic findings.
d. Thoracic, abdominal, on peripheral lymphadeno pathy .

56) A 71 year-old man with a medical history of CCL presents with new-onset fevers and night sweats, as well as
rapidly enlarging nodes in his right axilla. You are concerned about richter transformation.
Which of the following is the best diagnostic intervention of this patient?

a. Serum flow cytometry.


b. Fine-needle aspiration of axillary node.
c. Excisional lymph node biopsy.
d. Bone marrow biopsy.
57) A 32 year-old man presents with a testicular mass. His examination is unremarkable except for a palpable, firm
left testicular mass. The presence of which of the following markers signifies a nonseminimatous testicular cancer?

a. human chorionic gonadotropin(hcG).


b. CA-125.
c. CA 19-9.
d. α-fetoprotein (AFP).
e. carcinoembryonic antigen(CEA)

58) A 38 years old man with a history of testicular cancer presents with fatigue, dyspnea on exertion, and
generalized weakness, he underwent chemotherapy 10 years ago. Subsequent serial evaluations have not revealed
any remaining testicular cancer. On physical examination, the patient is found to have pale conjunctivae and nail
beds, as well as scattered lower-extremity petechiae.
What possible long-term side effects of chemotherapy could be responsible for this patient’s symptoms?

a. Cardiovascular disease.
b. Hypercholesterolemia.
c. Leukemia.
d. Cerebrovascular disease.
e. Retroperitoneal fibrosis.

60) A 72 year old woman complains of fatigue, easy bruising, weight loss, and abdominal distention. Her pulse is
slightly fast, and she has notable pallor and splenomegaly. Blood counts reveal anemia and thrombocytosis, you are
concerned that she may have myelofibrosis, what would you expect to see on this patient's peripheral blood smear if
she did have myelofibrosis?

a. Teardrop cells
b. Schistocytes
c. Burr cells
d. Acanthocytes

61. A 38 y/o woman with severe asthma is complaining that for the past 2 days she has been experiencing progressive
shortness of breathe. On examination, patients respiratory rate is 18 BPM, room air saturation: 86%. She had a few
expiratory wheezes with little air movement.
Results of arteriole blood gas: PH;7.2, PaO2; 62 mmHg, PCO2; 63 mmHg.
On the basis of the arterial blood gases measurements which of the following BEST describes this patients condition:

A) Acute hypoxemic respiratory failure


B) Acute hypercarbic respiratory failure
C) Chronic hypercarbic respiratory failure
D)Mixed hypoxemic and hypercarbic respiratory failure

62. A 37 y/o woman is brought for evaluation for altered mental status. She has
no medical problems and only takes OCP.
Her husband reports that the patient has had a fever for 2-3 days.
On examination, patient is febrile (38.9 C), conjunctivae are pale.
She has palpable purpura on her lower extremities .
Lab results: Hb; 6 g/dl, Plt count; 20,000/mm3, creatinine;2.8mg/dl.
Peripheral smear shoes evidence of microangiopathic hemolysis.
which of the following interventions is the MOST appropriate step to take next for this patient?

A) Platelets transfusion t o raise PLT > 50,000/mm3


B) platelets transfusion to raise PLT > 100,000/mm3
C) Initiation of aspirin therapy
D) Plasma exchange for 6 session
E) surgical for splenctomy
63. A 72 y/o woman with COPD and hypertension presents for a follow up visit.
She has recently been admitted for an acute exacerbation of her COPD.
Her most common recent spirometry results shows an FEV1/FVC ratio of 0.48.
Which of the following interventions has been shown to reduce the number of acute COPD exacerbation?

A) Short acting anticholinergic inhalers


B) short acting inhalational Beta-agonists
C) Inhaled corticosteroids
D) Oxygen therapy.

64.A 52 y/o woman smokes ½ pack of cigarettes per day and drinks 1 bottle of beer a week. She has GERD for
which she takes PPI. Last year her father was diagnosed with esophageal cancer.
For this patient which of the following risk factor for esophageal cancer?

A)Tobacco use
B)Alcohol
C) GERD
D) all of the above

65. A 47 y/o woman with SLE is complaining of chest pain, fever and cough of 5 days duration. The results of
physical examination are: significant temp. of 38.2C, BP; 95/50 mmHg, HR;114 BPM. Lung examination reveals
inspiratory crackles at the bases. CXR shows diffuse infiltrates and a right sided effusion. Patient undergoes
thoracosynthesis. Which of the following thoracosynthesis findings would be consistent with a lupus effusion?

A) excudate
B) transudate
C) very low glucose
D) + gram staining
E) high WBC count

66. A 40 year old women undergo a routine abdominal ultrasound, There is no inflammation but two stones
are found in size less then 5 mm she denies any symptoms and she has no past , what is the best next step:

A. Cholecystectomy
B. ERCP
C. Observation
D. PTC
E. MRCP

67. A 18 year old with easy brusing, lymphocytes 6000 platelets 200,000 hemoglobin 13 PT 13 PTT 70 which was
rechecked 67 . What is the most likely diagnosis?

A. Factor 7 deficiency
B. lupus
C. Factor 7 inhibition
D. Glanzmam thrombasthesis

68. A 63 year old man with irregular rhythm who walks 2 mile with no dyspnea was checked in a 12 lead ECG and
found to have mobits 2 heart block and LBBB , when Is a pacemaker indicated ?

A. In asymptomatic it is not indicated


B. indicated when there is a stable rhythm
C. Indicated only when there is a widen QRS
D. in all AV block it is indicated
69. A 65 year old man with intermittent palpitation for 2 months, the palpitation are unpredicted from
several minutes to hours. The paitents history has well controlled hypertension and he denies alcohol use.
On ECG there is atrial fibrillation with 94 BPM and left ventricular hypertrophy what is the best next step ?

A. Cardiac catherisation
B. Trans esophagus echocardiogram
C. Pulmonary function test
D. CT Angio of chest

70. A 78 year old man with hypertension has shows on ECG atria fibrillation what best describes the use of oral
anticoagulant ?

A. High risk pt over 75 y are contraindicated


B. low risk for thromboembolic are not indicated
C. The combination of aspirin and Clopidrogrele is equivalent to warfarin in high risk pt
D. Warfarin shown superiority to aspirin in patients over 75 years

71. a 35 years old woman who was diagnosed with atrioventriculare node reentry tachycardia but is not taking
medications is admitted to the hospital for a hysterectomy. The night before the surgery she
develops tachycardia with heat rate of 114 bpm. Which of the following is the best medication to slow this patient's
heart rate and establish sinus rhythm:

a. amiodarone
b. procainamide
c. adenosine
d. propafenon

72. a 23 years old man with type I DM presents to routine diabetes management. He had difficulty with
adherence and has been managed with twice daily fixed dose insulin. Today his HbA1 is 9%. Which of the following
insulin regimens would offer the best glycemic control for this patient:

a. long acting insulin injected daily


b. rapid acting insulin injected before meals
c. long acting insulin injected daily and rapid acting insulin before meals
d. continuing current fixed dose insulin mixture injected daily

73. an 82 years old woman with history of breast cancer present to the clinic with complains of headache for the past
2 weeks. She describes daily retro orbital headaches that are worse in the morning. In addition, she notes severe
nausea and vomiting. MRI of the brain reveals multiple metastatic lesion. Which of the following is the most
appropriate initial management for this patient:

a. Biopsy the lesions


b. Surgical resection
c. Dexamethasone
d. Repeat MRI in 6 months

74. a 42 years old man with a medical history significant for lung cancer is brought because of altered mental status.
On examination he has orthostatic hypotension and dry mucous membranes. Laboratory evaluation reveals Ca 14.5
mg/dl and creatinine 2 mg/dl. Which of the following is the most appropriate initial management of this patient:

a. 0.9% saline bolus


b. Furosemide
c. Pamidronate
d. 0.45% saline bolus
75. A 25 years old man was recently diagnosed with leukemia. Laboratory evaluation reveals
pancytopenia, hyperkalemia (K 6.8 Meq/l), uric acid 13 mg/dl, hyperphosphatemia 12 mg/dl and elevated lactate
dehyrogenase (LDH). What is the most likely cause of his electrolyte abnormalities:

a. Laboratory error
b. Tumor lysis syndrome
c. Acute renal failure
d. Bone metastases

76. a 75 years old man with lung cancer complains of progressive dyspnea for 2 months. On physical examination
he has distended jugular veins, plethoric face and venous collaterals on his chest wall and distant heart sounds. What
is the most likely diagnosis:

a. Pericardial tamponade
b. Constrictive pericarditis
c. Congestive heart failure
d. Superior vena cava syndrome

77. a 72 years old woman with history of hypertension and tobacco use is complaining about chest pain for the last 2
weeks. She describes the pain as pressure like with radiation to the left arm. It is exacerbated by exertion and relieved
by rest. There are no associated symptoms. Her physical examination is unremarkable. ECG shows signs of left
ventricular hypertrophy. Laboratory test including cardiac biomarkers are within normal limits. Which of the following
is the most likely diagnosis:

a. Non STEMI
b. Pericarditis
c. Unstable angina
d. Stable angina

78. a 47 years old woman is evaluated 2 hours after the onset of acute right sided weakness. A noncontrast CT of the
head showed no evidence of hemorrhagic event. Her blood pressure at this time is 190/120
mmHg. Which of the following statements concerning blood pressure and the treatment of acute ischemic stroke is
the most accurate:

a. Elevated blood pressure should be aggressively treated in this setting


b. Cerebral blood flow is independent of intracranial pressure
c. Antihypertension therapy is needed in this patient before giving thrombolytics
d. Hypertension should never be treated in the setting of acute ischemic stroke

79. a 45 years old woman is evaluated for nausea, vomiting and abdominal pain of 3-4 weeks duration.
Further history reveals polyuria and muscle weakness although physical examination does not reveal any
abnormalities. Her serum Ca level is 13 mg/dl, intact parathyroid hormone is elevated and she is diagnosed with
primary hyperparathyroidism. Which of the following is the next step in the management of this
patient's hypercalcemia:

a. Referral for parathyroidectomy


b. Prescribe hydrochlorothiazide
c. Biophosphonate therapy regardless of her bone mineral density
d. Repeat serum calcium in 3 months
80. a 70 years old man with squamous cell lung cancer is admitted with altered mental status. The patient's
wife reports he has become increasingly confused over the last 2 days. Physical examination is significant for heart
rate of 115 bpm and dry mucosa membranes. The patient is somnolent and unable to follow commands. Laboratory
serum Ca level is 15.1 mg/dl. What is the most appropriate therapy for his hypercalcemia:

a. Fluid restriction with loop diuretics


b. Intravenous zoledronic acid
c. Subcutaneous calcitonin
d. Intravascular volume expansion with intravenous normal saline

90. A 55 hypertension history , 1st step of manegment:

a.Thrombolytic therapy
b.i.v morphine
c.PCI
d.markers

91.50years old male with inferior Myocardial infarction :

a.lt heart cath’


b. angioplasty
c.surgical repair
d. angioplasty

92. A 65 years old with Diabetes mellitus, hyperlipidemia and tobacco abuse presents with chest pain for the past 3
days. The pain is worse with exertion and was relieved by sublingual nitroglycerin until today
when pain persisted. The pain radiates to the jaw and is associated with nausea and diaphoresis. On
examination the blood pressure is 90/50, HR 96, on ECG there are ST depression in the inferior leads and the CK-MB
and troponin levels are elevated. Which of the following medication should be avoided in thr management of this
patient?

a. heparin
b. aspirin
c. nitrates
d.ACE

93. A 40 year old female with a history of coronary artery disease, status post-percutaneous coronary intervention
with a drug-eluting stent 1 year ago, presents to the emergency department with fever and right upper quadrant pain
for 1 day. An abdominal sonogram reveals gallstones with likely cholecystitis and no biliary ductal dilation. She is
admitted for antibiotics and a general surgery consultation, the surgeon is concerned because the patient is on
clopidogeral for a drug-eluting stent. If possible, how long should surgery be delayed after stopping the patients
clopidogeral:

a. 1 day.
b. 7 days.
c. 1 month.
d. 2 weeks.
94. A 40 y old man with a history of IV drug abuse presents with fever and chills for 1 week. His examination
reveals a new holosystolic murmur at the left lower sternal border that increase with inspiration. He is febrile
at 38.9C. You plan to admit him for work up and treatment of infective endocarditis. what duration of IV
antibiotics is necessary for this patients illness.

a. 4 to 6 weeks.
b. 7 to 10 days.
c. 3 months.
d. 1 year.

95. A 70 year old man with a history of hypertension and diabetes presents to the management of chronic
angina for the past year. His chest pain now occurs one or 3 times per month, typically last 5 minutes, and
are relieved by rest and sublinguinal nitroglycerin. A recent pharmacologic stress test showed a reversible
perfusion defect. What is the most appropriate next step in the management for this patients angina:

a. Add morphine to the patients regimen.


b. Coronary angiography
c. Lifestyle modification including exercise
d. Continue with the current management.

96. A 59 year old female with a history of a heart murmur but no other symptoms or rheumatic fever. The murmur is
2/6 crescendo decrescendo that radiates to the carotid artery. The murmur decreases with
valsalva maneuver and on standing, the most likely diagnosis is:

a. aortic stenosis
b- tricuspid regurgitation
c- mitral regurgitation
d- hypertrophy obstructive cardiomyopathy

97. 60 years female ST elevation MI with Left ventricular ejection fraction of 25%, treatment:
a.furasamide
b. coronary bypass
c. icd implant
d. amlodipine

98. 29 old female with cardiac arrest most likely diagnosis:

a. brugada syndrome
b. long QT syndrome
c. MI
d. stroke

99. A 55 year old male with urosepsis started on IV fluid and pipercillin-tazobactam. His symptoms
gradually improved until hospital day-5, when he develops a rash and low grade fever. Lab tests reveal an
increased creatinine and a urinalysis with blood cells and eosinophils. What is the most likely diagnosis?

a. prerenal disease
b. Nephrotic syndrome
c. acute tubular necrosis.
d. acute interstitial renal disease
100. A 40 years old woman with history of hypertension & diabetes visit your clinic with the compliant of
weight gain. On examination you find - obese female , moon facies and supraclavic fat pads. You consider
cushing syndrome as diagnosis. for this patient, what is the best diagnostic test for cushing syndrome?

a. high dose dexamethasone suppression test


b. 24-h urine free cortisol
c. random cortisol level
d. plasma adrenocorticotropic hormone level

101- What is the recommendation concerning the administration of MMR vaccine during pregnancy?

a. Contraindicated
b. The same as in non-pregnant woman
c. Only for post exposure
d. Only if there is risk factors
e. Should be administer if there is no………

102- In which gestational age laboratory screening for diabetes should be performed?

a.10-16week
b.16-20 week
c. 20-23 week
d. 24-28 week
e. 32-36 week

103- 31 year old woman G2P1 in 40 week gestation has progress in labor from 5 to 6 cm cervical dilation over 2 hours.
Which of the following best describe the labor?

a. Prolonged latent phase


b. Accelerate active phase
c. Arrested active phase
d. Protected active phase
e. Normal labor

104- What is the most common complication of Para cervical block?

a. Maternal hypotension
b. CNS toxicity
c. Fetal bradycardia
d. Bleeding
e. arrest of dilation

105- Which deleterious effect is associated with oxytocin when given intravenously as a 10 unit bolus?

a. Cardiac arrhythmia
b. Hypotension
c. Neuromuscular block
d. Seizure
e. Uterine relaxation
106- With prima with uneventful cervix and estimated fetal weight of 3800g which is the most appropriate
management at 42+0 week gestation?

a. To re date the pregnancy


b. Cesarean delivery
c. Hospitalization and bed rest
d. Cervical ripening, than labor induction
e. Epidural analgesia prior to induction of labor

107- Zygotic division to form dichroniotic diammniotic twins accrue during which of the following time period
following fertilization?

a. <72 hours
b. >72 and <120 hours
c. >120 and <240 hours
d. >264 hours

108- Ultra sonographic evidence pointing to the diagnosis of monochronionicity in pregnancy with twins include
which of the following?

a. T sign
b. Two separated placenta
c. Twin with different gender
d. Dividing membrane of more than 2 mm thick
e. Lambda sign

109- At what time are most non obstetric surgical procedure performed when indicated during pregnancy?

a. 1st trimester
b. 2nd trimester
c. 3rd trimester
d. Post-partum

110- Chronically hypertensive woman develop superimposed severe preeclempsia at 36 week of gestation, all of the
following regarding delivery are acceptable except:

a. Magnesium administration
b. Epidural anesthesia
c. Wait for delivery at term
d. Immediate induction of labor
e. Immediate c- section
Part B

1. A pregnant woman with HIV was admitted to hospital which of the following regimen effect pneumoscystisis carnii
pneumonia:

A. AMPICILIN
B. ERYTHROMYCIN
C. COTRIMOXAZOLE
D. AZITHRHROMYCIN
E. VANCOMYCIN

2. WHAT IS THE most common cause of acute renal failure in pregnancy?

A. drug abuse
B.systimic lupus erythematus
c. Preeclampsia or eclampsia
d. Sickle-cell disease
E.dehydration

3. What pregnancy complication associated with ruptured appendix with peritonitis?

A. Fetal growth restriction


B. Oligohydroamnios
C. Chorioamnionitis
D. Preterm labor
E. Pylonephritis

5. What is the most likely course of rheumatoid arthritis in pregnancy:

a. Stable during and postpartum


b. Mild flare during and postpartum
c. Serious flare during, remission postpartum
d. Marked improvement during, flare postpartum.

6. Which of following correct regarding IVF complication?

A. with ivf pregnancy is an increase for multi fetus pregnancy but not for ectopic pregnancy
B. ONCE IVF IS ACHEVID THERE IS NO DIFFERENCE IN OBSTATRIC OUTCOME AS COMPARED TO SPONTANUS
PREGNANCY
C. OVARIAN HYPERSTIMULATION IS COMPLICATION OF IVF WHICH AGGREVATES WITH ACHIVED PREGNANCY
D. THERE IS SHORT INCREASE FOR GENITICS MUTATION OR DEFORMOTIES IN NEWBORN FOLLWING
ICSI

7.MARRID COUPLE OF 4 ATTEMPTS TO CONCIVE WITH NO SUCCES FEMALE EVALUATION NORMAL SPERMATIC
EVALUATION NORMAL SPERMATIC EVALUATION REVEALS 2ML, 2 MILION CELLS 50% MOTILITY AND 5% NORMAL
CELLS REPEAT SPECIMEN NOT SIGNIFICANT DIFFERENT WHICH OF THE FOLLWINGS IS APPROPIATE MANGMENT:

E. IN VITRO FERTILIAZATION WITH INTRA CYTOPLASMIC SPERM INJECTION


8. ALL OF THE FOLLWING ARE CLEAR complications OF IVF EXCEPT:

A. over all mobile sperm cell count lower than 2 millions cells
B. bilateral fallopian tube distress of the female partner
C. decrease ovarian reserve of the female partners
D. POS
E. SEVERE ENDOMETRIAL ADHESION

9. WHICH OF THE FOLLWING CANNOT BE USE SPERMOVULATION

A. human urinary goandotropins


B.recumbinant human FSH
C.recombinant human LH
D.highly purified human urinary goandotropin
E. recombinant androgen

10. Which of the following not considered a complication related to pregnancy achieved with fertility treatment?

A. multi fetus pregnancy


B.ectopic pregnancy
C.ovarian hyper stimulation syndrome
D.mullarian pregnancy
E. premature delivery

11) 12 years old female arrives with her upset mother to clinic that describe that her daughter has not
Mensterued yet. According to mother all the female spouses menstured by age and therefore this dealy is
concerning. On detailed history the child seems healthy asymptomatic. On physical physical examination the child has
appropriate growth, primary pubic hair, and early evidence of breast bud. What should be told to concerned mother?

A) This is primary amenorrhea that requires further evaluation.


B) This is primary amenorrhea but further evaluation should only be performed in 6 months.
C) This is not amenorrhea and therefore there is no reason for further evaluation.
D) The sex sign caused by estrogen administration and the exact preparation that she took should be evaluated.
E) Pelvic imaging is require to rule out mullerian agenesis.

12) 25 years old female seek consultation for amenorrhea. This is a healthy female that is sexually active of uses
condoms for contraception. History include regular menses 4/28, last menses about 3 month ago
amenorrhea. No abnormal event since. Which of following correct?

A) There is no reason for concern, progesterone should be administered to induce menses.


B) This is secondary amenorrhea and hormonal profile should be obtained to rule out premature ovarian failure.
C) It is likely that she has polycystic ovary syndrome and ultrasound should be performed to confirm the diagnosis.
D) This is secondary amenorrhea and therefore pregnancy should be ruled out first.
E) Hysteroscopy to demonstrated uterine cavity should be performed.

13) Which of the following defines primary amenorrhea?

A) Amenorrhea in the presence of secondary sexual development at age of 13.


B) Amenorrhea in the presence of secondary sexual development at age of 15.
C) Amenorrhea in the presence of secondary sexual development at age of 17.
D) Amenorrhea in the presence of secondary sexual development at age of 16.
E) Amenorrhea in the presence of secondary sexual development at age of 18.
14) Which of the following defined as primary amenorrhea?

A) Kallman syndrome.
B) Sheehan syndrome.
C) Asherman syndrome.
D) Pregnancy.
E) Premature ovarian failure.

15) Which of the following is a characteristic of female athletic syndrome?

A) Menses response to progestron administration.


B) Pituitary hypertrophy.
C) No menses is response to estrogen and progestron administration.
D) Hirsutism and acne.
E) Decreased bone mass.

16) Which of the following contraceptive methods shock syndrome has been associated?

A) Oral contraceptive pill (OCP).


B) Progestin only pill.
C) Male condoms.
D) Vaginal diaphragma.
E) Intrauterine contraceptive device (IUCD).

17) 36 years old multiparous female and her husband request information regarding vasectomy. You
should advice them that when compared to female sterilization the following is true (about vasectomy)?

A) Has the same failure rate.


B) Require a longer hospitalization.
C) Is effective sooner.
D) Carries a higher mortality rate.
E) Is less reversible.

18) A 49 years old female experiences irregular vaginal bleeding of 3 months duration. You perform an endometrial
biopsy, which obtains copious tissue with velvety, lobulated texture.The pathologist report show proliferation of
grandular and stromal element with dilated endometrial glands. Cyctologic atypia is absent. Which of the following is
the best way to advice the patient?

A) The presence of dilated endomaterial gland is usual.


B) The histological finding is called cystic atrophy.
C) The tissue may be weakly premalignant.
D) She requires a hysteroscopy.
E) No further therapy is needed.

19) What is the most common uterine sarcoma?

A) Leiomyosarcoma.
B) Endometrial stromal sarcoma.
C) Endolymphatic stromal myosis.
D) Malignant mixed mesodermal tumor.
E) Lymphoma.
20) Two days after surgery of 25 years women, you received the pathology report of the ovarian tumor. It is mucinous
cystadenoma of low Malignant potential mixed with well differentiated carcinoma. The tumor has not invaded the
ovarian capsule, lymphatic or mesovarium. Omental and retroperitoneal lymph node biopsies and peritoneal washing
are negative for tumor cells. How does you advice this patient?

A) Biopsy of the contralateral ovary.


B) Removal of the uterus and contralateral adnexa.
C) Postoperative chemotherapy.
D) Postoperative radiation therapy.
E) No further therapy.

21. 27 year old gravida 2 para1 is at your clinic on the 30th week of gestation for routine prenatal visit. She suffers
from “serosal fibroids”. The pregnancy is unremarkable thus far. The fundus is 37 cm from the
symphysis. In discussing possible complications of a fibroid uterus during pregnancy you mention that she is at a
highest risk for:

a. PROM
b. Placenta previa
c. Gestational diabetes
d. Breech presentation
e. Placental abruption

22. 55 year old woman in menopause presents to your office with a history of a 3 day light vaginal bleeding. You
should:

a. Prescribe vaginal estrogen for atrophic vaginitis and to come back if the bleeding isn’t better
b. A hysterectomy and bilateral salpingoophorectomy to rule out endometrial cancer
c. Take history, perform physical examination, endometrial tissue sampling and pelvic ultrasound
d. Recommend her to go on a diet, since there is increased production of estrone in obese women

23. Which of the following organs are first affected by spread and encroachement of advanced ovarian cancer:

a. Brain
b. Ureter
c. Intestine
d. Stomach
e. Spleen

24.52 year old woman after menopause complains of urinary frequency, urgency and urge incontinence. Otherwise
she is healthy. What should be included in behavioral treatment?

a. Relaxation techniques
b. Anticholinergic medications
c. Voiding every hour during daytime
d. Bladder retraining
e. Incontinence pad testing
25. 38 year old female with two year history of heavy menstrual flow of 9 days every period. Menses occur every 32
days. There is no bleeding between menses. Next step in evaluation:

a. Coagulation profile
b. Pregnancy test
c. Endometrial biopsy
d. Pelvic ultrasound
e. TSH evaluation

26. A 24 year old salesperson is referred for an evaluation of difficulty dealing with costumers because she
gets "so nervous and anxious that my mind gets blank". A careful evaluation revels that she has social
phobia. She is motivated to obtain treatment and continue doing her job. All of the following interventions, except:

a. Selective serotonin reuptake inhibition - SSRI's


b. Flooding
c. Modeling
d. Systematic desensitization
e. Electrical convulsion therapy - ECT

27. Factitious disorder with predominant physical signs and symptoms is also known as:

a. Hypochondriasis
b. Munchhausen
c. Munchhausen by proxy
d. Somatoform disorder
e. Factitious disorder not other specified

28. Which of the following is true about blood-injection-injury phobia?

a. Twice as common in women than men


b. Low familial inheritance
c. Sever tachycardia and hypertensive response
d. Easily treated
e. Bradycardia and hypertension often follow initial tachycardia

29. The term double depression is used to describe:

a. Major depressive disorder superimposed on grief reaction


b. Major depressive disorder superimposed on dysthemia
c. Major depressive patient not responding to treatment
d. Major depressive disorder with psychosis
e. Major depressive disorder with anxiety

30. A 26 year old Caucasian make is stable on lithium for bipolar disorder for the past 4 years. During his routine visit
with his family physician, he complains of fatigue and lack of motivation. He is also feeling very cold during this
winter and has gained weight. He wonders if he is getting depressed again. What is the next most appropriate step to
take in this case?

a. Diagnose depression and start the patient on antidepressants


b. Obtain patient lithium levels
c. Check patient Thyroid stimulating hormone (TSH) levels
d. Discontinue lithium
e. Consider electric convulsion therapy (ECT)
31- 46 female is 6 week postpartum complains of felling low in her mood and having crying spells, lack of motivation
and feeling hopelessness for the past 4 weeks what diagnosed to have postpartum depression?

a. True risk of infanticide is minimal


b. SSRI are contraindicated
c. Risk of depression during subsequent pregnancy is high
d. Suicide risk is low
e. Hospital admission is rarely required

32- Which SSRI is least likely to cause serotonin discontinuation syndrome ?

A. Paroxetine
B. Citalopram
C. Escitalopram
D. Fluoxetine (Prozac)
E. Sertraline

33- Of all tricyclic antidepressants which of the following most potent serotonin reuptake inhibition activity?

a. Clomipramine
b. Imipramine
c. Amitriptyline
d. Desipraminn

34- Which symptom has to be present to diagnose major depression disorder?

a. Depressed mood or loss of interest or pleasure


b. Lack of energy or motivation
c. Lack of appetite
d. Hopelessness
e. Gilt or worthlessness

35- 36 year old pregnant admitted due to possible infection associated with IV heroin, she is 5
month into her pregnancy and would like to quit heroin for good, what best course of action?

a. Allow her to continue to use heroin as withdrawal is dangerous


b. Methadone maintenance
c. Naltrexone
d. Advise her to stop heroin and start supportive therapy
e. Buprenorphine

36- Good test of recent memory?

a. Subtract 7 from 100


b. Ask Their date of birth
c. Ask How many sibling they have
d. Ask What they had eat for their last meal
e. Ask Who is the president
37- Most common case of delirium within 3 day of postoperative in a 40 year old man with history of alcohol
dependence?

a. Delirium tremens
b. Infection
c. Pain medication postoperative
d. Pain stress of surgery

38- Clinical feature which may be associated with delirium:

a. Disorganized thought process


b. Hallucination
c. Illusion
d. All of the above

39- Which of the following drug is best use to treat acute delirium?

a. Amobarbital (amytal)
b. Chlorpromazine
c. Diazepam
d. haloperidol
e. physostigmine

40- Mouth ulceration is associated with which of the following types of withdrawal ?

a. alcohol
b. benzodiazepins
c. cocain
d. nicotin
e. opoids

41. The postpartum blues:

A. occur in up to 50 percent of women after childbirth


B. are self-limited
C. begin shortly after childbirth and lessens in severity over the course of a week
D. are considered to be normal
E. all of the above

42. Depression and mania share which of the following symptoms?

A. Psychomotor acceleration
B. Low-self esteem
C.Grandiosity
D. Anger
E. Pessimism

43. The highest suicide rates are in which of the following age groups?

A. Under age 15
B. 15 to 24 year olds
C. 25 to 44 year olds
D. 45 to 64 year olds
E. Over age 65
44. The best indicator for increased risk of suicide?

a. Alcohol dependance
b. Marriage
c. Previous suicidal behavior
d. Schizophrenia
e. Major depressive disorder

45. Which of the following antidepressants would not be the best choice for the above patient with a
history of suicidal ideation?

A. SSRI
B. MAOI
C. Bupropion (Wellbutrin)
D. TCA
E. Venlafaxine (Effexor)

46. Generalized Anxiety disorder:

A. is least likely to coexist with another mental disorder


B. has a female to male ration of 1:2
C. is a mild condition
D. has about a 50 percent chance of a recurrence after recovery
E. has a low prevalence in primary care settings

47. Tourett’s disorder has been shown to possibly have a familial and genetic relationship with:

A. panic disorder
B. OCD
C. generalized anxiety disorder
D. social phobia
E. none of the above

48. Medical disorders to be considered in the differential diagnosis of somatization disorder include:

A. multiple sclerosis
B. systemic lupus erythematosus
C. acute intermittent porphyria
D. all of the above

49. The most accurate statement regarding pain disorder is:

A. It is diagnosed equally among men and women


B. Peak ages of onset are in the second and third decades
C. It is least common in persons with blue collar occupations
D. First degree relatives of patients have an increased likelihood of having the same disorder
E. Depressive disorders are no more common in patients with pain disorder than in the general public

50. Factitious disorders are best treated by which of the following?

A. Immediate discharge from the hospital


B. Confrontation about the patient’s deceit
C. Focusing on management rather than cure
D. Performing only minimally invasive procedures to satisfy the patient
E. Using low-dose neuroleptics to decrease the patient’s physical distress
61. Which of the following tumor markers is most closely associated with the diagnosis of Gastrointestinal Stromal
Tumor - GIST?

a. CEA
b. CD117
c. HER2/neu
d. CEA125
e. p53

62. A 67 years old woman presents with pelvic pain. CT shows a mass near her right pelvic wall. CT
guided biopsy reveal spindle type with few mitotic cells. At exploration 2cm pedunculated mass is noted on the
antimesentric border of mid ileum. Appropriate therapy would be:

a. ligation of the mass at the pedunculated stalk


b. wedge excision of the bowel containing the mass
c. wide excision, regional lymph node dissection
d. wedge excision, imatinib
e. wide excision, regional lymph node dissection, imatinib

63. The most appropriate treatment for a patient with 2cm diffuse large B cell lymphoma limited to the antrum of the
stomach would be:

a. total gastrectomy
b. chemotherapy, radiation followed by total gastrectomy
c. systemic chemotherapy
d. distal gastrectomy
e. chemotherapy, radiation.

64. A 27 years old female presents for evaluation after a recent diagnosis of cutaneous melanoma on her right leg. On
examination the lesion is irregular with discoloration but no ulceration. Physical examination of the right inguinal
shows no palpable nodes. Pathological analysis of the punch biopsy of the lesion reveals a 3.1mm thick lesion. In
addition to 2cm margin total excision of the primary, the next step in management of her inguinal nodes should be:

a. observation
b. sentinel lymph node biopsy
c. elective lymphadenectomy
d. radiotherapy
e. isolated limb perfusion

65. A 75 years old man taking NSAIDs for arthritis presents with an acute abdomen and pnumoperitoneum. His
symptoms are 6 hours old and his vital signs are stable after the infusion of 1L normal saline solution. What should be
the next step in the management of this patient?

a. Computed tomography of the abdomen


b. Esophagogastroduodenoscopy
c. Antisecretory drugs, broad spectrum antibiotics, and surgery if he fails to improve within 6hrs
d. Antisecretory drugs, antibiotics for H.Pylori, and surgery if he fails to improve within 6hrs
e.
66. Which of the following is NOT a risk factor for developing a surgical site infection - SSI?

a. Radiation exposure
b. Recent surgery
c. Prolonged hospitalization
d. Infancy

67. A 68 years old man is admitted to the hospital after having passed three large maroon colored stools. On arrival at
the hospital, he passes more bloody stools as well as clots. He is pale, orthostatic, and tachycardic. Nasogastric
aspirates are bilious. After resuscitation is begun, which of the following is the most appropriate initial test?

a. Angiography
b. Nuclear medicine red blood cell scan
c. Rigid proctoscopy
d. Colonoscopy
e. Barium enema

68. A 4 years old patient presents with diffuse scald wounds after being held in a hot tub of water. There are
circumferential blisters present over the right leg (from hip to toes) and circumferential blistering over the lower left
leg (from knee to toes). The right thigh, abdomen and back below the umbilicus, as well as the buttocks and perineum
are red hot but without blisters. What is the total burn surface area?

a. 25%
b. 36%
c. 46%
d. 54%

69. Epstein-Barr virus is associated with which of the following cancers?

a. Nasopharyngeal carcinoma
b. Non-Hodgkin’s lymphoma
c. Adult T cell leukemia
d. Kaposi’s sarcoma

70. Which of the following is the most significant risk factor for invasive breast cancer when screening a patient for
risk?

a. >2 first degree relatives with breast cancer


b. 2 previous breast biopsies in a patient <50 years of age
c. Age <12 at menarche
d. Atypical hyperplasia in a previous breast biopsy

71. (Schwartz?) Lymphatic drainage from the supraglottic larynx is primarily to:

a. The prelaryngeal (Delphian) node


b. Paratracheal nodes
c. Deep cervical nodes
d. Superior jugular nodes
72. (Schwartz?) Level V lymph nodes in the neck are located:

a. In the submental area


b. In the anterior triangle
c. In the posterior triangle*
d. In the inferior jugular chain

73. (Schwartz?) Heparin induces anticoagulation primary by:

a. Increasing production of antithrombin


b. Increasing the activity of antithrombin
c. Increasing conversion of factor X to Xa
d. Increasing conversion of factor XI to XIa

74. (Schwartz?)The most common location of gastrointestinal stromal tumors (GIST) is:

a. Stomach
b. Duodenum
c. Jejunum
d. Ileum

75. (Schwartz?) A high output enterocutaneous fistula is defined as draining more than:

a.100 mL/day
b. 500 mL/day
c. 1000 mL/day
d. 2000 mL/day

76. The most common cause of small bowel obstruction is:

a. Adhesion
b. Hernia
c. Malignancy
d. Crohn's disease

77. A 24-year-old woman in the 20th week of pregnancy experiences a single episode of biliary colic. The most
appropriate initial management is:

a. observation with plans to follow her after delivery for recurrent episodes.
b. Dietary changes
c. Elective laparoscopic cholecystectomy during 2nd trimester.
d. Elective open cholecystectomy during 2nd trimester.

78. Surgery is indicated in which of the following asymptomatic patients with primary hyperparathyroidism?

a. Mildly elevated urinary calcium excretion (>100mg/dl).


b. Reduction in creatinine clearance by 10%
c. Serum calcium> 0.8 above the upper limits of normal
d. Age < 50 years
79. Patient with carcinoma of 1 cm of diameter in thyroid glandule without adenopaty.
Treatment of medullary thyroid cancer:

a. Total thyroidectomy
b. Total thyroidectomy+ cervical LN dissection
d.

80. The most common cause of primary parathyroidism is:

a. Parathyroid adenoma
b. Multiple parathyroid adenomas
c. Parathyroid hyperplasia
d. Parathyroid carcinoma

81- which of the following would be the first diagnostic test ordered in patient with solitary thyroid noudle?

a. radioactive iodine scan


b. CT or MRI
c. fine needle aspiration
d. core needle aspiration

82 - a 35 year old woman experiences an acute onset of epigastric and right upper quadrant pain several hours after
large dinner. she has had similar episodes in the past that resolved after a few hours. This episode persists, and she
has fever and nonbilious vomiting. what is the most likely source of the abdominal pain?

a. perforated ulcer
b. acute appendicitis
c. perforation following bowel obstruction
d. acute cholecystitis
e. diverticulitis

83 - a 57 year old woman with a 1.5 cm infiltrating ductal carcinoma is found to be ER negative, PR
negative, and HER - 2/neu positive. she comes to your office to discuss treatment options.
what would you recommend?

a. modified radical mastectomy alone


b. wide local excision. radiation therapy and tamoxifen.
c. simple mastectomy, sentinel lymph node biopsy, trastuzumab (Herceptin) and tamoxifen.
d. modified radical mastectomy and adjuvant chemotherapy.
e. wide local excision, sentinel lymph node biopsy, radiation therapy and adjuvant chemotherapy with trastuzumab.

84 - a 35 year old woman has a 5 year history of poorly controled hypertension. she is taking three different
medications, including a diuretic, b-blocker, and potassium supplements. what would be the next step in establishing
the diagnosis of surgically correctable hypertension?

a. CT of the abdomen and pelvis


b. urine catecholamines.
c. serum aldosterone and renin levels
d. renal US
e. saline suppresion testing
85 - a 44 year old man is due to undergo laparoscopic adrenalectomy for pheochromocytoma. which of the following
agents shoud NOT be given as the initial oronly medication to help achieve appropriate
preoperative adrenoreceptor blockade?

a. phenoxybenzamine
b. amlodipine
c. prazosine
d. atenolol
e. metyrosine

86 - following surgery a patient develops oliguria. you believe the patient is hypovolemic, but you seek
corroborative data before increasing intravenous fluid. which of the following values supports the diagnosis
of hypovolemia?

a. urine sodium of 28 mEq/L


b. urine chloride of 15 mEq/L
c. fractional excretion of sodium less than 1
d. urine/serum creatinine ratio of 20
e. urine osmolality of 350 mosm/kg

87 - a 45 year old woman with Crohn's disease and a small intestine fistula develops tetany during the second week
of parenteral nutrition. the laboratory findings include:
Na - 135
K - 3.2
Cl - 103
HCO3 - 25
Ca - 82
Mg - 1.2
PO4 - 2.4
albumin - 2.4
an arterial blood gas sample reveals a pH of 7.42, PCO2 of 38, and PO2 of 84 mmHg.
which of the following is the most likely cause of the patients tetany?

a. hyperventilation
b. hypocalcemia
c. hypomagnesemia
d. essential fatty acid deficiency
e. focal seizure

88- a victim of blunt abdominal trauma undergoes a partial hepatoectomy. during surgery he recieves
twelve units of packed red blood cells. in the recovery room he is noted to be bleeding from intravenous
puncture sites and the surgical incision. which of the following statements regarding coagulopathy is most likely true?

a. the patient has n unknown primary bleeding disorder


b. the coagulopathy is secondary to the partial hepatectomy
c. the coagulopathy is secondary to dilutional thrombocytopenia and deficiency of clotting factors from the
massive blood transfusion.
d. the treatment is oral vitamin K
e. the treatment is intravenous vitamin K
89 - a patient with a solid malignancy is discussing chemotherapy with his oncologist. he is interested in the risks of
the treatment. what is the primary toxicity of doxorubicin (adriamycin)?

a. cardiomyopathy
b. pulmonary fibrosis
c. peripheral neuropathy
d. uric acid nephropathy
e. hepatic dysfuncion

90 - a 41 year old man complaining of regurgitation of saliva and undigested food. an esophagogram reveals a
bird's-beak deformity. which of the following statements is true about his condition?

a. chest pain is common in the advanced stages of this disease.


b. more patients are improved by forceful dilation than by surgical intervention
c. manometry can be expected to show high resting pressure of the lower esophageal sphincter (LEs)
d. surgical treatment consists primary of resection of the distal esophagus with reanastomosis to the stomach
above the diaphragm
e. patients with this disease are at no increased risk for the development of carcinoma

91. A 41 y/o man presents with food and saliva regurgitation , an esophagogram was performed and shown "beard
leak " phenomenon what is true ?

A .Chest pain is common in this condition


B. First perform forceful dilation than proceed to surgery
C. Manometry show increase LES resting tone
D. Resection above the diaphragm
E. There is no increased risk for carcinoma

92. 3 findings in achalasia :

A. Increased LES resting pressure , decreased LES relaxation , increased esophageal action
B. Decreased LES resting pressure , Increased LES relaxation , Decreased esophageal action
C. Increased LES resting pressure , Decreased LES relaxation , Decreased esophageal action
D. Decreased LES resting pressure , Increased LES relaxation , Increased esophageal action

93. An antireflux procedure unsuccessful because:

A. Gastric outlet obstruction was not identified


B. Well dilated previous stricture
C. Decreased saliva production
D. Transthoracic fundoplasty

94. Esophageal perforation frequently caused:

A. Spontaneous rupture
B. Instrumental perforation
C. Foreign body
D. Barrett's esophagus
95. Which one is not contribute to digestion of fats in small intestine:

A. Brush border enzymes


B. Pancreatic lipases
C. Bile salts
D.?

96. Which one is not characteristic of chronic Crohn's disease :

A. Rectal bleeding
B. Perianal disease
C. Diarrhea
D. Abdominal pain

97. Intussuspection in 20y/o man :

A .Idiopathic
B . Carcinoid
C. Adhesions
D . Lymphoma

98. Lesions associated with Peutz-Jegher syndrome :

A.Sessile adenomas
B. Cluster lesions in duodenum
C. Hamartoma
D. It has AR inheritance

99. What is Not an indicator for an open cholycestectomy :

a. poor pulmonary or cardiac reserve


b.third trimester gestation
c. child’s class C liver disease
d. suspected gallbladder cancer
e. Previous gastric bypass procedure

100. Primary sclerotizing cholangitis associated with which disorder :

A. Crohn's disease
B. Diabetes
C. Rheumatoid arthritis
D. Ulcerative colitis
E. Chronic pancreatitis

101. Which of the following does NOT stimulate bile flow?

A. Cholecystokining
B. bile salts
C. vagal stimulation
D. splachnic stimulation
E. secretin
102. Acute ischemia in lower extremity:

a. most often the result of femoral artery thrombosis


b. can never occur as a consequence of DVT
c. is never amenable to treatment with thrombolytic treatment
d. is most often the result of cardiac embolic events
e. rarely result in limb/(life? Not sure I wrote correct) threatening ischemia

103. Which of the following regarding aneurysms is true?

a. inflammatory aneurysm of the aorta rarely rupture.


b. splenic artery aneurysm are usualy atherosclerotic.
c. thrombosis of popliteal artery aneurysm may be treated with thrombolytic therapy
d. infected aneurysm of the aorta usualy require complete excision and extraanatomic revascularizaion
e. aneurysms of the hepatic artery are the most common visceral aneurysms

104. Breast conservation surgery in breast carcinoma:

a. resulted major imporovement in mortality and morbidity.


b. has resulted durable survival data comparable to those for mastectomy for certain breast cancer

105. ductal carcinoma of breast in situ:

a. is almost always bilateral.


b. has become more frequently diagnosed because of the use of mammogram.
c. cannot present as a palpable mass.
d. is frequently associated with microscopic lymph node metastases.

106) A 33-year-old woman pregnant for the third time present at 3 months with a 2-cm mass in the inner aspect of
the left breast. A needle aspiration reveals no fluid. You would:

a) arrange for a mammogram because multicentric lesions are common during pregnancy.
b) consider termination of pregnancy because chemotherapy has been shown useful in nodenegative premenupausal
patients.
c)expeditionsly obtain a histologic diagnosis of the mass.
d)wait until the third trimester because surgey is safer at that time.

107) Risk factors for the development of melanoma include all of the following except:

a)fair hair.
b)light complexion.
c)green eyes.
d)multiple nevi.

108) Which of the following endoscopic peptic ulcer characteristics has the highest risk for recurrent bleeding:

a) oozing ulcer.
b)clean based ulcer.
c)nonbleeding "visible vessel"
d)nonbleeding ulcer with an overlying clot.
e)Dieulafoy ulcer.
109) A 38-year-old man was taken to an emergency facility after his car skidded on the freeway and struck a pillar. His
vital signs were as follows: puls 88/min; respirations 20/min; blood pressure 100/70 mmHg. Physical examination
revealed nonprominent jugular neck veins, no indication of cyanosis, and symmetric breath sounds. A posterior-
anterior chest X-ray film revealed a widened mediastinum. Which of the following choises is the most likely diagnosis?

a)ruptured aortic aneurysm.


b)cardiac tamponade.
c)dissection of the thoracic aorta.
d)myocardial contusion
e)pulmonary contusion.

110) Which of the following statements is true about surgery for pheochromocytomas?

a)patient should receive preoperative alfa-blockade for 1 to 4 weeks prior to surgery.


b)the adrenal vein should be taken only after the arterial supply is isolated and ligated.
c)intravenous fluids should be restricted until the tumor is removed.
d)preoperativebeta-blockade should precede any alfa-blockes to avoid the precipitation of malignant hypertension.
e)bilateral adrenalectomy should be performed for patients with MEN2a.
<<01.03.2012
By: Alhag Abu Anzeh Muhammad

Part A:

1- A 25 y.o female at 41 wk gestation is noted to change her cervical dilatation from 6-9 cm within 2 hours .
<<Where of the following most diagnosis is ?

A. Normal labor progress


B. prolonget latent phase
C.protracted active phase
D.arest active phase
E.arest of descent

2. Which NON-obstetrical surgical procedure is most commonly performed in the 2-end trimester of pregnancy ?

A. appendectomy
B. cholecystectomy
C.ovarian cyestectomy
D.tensillectomy
E.splenectomy

3.During what period of gestation does high dos ionizing radiation exposure pose the most risk of mental
retardation in the fetus ?

A.4-7
B.8-10
C.20-26
D.28-37

4. Evaluation of uncomplicated long standing chronic hypertension early in pregnancy includes all EXEPET which
of the following ?

A.eachocardiography
B.serum creatinine level
C. ophtalmological evaluation
D.urinary catecolamine
E. Quantitive urinary protein analysis

5. B-blockers use in pregnancy in particular Atenolol is associated with which of the following perinatal
morbidities ?

A. preterm birth
B. neonatal hyperglycemia
C fetal growth restriction
D. infantile respiratory distress syndrome
E. neonatal polycythemia

6.which of the following actions improves oxygenation through the placenta in cases of fetal heart rate
deceleration ?

A. left lateral recumbent position


B. epidural anesthesia
C morphine sulphate administration
D. IV oxytocin
E. IV atropine
7.umbilical cord prolapse is most likely to occur with which of the following ?

A. face presentation
B. transverse lie
C.frank breech presentation
D.complete breech presentation
E. brew presentation

8. a multipara with previous major thromboembolic event is on heaparin treatment from the beginning of
pregnancy, long-term heaparin therapy may lead to which of the following complications ?

A.osteporosis
B.thrombophilia
C. fetal intracranial hemorrhage
D.diabetes mellitus
E. heparin induced thromocytopenia

9. which of the folloeing is best management of an 18 year old primigravida at 28 weeks gestation with BP
160/110 , elevated liver function tests and platelet of 60,000/mml ?

A.expectant management and oral antihypertensive therapy


B. platelet transfusion and liver function monitoring
C. magnesium sulfat therapy and prompt delivery
D. IV immunoglobulin therapy
E. plasmapharesis

10. what lower limit level for the 50 g glucose screen would improve its sensitivity to >90% for detection of
gestational diabetes:

A.130
B.135
C.140
D.145
E.150

11) A 20 year old woman nullparius patient was found to be sereonegative for CMV you counsel your patient
that the most effective prevention for maternal CMV infection during pregnancy involves which of the following :

A) pre pregnancy inactivated virus vaccine administration.


B) good hygiene and hand washing.
C) CMV Ig given with 72h of viral exposure.
D) ganciclovir administration within 72h of exposure.
E) un house isolation the entire pregnancy.

12) how many weeks after delivery does menstruation normally return in a non breast feeding woman?

A) 4-6 weeks .
B) 6-8 weeks .
C) 14-16 weeks .
D) 16-18 weeks .

<<
13) a repeated gradual 20 beats per mint deep smooth deceleration of the fetal heart rate that start after the
peak of each c-indication describes which of the following fetal heart beat patterns :

A) early deceleration .
B) late deceleration .
C) variable deceleration .
D) prolonged deceleration .
E) sinusoidal pattern .

14) your patient 30 weeks gestational experience premature of p-prom current recommended for the
management of prom include which of the following ?

A) broad spectru prophylactic parenteral antimicrobial therapy.


B) cesarian section if spontan labor occurs before 35 weeks .
C) gelatin sponge pulgging of the cervix.
D) second ultrasound measurements of the expansion of the cervix E) daily washing of the vagina with a
solution of iodine or Chlorhexidine

15) which of the following incorrect regarding semen analysis ?

A) sexual intercourse should be avoided 6-8 days prior the semen.


B) semen ejaculaion should be up to an hour prior to analysis.
C) there is high variable in the same male patient analysis .
D) diagnostic of an anomaly will never be given based on a single sample.
E) morphology of the sperm has the highest correlation with fertility

16. What of the following is not a significant detail in the anamnesis of the male infertility?

A. inborn defects
B.no omission of the testis
c. frequency of sex
D. frequency of Shaving
E. frequency of consumption of fatty foods

17) a female patient undergoes follicle sonograraphic follow up for the evaluation of primary infertility on cycle
day 14 a single follicle 22mm in diameter is identified, what is the significance of the finding:

a-normal finding prior to ovulation


b-corpus luteum
c-functional ovarian cyst
d-benign ovarian tumor
e-normal finding following ovulation

18) what is the main hormone that effects the cervical mucus after ovulation

a-progesteron
b-estrogen
c-cortisol
d-LH
e-FSH
19) following a primary infertility evaluation of a 41 years old couple the female partner was found to have
menses every 38 day progesterone level 2ng/ml on cycle day 30 and a normal uterine cavity with normal contrast
dye passage to the pelvis on hysterosalpingography the male semen analysis was: concentration 22 milion/ml
55%moyility and 30%normal morphology what is the etiology of the infertility in this case:

a-luteal phase infertility


b-male factor infertility
c-uterine factor infertility
d-idiopatic infertility
e-age related and ovulation disorder

20-A which of the following is correct regarding females with mullerian dysgenesis?

a-absence of the inferior third of vagaina


b-uterus existence
c- 46 xx karyotype
d-complete vaginal and uterus absence
e-menses appearance following estrogen and progesterone test

21-which of the following characterized the female athletic triad syndrome?

A-Menstrual bleeding following estrogen and progesterone administration .


B-Increase in over 10% in ideal muscle mass.
C-Highly increased gonadotropins.
D-Bone mass increase only in the presence of muscle mass increase.
E-Hirsutism and menses appearance following administration of progesterone only.

22-which of the following combination is accepted for the treatment of hirsutism?

A-Progesterone, GnRH antagonist, combined oral contraceptives.


B-Progesterone,combined oral contraceptives,GnRH agonist.
C-GnRH agonist, cyproterone acetate, SSRI.
D-GnRH antagonist , cyproterone acetate, flutamide.
E-GnRH agonist, cyproterone acetate, combined oral contraceptives.

23-In vitro fertilization is the only acceptable solution for achieving self ovum pregnancy in women with:

A-an obstructed fallopian tube following PID(pelvic inflammatory disease) and a normal another fallopian tube .
B-an obstructed fallopian tube following PID and another fallopian tube following pregnancy and salpingostomy.
C-a single fallopian tube following salpingectomy and male partner sperm count of cells/ml.
D-male sperm count ofcells/ml and an obstructed fallopian tube following PID.
E-a fallopian tube with hydrosalpinx and normal contrast media passage to the pelvis and another normal
fallopian tube.

24- all of the following are indications for IVF treatment except:

A-bilateral tubal obstruction .


B-severe oligoteratozoospermia.
C-decreased ovarian reserve.
D-repeated abortions during the first trimester.
E-genetic disorder that requires pre-gestational diagnosis(PGD).
25-which of the following is a test for ovarian reserve?

A-FSH level on period day 3.


B-estradiol level on period day 6.
C-progesterone level following ovulation.
D-LH level before ovulation.
E- inhibin A level.

26- what are the accepted indications for systemic estrogen treatment for post-menopausal women?

A-osteoporosis prevention, coronary artery disease prevention, treatment of vaginal and vulvar atrophy.
B-osteoporosis prevention, treatment of vasomotor symptoms, treatment of vaginal and vulvar atrophy.
C-coronary artery disease prevention, mood improvement, prevention of Alzheimer’s disease.
D-mood improvement, treatment of vasomotor symptoms, prevention of Alzheimer’s disease.
E-coronary artery disease prevention, treatment of vaginal and vulvar atrophy, prevention of Alzheimer’s
disease.

27-what are the contraindication of systemic estrogen treatment as a replacement therapy for post-menopausal
woman?

A-osteoporosis
B-undiagnosed vaginal bleeding
C-obesity
D-diabetes
E-hypertension

28-a 30 year old female , G5P4, is 10 weeks pregnant(intrauterine pregnancy) in the presence of an intrauterine
device (ID). She is interested in continuing the pregnancy. The IUD wire is visible at the cervical os.
Which of the following is the appropriate management ?

A-leave the IUD in place with no further treatment.


B-leave the IUD in place and treat with antibiotics.
C-remove the IUD now, followed by expectant management.
D-terminate the pregnancy due to the increased infection risk.
E-perform a laparoscopy to exclude heterotopic pregnancy.

29-which of the following is an absolute contraindication for oral contraceptive administration?

A-lower extremity varics.


B-tension headaches.
C-seizure disorder.
D-smoking over the age of 35 .
E-mild hypertension.

30-which of the following is correct regarding uterine leiomyomas?

A-50% of the patients will suffer from abnormal uterine bleeding.


B-there is a strong association between leiomyomas and infertility.
C-leiomyomas diminish and are mostly asymptomatic during pregnancy.
D-anemia is the most common complication of leiomyomas.
E-there is no recurrence following the resection of leiomyomas.

<<
31. woman with mature cystic benign teratoma . which of the following is correct:

a.10% of the benign tumors


b. more related with menopause
c. associated with pain and ascites
d. composed of the 3 embryonic layers
e. make diagnosis with MRI

32. woman with HGSIL on pap smear positive 3.next management

a. histeroscopy abdomen
b. colposcopy and biopsy
c. HPV vaxin
d- tracheletomy

33. 17yo girl with pain in abdomen after ultrasound 16cm ovarian adnexal mass:

a.gonadoblastoma
b. linfoma
c. Brenner
d, disgerminoma
e. serous epithel carcinoma

34- anatomy injury to ureter smallest chance in gynecologic procedure:

a- uterus artery
b. cardinal ligament
c.bilateral vaginal fornices
d.
e.ovarian ligament

35. 70 yo women with watery vaginal discharge:

a-metastatic uterine cancer


b-epitel ovaric cancer
c-epitel fallopian carcinoma
d-benign ovaric cancer
e- mtx colon cancer

36. 45yo non productive cough with hypertension with lisinopril + statin + ACE inhbitorP:

a. change ACE inibitor to angiotensin 2 inibitor

36. Male 45 years old complained of nonproductive cough for the past 6 months.In addition, he is healthy, a
history of hypertension, hyperlipidemia. He takeslisinopril, atorvastatin. In addition, he has no symptoms.
Auscultatory chistyes lighton both sides. Which of the following is the most appropriate next step in the conduct
of this patient?

A. immediately transferred to the treatment of ARBs 2 \ ARB \


B. because the cough induced by angiotensin-converting enzyme usually occurswithin one month from the
beginning of acceptance should continue to takelisinopril and be screened for other causes of cough.
C. should not take lisinopril 48 hours and see whether the host-cough.
D. has to take lisinopril and assign a cromolyn, will see whether the cough.
37. 70yo with lisinopril amlodipine ,Blood Pressure 138/74/ weight ,2 kg creatinie 2.5,
GFR 28. what stage?

a. I
b .III
c. IV
d. V

38. 60 yo V stage CKD with increase creatinin, BP 120/70, creatinin 6.3, K+ 4.8, HCO3 20, next step ?

a change fistola
b dialysis
c biopsy
d 24h urine protein collection

39. 50 yo man with diffuse bone pain for 6 wk now . dialysis for 15 year:

a. amiloidosis
b. renal osteodistrophy
c. mieloma multiple
d. osteoarthritis

40. woman 29yo 6 wk with morning stiffness , swelling hands 3th and 4th metacarpal ,small nodules . diagnosis
test:

a. anti ccp (anti-cyclic citrullinated protein antibodie)


b. rheumatoid factor
c. cANCA = citoplasmatic Anti-neutrophil cytoplasmic antibodie
d. pANCA =perinuclear Anti-neutrophil cytoplasmic antibodie

41)56 year old man with achalasia and hypertention is evaluated for worsening dyspnea and 10-pounds weight
loss over the past 6 weeks.he has 50 pack \year smpking historyand consumes 2 liqour drinks per day .physical
examination is unreveling subsequent esophagogastrodoudenoscopy reveals ulcerating lesion in the proximal
esophagusbiopsy of the mass shows squamus cell carcinoma which of the following are risk factor for
development squamus cell carcinoma?

a)tobacco
b)alcohol
c)achalasia
d)all the above

42) A47 year old man is evaluated because of 4 months history of fatigue ,nausea, and mild epigastric pain ,
upper endoscopy shows thickened nodular forms in the distal stomach . biopsy specimen revelas h.pylori
organism and low grade gastric mucosa associated lymphoid tissue lymphoma .ct scan of the abdomen shows no
evedance of metastasis or lymnodes involvment which of the following is the most appropriate treatment for this
patient ?

a)chemotherapy ,radiation ,surgical resection


b)total gastrectomy
c)radiation
d)omeprazol,clarithromycin .amoxicillin

43)which of the following about angina pectoris is true?

a) typically lasts 1-2 h


b) it may associated with epigastric pain
c) it causes perdictable ECG changes
d) it typically associated with chest wall tenderness
e) rarely radiates to neck jaw or shoulder
44)18 year old women present for evaluation of facial and lower extremity edema 2weeks ago ,she was
diagnosed with upper respiratory tract infection that has since resolved with suppurative treatment on physical
examination B\P 150\100 cardiac normal ,bilateral lower pitting edema lab test : creatinine 1.7 albumin 3.5 lower
serum complement urinealysis 1+ protiens dysmorphic rbcs with casts which of the following is most likely
<<diagnosis ?

a)Iga glumerulonephritis
b)wegner disease
c)SLE
d)post strept glumerulonephritis

45)52 year old man complain of dyspnea for 8 weekshe has minimal prior medical care notes no prior medical
problems and denies any medication use he admits to have smoked apack of ciggarets daily for 30 years .he is
currently stable and physical examinattion is remarkable for prolonged expiratory phase and diminished
braething sounds bilaterally in which of the following most likely cause for chronic dyspnea ?

a)pulmonary embolism
b)copd
c)diabetes
d)myocardial infarction

46)20 year old man present with hematuria he has noticed blood in his urine twice in the past 6 months both
occation 3-5 days after upper respiratory infection , his physical ex is unremarkable , lab normal craetinine
urinelysis :trace protinuria 2+ blood 10-15 rbcs ...most likely diagnosis?

a)goodpasture
b)henoch schenloin purpra
c)post infection glumerulonephritis
d)Iga nephropathy

47)65 year old women with prosthetic mechanical mitral valve present of 1 week history of fever up to 39.5,
maliase diffuse myalgia , blood culture return positive for coagulase negative staph which of the following is
indicated for this patient?

a.transthoracicecho
b.trans esophgeal echo
c.multi slice ct
d.24 hour holter

48)58 year old woman previous history of infective endocarditis present for advice regarding antibiotic
prophylaxis prior to dental prosedsure it will involve significant gingival manipulation she has no allergy .
which AB is indicated ?

a)levofloxacin
b)clarithromycin
c)no antibiotic
d)amoxicillin

49) 32 year old man history of iv drug use present with 3 days history of intermitten fever up to 38.3 and maliase
on examination T:38.3 B\P 130\85 heart rate 94 RR:19 he has new 2\6 systolic murmur at left sternal chest
radiograph normal urinelysis 5-10rbcs for this patient which more likely etiology of fever?

a)coagulase neg staph


b)strept
C)mecthillin resistance staph
d)gram negative enteric bacteria
50)72 yEar old woman with history of hypertention and tobacco present with chest pain for 2 weeks she discribes
pain as pressure like with radiation to left arm it incrase by exertion and relived by rest there is no association
she deniy chest pain history physical examination unrevealed ECG shows LVH .labs cardiac enzymes are normal
which of the following is most likely diagnosis ?

a)non st segment elevation myocardial infarction


b)pericarditis
c)unstable anginal
d)stable anginal

51. a 63 year old male presents with productive cough. The ptient reports feeling well until he developed
dyspnea and a productive cough of green sputum 2 days ago. His symptoms have progressed and now he also
reports fever and chills. On examination: Decreased breath sounds associated with rhonchi and increased
fremitus in his right lung base. A chest x-ray reveals consiladation in the right lower lobe.
Which of the following is the most common organism identified in such adult patients:

a. H. Influenza.
b. strep. Pneumonia.
c. Moraella catarrhalis.
d. Pseudomonas aerginosa.
e. Legionella pneumophilia.

52. a 76 year old man complains of progressively worsening fatigue over the preceding 6 months. He has
difficulty walking 2 blocks because of this. He also has a poor apetite and has lost 15 pounds.
Physical examination revealspale conjunctiva. The lungs are clear to auscultation bilaterally.
The hert rhythm is regular an the rate is 96 bpm. There are no heart murmurs. The CBC shows hemoglobin of
7g% with a decreased MCV. The total iron-binding capacity is incresed and the ferritin level is low.
This patients anemia is most likely secondary to:

a. thalassemia.
b. anemia of chronic disease.
c. folate deficiency.
d. B12 deficiency.
e. iron deficiency anemia.

53. a 60 y old female with a history of coronary artery disease presents with chest pain for 1 hour. Her pain is
similar tpo the symptoms that she ex[erienced last year when she was diagnosed with a non-ST elevatipon MI.
She has been taking her medictions as instructed. The nearest catheterized laboratory is 3 hours away.
Which of the following is the most appropriate next step in the management of this patient:

a. Transfer the patient for PCI.


b. consult cardiovasular surgery for coronary bypass surgery.
c. adminster thrombolytic is there is no contraindication.
d. medical management.

<<
54. a 40 year old female with a histroy of coronary artery disease, status post-percutaneous coronry intervention
with a drug-eluting stent 1 year ago, presents to the emergency department with fever and right upper quadrent
pain for 1 day. An abdominal sonogram reveals gallstones with likely cholecystitis and no biliary ductal dilation.
She is admitted for antibiotics and a geeral surgery consultation, the surgeon is concerned because the patient is
on clopidogeral for a drug-eluting stent. If possible, how long should surgery be delayed after stopping the
patients clopidogeral:

a. 1 day.
b. 7 days.
c. 1 month.
d. 1 year.

55. a routine examination in a 67 year old woman revelaed leukocytois. She denies fever, night sweats or weight
loss. Herphysical examination is normal, in particular she has no lymphadenopathy or hepatosplenomegaly.
A repeat blood cell count shows leukocytosis with an absolute lymphocyte count of 7.5X109/L. Her hemaglobin
<<and platelet count are normal. Flow cytometry reveals a monoclonal B cell population.
Which of the following is the most likely diagnosis for this patient:

a. Acute myelogenous leukemia.


b. acute lymphocytic leukemia.
c. chronic myelogenous leukemia.
d. chronic lymphocytic lekemia (CLL).
e. Lymphoblastic lymphoma.

56. a 44 y old woman with acute myelogenous leukemia, currently undergoing induction chemotherpy, was
noted to hve a platelet count of 9000/ MacroL.
On physical examination: Afebrile and hemodynamically stable, no evidence of bleeding.
She is tranfused 1 unit of pooled platelets.
Repeat laboratory testing 1 hour post transfusion shows a platelet count of 18000/MacroL.
Which of the following is the most appropriate next step in management of this patient:

a. transfuse 1 unit of pooled platelets now.


b. transfuse 1 unit of platelets and 1 unit of packed RBC now.
c. No transufussion t this time, initiate platelet cross match prior to next transfusion because of inappropriate
response to transfusion.
d. No tranfussion at this time, no further testing is required prior to next transfusion because response was
appropriate.

57. a 40 y old man with a history of IV drug abuse presents with fever and chills for 1 week. His examination
reveals a new holosystolic murmur at the left lower sternal border that increase with inspiration. He is febrile at
38.9C. You plan to admit him for work up and treatment of infective endocarditis.
what duration of IV antibiotics is necessary for this patients illness.

a. 4 to 6 weeks.
b. 7 to 10 days.
c. 3 monhts.
d. 1 year.
58. a 70 year old man with a history of hypertension and diabetes presents to the management of chronic angina
for the past year. His chest pain now occurs one or 3 times per month, typically last 5 minutes, and are relieved
by rest and sublinguinal nitroglycerin. A recent pharmacologic stress test showed a reversible perfusion defect.
What is the most appropriate next step in the management for this patients angina:

a. Add morphine to the patients regimen.


b. Coronary angiography,
c. Lifestyle modification including exercise.
d. Continue with the current management.

59. which of the following is the most likely factor contributing to chronic myeloid leukemia:

a. Exposure to radiation.
b. Family histroy of cancer.
c. Tobacco abuse.
d. Philadelphia chromosome (Ph).

60. a 60 year old woman has just had the diagnosis of CML confined. She is completely asymptomatic and is very
active. Her laboratory examination: WBC-30X10/9/L, hematocrit-36%, platelet count- 435000/MacroL. Which of
the following would be the most appropriatetreatment at this time:

a. Monitor the patient closely and initiate treatment when she is symptomatic.
b. Admint the patient to the oncology ward and initiate 7+3 (cytarbine and daunorbicin).
c. Start the tyrosine kinase inhibitor (TKI) imatinib.
d. Begin interferon alfa and hydroxy urea.
e. Begin testing the family and searching data bases for stem cell donor.

61.67yrs woman present with painful burning of hand accompained by erythema and new heat , on exam:
obvious erythema of both hands, the tip of spleen palpable , lab: platlet 1650000, wbcs 12000, haemoticret 40%
mutation of jak2 is detected , which of following is well documented complications associated with this disorder:

a- biddi-chiari syndrome
b- hypertension
c- myopathy
D-interstitial lung disease

62.60 yrs male with history aortic valve regur , present with 3 days history of intermittent fever 38.3 , b.p. 130/85
, h.r. 94 , RR 19 , HEART SOUND ARE CRISP & typical of AVR , most likely etiology of fever:

a- methicillin sensetive coagulase -ve strep


b- methicillin resistance coagulase -ve strep
c- methicillin sensitive staph aureous
d- gm -ve enteric bacteria

63. Which of the following proposals for enoxaparin is true:

A. It is proved that its use in ambulatory patients determines the cost


b. The use of this drug does not require periodic testing
c. The frequency of thrombocytopenia, such as the use of heparin
D. Be assigned to intravenous
E. Its use in patients with renal failure is safe
64. Which of the following is the most appropriate next step in the examination of patients with endocarditis, the
causative agent is Streptococcus bovis:

A. Complete examination of the teeth to prevent a possible abscessed tooth


B. KLA and liver function tests every 6 months
C. KLA and liver function tests every 6 months, eye examination every year
D. Liver function tests every 6 esyatsev, eye examination every yeaR.

65. Patient 44 years old, suffering from rheumatoid arthritis 2 years, undergoing tests on the complaints
of aggravated symmetrical pain in the joints of hands and feet 2 weeks. Temperature 37.3. PS-90 min, blood
pressure, 132/90, heart and lungs are normal, marked hepatosplenomegaly, active synovitis
fingermetacarpophalangeal joints of the hands. Hemoglobin 12 g / l, leukocytes 1.7 × 10 9 degrees, platelets 185
000. The total metabolic rate analysis.
<<What is the most likely diagnosis of this patient?

A. Lupus erythematosus, drug induced


B. Cindrom Felty
C. Evans Syndrome
D. Infection with parvoviruS.

66) 35 year old man was diagnosed with pancreatitis a weak ago, right upper quadrant sonogram showed the
evidence of cholelithiasis, his symptom resolved with conservative treatment, management after 2 days of
hospitalOn physical e...xamination raised waxy appearance yellowish coloured skin lesion on the extensor
surgace of bilateral arm, having had this for many years.A lipid panel revealed total cholesterol 218, HDL = 42,
LDL = 128, Triglycerides = 1840.
What is the most appropriate next step?

a) No further workup
b) Prescribe gemfibrozi
lc) Prescribe atorvastatin
d) Refer to cholecystectomy
e) Prescribe niacin

67) 35 year old female came for a routine visit her past medical history is significant for poorly controlled type II
Diabetes mellitus last HB A1c is 8.7% has obstructive sleep apnea, obesity and hypertension, dyslipidemia.Her
medication metformin, insulin, hydrochlorothiazide, and atorvastatin, on physical examination BMI 42 kg/m and
blood pressure 148/89, treatment which of the following underlying condition Is primary approach?

a) Hyperglycemia
b) Hyperlipidemia
c) Hypertension
d) Inflammatory cytokines
e) Obesity

68) 60 year old man presend with witnessed acute onset of right sided weakness and slurred speech o f 30
minute prior to presentation he has a history of hypertension, type II diabetes mellitus and hyperlipidemia. On
examination blood pressure 200/116 mmhg, heart rate 98 beat/min his speech is slurred and he has 3\5 strength
on the right, CT scan shows ischemic stroke, a basis metabolic panel, CBC and coagulation test are all normal, the
patient has no history of stroke, and intracerebral bleeding, head trauma or recent surgery.
What is the next immediate step:

a) IV labetalol to lower the BP to below 185\110 mmhg and then thrombolytic therapy
b) immediate thrombolytic therapy
c) permissive hypertension, aspirin, statin and close monitor
d) IV labetalol to lower Blood pressure by 25%, aspirin, statin & close monitor
69) a 34 year old man presents for a routine visit, he has no complain today, he has history of obesity (BMI = 34)
hyperlipidemia and tobacco use, only medication is statin, blood pressure 136\86, lab examination reveal
creatinine of 1.0 mg and Hg A1c of 5.7%, what is the next appropriate action to control blood pressure?

a) order 24 hr ambylatory blood pressure monitoring


b) Council patient on life style modification
c) start hydrochlorothiazide 25 mg daily
d) order genetic studies to determine the patients risk of hypertension

70) 35 year old white man is admitted for an unprovoked DVT of the left femoral vein he has 1st degree relative
of unprovoked DVT he is started on low molecular weight heparin and warfarin, he is interested in understanding
the cause or any precipitating factors for DVT, for this patient, what is the most appropriate?

a) anti-thrombin functional assay


b) protein C functional assay
c) factor V leiden genetic testing
d) protein S functional assay
e) none of the above

80.A 26 y women reports experiencing generalized pain ,which become progress, worse over the past several
months, her pains specifically worse in the joints. Symptoms have coincided with difficulty sleeping and
generaliezad walars.and lethargy, on examination she has a hyperpigmented malar rash , moderate synovitis of
the hands, and pale conjuctiva,
Which of the following is the most likely diagnosis for this patient?

a.reumatoid aritritis
b.systemic lupus
c.fipromyal;igia
d.polymialgia
e.hypothiroidism

81.A 47 y old man is complaining of fatigue ,lethargy and 4.5kg wheight gain for last 2 months .his past medical
history consists of hypertension and hyperlipidemia for which is take lisinopril, simvastin,on examination,blood
pressure 130/78 and heart rate 54 beats , mild nonpiting edema in his bilateral lower extrimties ,deep tendon
reflexs and delayed the rest of his examination in unmarkable, which of the following is the best screening testing
for this patient?

a.T4
b.TSH
C.T3
D.T4+T3

82.A43 years old man presents whith palpitation an shortness of breath,on examination blood pressure 124/80
h.rate 128 b/min irregular and T36.6c ,moderate bilateral exophtalmos ,a palpable goiter,a 2/6 systolic flow
murmur and mildweakness in bilateral lower extremities.An ECG confirm atrial fibrillation with rapid,ventricular
response and tsh at 0.003mm/ml los-4.7mm/ml (0.5-4.7mm/ml) which of the follwing is the most appropriate
treatment for AF. In THE PATIENT?

a.propylthiouracil
b.iodide
c.propranolo
d.prendison

<<
83.a 57 years old complaining of a mass in the neck she has not had any systimacly symptoms and has not been
losing weight . blood pressure , heart rate T normal. She does not have palpable nodule in the left pole of the
thyroid gland. She has no cervical or supraclavicular lymphoadenopathy. TSH levels are whith normal limits ,and a
sonogram of the thyroid confirms 1.8 cm cystic nodule in the left pole for this patient.
Which of the following is the best next step?

a.fna
b.radionuclide uptake imaging
c.ct of the neck
d. serum thyroglobulin assay

84.a 41 years old business excutive presents to your office and complaints of palpitation and shortness breath ,
after further questioning ,he admits to heavy alcohol consumption the previous evening ,on examination he is
found to have an irregular heart beat of 130 b/min, the most likely diagnosis is?

a.Ventricular tachycardia
b.ventricular fibrillation
premature V, contractions
c.atrial fibrelation
d.wPw syndrom

85) a healthy 26 yrs. Old man presented with abdominal cramping, fever for 2 days and diarrhea, 10 stools in the
last 24h with blood and WBCs. The most likely diagnosis is:

a- staphylococcal food poisoing


b- rotavirus
c- chron's disease
d. shigellosis
e- IBS

86) 59 yrs. Old female with a history of a heart murmur but no other symptoms or rihomatic fever. The murmur
is 2/6 crescendo decrescendo that radiates to the carotid artery. The murmur decreases with valsalva maneuver
and on standing, the most likely diagnosis is:

a. aortic stenosis
b- tricuspid regurgitation
c- mitral regurgitation
d- hypertrophy obstructive cardiomyopathy

87) a previous healthy 15 yrs. Old boy was at a party. he presented to the emergency room after a sudden onset
of anger, agitation and paranoia. He clam that a government agent is spying on him. The first thing to do is:

a- EEG
b- glucose rolerance test
c- PET for the head
d- thyroid function test
e. urine toxicology screen

88) flouxetine's most common adverse effect:

a- hypotension
b- liver toxicity
c. nausea
d- sedation
e- weight gain
89) appropriate medication for Tourette disorder:

a- bupropione
b. clonidine
c. haloperidol
d- paroxetine
e- venlafaxine

90. Which of the following is the most common adverse effect of methylphenidate ?

A. Hypotension.
B. Insomia.
C. Liver toxicity.
D. Tremor
E. Weight gain.

91. Which of the following qualities would be most commonly associated with a more favorable prognosis of
autistic disorder?

A. Easy toilet training.


B. Interested in mechanical toys.
C. Organized play.
D. Reciprocal conversion.
E. Reciting songs and poems from memory.

92. Which of the following laboratory abnormalities are most likely to be found in a patient with anorexia
nervosa?

A. Hypercholesterolemia.
B. Hyperkalemia.
C. Hypocarotenemia.
D. Increased thyroid stimulating hormone (TSH).
E. Leukocytosis.

93. Which of the following disorders is rarely confused with anxiety that stems primarily from medical disorder?

A. Panic disorder
B. Specific phobia
C. Obsessive compulsive disorder.
D. Post traumatic stress disorder
E. Generalized anxiety disorder

94.a 28 year old female was diagnosed to have (post stress disorder) PTSD after she was assaulted and raped 6
months ago. Over the next2 years, despite extensive therapy and pharmacological treatment, the response was
suboptimal. She was later diagnosed to have major depressive disorder and substance abuse problems. Which of
the following is the most common comorbid condition in women with PTSD?

A. Substance abuse
B. Depression
C. Obsessive compulsive disorder
D. Psychotic disorder
E. Eating disorder

<<
95. Which of the following is the most common somatoform disorder?

A. Conversion disorder
B. Somatization disorder
C. Pain disorder
D. Hypochondriasis
E. Body dismorphic disorder

96. Which of the following helps in distinguishing malingering and factitious disorder?

A. Age
B. Secondary gain
C. Deliberate production of symptoms
D. Course of illness
E. Response to conformation

97. A 46 year old women is frustrated and angry with her neurologist because he recommended a psychiatric
evaluation for what she describes as “medical problems”. She complains of dizznes, nausea, palpitations,
abdominal pain and sweating , but no one is able to give her the right ttreatment. She was seen by an ear
specialist , cardiologist , and a gastroenterologist , but nothing abnormal was detected. She was hoping that a
neurologist would find a cause for her symptoms. She also states that these symptoms occur with symptoms
without any precipitating factors, last for 20 to 30 minutes, and she feels as if she is “about to die”. The most
probable diagnosis in this women is:

A. GAD.
B. Major depressive disorder.
C. MVP.
D. Panic disorder.
E. Meniere’s disease.

98. A 76 year old male patient is admitted to an acute psychiatric unit for severe suicidal ideation. He admits to
feeling hopeless and refuses to contract for safety. He has a history of noncompliance with treatment and is
refusing to take any medications. He has severe psychomotor retardation and stops eating and drinking. A
reasonable choice of treatment in this patient would be:

A. Persuade the patient to take antidepressant.


B. Wait and watch for the patient to change his mind.
C. Consider feeding against his well.
D. Electroconvulsive treatment (ECT).
E. Intensive psychotherapy.

99. One of the criteria for the diagnosis of major depressive disorder is weight loss that is unintentional. The
criteria for weight loss specify that:

A. Loss of any amountof weight is significant.


B. Loss of 5% or more of the body weight in the past one month.
C. Loss of 10% or more of the body weight in the past one month.
D. Loss of 10% of body weight in the past 2 weeks.
E. Loss of 5% of body weight in the past 2 weeks.

100. Mania can be associated with all of the following features except:

A. Sometimes associated with depression symptoms.


B. Can be triggered by antidepressants.
C. Irritability is a recognized symptom.
D. Is always followed by depression.
E. Can occur during bereavement.
101. Which of the following illicit drugs can lead to a full range of symptoms similar to that seen in patients with
schizophrenia?

A. Ecstasy.
B. Alcohol.
C. Marijuana.
D. Phencyclidine.
E. Opiates.

102. A 46 year old male is stable on low dose haloperidol for chronic schizophrenia. Recently he was diagnosed to
have comorbid depression and was started on fluxetine 20mg. he calls the physician with complains of stiffness
in his arms and legs. What is possible cause of these symptoms?

A. Malingering, the patient would like to stop taking antidepressant medicine.


B. Increase level of haloperidol because of fluxetine.
C. Fluxetine is causing extrapyramidal side effect symptoms.
D. Decrease levels of haloperidol are causeing extrapyramidal side effects.
E. Increase level of fluxetine because of haloperidol.

103. What is the characteristic feature of chronic marijuana abuse?

A. Motivational syndrome.
B. Seizure.
C. Wernick’s encephalopathy .
D. HPPD.
E. Stroke.

104. A 52 year old male is admitted to medical unit for an ulcer on his foot. 2 days after admission, he was found
to tolerate the Intravenous antibiotics well and has no fever. However he becomes restless and appears slightly
confused. He also complains of insomnia and the nurse noticed he is diaphoretic with an increase heart rate and
blood pressure, physical examination reveals a course tremor of the upper extremities and his reflexes are
described as brisk , what is the most likely diagnosis?

A. Alcohol withdrawal.
B. Opiate withdrawal.
C. Methamphetamine withdrawal.
D. Nicotine withdrawal.
E. Marijuana withdrawal.

105. Which of the following is considered to be a poor prognostic factor with patient with anorexia nervosa?

A. Late age of onset.


B. Family members open to participate in patient care.
C. Early age onset.
D. No suicidal behavior.
E. No previous hospitalization.

106. Which of the following is the most significant risk factor for suicide?

A. Psychiatric disorder.
B. Death of a loved one.
C. Loss of job.
D. Lack of home.
E. Conflict at work
107. Which of the following medical disorder is associated with the highest risk of suicide?

A. Epilipsy.
B. Huntingtons disease.
C. AIDS.
D. Cancer.
E. Brain injury.

108. cortisol has a significant impact on mood and cushing’s syndrome is often associated with psychiatric
disturbance. What is the most common psychiatric manifestation in patient with cushings syndrome?

A. Mania.
B. Psychosis.
C. Anxiety.
D. Panic attack.
E. Depression.

109. A majority of the psychotropic medications are metabolized in the liver. Which of the following statements
is true regarding the treatment of depression in hepatic disease?

A. Fluxetine is not cleared by hepatic enzymes.


B. Lithium is the mood stabilizer of choice in the presence of hepatic failure.
C. SSRI’s are contraindicated in liver disease.
D. Paroxetine causes minimal inhibition of hepatic enzymes.
E. Half lives of drugs are reduced in liver disease.

110. Which of the following statements is true about suicide in medically ill patients ?

A. Most terminally ill patients develop a psychiatric disorder.


B. Most terminally ill patients are at risk of suicide.
C. Anger is an important factor in suicide.
D. Medical illness is not common in patients who commit suicide.
E. suicide in medically ill patients is not preventable.

<<

<<

<<

<<

<<

<<

<<
Part B:

1) in distinguishing schizophrenia from amphetamine induced toxic psychosis the presence of which of the
following in the most helpful?

a- tectile or visual hallucination


b- paranoid delusion
c-intact orientation
d- clear consciousnes
e- auditory hallucination

2) which of the following electrolyte abnormality is associated with bulumic patient ?

a- metabloic acidosis
b- respiratory acidosis
c-metabloic alkalosis
d- respiratory alkalosis
e-normal

3) PTSD differs from acute stress disorder(ASD):

a- ASD occurs more commonly than PTSD


b- PTSD is associated with at least 3 dissociative symptoms
c- reexpercing the trauma is not found in ASD
d- avoidance of stimuli is associated with the trauma is only found in PTSD
e- PTSD lasts less than 1 month after trauma

4) double depression:

a- 2 family members with depressive disorder concurrently


b-recurrent major depressive disorder with current symptoms twice as disabiling
c- two episodes of major depressive disoder at month
d-superimposed bipolar 2 disorder with atypical depression
e- recurrent major depression disorder superimposed with dysthmic disorder

5) clozapine:

a. Is associated with few, if any, extraperamidal side effects.


b- is believed to exert its therapeutic effect mainly by blocking the dopamine receptors.
c- causes significant increase in prolactin level
d- is associated with 10-20% incidence of agranolocytosis
e- requires monthly monitoring of blood chemistry

6) late onset schizophrenia:

a- is more common in men


b. is associated with a preponderance of paranoid symptoms
c- is clinically distinguished from early onset schizophrenia d- results in
poorer response to antipsychotic medication
e- has an onset after 60 years of age.

7) minor signs and symptoms of discontinuation on benzodiazepine:

a- hyperpyrexia
b- grand mal seizure
c- psychosis
d. nightmares.
e- death
8) which of the following complications would be the best indication for admitting patient with anorexia
nervoasa to the hospital:

a- anemia
b. arrhythmia
c- bradycardia
d- hypotention
e- lanugo

9) Which of the following would be the most appropriate treatment for panic disorder:

a- aripiprazole
b- carbamazepine
c- resperidone
d. sertraline
e- valporic acid

10) in bipolar disorder, which tests should be done before treatment with lithium?

a-thyroid
function test, creatinine, pregnancy test
b-thyroid function test, creatinine, liver
c-thyroid function test, creatinine, complete blood count
d- thyroid function test, liver function test pregnancy test

11. You are seeing a 4 week infant who appears jaundiced he is growing well without any
problem and is exclusively breastfed ,his physical examination is notable only for some
scleral icterus and jaundice to the appear chest your next course of action would be?

a. abdominal ultra sound examination


b. fractionated serum bilirubin
c. observation with follow up appointment in 1 month
d. liver function tests urine culture

12. what would be the best first line management in preventing exacerbations in patient with chronic
asthma?

a. inhaled albuterol (ventolin)


b. leukotriene inhibitors
c.antiimmunogglobulin E
d. inhaled corticosteroids
e. cromolyn sodium

13. You are seeing a 12 year old female with h fever arthritis oral ulcers malar rash and
photo sensibility you suspect she may have an immunologic disease, which of the following tests would be
most specific for her disease:

a. antiendomesial antibody
b. anti double sranded DNA
c. anti strepolysin antibody
d. serum immunogobulines levels

14. Which of the following co morbidity complications is associate with overweight in adolescents?

a. slipped capital femoral epiphysis


b. hypoglycemia
c. angina
d. osteomalasia
e. fat necrosis
15. nasal polyps in children:

a. may cause serous otitis


b. are common in early infancy
c. may cause suffocation or apnea
d. are associate with cystic fibrosis
e. are most often cancerous

16. Which of the following causes of congenital infection is associate with cats?

a. cytomegalovirus
b. rubella
c. toxoplasma gondi
d. syphilis
e. parvovirus B19

17. Patient who should receive prophylaxis for pneumocystis carini pneumonia mor than others include
those with:

a. X linked agammaglobulinemia
b. HIV infection
c. chronic granulomatous disease
d. sickle cell disease
e. congenital neutropenia

18. A worried mother calls into report that her 2 days old newborn is bleeding from her bottom, her
prenatal and birth history were unremarkable. On physical examination you noticed a bloody
serosanguious fluid oozing from the vagina. There is no bruising or
patechia. The most likely diagnosis is:

a. hemophilia A
b. child abuse
c. birth trauma
d. hemangioma of the vulva
e. withdrawal bleeding

19. A 10 years old white female presents with pain and swelling of the right elbow,
accompanied with fever of 39 c, she reports that 3 days ago she had painful swelling of the left ankle the
subside this morning. 2 weeks ago she had sore throat. Laboratory results reveal elevated sediment rate, a
negative antinuclear antibody, and an elevated anti
streptolysine. The most likely diagnosis is:

a. SLE
b. juvenile idiopathic arthritis
c. acute raumatic fever
d. gonoccocal arthritis
e. septic arthritis

20. A 12 months old boy is examined because of refusal to walk. He started to walk at 10
month and has been doing well until this point. On physical examination he is moderately ill child with
fever of 39.2 c , externally rotated left hip with limit to passive and active motion. The CRP level is high.
The most likely diagnosis is:

a. osteomyelitis of the left femur


b. septic arthritis of the left knee
c. legge perthes disease
d. septic arthritis of left hip
e. ewing sarcoma of the left femur.
21-3 years -old boy presents to an urgent care unit with 3 days history of abdominal pain and difficulty
walking. Abdominal findings includ blood pressure of 120/80 mm Hg , diffuse abdominal tenderness,
purpuric rash of hands and ankles, and diffuse periarticular tenderness and swelling of the ankles.the mos likele
diagnosis is:

a. Systemic lupus erythematosus


b. Kawasaki desease
c. Juvenile rheumathidarthritis
d. Henoch -Schonlein purpura
e. Stevens _ Johnson syndrome

22-A 10 years-old boy present to clinic routine preparticipation sports physical. He reports no recent illness
or unusual syntoms.on examination you hear a distinct short systolic eyection murmur. It is l oudest at the left
lower sternal border and is low pitched and somewhat musical nature. When the patient sits upright the murmur
is less intense.Which of the following is the most likely diagnosis?

a-Aortic valve regurgiatation


b- ventriculo septal defect
c-Atrial septal defect
d- pulmonary valve stenosis
e-vibratory innocent murmur

23-Trisonomy 21 is most commonly associated with:

a-malrotacion
b-atrioventricular canal
c-cleft palat
d-renal failure
e-sensory neural hearing loss

24- What is the most significant serious complication arising from kawasaky desease?

a-coronary aneurisms
b- kidney failure
c- gallbladder stones
d- gastrointestinal bleeding
e-hypertension

25- a 20 months-old toddler is seen in the emergency department because of an 8 hours


history of difficult breathing. Previously he had been perfectely well. During supper he began having
paroxysmal coughing and wheezing. He has not hat any previous similar episodes .
He has moderately increaded work of breathing. On auscultacion the wheezing is heard only over his right
chest. His vital signs are: temperature 37C, pulse 136, respitarory rate 60. The most lakely diagnosis is:

A. Asthma
B. Cystic fibrosis
C. Pneumonia
D. Poreign body aspiration
E. Peanut allergy with anaphylaxis

26-A 10 months-old infant has poor weight gain and persistent cough. The mother describes the child as
having very large , foul smelling stools for months . Which of the following diagnostic modalities is lakely to result
in correct diagnosis of the child?

a-CT examination of the chest


b-serum inmunoglobulins
c- PPD. Skin test
d-inspiratory and expiratory chest X-ray
e-Sweat chloride test
27- 15 years -old girl with short stature ,neck webbing and sexual infantilism is found to have heart
murmur , and weak femoral pulse. Which of the following is the most likely diagnosis?

a-Achondroplasia
b-Turner Syndrome
c- Di George Syndrome
d- congenital hipotiroidism
e-Celiac desease

28-a 6 year- old is hospitalized for observation because of short period of unconsciousness after fall from
plauground swing. He had developed unilateral papillary dilatation , focal seizures, recurrence of depressed
conscioness and hemiplegia. Which of the following is the most appropriate measure at this time?

a-spinal tap
b- CT scan of the head
c-rapid fluid hydratation
d-naloxone
e-bilateral burr holes

29-6 year- old boy present with the sudden set of ataxia which of the following is the most likely cause?

a-drug intoxication
b- agenesis of corpus callosus c-
ataxia telangiectasia
d-muscular dystrophy
e-Friedrich Ataxia

30-a 3 year- old boy parent’s complain that their child has difficult walking Tha child perfomed adequately until a
few months ago when the family notical an increased inward curvature of the lower spine as he walks and gait
became wadding On examination he has anlargedcalves. Which of the following is the most likely diagnosis?

a-spina bifida oculta


b-muscular dystrophy
c-brain tumor
d-Guillain Barre syndrome
e-Botulism

31- A 14-year-old girl awakens with a mild sore throat, low-grade fever, and a diffuse
maculopapular rash. During the next few hours, she develops tender swelling of her wrists and redness of
her eyes. On examination there is marked swelling with mild tenderness of her posterior cervical and
occipital lymph nodes. Four hours later the rash has vanished.
Which of the following is the most likely diagnosis?

a. Measles
b. Roseola
c. Erythema multiforme
d. Rubella
e. Erythema infectiosum

32- Which of the following conditions is contraindicated to a diagnostic lumbar puncture?

a. Thrombocytopenia
b. Bulging fontanelle
c. Lumbar puncture 2 days before
d. Marked uncooperativeness on the part of the patient
e. Significantly elevated WBC count consistent with bacteremia Correct
33- 2 weeks ago, 2 year old boy developed diarrhea, which has persist to the present time
despite dietary management. His stools have been watery, pale and frothy without traces of
blood. He has been febrile. Examination of his stool is likely to reveal which of the following?

a. Salmonella sonnei
b. Enterobius vermicularis
c. Cryptosporidium jejuni
d. Toxoplasma gondii
e. Rotavirus

34- An 18 month child presents with a brief, generalized tonic-clonic seizure. He is now
postictal and has a temperature of 40°C. During the lumbar puncture (which ultimately
proves to be normal), he has a large, watery stool that has both blood and mucus in it.
Which of the following is the most likely diagnosis in this patient?

a. Salmonella
b. Enterovirus
c. Shigella
d. Campylobacter
e. Rotavirus

35- 12 years old ashkenazy jewish girl has mild anemia , leukopenia and thrompocytopenia . pfysic
examinations revealsan enlarged spleen. An x-ray of the femur is described as appearing to be “ Erlenmeyer
flask”. Bone marrow examination shows abnormal cells. Which of the following the most likely diagnosis?

a. Tay-sachs disease
b. Gaucher disease
c. Mucopolysaccharidosis
d. Canavan disease
e. Glycogen storage disease

36- Which of the following cancer has the highest incidence in young children (<7 years)?

a. Ewing sarcoma
b. Hodgkin disease
c. Testicular cancer
d. Retinoblastoma
e. Osteosarcoma

37- A12 month-old female infant with failure to thrive is brought to the office. Her parents
note that she is very fussy and often spits up after feeding. she also has two loose foul- smelling stool each day.
Which of the following foods can she eat safely without aggravating her symptoms?

A) Rice
B) Wheat
C) Oats
D) Barley
E) Rye

<<

<<
39- An infant born 35 weeks gestation to a mother with no parental care is noted to be
jittery and irritable and having difficulty feeding. You note coarse tremors on examination with high
pitched cry. Diarrhea and enuresis. You suspect the infant is withdrawing from which of the following?

a.Alcohol
b.Marijuana
c.Heroin
d.Cocaine
e.Tobacco

40- A primiparous woman whose blood type is O positive gives birth at term to an infant who has A-positive
blood and a hematocrit of 55%. A total serum bilirubin level obtained at 36 hours of age is 12 mg/dL. Which
of the following additional laboratory findings would be characteristic of ABO hemolytic disease in this infant?

a. A normal reticulocyte count


b. A positive direct Coombs test
c. Crescent-shaped red blood cells in the blood smear
d. Elevated hemoglobin
e. Petechiae

Q41 - a 3 year old boy is evaluated because of pallor and weakness. Laboratory tests reveal microcytic and
hypochromic anemy. The ferritin level is low. Which of the following is the most likely diagnosis?

a - folate deficiency
b - thalassemia minor
c - aplastic anemia
d - iron deficiency
e - vitamin B12 deficiency

Q42 - a 6 year old boy is examined because of nighty bedwetting. The boy has never had a prolonged
period of nighttime dryness and never has "accidents" while awake. He has no dysuria and his urinary
steam is normal. What is the most common cause of his primary nocturnal enuresis?

a - urinary tract infection


b - vesicoureteral reflux
c - psychological problem
d - normal developmental variant
e - unstable bladder

Q43 - a 5 year old boy presents with a history of grossly bloody urine, puffy eyes and
headache for a 1 day. He has been a well child but did have a fever and sore throat 10 days ago, which
resolved without treatment. The most likely diagnosis is:

a - acute cystitis
b - Ig A nephropaty
c - acute pyelonephritis
d - postinfectious glomerulonephritis
e - benign hematuria

Q44 - which of the following is regarded as a known complication of treatment with total parenteral
nutrition (TPN)?

a - sepsis
b - renal failure
c - chronic diarrhes
d - vitamin A deficiency
e - irreversible athropy of the mucosa of the small intestine Correct
Q 45 - a 3 month old infant presents for a well child evaluation. Which of the following would be a
cause for concern in the infant?

a - regurgitation of 15-30 ml of formula 3 times a day


b- one bowel movement every other day
c - 3 bowel movements per day
d - liver span of 5 cm
e - hemoccult-positive stool

Q46 - during routine screening CBC, a 1 year old is noted to have pronounced eosiniphilia.
Which of the following is the most likely explanation?

a - bacterial infections
b - chronic allergic rhinitis
c - fungal infections
d - helminth infestation
e - tuberculosis

Q47 - an 17 year old male is seen in the student clinic for urinary frequency, dysuria and uretheral
discharge. Which of the following is not likely to explain this condition?

a - herpes simplex
b - Escherichia coli UTI
c - chlamydial urethritis
d - syphilis
e - HIV infection

Q48 - a 6 week old baby presents to the office. His weight is still near his birth weight. He
had a normal gestation and delivery and has not had any signs of illness. The physical
examination is normal. A diagnosis of failure to thrive (FTT) is made. Which of the following is indicated?

a - immediate hospitalization and extensive laboratory tests.


b - increasing the caloric content of formula and frequent weight measurements
c - starting solid foods since formula is not resulting in good growth.
d - report the child to the health services and immediate placement in foster care
e - switching formula to a different cow's based formula

Q49 - you are evaluating a 5 year old for the first time. His height just below the 5th
percentile. Previous growth chart reveals that since 9 months his weight and height are just below the 5th
percentile. His mother height is 155cm and his father's height is 163. History and physical examination is
otherwise normal. What is the most likely diagnosis?

a - constitutional growth delay


b - growth hormone deficiency
c - noonan syndrome
d - familial short stature
e - hypothyroidism

Q50 - a 15 month old male is brought to the pediatrician because he seems much smaller
than his two older brothers were at that age. He has been generally healthy exapt for 2
episodes of otitis media and occasional "colds". He began walking at 11 months and can now say "mama"
"dada" and names of his brothers. What is the most appropriate next step?

a - perform a denver development screening test.


b - ask the mother to complete a 3 day diary of all the food that the child has eaten.
c - send blood for equantitative immunoglobulins
d - plot his height and weight on growth chart and compare to previous charts.
e - obtain a sweat chloride test.
51 . a 4 year old presents with a dry scaling rash wish recurs intermittently. The family history is positive for
asthma ,allergy and eczma. Treatment of his eczma includes which of the following?.

a . keeping the skin scrupulously clean with baths twice daily


b . allergy testing and allergy shots
c . a strict elimination diet
d . steroid cream and moisturizing lotion
e . oral steroid

52 . a teenager who gets stung by a bee develops uriticaria within 30 minutes of the sting this reaction is
most likely mideted by ?

a . complement C3
b . IgE antibodies
c . neutrophilis
d . T cells
e . Ig A

53 . a 1 month female is examined because she spits up about a tablespoon of milk-like


material after each feed .it does not come out forcefully and contains no blood or bile . she
takes 50 cc of formula every 2-3 hours the spitting up often occurs after she is placed on her back in the crib
the physical examination and growth percentiles are normal .the most likely diagnosis is ?

a . pyloric stenosis
b . volvulus
c . gastroesophageal reflux
d . gastroenteritis
e . tracheoesophageal fistula

54. malrotation with volvulus is most likely to be present in which of the following patients?

a . 15 months old with severe paroxysmal abdominal pain and vomiting


b . 15 year old sexually active girl with lower abdominal pain
c . 3 day old term infant with bilious emesis .lethargy and abdominal distention
d . 4 day old premature ( 33 w ) baby who just started nasogastric feedings he has abdominal
distention bloody stools and thrombocytopenia
e . 7 year old girl with abdominal pain vomiting fever diarrhea Correct

55 . in female which of which of the following events signifies the onset of puberty ?

a . menarche
b . growth spurt ( height )
c . pubic hair
d . thelarche
e . voice dranges

56 . a full term neonate is noted to have circumoral cynosis and twitching of his left hand at 12 hours of age
on physical examination he is found to have an absent pupillary response to light and a small penis
which of the following is the most likely diagnosis:

a .hypocalcemia
b . hypoglycemia
c . congenital hypothyroidism
d . congenital heart disease
e . idiopathic epilepsy
57 . a 14 year old adolescent female has insulin dependent diabetes mellitus for the past 7 year she now
has a hemoglobin A1C of 14.9% this laboratory test indicates which of the following ?

a . her glucose control is poor


b . she does not have insulin dependent diabetes mellitus
c . she has entered the honey-moon phase of her diabetes
d . she has an underlying infection
e . she is demonstrating the somogy phenomenon

58 . a newborn infant has mild cyanosis , diaphoresis ,poor peripheral pulses, hepatomegaly and
cardiomegaly . respiratory rate is 60 breaths per minute and heart rate is 250 bmp the child most likely has
congestive heart failure caused by which of the following ?

a . a large atrial septal defect and valvular pulmonic stenosis


b . transposition of great vessels and ventricular septal defect
c . total anomalous pulmonary venous return
d . Hhypoplastic left heart syndrome
e . paroxysmal atrial tachycardia

59 . a 6 year old girl underwent removal of a craniopharyngioma 3 mo previously. she is on a thyroid


replacement regimen but her mother reports she is very fatigued the next step would be to ?

a . increase the dose of her thyroid medication


b .schedule a growth hormone stimulation test
c . measure ACTH and cortisol
d . obtain an M.R.I study of her hand
e . measure IGF-1 and IGFBP-3

60 . a 20 mo.old child has fever irritability and refusal to move his right lower extremity physical examination
reveals a swollen and tender right knee that resists passive motion which of the following is the most
important test to confirm the impression of septic arthritis?

a . examination of joint fluid


b . x - ray of the knee
c . erythrocyte sedimentation rate
d . complete blood count
e . blood culture

61) A 27 year old man presents to the Emergency room after a high speed motor vehicle collision, with
chest pain and marked respiratory distress. Physical examination, he is hypotensive with distended
neck veins and absence of breath sounds in left chest. Which of the following is proper initial treatment?

a. Intubation
b. Chest X-ray
c. Pericardiocentesis
d. Chest decompression with needle
e. Emergent thoracotomy

<<

<<
62) A 33 year old man who had been morbidly obese underwent successful Roux-en-Y
gastric bypass 33 month earlier. However, recently he has been having disturbing
symptoms. These include insomnia, sensory disturbances, such as numbness and tingling in
his hands and feet, as well as inflamed tongue, dizziness, poor coordination and balance. In
addition he developed strange way of walking, a stomping gait striking first heavily with his
heel. The deficiency that would most likely cause symptoms described is which one of the following?

a. Iron
b. Vitamin B12
c. Folate
d. Zinc
e. Copper

63) A 65 year old man with long history of Coronary artery disease and recent history of
Myocardial infarction is seen in Emergency room with history of sudden onset of pain in right leg. The
patient also complains of numbness in the leg. Physical examination revealed pallor of right leg, the skin was
cool to touch, and the patient has difficulty in moving his toes.
Raising his leg increase the pallor and dorsalis pedis pulse could not be felt.
These symptoms most likely represent which of the following abnormalities?

a. Superficial thrombophlebitis
b. Herniation of a lumbar disk
c. Arterial occlusion
d. Deep venous insufficiency
e. Hypovolemic shock

64) A 38 year old man was taken on an Emergency room after his car skidded on the free way and stuck
a pillar. His vital signs were as follows: pulse- 88/min., respiratory rate -20/min., blood pressure- 100/70 mmHg.
Physical examination revealed non prominent jugular neck veins, no indication of cyanosis and systemic breath
sounds. A posterioranterior chest X-ray film reveals a widened mediastinum. Which of the following choices is
most likely diagnosis?

a. Ruptured Aortic Aneurysm


b. Cardiac temponade
c. Dissection of the thoracic aorta
d. Myocardial contusion
e. Pulmonary contusion

65) A 68 year old man presents at the Emergency department because of such severe
abdominal pain that he “ Just could not stand it any longer “ . He tells the triage nurse that
he had not been feeling well for the past couple of months, primarily because he had been
having abdominal pain about 30 minutes after eating and as a consequence lost almost 10
pounds, but last night he suddenly developed “ a stomach ache from hell “. He also has been
vomiting and has had several episodes of bloody diarrhea. Upon physical examination the
physician notes hypotension and notices abdominal distention. Bowel sounds are absent,
and there is no rebound tenderness present or other relevant findings upon abdominal
examination. Laboratory data reveal an absolute neutrophilic leukocytosis and mild left shift plus lactic
acidosis, and elevation of the serum amylase level. Which of the following is the most likely diagnosis?

a. Acute ulcerative colitis


b. Hemorrhagic pancreatitis
c. Aortoenteric fistula
d. Acute small bowel infarction
e. Toxic megacolon
66.A grossly underweight 52 y/o women with chronic malnutrition is due to undergo major surgery . it
decide to start total parental nutrition (TPN)nas part of intial therapy . while introducing a central venous
catheter into right subclavian vein she develop sudden dysnea . What is the most diagnosis ?

a. Air embolism
b. pulmonary embolism
c. fat embolism
d. acute anxiety
e. pneumothorax

67.72 y/o female is seen in emergency department with a history of sudden sever colick
midabdominal pain and nausea and vomiting over the past 3 hours . during examination she throw up a bile-
stained vomits . physical examination reveal generalized abdominal tenderness with decreased bowel sound ,
radiography of abdomen show radioopaqe mass in distal small bowel on right side with dilated loops proximally .
no free air is noted under diaphragm but air is present in biliary tree . What is mostly diagnosis ?

a. lymphoma for small bowel


b. acute pancreatitis
c. acute appendicitis with radiopaque fecalith
d. gallstone ileus
e. intussusception

68. A 35 y/o fire man was badly burned when the roof of house collapsed on him .
He had second-degree burn involving 15% of his body surface and third degree burn
involvig 20% of body surface . he was rushed to the nearest emergency department and
then transferred to burn unit . a week later he develop fever , black patches are noted in the burn wound .
Biopsy and culture of one of wound site would most likely reveal which of the organisms ?

a. staph aureus
b. pseudomonas aurgensa
c. candida albicans
d. group a streptococcus
e. streptococcus pneumonia

69. A 55 y/o alcoholic man with chronic pancreatitis has recurrent attack of abdominal pain that radiate to back
. pain is controlled with medical therapy , he lost 30 Ib in past 3 months because of chronic diarrhea .
examination of abdominal show no masses , CT shoe multiple calcification but no mass lesion , serum glucose
normal , qualitive stool test for fat normal , antiglidian antibody are not present , most appropriate treatment is ?

a. total pancreatectomy
b. broad spectrum therapy
c. gluten free diet
d. oral pancreatic enzyme before , during , and after meals
e. administration of lactulose

70. A 55 y/o man present to emergency with massive hematemessis . physical examinasion reveal abdominal
distension , shifting dullness on percussion and spiderangiomato over the faceand upper chest , an emergent
endoscope reveal bloody rapidly filling distal esophagus . Hematemessis is most likely due to ?

a. pyloric stenosis
b. ruptured esophageal varices
c. gastric ulcer
d. esophageal carcinoma
e. duodenal ulcer
71) a 68 years old man with intermittent cramping abdominal pain , also have difficulty
defecating , he is managing bowel movement every third or fourth day only , with grait *** of straining and
then only pass small hard feces with lot of mucus and sometimes fresh blood . his physician suspects obstipation
and orders a barium enema that reveals massively dilated sigmoid
colon with column of barium resembles a beards peak . what is the most likely diagnosis?

a- intusseption
b- volvulus of sigmoid colon
c- toxic mega colon
d- ogilvie syndrome
e- impacted stool

72) a 18 years old woman presents to the emergency department with a history of severe
retrosternal chest pain that is aggravated by swallowing and deep breathing . the patient
appears anxious , she is afebrile , has sinus tachycardia, slightly elevated blood pressure and tachypnea ,
mild pallor is noted , but she seems well hydrated and has no icterus .
examination of the cardiovascular system had normal findings except for sinus tachycardia , the abdomen is
schaphoid and no tenderness,masses or organomegaly is noted. her wight is less than the norm for her age
and hight . the patient however feels that she is obese and has taken to binge eating followed by self induced
vomiting . which of the following is the most probable cause of her problem ?

a- gasroreflux desease
b- boerhave syndrome
c- tension pneumothorax
d- gastric ulcer desease
e- esophegeal cancer

73) a 65 years old woman presents to the emergency department with diffuse abdominal
pain and vomiting , she has not had a bowel movement in the past 3 days , physical exam
reveals hyper peristalsis , tympany to percussion , and no rebound tenderness , her
temperature is 38 c (100.4f). an abdominal x-ray film reveals distended loops of small bowel with a step
ladder pattern of differential air-fluid levels. which of the following is the mechanism that most likely
produce these symptoms?

a-diverticulosis
b- adhesions from previous surgery
c- torsion of the bowel around the mesenteric root
d- intussusiption of the terminal ilium into cecum
e- ischemia secondary to thrombosis of the superior mesenteric artery Correct

74) a 35 years old man with a long history of dyspepsia, experiences sudden onset of sever epgastric distress
with associated pain in the right shoulder . physical exam reveals a patient who appears ill and has rigid, quit
abdomen with rebound tenderness. which of the following is the most appropriate first step in the management
of this patient?

a- order a barium study of the upper GI


b- order upright and supine abdominal films
c- perform a peritoneal lavage
d- administer anti-acids
e- do an exploratory laparatomy
75) 20 years old man is stapped in the left side of his chest, medial to the nipple . upon
examination his blood pressure is 90/60 mmHg , and his pulse is 130/min , his jugular
venous pulse increases on inspiration , whereas his peripheral pulse and blood pressure
decreases on inspiration . breath sounds are normal bilaterally . the patient's chest x-ray film is unremarkable
. after receiving 2L of isotonic saline , his blood pressure remains low ,
whereas his central venous pressure rises to 32 cm H2O .
which of the following is the most appropriate next step in the managing of this patient ?

a- insert a chest tube into the left pleural cavity


b- increase parenteral fluids until the blood pressure increases
c- order an echocardiogram
d- decrease venous pressure by administering ventilator
e- decrease venous pressure by administering loop diuretics

76. A 50 years old womam arrived at the E.D brought by his sun ,tell to the triage nurse that after
consuming a hearty thankgiving meal felt ill ,pain and symptoms grew worse during
the night and early morning and have 2-3 episodes of gallbladder trouble pain but non so
severe ,t×102f ,BP 90/42 ,sever right cuadrant pain ,jaundice ,problems with answer
question and babble incoherent ,mild hepatomegaly .PR125/min ,WBC 13,6*10-3 (N4,8-
10,8)billirubina total 6,6mg/l ,tranpeptidase activity rate is 105 ,alcline phosphatase value 176umts/l ,urine
analysis is normal.

a-amebic liver absces


b-acute hepatitis.
c-acute pancreatitis
d-ascending colangitis.
e-sclerosis pericolangitis.

77- 67 y. old woman with pelvic pain ,ct shows mass near her right pelvic wall ,ct guidede biopsy
revealed spidle type with few mitotic cell , at explor. ,a 2cm´ pedunculated mass is noted on the
anteromesenteric border of the mild ileum ,appropiate therapy...

a-ligated of the mass at the pedunculated stak.


b-widge excision of the mass .
c-widge excision ,regional lymph nodes disection.
d-widge excision ,imatinib
e-wide excision regional ltmph nodes disection ,imatinib.

78- refeedind in surgery related to:

Hypophosphatemia

78) WHAT is the hallmark biochemical feature of the resumption


of feeding syndrome(re-feeding syndrome), associated with increased mortality?

A) hypokalemia
B) hypophosphatemia
C) gipomagnezemiya
D) hypoglycaemia
E) hypocalcemia

79-a patient with cutaneous melanoma in the right leg ,irregular with discoloration ,but not ulceration ,right
inguinal region without palple nodes ,punch biobsy of the lesion reveal3`1mm thick lesion . 2cm`local excision of
the primary lesion ,next step…

a-observation .
b-sentenel lymph node biopsy .
c-elective lymphadenopathy .
d.RT(radiotherapy).
e-isolated lymph perfusion.
80- 50 y.old patient has a 3cm villous appearing tumor at 6 cm´ of the anus next step…

a-observe thelesion of sign of malignancy .


b-resure the patient that this are not premalignant lesio
c-fulgurate the lesion.
d-locally irradiate of the lesion .
e-surgically resect the lesion.

81- A 60 years old man, presents with jaundice for 2 weeks, there is no history of abdominal pain, But there
is a markedly distended gallblader seen on ultrasound. what is the most likely diagnosis :

a. Common bile duct obstruction from a stone


b. Common bile duct obstruction from pancreatitis
c. Common bile duct obstruction from pancreatic carcinoma of the head of the pancreas
d. acute cholecystitis
e. alcoholic hepatitis

82- A 29 years old man with a history of crohn’s disease involving the terminal ileum develops pneumaturia and
fecaluria. Witch of the following complications of crohn’s disease this patient have ?

a. ileodoudenal fistula
b. ileosigmoid fistula
c. recurrent disease involving a previous ileocolon anasromosis
d. colovescical fistula
e. acute cystits

83- A 65 years old man remains in the ICU for 1 month after coronary artery bypass grafting (CABG), he
has been treated for 2 episode of pneumonia and continues to require ventilatory support. He develops a
watery diarrhea and abdominal distension, stool sent for Clostridium difficile is positive and oral
metronidazole is initiated. 5 days later the patient develops hypotension, requiring vasopressors, and renal
failure, WBC= 36000/mm3. Which of the following is the most appropriate management ?

a. intravenous vancomycin
b. vancomycin enemas
c. cecostomy tube placment
d. diverting transverse loop colostomy
e. total abdominal colectomy and ileostomy

84- Small bowl perforation in blunt trauma is detected by:

a. CT abdominal scan
b. Focused Assessment with Sonography for Trauma (FAST)
c. Diagnostic peritoneal lavage (DPL)
d. physical examination
e. none of the above

85 - 23 years old patient carries a total thyroidectomy on carcinoma of the thyroid gland. On the 2nd day
after surgery, she began to complain of feeling in the bones of the hands pokalivaniya. It looks pretty excited, and
then begins to complain of muscle cramps. Which of the following is most correct?

A. 10 ml/10% of magnesium sulfate / in.


B. Vitamin D orally
C. Levomiroksina 100mcg orally. ???
D. Continuous infusion of calcium gluconate.
E. Glyukanat calcium glocunate orally.
86-A 10 years old boy was the backseat belled passenger in a high speed motor vehicle
collision . on presentation to the ER he is awake , alert,and hemodynamically stable. He is
complaining of abdominal pain and has an ecchzmosis on his anterior abdominal wall where the seatbelt
was located. Which of the following statements is true regarding need for additional workup?

a-the boy can be safely discharged home without any other workup , since his abdominal pain is
probably secondary to his abdominal wall ecchzmosis
b-the boy can be safely discharged home if his amylase level is normal
c- the boy can be safelz discharged home if abdomen plain films are negative for the presence of free air
d-the boy can be safelz discharged home if an CT is negative
e-the boy should be observed regardless of negative test result Correct

87- a 35 year old with a history of previous right thzroidectomy for a benign thyroid noduls
now , undergoes completion thyroidectomy for a suspicious thzroid mass several hours
postoperatively, she develops progressive swelling under the incision, stridor and difficulty breathing
.orotracheal intubation is successful -which of the following is the most appropriate next step?

A-fiberoptic laryngoscopy to rule out bilateral vocal cord paralysis


B-administration of IV cacium
C-administration of broad spectrum antibiotics and debridment of the wound
D-wound exploration
E-administration of high dose steroids and antihistamines Correct

88- a15 yaer old otherwise healthy female high school student begins to notice galactorrhea A pregnancy test
is negative . which of the following is frequently associated physical finding:

A-gonadal atrophy
B-bitemporal hemianposia
C-exophthalmos and lid leg
D-episodic hypertension
E-buffalo hump

89- A 46 years old woman ask about the need for surgery ,since she was recently diagnosed with crohns
disease .indication for operation in crohns include all except?

A-intestinal obstruction
B-enterovesical fistula
C-prolonged use of steroids
D-enterovaginal fistula
E-free perforation

90- A 41 years old complains on regurgitation of saliva and undigested food ,an
esophaogram reveals birds beak deformity. Which of the following statements is true about the condition?

A-chest pain is common in advanced stage of the disease


B-more patients are improved by forceful dialation then by surgical intervention
C- manometry can be expected to show increase resting pressure of the LES
D-initial surgical treatment consist primarily of resection of the distal esophagus with reanastomosis
of the stoma above the diaphragm
E-patients with this disease are at no increased risc for the devolpment of carcinoma Correct
91. A 55 years old man presents with his first episode of Diverticulitis. He is hemodynamical stable and
improves within 48 hours. He gets antibiotica. CT reveals stranding of sigmoid mesentery, no free air and
no abscess. The next Step in treatment should be:

a. Resection and anastomosis.


b. Resection and colostomy.
c. Discharge and elective colectomy.
d. Discharge, oral antibiotica and observation.
e. Colnoscopic intraluminal stent.

92. A 44 years old woman has papillary thyroid carcinom of the right lobe confirmed by fine-needle aspiration.
Ultrasoundstudy stages the tumor as T1 (1,5 cm). What schould bet he surgical procedure?

a. Right thyroid lobectomy, selective node dissection of the right side.


b. Right thyroid lobectomy, level 6 node disscection.
c. Total thyroidectomy, selective node dissection of the right side.
d. Total thyroidectomy, level 6 dissection and selective dissection of the right side.
e. Total thyroidectomy.

93. A 36 years old man has a blood presure of 70/60 mmHg after a motor vehicle crash.
Resuscitation is initiates. Focused Assessment with Sonography for Trauma (FAST) was
positive for fluid in the abdomen. On examination of the pelvic, fracture-related- cerpitus and an enlarging
perinal hematoma are appreciated. The blood oresure dose not improve with resuscitation.
What is the next step?

a. CT of the abdomen and pelvic.


b. Diagnostic peritoneal lavage.
c. Placement of external pelvic fixator.
d. Explorative laparotomy.
e. Pelvic angioemobilization.

94. A 19 years old man, who fell while playing basketball fractured the shaft of the right radius and ulna
subsequently underwent closed reduction and application of a cylindrical cast. He now presents with
complaints of pain in the right index finger. Thereis no tingling sensation. Clinical examination reveals no
diminution in capillary refill time, no hypoesthesia, cyanosis ist absent. Passive movements of the finger elicits
pain. Cast is intact. What should be your next step?

a. Reassure the patient.


b. Prescribe analgesis.
c. Prescribe corticosteroids.
d. Cleave the cast.
e. Leave the cast intact and raise the arm for 24 hours. Correct

95. A 14 years old boy come to the office because his breasts have recently become tender and slighty
swollen. Upon examination a tender, 2 cm mass is found to be palpable in the subareola region of both
breasts. What should be your next step?

a. Excise the masses by performig a subcutaneous mastectomy.


b. Incise and drain the mass
c. Treatment with steroids.
d. Aspirate the mass for culture and cytology.
e. Leave alone.
96.A 75 y.o. man was to the emergency room of local hospital after having collapsed at home.
the patients sapouse related that he had complaned of sadden sever pain over the left flank. he informed the
physician that he had been complaning of back itch for few years.
physical ex. revealed a conscions male ,acute distress, with definit signs of hypotention
,tachycardia,tachypnea and hypothermia.he had mucosal pallor but no cyanosis.the
abdomen was tender to palpitations,and a pulsatile supraumbilical mass was present.bowel sounds were
diminished and peripheral pulses were equal and moderately strong.the patient was given 100% O2 by
mask,at 10L per minute. the cardiac minotor showed sinus
tachycardia with mild ischemia.
Which of the following mechanisims is most likely involved in the pathogenesis of this patient clinical disorders?

A.cystic medial degeneration.


B.defect in collagen.
C.defect in fibrillin.
D.an immunologic reaction.
E.athrosclerosis.

97.A 65 y.o. women had a 5 years history of pain in the right hip that had qradually gotten worse.she had
reached a point where she hobbled with the help of a cane.nonsteroidal
analgetics did not help her very much.she finaly agreed to undergo surgery.an x-ray film of the right hip
confirmed severe osteoarthritis.the orthopedic surgean recomended hip
arthroplasty.she was considered at hish rist for deep venous thrombosis and heparin
prophylaxis was begun.
Which of the following is the best test to monitor heparin therapy?

A.thrombin time(TT).
B.prothrombin time(PT).
C.prothrombin and proconvertin test.
D.activated partial thromboplastin.
E.bleeding time. Correct

98.A 65 y.o. women complans of diarrhea and weight loss for several months.on physical ex. she is noted to
be slim with evidence of recent weight loss and a systolic heart murmar can be heard to the right sternal
border in the 4th intercostal margin.laboratory findings include urinary collection for 5HIAA increased to 4
fold the upper normal limit,and chromogranin A level of the 1000ng/m l(n<92 ng/ml).
Which of the folloing is the initial recomended managment of thes patient?

A.surgical resection of the bowel tumor.


B.chemotherapy due to metastatic tumor.
C.somatostatin analog.
D.interferon alfa.
E.administration of specific chromogranin antibodies.

99.A 74 y.o caucasan man recently made an appointment to see his physician becouse he
had difficulty in swallowing his pills .it took 2 weeks to get an appointment .by the time his
problem had progressed to the point where he was having problem with solids in general but
noproblem with liqids.in the past he had also a longstanding history of GERD wich was
treated with omeprazole starting last year. at present he doesnot smoke but he had started
smoking at the age 14 and quite some 10 y ago.he did drink ,but primarily beer and only at social occasion.

A.diffuse esophageal spasm.


B.achalasia.
C.bengin esophageal stricture.
D.zenker's diverticulum.
E.esophageal carcinoma.
100.A 73 y.o man with scandinavian ancestors was undergoing a routin physical ex. ehen his
physician noted a rounedish spot about 6 mm in diameter that was slightly pinker that the
remainder of the man face.when he ran his finger across it, he also noted that was dry and
rough.the patient informed him that something had been going on with that spot for a year
or so its apperance seemed to wax and wane, essentially disappearing once in a while but
then retarning,sometimes covered with white scales that then dropped of.the patient
admitted that as a boy and a young man he spent countless houres at athe beach.the
physician cauterited the lesion with liquid nitrogen within 2 weeks the area became crusted ,shrank and fell
off.2 weeks therefore,the rough spot reterned and after a month ,the patient developed an open sore at the
same location that would not heal.The sore is most likely to be wich of the following?

A.actinic keratosis.
B.basal cell carcinoma.
C.squamous cell carcinoma.
D.nodular melanoma.
E.herpes melanoma.

101- A 24 year old male playing football fell on his outstretched hand hoping to make a
touchdown. The patient complains on severe pain in his right arm. His vital signs are: BP
140/80, PR 98 regular, Temp 37C and RR 22/min regular. The right arm is swollen and
angulated in the mid arm area, it's tender to touch and movement is painful. The humeral
appear to be fractured, capillary circulation in the nail bed is normal. However, there is a
clinical evidence of nerve damage. X-ray imaging confirms fracture of the humeral shaft with some
angulated displacement. Which of the following is the nerve injury associated with this fracture:

a. Axiliary nerve
b. Median nerve
c. Ulnar nerve
d. Radial nerve
e. Brachial nerve Correct

102. While riding his bicycle 9 year old boy loss control and falls during the process. His
abdominal strikes the handle bar. His parents bring him to the Emergency Room because he has a vague
mid-abdominal pain and some bruising of the anterior abdominal wall. His vital signs are stable and there
are no other visible injuries.
Which of the following is the most likely injury:

a. Ruptured Spleen
b. Ruptured Liver
c. Ruptured Pancreas
d. Hematoma of the Rectus Muscle

103. 48 year old obese woman was admitted to the hospital with upper right quadrant pain and vomiting.
She has no diarrhea and no constipation. Clinical examination reveals
tenderness and appropriate investigation demonstrate a presence of stone in the Common Bile Duct
(CBD). Attempt to dislodge the stone endoscopically proved futiled, so she
underwent CBD exploration. 6 days post operation, the patient developed fever of 38÷C. Which of the
following is most likely the cause of fever:

a. Accumulation of blood from the peritoneum


b. Endotoxic shock
c. Wound infection
d. Renal failure

104. What is the cause for rise the rise in lactate levels during stress?

a. Muscle protein degradation


b. Anaerobic cell respiration
c. Increase ATP degradation
d. Increase gluconeogenesis
105. Which of the following is the most important to evaluate the risk for intra-operative bleeding?

a. Bleeding time
b. aPTT
c. INR
d. Medical history taking

106) during her visit to a primary care physician, 39 years old woman relates that she has
recently had bouts of hoarsenes aften associated with difficulty in swallowing and
breathing. Upon taking a history the physician also determines that she emigrated with her
parents from the Ukraine in 1988 at the age of 17 year. Upon examination the physician discovers a firm nodule
near her Adam’s apple. The next step in obtaining a diagnosis is which of the following?

a) analyzing the results of an ultrasound and scan.


B) doing a fine needle aspiration biopsy
c) a thyroid nuclear scan using 123I
d) determining the thyroid serum level
e) a formal surgical biopsy.

108) 65 years old man was sean by his family physician with a 6 month history of
constipation and recent history of pain in the left lower quadrant of his abdomen. The
patient complained of tenderness and fever as well. Physical examination revealed a
moderately obese man, blood pressure 138/90, pulse 76, respiration 16/min, temperature
37,8 °C. his cardiovascular and respiratory system was normal, examination of the abdomen
revealed tenderness in the left lower quadrant with a positive rebound. In addition a tender
mass was felt on rectal axamination. A complete blood count revealed neutrophylic
leucocytosis and the stool guaiac test results was negative. Most likely diagnosis?

a) ulcerative colitis
b) irritable bowel syndrome
C) acute diverticulitis
d) carcinoma colon
e) aschemic colitis

<<
109) 65 years old women went bicycling into the country near lake Tahoe/ the front wheel
struck a pathole and she fell to the ground. While doing so, her chest struck the handlebar.
She was immediately rushed to the local hospital, on arrival she is conscious and appears to
be in moderate distress . blood pressure 140/95 mmHg, pulse 96, respiration are shallow
and rapid at 22/min, bruising over the left anterior hemithorax is observed upon
examination of the chest. There is no jugular venous dilation. The rest of her examination
findings are unremarkable. Further examination would most likely yield which of the following?

a) increased breath sounds of the chest.


b) egophony on the left
C) increased hyperresonace
d) increased width of intercostals on the left
e) mediastinal structures shifted to the left.

110. A 38 year old woman visits her family physician because of her concern about a lump in
her left breast which she noticed after a fall approximately one week previously. She doesn’t
smoke, drinks wine on social occasions, and takes birth control pills. There is no family
history of breast cancer. Physical examination reveals a rather stocky women with
pendulous breasts. She has a moderated sized, nontender, irregular lump in the lower outer
quadrant of her left breast. The overlying skin appears thickened. No nipple discharge is
present, and she has no axillary lymphadenopathy. The right breast and axilla are normal.
Mammography reveals increased density in the affected region. Ultrasound guided fine
needle aspiration (FNAB) was performed. The pathology report was consistent with a
diagnoses of fat necrosis.
Which of the following is the most appropriate next step in the management of this patient?

a. Excise the mass.


b. Repeat fine needle aspiration biopsy after one month.
c. Repeat mammography in a month.
d. Reassure the patient and follow up in a few weeks. Correct
03.10.2011
By: Alhag Abu Anzeh Muhammad

Part A:

1. A 75 year old man with a history of myocardial infarction2 year ago, peripheral
vascular disease with symptoms of claudication after half block, hypertension, and
diabetes presents with a large ventral hernia, he wishes to have the hernia repaired.
Which of the following is the most appropriate next step in his preoperative workup?

a. A normal ECG precludes the need for further cardiac testing.


b. He should undergo an exercise stress test.
c. He should undergo coronary artery bypass prior to operative repair of his ventral hernia.
d. He should undergo a presntative thallium stress test and echocardiography.
e.His history of a myocardial infraction within 3 years is prohibitive for elective surgery.No further testing isnecessary.

2. A victim of blunt abdominal trauma undergoes a partial hepatectomy. During surgery he receives twelve units of
packed red blood cells. In the recovery room, he is noted to be bleeding from intravenous puncture sites and the
surgical incision. Which of the following statements regarding the coagulopathy is most likely true?

a. The patient has an unknown primary bleeding disorder.


b. The coagulopathy is secondary to the partial hepatectomy
c. The coagulopathy is secondary to delusional thromnocytopenia and deficiency of clotting
factors from the massive blood transfusion.
d. The treatment is oral vitamin K
e. The treatment is intravenous vitamin K.

3. A 65 year old man has an enterocutaneous fistula originating in the jejunum secondary to inflammatory bowel
disease. Which of the following would be most appropriate fluid for replacement of his enteric losses?

a. D5W
b. 3% normal saline
c. Ringer lactate solution
d. 0.9% sodium chloride
e. 6% sodium bicarbonate solution

4. A 23 year old woman undergoes total thyroidectomy for carcinoma of the thyroid gland. On the second
postoperative day, she begins to complain of a tingling sensation in her hands. She appears quite anxious and later
complains of muscle cramps. Which of the following is the most appropriate initial management strategy?

a. 10 ml of 10% Magnesium sulfate IV


b. Oral vitamin D
c. 100 micro gram oral synthroid
d. Continuous infusion of calcium gluconate
e. Oral calcium gluconate.

5. A 24 year old firefighter sustains 30% total body surface area burns to his torso, face, and extremities. His wounds
are treated topically with silver nitrate. Which of the following complications is associated with use of this agent?

a. Hypernatremia
b. Metabolic acidosis
c. Hyperchloremia
d. Neutropenia
e. Hyponatremia
6. A 10 year old boy was the backseat belted passenger in a high speed motor vehicle collision. On presentation to
the ER he is awake, alert, and hemodynamically stable. He is complaining of abdominal pain and has an ecchymosis
on his anterior abdominal wall where the seatbelt was located. Which of the following statements is true
regarding need for additional workup?

a.The boy can be safely discharged home without any other workup, since his abdominal pain is
probably secondary to his abdominal wall ecchymosis.
b. The boy can be safely discharged home if his amylase level is normal
c. The boy can be safely discharged home if abdomen plain films are negative for the presence of free air.
d. The boy can be safely discharged home if an abdominal CT is negative
e. The boy should be observed regardless of negative test result.

7. A patient is brought to the ER after a motor vehicle accident. He is unconscious and


has a deep scalp laceration and one dilated pupil. His heart rate is 120 beats per minute blood pressure is 80/40
mm/Hg, and respiratory rate 35 per min. Despite rapid administration of 2 L normal saline, and patients vital signs do
not change significantly. Which of the following is the most appropriate next step in the workup of his hypotension?

a. Neurosurgical consultation for emergent ventriculostomy to manage his intracranial pressure


b. Neurosurgical consulation for emergent craniotoymy for suspected subdural hematoma
c. Emergent burr hole draining at the bedside for suspected epidural hematoma
d. Administration of mannitol and hyperventilation to treat his elevated intracranial pressure
e. Abdominal ultrasound (focused assessment with sonography in trauma, FAST)

8. A 25 year old woman is diagnosed with bilateral breast CA. sh undergoes genetic testing to assess whether she
has a mutation in either BARCA1 or BARCA2. Which of the following statements is true regarding BARCA mutation ?

a. Her risk as a woman under the age of 40, for having a mutation in BARCA1 or 2 is 40%.
b. BARCA1 tumors are more likely to be estrogen recetor positive.
c. A mutation is BARCA 2 is associated with an increased risk of colon CA.
d. A mutation in BARCA 1 is associated with an increased risk for pancreatic, bile duct and galdbladder CA.
e.Both BARCA 1 and BARCA 2are associated with an increased risk of ovarian CA by age 70.

9. A 35 year old woman with a history of previous right thyroidectomy for a benign thyroid nodule now undergoes
completion thyroidectomy for a suspicious thyroid mass. Several hours postoperatively she develops progressive
swelling under the incision, stridor and difficulty breathing. Orotracheal intubation is successful.
Which of the following is the most appropriate next step?

a. Fiberoptic laryngosopy to rule out bilateral vocal cord paralysis


b. Administration of IV calcium
c. Administration of broad spectrum antibiotics and debridement of the wound
d. Wound exploration
e. Administration of high dose steroids and antihistamines.

10. A 15 year old otherwise healthy female high school student begins to notice galactorrhea. A pregnancy
test is negative. Which of the following is a frequently associated physical finding?

a. Gonadal atrophy
b. Bitemproal hemianopsia
c. Exophthalmos and lid lag
d. Episodic hypertension
e. Buffalo hump
11. A 72 years old man status post CABG 5 years ago present with hematochezia,
abdominal pain and fever. colonoscopy reveals dusky appearing mucosa at splenic
flexure without active bleeding. which is the most appropriate manegment of this patient?

a. angiography with administration of intra-arterial papaverine


b. Emergent laprotomy
c. Aortomesenteric bypass
d. Exploratory laprotomy with Thrombectomy of the inferior mesenteric artery
e. Expectant management

12. A 46 years old women ask about the need for surgery, since she was recently
diagnosed with Crhon's Disease. Indication for operation in Chron's include all except:

a. Intestinal obstruction
b. Enterovesical fistula
c. Prolonged use of steroids
d. Enterovaginal fistula
e. Free perforation

13. A28 years old women is 15 week pregnant, has new onset of nausea, vomiting and right sided abdominal pain.
She has been free of nausea since early in her 1st trimester. the pain has become worse over the past 6 hours. Which
of the following statments about appendicitis during pregnancy is correct?

a.Appendicitis is the most prevalent extra uterine indication for celiotomy during pregnancy.
b.Appendicitis accure more commonly in pregnant women then in nonpregnant women.
c.Suspected appendicitis in pregnant women should be manged with period of observation due to the risk
of laprotomy to the fetus.
d. Non complicated appendicitis result in 20% fetal mortality and premature labor rate.
e.The sevirity of appendicitis correlates with increase gastational age of the fetus.

14. A 41 years old men complains on regurgitation of saliva and undigested food, an esophaogram reveals "birds
beak" deformity. Which of the following statments is true about the condition:

a. Chest pain is common in advanced stage of the disease.


b. More patients are improved by forcefull dialatation then by surgical intervention.
c. Manometry can be expected to show increase resting pressure of the LES.
d. Initial surgical treatment consist primarily of resection of the distal esophagus with reanastomosis
to the stoma above the diaphragm.
e. Patients with this disease are at no increased risk for the development of carcinoma.

15. A 72 years old men undergo an aortobifemoral graft for symptomatic aortoiliac
occlusive disease. The inferior mesenteric artery (IMA) is ligated at its aortic
attachment. 24 hours after surgery the patient has abdominal distention, fever and
bloody diarrhea. Which of the following is the most apropriate diagnostic study:

a. Aortogram
b. MRI
c. CT
d. Sigmoidoscopy
e. Barium enema
16. A 38 years old women has 1.5cm mass in left Brest, clinical exam reveals no suspicious adenopathy.
Mammogram guided biopsy reveals infiltrating ductal carcinoma. The patient prefer lumpectomy followed
by radiation. Which of the following statements about sentinel lymph node is true?

a. it is only indicated in clinically + axilla


b. it is associated with survival advantage over routine axillary dissection .
c. neoadjuvant chemotherapy does not affect reliability of SLNB.
d. it is best preformed with blue dye alone.
e. it is not indicated because of her premenopausal state.

17. Postmenopausal women choose mastectomy and sentinel lymph node biopsy
(SLNB) for a new diagnosed Brest carcinoma. Final pathology reveals 1cm infiltrating ductal carcinoma with
negative SLN. It is estrogen receptor/progesterone receptor (ER/PR) positive and HER2/neu negative
The most appropriate adjuvant therapy would be:

a. aromatase inhibitor
b. tamoxifen
c. postoperative Brest radiation therapy
d. prophylactic oophorectomy
e. cytotoxic chemotherapy

18. 38 years old women with right colon cancer, her history meet Amsterdam criteria, also has a 10 years old son.
Which of the following statements is true?

a. son should begin colonoscopy at age 10-12 years.


b. son should have colonoscopy every 5 years.
c. son have 25% risk of receiving the mutation.
d. tumor should be tested for microsatelite instability.
e. patient should have right hemicolectomy

19. 55 years old with 1st episode of diverticulitis, hemodynamicaly stable and improve within 48h receiving
antibiotic and nothing by mouth. CT show stranding of sigmoid mesentery, but no free air evidence of an abscess.
The next step should be:

a. resection and anastomosis


b. resection and colostomy
c. discharge + elective colectomy
d. discharge + oral antibiotic
e. colonoscopic intraluminal stench

20. 40 years old women, history of chron's disease and many prior small bowel
resections, present with intractable non bloody diarrhea, weight loss and several recent
admissions for vol depletion and electrolyte disturbance. The most likely diagnosis:

a. recurrence of active Chron's


b. short bowel syndrome
c. chronic renal insufficiency
d. villus adenoma of rectum
e. inf. Gastroenteritis
21. A 48 years old man with a history of alcohol use presents evaluation reveals a
linear disruption of the gastric mucosa high on the lasser curvature at the gastroesophageal junction.Endoscopic
attempts to control the bleeding fail. The patient has received 6 units of packed RBCs and his heart rate is115/min
with a systolic blood pressure of 85mmHg. The most appropriate management would be:

a. Embolization of the left gastric artery


b. placement of a Sengstaken-Blakemore tube with inflation of gastric balloon
c. Anterior gastrotomy and eversewing of the bleeding site.
d. Antrectomy ,vagotomy,and biopsy of the bleeding site.
e. Total gastrectomy

22. A 51 years old man with a history of peptic ulcer disease presents with abdominal pain and free air.during
operation a duodenal perforation is identified with significant associated peritonitis .A modified Graham patch is
performed.Acid reduction therapy is initiatedand serology for Helicobacter pylori is obtained.seven days after
operation an operativedrain begins draining copious amounts of bile.Management of this duodenal fistula includes all
of the following except:

a. cholestyramine
b. total parenteral nutrition
c. somatostatin enteral nutrition
d. Nasojejunal enteral nutrition
e. Bile re-feeding

23. A 75 years old woman presents with the acute onest of abdominal pain . on
physical examination ,she has a palpable abdominal mass that is painful with
contracture of the abdominal muscles. She has no peritoneal irritation .she is receiving anticoagulation for recent
onest atrial fibrillation . the underlying vessel causing this problem is most likely the:

a. Inferior epigastric artery


b. Superior mesenteric arery
c. Splenic artery
d. Superior mesentric vein
e. Left gastroepiploic artery

24. Six months after laparoscopic splenectomy for immune thrombocytopenic purpura ,a 38 years old woman with
lupus remains thrombocytopenic (platlet count42,000 cell/mm^3). The next step should be:

a. platlet transfusions
b. Evaluation for accessory spleen
c. Plasmapheresis
d. Bone marrow biopsy
e. Re-laparoscopy

25. A 20 years old man has a 2 cm right adrenal mass found during a work up for low back pain.the left adrenal is
of normal size .the next step should be:

a. Repeat imaging study in 6 months


b. 24 -hour urinecolection for catecholamines
c. CT - guided fine -needle aspiration
d. high -dose dexamethasone test
e. Elective right laparoscopic adrenalectomy
26. A 56 years old postmenopausal woman has serium calcium of 10.9 mg/dl (normal 7.5 to 9.5) and a serum PTH of
85 pg/ml(normal 20 to 60 pg/ml) . She has no family history and is asymptomatic .she is an avid jogger . Bone
densitometry (DEXA scan) shows minimal osteoporosis. Which of the following is true?

a. Calcium supplementation will decrease her risk of stress facture.


b. the NIH consensus conference recommends observation
c. 24 -hour urine collection for calcium will be diagnostic
d. she is a candidate for parathyroidectomy
e. treatment with biphosphonates is warranted

27. A 44 years old woman has papillary thyroid carcinoma of the right lobe confirmed by fine-needle aspiration.
Ultrasound study stages the tumor as T1 (1.5 CM). Optimal surgical management would be:

a. Right thyroidlobectomy , selective mode dissection of the right side


b. Right thyroidlobectomy, level VI (central compartment) nod dissection
c. Total thyroidectomy, selective dissection on the right side
d. Total thyroidectomy, level VI dissection and selective dissection of the right side
e. Total thyroidectomy

28. Which of the following is the most useful in identifying patients at high risk for familial medullary thyroid cancer?

a. RET oncogene
b. calcitonin levels
c. calcium levels
d. K-ras oncogene
e. Urine metanephrines.

29. Abnormalities in all of the following variables have been used to suggest the need for damage control
(or abbreviated) laparotomy in trauma patients except :

a. Temperature
b. PH
c. Base deficit
d. Prothrombin time
e. Platelet count

30. A 36 years old man has a blood pressure of 70/40 mm/Hg after a motor vehicle
crash .resuscitation is initiated. Focused assessment with sonography for trauma
(FAST) is positive for fluid in the abdomen. on examination of the pelvis ,fracture -
related crepitus and an enlarging perineal hematoma are appreciated .the patients
blood pressure does not improve with resusciatation .the next step in management should be:

a. CT of the abdomen and pelvis


b. Diagnostic peritoneal lavage
c. Placement of an external pelvic fixator
d. Exploratory laparotomy
e. pelvic angioembolization
31. 38 y.old woman should go laparoscopic cholicystectomy, no family history of thromboembolic events and
no personal medical history of thromboembolic events. Which is indicated for perioperative DVT prophylaxis?

a. no prophylaxis is needed
b. sequential compression
c.subcutaneus unfrictioneted heparin
d. subcutaneous LMWH
e. sequential compression device and subcutaneous unfrictionated heparin

32. Preoperative biochemical evaluation of patient for parathyroidectomy is dune for PHPT (primary
hyperparathyroidism) show Ca+ 12 and intact PTH (200pg/ml). 1cm hypercellular parathyroide adenoma is
resected . Postoperatively the patient complaint on perioral tingling, numbness and carpopedal spasm.
The ECG will show:

a. atrial fibrillation
b. prolong QT
c. pecked T wave
d. torsa des pointes
e. U wave

33.Common complication of RYGB:

a. vitamin D deficiency
b. primary hyperparathyroidism
c. anemia due to lack of transferin
d. symptomatic thyamin deficiency
e.hypocuagulability

34. 60y.o. man completed chemotherapy for hepatic metastasis related to colorectal
cancer WBC 1200, fever 39c, diffused abdominal pain. CT of the abdomen and pelvic show inflammation and
thickening of the ilium, cecum and right colon. Most likely diagnosis is:

a. clostridium difficilis colitis


b. neutropenic enterocolitis
c. toxic megacolon
d. ulcerative colitis
e. colon pseudoobstraction

35. 18 year old men went inguinal hernia reaper with spinal anesthesia. What is correct regarding preoperative:

a. oral ceforal on the morning of the surgery


b.oral ceforal in the evening be for the surgery
c. no preoperative is needed
d. 40mg clexan evening be for the surgery

36. 88y.old man has sever inguinal pain. 2 weeks ago he had acute M.I. and since than using aspirin, on examination
he has pain, no fever, heart rate 115, abdomen is not distended, no peritonitis. Incancerated no reducible right
inguinal hernia is found. The most appropriate next step will be:

a. urgent cardiac catheterization


b. exercise thalium
c. surgical hernia repaire
d. hernia reduction attempt under deep sedation
e. CT of the abdomen and pelvic
37. Small bowl obstruction from bezoar is more common after:

a. colon resection due to cancer


b. bariatric surgery
c. vagotomy
d. multiple abdominal surgery

38. All are compensatory mechanism in hypovolemic shock except:

a. increased peripherial resistance secondary to increase catecholamine level


b. increase DPG 2,3
c. right shift of hemoglobin dissociation curve to acidosis
d. decrease cardiac contractility

39. Recommended to colorectal liver metastasis that localized to one liver lobe is:

a. lobectomy of the lobe which containing the metastasis


b. lobectomy of the lobe which containing the metastasis and systemic chemotherapy
c. systemic chemotherapy with cytotoxic and biologic drug
d. palliative treatment only

40. A 55y.old woman no history evaluation for melena has 2cm GIST (gastrointestinal stromal tumor) in the anterior
wall of the stomach. The diagnosis confirmed by immunohistochemical, no evidence for adjacent organ invasion or
metastasis. The recommended treatment is:

a. subtotal gastrectomy and regional lymph node dissection


b. give tyrosin kinase inhibitor (imatinib) till complete response and offer surgery if medical treatment is fail
c. open or laparoscopic margin free tumor excision and fallow up
d. only imatinib treatment

41. In the ER, 60 year-old male with no previous medical history is found to have an obstructing tumor in the
sigmoid colon on combined CT and gastrografin enema , no distant metasteses are identified and the cecum's
diameter is 15 cm. What is the recommended treatment?

a. mecchanic bowel preparation followed by surgery that includes sigmoidectomy, regional lymph node dissection,
and anastomosis.
b. b.immediate surgery that includes sigmoidectomy, regional lymph node dissection, and anastomosis
with no further work-up.
c. immediate colonscopy with biopsy and after the diagnosis is histologically confirmed, sigmidectomy, regional
lymph node dissection, and creation of proximal stoma(Hartman resection).
d.immediate surgery that includes sigmoidectomy,regional lymph node dissection, and creation of proximal stoma.

42. A cirrotic patient that is a liver transplant candidate is admitted for esophageal bleeding varices .
Treatment with somatostatin and endoscopy including sclerotherapy and failed banding don't stop the bleeding.
What is the next step?

a. TIPS
b. repeat endoscopy
c. distal splenorenal shunt
d. surgical procedure (porto-azygos disconnection).
43. A 45 year-old man with a history of chronic peptic ulcer disease undergoes a truncal vagotomy and antrectomy
with a Billroth ll reconstruction for gastric outlet obstruction. 6 weeks after surgery, he returns , complaining of
postprandial weakness, sweating ,lightheadedness, crampy abdominal pain and diarrhea.
Which of the following would be the best initial management strategy?

a. treatment with a long acting somatostatin analog.


b. dietery advice and counseling that symptoms will probably abate within 3 months of surgery.
c. dietery advice and counseling that symptoms will probably not abate but aren't dangerous.
d. work-up for neuroendocrine tumor(carcinoid).
e. preparation for revision to Roux-en-Y gastrojejunostomy.

44. A 58 year-old man with a mass in the left side of his neck. The patient noticed the mass several weeks ago, and
was given antibiotic by his primary care physician. The mass appeared to subside somewhat, but is still present. U.S.
shows a 2 cm solid mass. On examination, the mass is 2 cm, firm and fixed to the underlying tissue.
The best step now would be:

a. color flow Doppler


b. CT of the head and the neck.
c. MRI of the head and the neck.
d. Fine-nedeel aspiration of the mass.
e. surgical biopsy.

45. 14 year-old girl with a six month history of sudden , rapid ,asymmetric enlargement of the right breast. On
physical examination, there is a large palpable mass occupying the breast with marked nipple-
areolar stretching, prominent dilated veins, and skin ulceration. Mammography and US show a
dense, well-circumscribed 7 cm homogenous mass occupying the left breast. The most likely diagnosis is ?

a. invasive ductal carcinoma


b. hemangioma
c. giant fibroadenoma
d. juvenile breast hypertrophy
e. phyllodes tumor

46. a 51 year-old blond, light ,complexioned man had undergone laparotomy for progressive small bowel
obstruction. A desmoplastic reaction is causing this small bowel obstruction. Which of the following best
describes the diagnosis?

a. lymphoma
b. carcinoid
c. GIST (leiomyosarcoma)
d. melanoma
e. primary small bowel adenocarcinoma

47. Which of the following statements about laparoscopic Roux-en-Y gastric bypass
(RYGB) compared with laparoscopic adjustable gastric banding (AGB) is not true?

a. both decrease long term mortality compared with controls.


b. AGB has lower perioperative.
c. RYGB is associated with higher risk of iron deficiency anemia.
d. AGB is associated with fewer internal hernias.
e. AGB is associated with more sustainable long-term weight loss.
48. Herediatry spherocytosis:

a. necessitates splenectomy for all patients with the diagnosis.


b. results in a loss of spherocytes on the peripheral smear after splenectomy .
c. is associated with increased red cell fragility.
d. results in splenic infarction.
e. is caused by a defected in the bilayer of the cell membrane.

49. A 64 year-old man has a solitary thyroid nodule discovered during a screening
carotid duplex. Fine-nedeel aspiration of the nodule shows a predominance of Hurthle cells with scant amounts of
colloid and no evidence of malignancy. the next step should be:

a. total thyroidectomy
b. repeat biopsy in 6 months.
c. thyroid suppression therapy.
d. hemithyroidectomy with isthmusectomy.
e. total thyroidectomy with central node dissection.

50. A healthy 38 woman is rescued from a house fire after 20 min.in emergency department, she has an anterior
chest burn and is weak, confused and lethargic. the most likely diagnosis is:

a. CO and pelvis.
b. inhalation lung injury.
c. hypercabic respiratory failure.
d. pneumothorax.
e. upper airway obstruction.

51. 31 year old G2P1, 39 weeks complains on abdominal pain, her uterine contraction recorder every 3-4 min, her
cervix changed only 1 to 2 cm in diameter in over 3 hours. Which of the following management of the patient?

a. C- section
b. IV oxytocin
c. Observer
d. Fetal scalp PH monitoring
e. Intra nasal gonadotropine

52. Which of the following placental implantation most like predisposed to inverted uterine in 3rd stage of labor?

a.Fundal
b. Anterior
c. Posterior
d. Lateral
e. Lumbar stage of labor

53. 23 year old G2P1 delivering 42 weeks gestational, is noted to be in second stage when fetus head is retracted
back to the introitus. The shoulder is not deliver with the push of the mother. Which of the following maneuvered
is best?

a. Internal podalic version


b. Suprabubic pressure
c. Fundal pressure
d. Intentional fracture of the humerus
e. Delivery of the anterior scalp
54. 33 year old in 29 weeks of gestation hers blood pressure is 150/90, proteinurea +2,
platelets and liver enzyme are in the normal range. Which BEST management of patient?

a. Induction of liver
b. C- section
c. Antihypertensive therapy
d. Expected management
e. Magnesium sulfate

55. 19 year 28 weeks of gestation complain of intermediate abdominal pain. On cardiotocography the uterine
contraction 3-5 min, cervix 3cm dilation, with 90% effacement. Fetal vertex presentation at spinal 1 station.
Which most likely to be contraindication for tocolytic?

a.Suspected placental abruption


b. Group B streptococci
c. Recent elective laparotomy
d. Uterine sub serous fibroid
e. The current status

56. Which of following most potent natural estrogen?

a. Esterone
b. Estroadiol
c. Estriol
d. Ethinyl estradiol
e. Phytoestrogens

57. Which of the following change in nasal bone have for Down syndrome in US?

a. Absent
b. Increase in size
c. Decrease in size
d. Increase in opacitaiton
e. Decrease in opacitation

58. You are monitoring patient contraction puttern, during 20 min she has 6 equally
contraction. Base line uteine tone is 10 mmHg and peak intensity of each contraction
reaches 50 mmHg. Which following number of Montevideo units define this activity?

a. 120
b. 150
c. 240
d. 300
e. 360

59. What is the most common side effect of intrapartum epidural Anastasia?

a. Increase in blood pressure


b. Decreases in blood pressure
c. CNS stimulation
d. Ineffective blacken
e. CSF leak
60. Forceps application when fetal head (left occipital antero-posterior) has reached the pelvic floor is at partum
classier at what type delivery?

a. Outlet forceps
b. Low forceps
c. Mid forceps
d. High forceps
e. Non of above

61. The best pregnancy management of a 40 years old nullipara, prior myomectomy during which the endometrial
cavity was anterior:

a. Labor allowed
b. Labor allow, low forceps/vacuum to shorten the 2sd stage
c. Caesarian section near prior term of labor
d. Oxitoxin at 38 weeks
e. Caesarian section at 40-41 week

62. Most common pathogen associated with intra uterine growth restriction:

a. Rubella
b. Varicella zoster
c. Influenza
d. Cytomegalovirus
e. Parvovirus

63. A couple, diagnosed as primary infertility 2 year old, in sperm count – sever
OligoTeratoAsthenoSpermia. Most appropriate next diagnostic test:

a. Repeat sperm count in 1-3 month


b. Male blood karyotype
c. Male full hormonal profile
d. Prolactin + thyroid function
d. Y chromosome deficiency

64. A 30 years old male, diagnosed as non obstructive azospermia, small testicle, lack of facial and pubic hair and
gynecomastia. Most probable hormonal status :

a. High LH, high FSH, low testosterone


b. Low LH, low FSH, low testosterone
c . High LH, low FSH, high testosterone
d. High LH, high FSH, high testosterone
e. High LH, low FSH, high testosterone

65. True regarding the difference between water and fat soluble contrast media for performing
HisteroSlapingoGraphy:

a. Chance of pregnancy increase with water soluble compare to fat soluble


b. Chance of pregnancy increase with fat soluble compare to water soluble
c. Uterine structure is better outlined with water soluble
d. Risk of granuloma is decrease with fat soluble
e. Abdominal pain slightly increase with water soluble
66. A couple is diagnosed with unexplained infertility, what is not correct:

a. 80% chance of pregnancy in 3 years without treatment.


b. IVF is the primary treatment of choice.
c. Ovulation induction with gonadotropine is a treatment option
d. Intra-uterine insemination after washing is a treatment option
e. IVF is an option if other treatments have failed.

67. Which of the follow cause primary amenorrhea?

a. Premature ovarian failure


b. Pregnancy
c. Kallman syndrome
d. Sheehan syndrome
e. Asherman syndrome

68. Which can result amenorrhea?

a. Arcuate nucleus injury


b. Olfactory nerve injury
c. Post Hypophisis injury
d. Medio-lateral Hypophisis injury
e. Prolactine deficiency

69. Which of the follow characterizes syndrome known as “female athletic Triad”:

a. Period bleeding in response to progesterone treatment


b. Increase in over 10% in ideal muscle mass
c. Hypophisis atrophy
d. Osteopenia
e. Hirsutism

70. In which of following will differentiation to female Phenotype occur?

a. Deficiency at chromosome Y long arm


b. 45 X0 karyotype
c. AMH gene mutation
d. 47 XXY
e. 47 XYY

71. A 25 y/o female is been evaluated for infertility with regular periods. Which of the following confirms the
existence of ovulation?

a. progesterone level >5ng/ml


b. proliferative histology in endometrial biopsy in the luteal phase viscous
c. cervical mucus
d. increase in corporal temperature after the menstrual period
e. increase in LH and decrease in progesterone in the luteal phase
72. Which one of this ovaric tumors doesn’t produce hirsutism?

a. sertoli-leydig tumor
b. don’t remember
c. quistic teratoma
d. gonadoblastoma
e. luteoma

73. Women with 50 y/o menopausic with ovaric insufficiency in treatment with ovum
donation, which of the following precautions for the implantation of the fetus is correct?

a. estrogen + progesterone en el comienzo de la menstruacion


b. she doesn’t need preparation of the uterin mucus
c. preparation of estrogen and ADD progesterone 2 weeks later
d. stop the hormonal support after implantation
e. the number of implanted fetus is associated with the age of the donor

74. A 38 y/o patient who since the last 2 years has been bleeding for more that 9 days in every period. She has
periods every 32 days and no bleeding between periods. What is the best nest step to do?

a. coagulation test
b. lhand test
c. endometrial biopsy
d. pelvis ecografy
e. TSH test

75. Which of the follows doesn’t produce hirsutism?

a. 5 alfa reductase deficit


b. 21 hidroxilase
c. ovaric aromatase deficit
d. 11 hidroxilase deficit
e. don’t remember

76. A 49 y/o with myomatosis and vaginal bleeding between the periods and in the periods, with periods every
5-6 weeks that lasts from 6 to 10 days. Which the next management to do?

a. GNRH agonists for 3 months


b. GNRH for 6 months + hormones
c. histerectomia
d. endometrial biopsy
e. ablation

77. A 23 y/o woman with abdominal pain that intensifies during periods. She on birth control pills, because of the
pain she goes to laparoscopy where they find where they see lesions of endometriosis and they treat it with laser.
What is the next step of the treatment in this patient?

a. continue with birth control pills


b. GNRH agonists for 6 months
c. aromatase + birth control pills
d. total hysterectomy + bilateral salpingooforectomy
e. danazol
78. A 24 y/o patient with no children presents with vaginal bleeding. She has a pregnancy by BHCG of 3500 mUI/ml
with normal vital signs, with an ecogafy that doesn’t show a gestational sac but shows a mass of 3.2 cm in the rifht
adnexal. Which of the following is the best treatment for this patient?

a. observation
b. metotrexate IM
c. laparoscopic salpinguectomy
d. laparotomy salpinguectomy
e. laparoscopic salpinguectomy

79. In the pelvic inflammatory disease you need to diagnose and treat early to prevent which of the following?

a. pelvic paint SD
b. infertility
c. ectopic pregnancy
d. tuboovaric abscess
e. adhesion

80. A 35 y/o woman nulliparous who has hemoptysis. She has a history of abortion 3 months ago and now has a
BHCG of 35000. On the thorax X ray there is a mass in the middle lobe of the right lung.
Which is the best initial treatment in this patient?

a. lavage and curettage


b. histerectomy
c. methotrexate and leucovirine
d. actinomicine
e. metotrexate + actinomicine + etoposide

81. Which of the following is correct regarding intrauterine device?

a. the risk of PID increases with the use duration.


b. the risk of PID increases during the first month after insertion.
c. the absalute risk for infection with proteus vaginalis is common.
d. ''mirena'' device contains medroxy-progesterone.
e. risk for ectopic pregnancy is increased.

82. Which of the following is correct regarding progesterone-only oral contraceptives?

a. cervical mucus viscosity is decreased


b. it is more effective than other oral contraciptives.
c. there are LH but no FSH surges during treatment.
d. there are FSH but no LH surges during treatment.
e. mood and libido are improved.

83. Which of the following stataments concerning usage of combined oral contraceptives and cancer is true?

a. usage of combined oral contraceptives increases the prevalence of ovarian carcinoma


b. usage of combined oral contraceptives protects from endometrialn carcinoma
c. usage of combined oral contraceptives protects from breast cancer
d. usage of combined oral contraceptives protects from cervical carcinoma
e. usage of combined oral contraceptives increases the prevalence of gastric carcinoma
84. Which of the following tissues is not routinely resected during radical hysterectomy for sqamous cell carcinoma of
the uterine cervix?

a. parametrium.
b. socro-uterine ligamints.
c. ovaries and fallopian tubes.
d. pelvic lymph nodes.
e. vaginal dome.

85. Which of the following is unique in ovarian endodermal sinus tumor?

a. estrogen secretion.
b. androgen secretion.
c. lymphocytic infiltration.
d. neuroectodermal tissue.
e. schiller-duval body.

86. A laparoscopy is done to an 18 years old female with peritonitis, it shows rupture and bleeding of a 10 cm cyst
that encompasses the entire left ovary. Dysgerminoma is diagnosed on frozen section biopsy. Correct treatment:

a. Oophorectomy of the involved ovary.


b. Full staging including hysterectomy, oophorectomy lymph node biopsy.
c. Chemotherapy+carboplatine+taxol.
d. Brachytherapy of left pelvis.
e.Resection of involved ovary and resection of the other ovary with cryo- preservation to maintain fertility.

87. A 54 year old male complains of dyspnea for 8 months. He has minimal prior medical care, non prior
problems and denise taking any medications. He smokes 1 package of cirgarrets for the last 30 years.
On physical examination stable prolonged expiratory phase an diminished breath sounds bilaterally. The
most likely cause of this chronic dyspnea:

a. Pulmonary embolism.
b. COPD.
c. DM
d. MI
e. Pneumonia.

88. An 40 y old male presents with painful, swollen right knee and low temperature. The most useful test is:

a. Complete blood count.


b. Uric acid level in blood.
c. ESR.
d. Rheumatic factor.
e. Joint fluid analysis.

89. A previously healthy 26 year old male complained of abdominal crsmps for the past 2 days.Loose stool in a
24h stool specimen reveals presence of blood and WBC. The most likely diagnosis;

a. Stasphylococus food poisoning.


b. Rotavirus.
c. Crohn’s disease.
d. Shigellosis.
e. Irratibal bowel syndrome.
90. A 76 year old male complains of progressive worse fatigue over the past 6 months, difficult in walking 2 blocks
and poor appetite and loose of 15 kg. Physical examination shows pale conjunctiva, clear lungs bilaterally, HR: regular
96bpm, no murmurs. CNC hemoglobin 7g% and decreased MCV. Increase in total iron binding capacity and low
ferritin. Anemia is most likely due to secondary:

a. Tthalassemia.
b. Anemia of chronic disease.
c. Folate deficiency.
d. Vitamin B12 deficiency.
e. Iron deficiency anemia.

91. 60 y\o woman compalins of fatigue for 6 month. Condition worsens progresivly.
now she begins to notice weakness and unsterdiness in her gate. On examination: pale
conjunctiva, heart\ lungs normal. On neurological examination:decreased vibratory
sense. Hgb: 8 MCVis increased. The most common cause of this condition is:

a. a.iron deficiancy
b. b.folate deficiancy
c. c.pernicious anemia
d. d.anemia of chronic disease thalassemia

92. A Patient is diagnosed with Grave's disease. What would be the findings?

a. low TSH and high free T4


b. Hot nodule"on a thyroid scan
c. Antibodies for Thyroid
d. High TSH & Free T4
e. Fine needle aspiration biopsy is necessary.

93. 40 year old female patient is presented with left chest pain. the pain is located mid clavicular on the 4th
intercostal space. There is a trigger point and the pain is increased with inspiration. What is the most likely diagnosis?

c. chostochondritis

94. A 68 year old woman comes to discuss result of her dual-energy x-ray
absorptiometry (DEXA) scan. Her T-score is 2.7 in spine femur. you explain that she has osteoporosis and requires
treatment management of osteoporosis includes:

a. Daily intake of 500 mg calcium.


b. avoidance of weight-bearing exercise.
c. Take bisphosphonates on a full stomach.
d. stop smoking.
e. monthly use of newly formulates bisphosphonates.

95. 60 y\o woman presents with chronic dyspnea and a long history of smoking. you diagnose COPD and treat to
relieve the symptoms. In addition to this therapy, other recomandetion must include:

a. spiral CT of chest
b.exercise avoidance
c.stop smoking
d.Haemophilu influenza b vaccination
e.refer to bronchoscopy.
96. 45 y\o healthy male come for physical examination. His last examination was 10
years ago and was normal. No history, no medication, no smoking, drinks 2-3 alcoholic drinks weekly. Family history is
positive for hypertension from the maternal side. The patient has no physical complains. His eamination is normal,
but, the BP is 148\98. You recomend recheck of BP in 1 month and BP is 150\100. Further evaluation should include:

a.serum and urine chatecholamine


b.cardiac stress test
c.ECG
d.no test
e.renal scan

97. 20 y\o woman with concerns that her thyroid is overactive. because her mother has similar condition and she has
not been feeling well. in evaluating this patient, what would strongly suggest hyperthyroidism?

a. palpitations
b.increased weight
c. constipation
d.depression
e.fatigue

98. 26 y\o man arrives to the ER with complaints of acute onset of the "worse headache of my life". Appropriate
diagnostic evaluation. Treatment would include:

a.sumatriptan 6mg subcutanous


b.head CT and if negative LP
c. head CT
d. ibuprofen 600mg per os every 6 hours
e.ESR test and while awaiting results corticosteroid therapy.

99. 68 y\o man presents for a routinly physical examination. in addition to obvious history and examination, which
one of the following would be routinly reccomended?

a.chest X-Ray
b. HBV vaccination
c.DEXA scan
d.pneumoccocal vaccination
e.ECG.

100. 42 y\o woman returnes from a buisness trip and notes the sudden onset of
dyspnea with pleuritic right sided chest pain. Medical history is unremarkable, she is currently use oral
contraceptives. Vital signs: BP120\70, HR100, RR24 and regular. temp 37C. Heart and lungs are unremarkable, no
chest wall tenderness, no cyanosis\clubbing of the extrematies, but, pitting edema in her right leg.
testing should be done promptly to exclude the following:

a.fibromyalgia
b.costochondritis
c. pulmonary emboli
d.lympedema
e.vericose veins
101. A 50 year-old man complain of recurrent chest pain that radiates down his left arm and has been occurring
over the past 9 months.The pain is described as a retrosternal pressure. It is brought on by walking or other
strenuous exercise and is relieved by 2 minutes at rest.
His vital signs are stable and the physical examination is unremarkable. The most likely diagnosis:

a. myocardial infarction
b. stable angina
c. unstable angina
d. pericarditis
e. pleurisy

102. Of the following conditions, wich is related to the development of osteoporosis in men?

a. prolactinoma
b. hypogonadism
c. prostate cancer
d)renal stones
e. inguinal hernia

103. A 65 year old with a history of chronic atrial fibrillation in being monitored while warfarin therapy. The nurse
calls to inform you the patient”s INR is measured at 7. He has no active signs of bleeding.but manangment at this
time includes:

a. stop warfarin,observe and repeat INR in 3 days


b. stop warfarin and observe,repeat INR in 24 hour
c. stop warfarin,give vitamin K and repeat INR in 24 hour
d. stop warfarin,give vitamin K and fresh frozen plasma with daily INR

104.When is a comprehensive evaluation necessary when a patient is affected by deep vein thrombosis (DVT):

a. 45 year-old male with an idiopatic DVT


b. 65 year-old with a recent transatlantic flight and DVT of the left thigh
c. 55 year-old who develops a calf DVT after a 4 hour car ride
d. 75 year-old with a history of non-small cell lung cancer with left leg DVT
e. 72 year-old with a right thight DVT and no history of travel

105. Anemia that is scan in patient with chronic renal disease is usually caused by insufficient:

a. Iron stores
b. vitamin B12
c. renin levels
d. erythropoietin levels
e. folat st

106. Iron deficiency anemia is associated with:

a. hyperchromic , macrocytic features.


b. elevated serum iron levels.
c. increased total iron-binding capacity (tibc).
d. increased ferritin levels.
e. normal bone marrow biopsy results.
107. Which of the following ECG findings is associated with sudden cardiac death:

a. prolonged QT interval.
b. first-degree AV block.
c. sinus arrhythmia.
d. right bundle branch block.
e. premature ventricular contractions.

108. The best medication to use in the emergent treatment of supraventricular tachycardia is:

a. digoxin.
b. verapamil.
c. adenosine.
d. diltiazem.
e. isoproterenol.

109. Treatment of a severely infected diabetic foot ulcers should involve:

a. topical antibiotics.
b. debridement only.
c. debridement with systemic antibiotics.
d. debridement with topical antibiotics.
e. none of the above.

110. A 41 year old business executive presents to your office and complains of palpitations and shortness of breath.
After further questioning, he admits to heavy alcohol consumption the previous evening. On examination, he is found
to have an irregular heartbeat of 130 beats per minute. The most likely diagnosis is:

a. ventricular tachycardia.
b. ventricular fibrillation.
c. premature ventricular contractions.
d. atrial fibrillation.
e. wolff-parkinson-white syndrome.
Part B:

1. A 36 years old woman with SLE presents with acute onset of lethargy and jaundice.
On initial evaluation, she is tachycardic, hypotensive, pale, dyspneic and is difficult to
arouse. On physical examination: splenomegaly. Initial Hb 6 g/dl, WBC - 6300/, PLT
- 294 000/, total bilirubin - 4 g/dl. Reticulocytes - 18%, haptoglobin is not detectable,
normal renal function and normal urinalysis. What would u expect in peripheral blood smear:

a. Macrocytosis and PMN with hypersigmented nuclei.


b. Microspherocytes
c. Schistocytes
d. Sickle cells

2. A healthy 62 years old woman returns to your clinic after a routine colonoscopy. Were found two 1.3 cm
sessile (flat- based) villous adenomas in ascending colon, which were removed completely during the procedure.
Next step:

a. Colonoscopy - 3 month
b. Colonoscopy - 3 years
c. Colonoscopy - 10 years
d. CT scan of the abdomen
e. Partial colectomy

3. Which of the following should promt investigation for the hereditary nonpolyposis colon cancer screening in a
32 year old man?

a.father, paternal aunt and paternal cousin with colon cancer with ages of diagnosis of 54, 68, 37 years, respectively
b. Innumerable polyps visualized on routine colonoscopy
c. Mucocutaneous pigmentation
d. New diagnosis of Ulcerative colitis
e. None of the above

4. A 30 year old woman presents to ur office complaining of parasthesias, weakness. Lack of coordination, and
difficulty with gait. Her symptoms are worse after a hot shower. Examination of the CSF shows oligoclonal bands of
IgG. The most likely diagnosis is:

a. Multipe Sclerosis
b. Huntington’s disease
c. Parkinson’s disease
d. Neurofibromatosis
e. Amyotrophic lateral sclerosis

5. A 56 year old, an presents with chronic diarrhea. He stats he has loose stools for last 2 days. No blood in stool,
temperature, weight loss or recent travel. Ur next step:

a. Observation
b. Stool culture
c. Colonoscopy
d. Stool fat studies
6. A 72 years old female with severe osteoporosis presents for evaluation of shortness of breath. She is a lifetime
nonsmoker and has had no exposures. On physical examination you note kyphoscoliosis.
All the following pulmonary abnormalities are expected except:

a. Restrictive lung disease


b. Alveolar hypoventilation
c. Obstructive lung disease
d. Ventilation - perfusion abnormalities with hypoxemia
e. Pulmonary hypertension

7. A 23 years old male is climbing Mount Kilimanjaro. He has no medical problems and takes no medicine. Shortly
after beginning the climb. He develops severe shortness of breath. Physical examination shows diffuse bilateral
inspiratory crackles. Which of the following is most likely etiology?

a. Acute intestinal pneumonitis


b. Acute respiratory distress syndrome
c. Cardiogenic shock
d. Community acquired pneumonia
e. High altitude pulmonary edema

8. Which of the following sequences represents how a typical anteroseptal myocardial infarction progresses on ECG?

a. Q- wave development, peaked T waves, ST segment elevation, T wave inversion


b. T wave inversion, Q- wave development, ST segment elevation, peaked T waves
c. peaked T waves, ST segment elevation, Q- wave development, T wave inversion
d. peaked T waves, Q- wave development, ST segment elevation, T wave inversion
e. ST segment elevation, T wave inversion, Q- wave development, peaked T waves

9. A 58 year old secretary presents with asthenia and hyperpigmented changes on her
elbow and inner cheek. She also had noted het blood pressure is low and she is dizzy
when she stands. She has also lost 5 kg and has some no nausea and no vomiting. A
recent test for coccidiomycosis was positive. Appropriate testing this time includes:

a. CT of the abdomen
b. Esophagoduodenoscopy
c. Glucose tolerance test
d. ACTH stimulation test
e. Colonoscopy

10. Which of the following statements about polycythemia vera is true?

a. It’s a chronic myeloproliferative disorder that is associated with increased levels of Hb concentration and red blood
cells masses (erythrocytosis).
b.It is associated with neurodegenerative condition of the thalamus
c. Physical examination usually shows decreased peripheral reflexes.
d. Leucopenia and thrombocytopenia are common.
e.The disease is associated with an increased life span of the re blood cells
11. Which of the following is used in treatment to congestive heart failure with systolic dysfunction?

a. verapamil
b. diltiazem
c. ramipril
d. nifedipine
e. isoproterenol

12. A 65 y.o. Man present to the hospital with shortness of breath. He has no prior medical care. Laboratory is
BNP 1500 pg/l. You suspect?

a. asthma exacerbation
b. copd exacerbation
c. pulmonary embolism
d. DVT
e. CHF

13. Which of the following can use to distinguish atrial flutter and sinus tachycardia?

a. carotid sinus massage


b. administration of diltiazem
c. administration of isoproterenol
d. temporal artery massage
e. administration of adenosine

14. Hospitalized patient receive blood transfusion. The patient is flashed, has abdominal discomfort and
38.4 t fever. What is the best management?

a. administration of acetaminophen and to slow the transfusion rate


b. administration of diphenhydramine and continue the transfusion
c. 100mg hydrocortisone i.v. And slow transfusion rate
d. stop transfusion and increase i.v. Fluid
e. i.v. Ranitidine and order abdominal series x-ray

15. Subclinial hypothyroidism is diagnosed with?

a. normal T4 and low TSH


b. normal T4 and high TSH
c. low T4 and high TSH
d. normal T4 and normal TSH
e. low T4 and borderline TSH

16. Which of the following is e severe complication of warfarin which is unrelated to bleeding?

a. pancreatic neoplasm
b. hepatitis
c. skin necrosis
d. peripheral neuropathy
e. pulmonary fibrosis
17. A 24 y.o. Man i.v. Drug user present to the hospital with fever, night sweats chest pain, arthralgia, painless
erythematous lesion on the palm of his hand, round erythematous lesions with central cleaning on the retina and
splinter hemorrhage on the finger nails. The most likely diagnosis?

a. HIV
b. bacterial endocarditis
c. syphilis
d. infectious hepetitis
e. lyme disease

18. Which of the following is associated with C.M.L?

a. leukopenia
b. philadelphia chromosome
c. elevated leukocyte alkalosis phosphatase
d. trombocytopenia
e. decreased b12 levels

19. Definition of chronic bronchitis?

a. chronic productive cough for 3 months in 2 consecutive years.


b. chronic productive cough not responsive to antibiotics.
c. associated with distraction of lung tissue and development of blebs
d. chronic productive cough for 6 months in 5 consecutive years.
e. reduction of lung compliance of 30% or more

20. A 25 y.o. Man present to the hospital with back pain and generalized fatigue laboratory test shows
anemia with rouleau formation, monoclonal spike on electrophoresis and hypercalcamia. Radiograph of the lumber
spine shows lytic lesions. What is the most probable diagnosis?

a. metastatic prostate cancer


b. paget disease
c. osteitis fibrosa cystica
d. multiple myeloma
e. colon cancer

24. Mitral valve prolapse (MVP) is associated with:

a. Elderly, obese men.


b. diastolic click that disappears with valsalva maneuver.
c. Chest pain, dyspnea and sycope.
d. rheumatic heart disease.
e. myxomatous transformation of the valve leaflet.

25. A 28 year old homosexual man, complians of non-productive cough, shortness of breath, high temperature and
chills. chest X-ray shows bilateral interstitial infiltrates. the best treatment would be:

a. Oral azithromycin.
b. IV penicillin.
c. IV TMP-SMX.
d. IV amphotericin.
e. Observation only.
29. A 6 year old girl is sent home from summer camp with a fever of 38.5C, stiff neck, photophobia and headache.
LP shows WBC 380 cells/mm3 with 65% polymorphonucleat cells and 35% lymphocytes, normal protein and glucose
level, negative gram stain. Which pathogen is most likely to cause her symptoms:

a. Nisseria meningitidis.
b. Strep. pneumonia.
c. Enterovirus.
d. Borrelia burdorferi.
e. Mycobacterium tuberculosis.

31. A 10 year -old girl has had a cold for 14 days prior to the visit, she has developed a fever of 39*C, purulent
discharge, facial pain, and daytime cough. Examination of the nose after topical descongestants shows pus in the
middle meatus. Which of the following is themost likely diagnosis?

a.brain abscess
b.maxillary sinusitis
c.streptococcal throat infection
d.retrofaringeal abscess
e.middle ear infection

32. Boy of 2 years-old, who developed noisy breathingon inspiration, marked retractions of the chest wall, flaring of
the nostrils , and barking cough, He has had mild apper respiratory infection for the past 2 days .
Which of the following is the most likely diagnosis?

a.Asthma
b.epiglotitis
c. bronchiolitis
d. viral croup
e.foreing body in the right bronchus

33. In which of the following a 10 years-old patient would isolation of pseudomona Aeruginosa from the sputum be
likely?

a. Child with tetralogia of Fallot undergoing cardiac repair


b. Child with fibrosis cystic who has moderate cough severe lung desease
c. a previously well child presenting with temperature of 40*C, a left lower lobe infiltrate, and white
blood cell count of 20.000/m3
d. Child with asthma previouslu under good control, who presents with wheezing unresponsive to bronchiodilators
e. An adolescent who has recently been using hot tub

34. 2 month old child is seen at your clinc for the first time. The child was born at home , and this is the first well child
visit. Risck factorsfor infant botulism that should be communicated to the pearents include:

a. gardening
b. home construction
c. frozen vegetables
d. honey
e. all of the above
35. A 15 years-old white girl develops bloody diarrhea on the 6 day of hospitalization
for therapy of presumed bacterial neumonia.She has recived intravenous Ampicilin.
Fever and tachipnea resolved and her oxygenation now is normal. In fact, she was
scheduled for discharge. Which of the following action would be most apropiate?

a. Swicht tha antibiotic to 3erd generarion cephalosporines


b. Discontinue Ampiciline
c. Obtain ventilation -perfusion scan
d. Obtain CT scan of the abdomen
e. Add erythromycin or doxycyline

36. Newborn develops sepsis and shock , the pathogen that most commonly causes systemic and focal infection
in newborn is :

a-staphylococcus Aureus
b- group A streptococci
c-group B streptococci
d-eschearichia coli
e-herpex simplex virus

37. Which of the following tumors is associated with Epstein -Barr virus infection

a. Kaposi sarcoma
b. Burkit lynphoma
c. neuroblastoma
d. wilm’s tumor
e. Carcinoma of colon

38. Which of the following cancers occurs primarity during childhood?

a. Breast cancer
b. Renal cell cancer
c. Wilm’s tumor
d. Prostate cancer
e. Colon cancer

39. A 4 year- old boy with anemia has a bone marrow biopsy that shows 4% blasts . No blasts are seen on
peripheral blood smear. Finding on physical examination and another studios are normal. Most likely diagnosis?

a. Acute Lymphoblastic leukemia


b. Acute myelogenous leukemia
c. chronic myelogenous leukemia
d. preleukemia Syndrome
e. this is a normal bone marrow biopsy

40. Most important extramedullary site of relapse in childhood acute lyphoblastic leukemia (ALL):

a. Adrenal glands
b. Kidney
c. Heart
d. Lung
e. Central nervous system
41. The developmental appropriate age of child who scribbles, walks alone, speaks one real word and pretends to
drink from a cup is:

a. 7 months.
b. 13 months.
c. 20 months.
d. 24 months.
e. 30 months.

42. The developmental appropriate age of child who rolls back to front, has a thumb finger grasp, self inhibits to
¨no¨ and bangs two cubs is:

a. 8 months.
b. 12 months.
c. 15 months.
d. 3 months.
e. 18 months.

43. A 10 years old girl has had diplopia, ptosis and weakness of her neck flexors for 2 months. Symptoms are
worse in the evening and usually less severe awaking on the morning. No fasciculation or myalgia, her deep tendon
reflexes are normal. The most likely diagnosis:

a. Hysterical weakness.
b. Muscle dystrophy.
c. Spinal muscular atrophy.
d. Botulism.
e. Myasthenia Gravis.

44. A 4 year old boy is evaluated for his first generalized tonic-clonic seizure which lasted 10min. No history of
illness or fever and findings on examination after the seizure are completely normal. The most appropriate
management is:

a. Begin therapy with Carbamazepine.


b. Order EEG.
c. Order CT scanner of the brain.
d. Order MRI of the brain.
e. Order psychometric testing.

45. A 15 months old girl is evaluated for a 10 min long generalized seizure associated with a temperature of 40÷C.
Which of the following factors in the list is most likely to increase the risk of future seizers?

a. Apgar 3-5.
b. Family history of epilepsy.
c. Clinical evidence of roseola.
d. Female gender.
e. Presence of 2-caf‫ י‬au lait spots.
46. A 3 months old infant presents for a well-child evaluation. Which of the following complaints would be a cause
of concern?

a. Regurgitation of 15-30ml formula 3 times a day.


b. One bowel movement every other day.
c. Three bowel movements per day.
d. Liver edge palpable 2 cm below the right costal margin.
e. Hemoccult-positive stool.

47. A 3 years old hearing impaired child playing with his hearing aid. No battery can be found, and the child is
noted to be drooling. The most appropriate next step in treatment would be:

a. Ensuring consumption of fluids.


b. Ipecac administration.
c. Activated charcoal administration.
d. Chest and abdominal X-ray studies.
e. Observation and follow up in 24hours.

48. Nosebleeds in children arise most commonly from:

a. Turbinates.
b. Nasopharynx.
c. Post septum.
d. Kiesselbachs plexus (ant septum).
e. Maxillary sinus.

49. A 30 minutes old term 3-4kg baby born of spontaneous vaginal delivery is noted to have acrocyanosis.
Most important next step:

a. Perform a sepsis evaluation.


b. Perform a CBC followed by a sepsis work up if neutropenia is found.
c. Keep infant warm.
d. Begin oxygen administration.
e. Hold oral feedings until acrocyanosis resolves.

50. Bilateral and multiple retinal hemorrhages, presenting in the sixth weak of life in a previously normal lethargic
neonate, without any other physical founding. Are most likely due to:

a. Birth trauma.
b. Nuchal cord.
c. Vacuum extract.
d. Being large for gest age.
e. Child abuse.

51. Very low birth weight <1500 gr infants are best described :

a. predominantly growth restricted


b. predominantly premature
c. predominantly post dates
d. the result of maternal illness
e. the result of placental infarction
52. A 3 year old boy presents to your office with sudden onset of cola colored urine
progressive facial swelling over the past 3 days, and decreased urine volume over the
past day. On examination: blood pressure 130/80 mmHg, periorbital edema, bibasilar
rales, and ankle swelling. His urinalysis is remarkable for 3+ hematuria, 1+ proteinuria, 100 red blood cells, and red
blood cell casts, his serum electrolytes are normal and the serum albumin is 3.2 g/l.
this clinical presentation is most consistent with:

a. acute renal failure


b. acute pyelonephritis
c. nephrotic syndrome
d. acute glomerulonephritis
e. chronic renal failure

53. A 3 year old boy presents to an urgent care clinic with a 3 day history of abdominal pain and difficulty walking.
Abnormal findings include blood pressure of 120/80 mmHg diffuse abdominal tenderness. Purpuric rash of the hands
and ankles and diffuse periarticular tenderness and swelling of the ankles. The most likely diagnosis is :

a. systemic lupus erytimatosis


b. Kawasaki’s disease
c. juvenile rheumatoid arthritis
d. henoch-schonlein purpura
e. stevens jhonson syndrome

54. A 3 year old girl presents acute onset of lethargy and pallor the child has bloody diarrhea for 5 days that
cleared one day prior to presenting to your office, she also notes acute onset of cola colored urine. On
examination the patient is pale and lethargic pressure 120/80 mmHg. The most appropriate next step:

a. urinalysis
b. x ray of the abdomen
c. midstream urine culture
d. blood cell count and smear
e. prothrombine time

55. 1 month female infant is presented with chief complaint of poor feeding and lethargy, the parents report that the
child was well until 3 days earlier when poor feeding began pulse rate is 280 beats/min, respiratory rate is 50/min
blood pressure is 80/50 mmHg. Physical examination shows a gallop rhythm and an enlarged liver palpable 2-3 cm
below the right costal margin. The most appropriate diagnostic test for this patient would be:

a. chest radiograph
b. CBC complete blood count
c. ECG
d. arterial blood gases
e. blood culture

56. 2 month old infant with supraventricular tachycardia a initial management:

a. vagal stimulation
b. cardioversion
c. defibrillation
d. digitalization
e. intravenous verapamil
57. A previously healthy 7 year old girl has a 3-wk history of fever, myalgias, and appositive blood culture for
staphylococcus aureus . the past medical history is negative including a normal camp physical examination 6-mo ago,
which did not reveal any heart murmur. After repeating a blood culture, the next step in her evaluation should be:

a.chest radiograph
b.complete blood count
c. echocardiogram
d. bone scan
e. dental clinic appointment

58. The most frequent presenting manifestations of pheochromocytoma in children is :

a. night mares
b. abdominal pain
c. polyuria and polydipsia
d. weight loss
e. hypertension

59. A 7 day is admitted to the hospital for evaluation of vomiting and dehydration, physical examination is normal
except for minimal hyperpigmentation of the nipples. Serum sodium and potassium concentrations are 120
meq/l (low) and 9 meq/l (high), respectively. Which of the following is the most likely diagnosis :

a. pyloric stenosis
b. congenital adrenal hyperplasia
c. secondary hypothyroidism
d. panhypopituitarism
e. hyperaldosteronism

60. A 6-wk has gained no weight since birth. Her skin appears mottled, and an indirect bilirubin level is measured at
15 mg/dl (high) her extremities are cold and her temp. is 35c, the most likely diagnosis is:

a. kernicterus
b. sepsis
c. galctosemia
d. hypothermia
e. hypothyroidism

61. A 12 years jewish girl suffer from anemia , leukopenia ,thrompocytopenia, large spleen on x-ray femur show
erlenm eyer flask, abnormal bone marrow the most likley diagnosis?

a. tay-sachs disease
b. guncher disease
c. mucopolysaccheridosis
d. canavan disease
e. glycogen storage disease
62. The disease pass through mother affect sons and daughter in variable manifestation in affect sibling. most likely ?

a. mitochondrial inheritance
b. multifactorial inheritance
c. x-linked recessive inheritance
d. autosomal rcessive
e. enviromentaly induce

63. RAST determinds?

a. bronchial reactivity serotonin


b.bronchial reactivity after inhalation bronchial provocation test
c.the proportion to the allergic immunoglobiulin
d. antigen specific serum ige concentration
e.overal allergic risk profile based on absolute eosinophil , total ige and skin test

64. A 12 years boy complain from sneezing, clear rhinorrhea , itching physical exam reveals boggy pale nasal
edema clear discharge most likely diagnosis ?

a. foreign body
b. vasomotor rhinitis
c. netrophilic rhinitis
d. nasal mastocytosis
e. allergic rhinitis

65. 3 year old female pt. coughing and wheezing what the strong risk factor for persistent asthma in toddler
with recurrent wheezing?

a. eczema
b. colic
c. living on farm
d. female gender
e. OM with effusion

66. What the effective screening test for T cell function?

a.absoult lymphocyte count


b. flow cytometry for CD4 (helper) + CD8 (cytotoxic)
c. respiratory burst assey
d. candida skin test
e. mump AB titer after mumps vaccination

67. Infection typical to pt. with cellular immunodeficiency?

a. plasmodium vivax
b. strep. Pneumonia
c. staphy. Aureus
d. pneumocytocysitis carini pneumonia
e. hemephilus influenza
68. 4 year white female pt. joint swelling with multiple joint for 6 month slow to move in morning , move as if
stiff for first hour of day , very active child , No rash very little limitation of range motion , ESR= 4 what the
diagnostic ?

a. hypermobility syndrome
b. Dermatomyositis
c. SLE
d. JRA
e. HSP

69. Component of diagnostic criteria SLE in children ?

a. leukocytosis (more 15000 WBC)


b. persistent proteinuria
c. subcutaneous nodule
d. false-positive heterophile test
e. erosive arthritis involving two or more peripheral joint

70. Newborn , congenital heart block, most like etiology ?

a. GB strep.
b. neonatal Echovirus infection
c. gestational DM
d. maternal lupus
e. gray baby syndrome

71. Regarding resuscitative efforts in child, the most important goal is:

a.Restoration of age-appropriate heart rate.


b.Appropriate movement of the chest wall.
c.Auscultation of equal breath sounds in both lungs field.
d.Adequate oxygen delivery and utilization for the body tissues.
e.Palpation of equal pulses in all four extremities.

72. Which statement is true relative to childhood atopic dermatitis?

a.The prevalence of atopic dermatitis is decreasing.


b.It's usually caused by dietary protein allergy.
c.The condition usually improves by age 5 years.
d.Atopic dermatitis is associated with dry skin.
e.Atopic dermatitis is usually asymptomatic.

73. A 2 day old well-appearing full term white neonate experience multiple firm,
yellow-white, 1 - to 2 mm pupules or pastules with a surrounding erythematous flare
on the trunk. wright strain of the lesion shows numerous eosinophils. The most likely diagnosis:

a.Erythema toxicum.
b.Pustular melanosis.
c.Acropustulosis.
d.Eosinophilic pustular folliculitis.
e.Herpes simplex virus infection.
74. A 2-year old boy is noted to be drinking from a container filled with kerosene. He immediately coughs,
becomes tachypnic, and is brought to the hospital. The best approach to his treatment is to:

a.Induce emesis.
b.Perform nasogastric tube lavage.
c.Instill mineral oil.
d.Administer steroids.
e.None of the above.

75. A Previously healthy 5 months-old infant develops bronchiolitis. On the fourth day of illness she is noted to have
bulging, opaque, white ear drums bilaterally. which of the following treatment regimens is the most appropriate to
institute?

a.High dose oral amoxcillin.


b.Intramuscular Ceftriaxone.
c.Oral Cefixime.
d.No initial antibiotic treatment.
e.Oral azithromycin.

76. The use of which of the following medications is limited by patentally life-threatening rashes?

a.Phenytoin.
b.Lamotrigine.
c.Carbamazipine.
d.Gabapentin.
e.Tramadol.

77. Urges of impulses for repetitive intentional behavior performed in a serotyped manner in an attempt to
relieve anxiety is termed:

a.Adaptation.
b.Congruence.
c.Compulsion.
d.Obsession.
e.Transference.

78. Which of the following is the greatest risk factor of suicide?

a.Unemployment.
b.Single status.
c.Low income status.
d.Resident of urban area.
e.History of admission to a psychiatric hospital.

79. In a dying patient, which of the following is used to detect depression?

a.Pervasive hopelessness.
b.Weight loss.
c.Crying.
d.Dependency.
e.Concern over being a burden.
80. Which of the following electrolyte abnormalities is associated with bulimic patients?

a.Metabolic acidosis.
b.Respiratory acidosis.
c.Metabolic alkalosis.
d.Respiratory alkalosis.
e.Normal electrolytes.

81. A 42 year old man is seen in your office.he has recently seen a psychiatrist and is being treated for severe
depression.the patient cannot recall the medication that he is taking,but he remembers that the psychiatrist told him
not to eat cheeses or aged meats.which of the following agents is this patient most likely taking?

a. tricyclic antidepressant
b. ssri
c. mao inhibitor
d. neuroleptic
e. anxiolytic

82. High fever,tachycardia,tachypnea,diaphoresis,hypertension and seizuers develop in a psychiatric patient who


receiving haloperidol.the most likely diagnosis is:

a. malignant hyperthermia
b. rhabdomyolysis
c. neuroleptic malignant syndrome
d. sepsis
e. serotonin syndrome

83. Which of the following is most likely to cause withdrawal symptoms with abrupt discontinuation?

a. fluoxetine (prozac)
b. sertraline (zoloft)
c. paroxetine(paxil)
d. citalopram(celexa)
e. none of the ssri’s cause withdrawal symptoms

84. Children who exhibit symptoms of school avoidance with nausea, vomiting and abdominal pain may benefit from:

a. antidepressent medication
b. Methylphenidate
c. Psychotherapy
d. bezodiazepines
e. Mood stabilizers

85. Which of the following statements about somatization disorder is true?

a. men are more affected than women.


b. Patients exhibit multiple physical comlaints that usually have an identifiable physiologic basis
c. The condition usually develops after age 50.
d. Symptoms rarely affect the patient’s interpersonal relationships.
e. Treatment involves frequent office visits and reassureance to the patient.
86. Which of the following statements about suicide is true?

a. Males make more suicide attempts


b. Women are more succsessful
c. Anxiety is the most common contributing factor
d. Most suicides occur during december
e. Married people have the lowest risk for suicide

87. Galactorrhea is associated with which of the following medications?

a. benzodiazepines
b. tricycle antidepressants
c. Mao inhibitors
d.antipsychotic medication
e. Dopamine agonist

88. Which of the following is most commonly associated with tricyclic antidepressant toxicity?

a. 3rd degree AV block


b. PR interval prolongation
c. QT interval prolongation
d. Widened QT
e. T elevation

89. A 25 year old male was hospitalized for his first episode of psychosis, started on haloperidol. After 3 days
patient’s upper body was contracting, he is in pain and is frightened. What is the best medication to help relieve
his suffering?

a. Propranolol
b. Olanzepin
c. Quetiapine
d. Benztropine
e. Risperidone

90. Most common type of obsession in patients with OCD?

a. Worries about contamination


b. Intrusive sexual thoughts
c. Religious obsessions
d. Hoarding
e. Preoccupation with body image

91. A 18 yrs single man recently diagnosed with schizophrenia referred for consultation was recently fired from his job
at afast food restaurant,clinical pictures has been dominated by positive symptomatology,his cousin recently
diagnosed with schizophrenia,patient asking about his prognosis,good prognosis in his case?

a. predominant to symptom
b. young age at onset
c. family history of schizophrenia
d. bizarre delusion
e. poor premorbid psychosoied functioning
92. A 74 yr women with history of Parkinson disease refered for psychiatric consultation due to worsening psychotic
symptom,acomplete medical workup failed to find any organic etiology for her audiotry hallucination and paranoid
delusion,which of the following anti psuchotic appropriate in treating patient symptom?

a. haloperidol
b. respiridone
c. quetiapine
d. ziprasidone
e. perphenazine

93. The most common location of imagined defect in body dysmorphic disorder?

a. stomach and waist


b. breast and genitals
c. arms and legs
d. face and hair
e. neck and shoulder

94. Which of the following most appropriate step in initial management of patient with neuroleptic malignant
disorder (NMS)?

a. hydration,cooling blanket,cardiac and renal function monitoring


b. cooling blanket,dialysis,hydration,cardiac monitoring
c. hydration,administer baclofen IV,restraints,cardiac monitoring
d. administer lorazepam IV,hydration,measure urine output.
e. adminster diphenhydramine IV,cooling blancket,cardiac monitoring

95. According to data from (NIHM) clinical anti psychotic trials effectiveness
study, which one where patient least to discontinue due to lack of efficacy?

a. olazipine
b. quetiapine
c. ziprasidone
d. risperidone
e. pephenazine

96. Regarding use of SSRIs for the treatment of OCD?

a. the starting dose for OCD IS LESS than the starting dose for depression
b. target symptom worsen befor theyimprove once treatment with SSRI is started
c. the maintenance dose for OCD may be higher than maintenance dose for depression
d. the onset of response is quicker in OCD than in depression
e. if the patient fails to respond to one SSRI they will fail to respond to all SSRI

97. Belief in clairvoyance and telepathy fall in the list of diagnostic of which of the following?

a. histrionic personality disorder


b. schizoid personality disorder
c. schizotypal personality disorder
d. narcissistic personality disorder
e. paranoid personality disorder
98. The best indicator for increased risk of suicide?

a. alcohol dependence
b. marriage
c. schizophrenia
d. previous suicidal behavior
e. major depressive disorder

99. A 30 year old man with history of bipolar disorder on combination of drugs, present wit fever, headache, hacking
cough, this progress to a spreading red rash, which of the following most likely to have caused this condition?

a. lamotrigine and valporic acid


b. fluxetine and valporic acid
c. lamotrigine and fluxetine
d. lithum and lamotrigine
e. lithium and valporic acid

100. A 75 yr women widow leaves her home for annual vication in florida,she forget to take her medication,2 days
later develop( tremor,sweating,nausea,increase anxiety),does not use alcohol or street drug,cessation of which most
cause her symptom?

a. lorazepamb
b. lithium
3. buspirone
d. trazodone
e. valporate

102. Which one of the following disorders is most often comorbide with anorexia nervosa ?

a. Depression
b. OCD
c. Social phobia
d. schizophrenia
e. Avoidant personality disorder

104. A 12 year old boy performs a 3 hour of daily compulsive hand washing and has extreme difficulty going to
school due to contamination fear. most appropriate treatment:

a. Interpersonal, cognitive or psychodynamic therapy plus fluoxetine.


b. Response prevention plus sertraline.
c. Social skill group plus methylphenidate.
d. desmopressin (DDAVP) nasal spray plus behavioral conditioning.
e. Family therapy.

105. Which of these substances is most likely abused by adolescents?

a. Marijuana
b. Alcohol
c. Cocaine
d. Ecstasy
e. LCD
107. Which of the following is common in the childhood history of patients with borderline personality disorder?

a. Shyness
b. Psychosis
c. Conduct disorder
d. Sexual abuse
e. Magical & odd beliefs

110. Hypothyrodism in the eldery may commonly present with all of the following except?
a. lassitude
b. fatigue
c. cognitive impairment
d. anorexia
e. constipation
05.06.2011
By: Alhag Abu Anzeh Muhammad

Part A:

1. Which of the following changes associated with the cardiovascular system occurs during pregnancy?

a. a significant increase in blood volume during the third trimester ,towards labor
b. a right shift of the heart axis due to diaphragm elevation
c. increase in cardiac output towards the end of the third trimester and labor
d. decrease in blood pressure mainly in the second trimester
e. most of the increase in blood volume happens during the first trimester

2. Which of the following is correct regarding the gastrointestinal tract during pregnancy?

a. increase pressure of the lower esophageal sphincter


b. reduced motility of the small bowel
c. decreased fluid evacuation rate from the stomach
d. increased contractility of the gallbladder
e. there is no change in the evacuation rate of solids from the stomach

3. In pregnancy, what is the Chadwick sign?

a. bluish discoloration of the hyperemic vaginal mucosa


b. lower uterine segment softening
c. tenderness of the breasts with enlargement
d. uterus palpable above the pubic symphysis

4. Which of the following is the most substantial component for the maternal weight gain during pregnancy?

a. maternal fat
b. fetus
c. placenta
d. amniotic fluid
e. intra and extra cellular fluid

5. At what frequency is it recommended to check fetal heart rate during low risk labour?

a. every 15 min, during the first stage and every 5 min during the second stage of labour
b. every 5 min during the first stage and continuously during the second stage of labour
c. every 30 min during the latent phase ,every 15 min during the first stage ,and every 5 min during the second stage
of labor
d. every 30 min during the first stage and every 5 min during the second stage of labour
e. every 60 min during the first stage and following every contraction during the second stage

6. A 32 year old female, 2 days following vaginal delivery, is complaining of contractions like pain and mild bloody
discharge. Blood pressure and tempreture mesearment normal. Plus examination, uter is palp below umbilicus,
appropriate step?

c. observation alone at this point due to lack of other physical findings


7. A sustained fetal heart rate deceleration is identified in a 32 week pregnant female fully dilated, with fetal head at
spina +3. What is the most appropriate management?

a. vacuum delivery
b. forceps delivery
c. oxytocin administration
d. right side up positioning and oxygen
e. a wait natural delivery

8. What is the most common fetal complication of vacum delivery?

a. retinal hemorage
b. subgleal hemorage
c. intracerebral hemorage
d. brachial plexus injury
e. cephalohematoma

9. What is the mean duration of pregnancy from the first day of the last menstrual period?

a. 250 days
b. 260
c. 270
d. 280

10. A pregnant patient with sickle cell anemia is at increased risk for which of the following?

a. urinary tract infection


b. gestational HTA
c. placenta accrete
d. gestational dyabetis
e. labor dystocia

11. Which of the following is present in cases of intrauterine grouth retardation (IUGR)?

a. increased venous flow through the ductus venosos.


b. decreased resistance in the umbilical arteries.
c. decreased resistance in the main cerebral artery
d. polyhidramnios in most cases.
e. postterm delivery in 20% of the cases.

12. A 14-years-old female with menarche about a yaer ago in being evaluated for heavy vaginal bleeding. Blood
pressure is 116\73 and heart rate is 86. Her hemoglobin is 10.5g/dl, βHCG testing is negativ. Pelvic ultrasound is
normal with uterine mucos 5 mm thick. What is the most appropriate treatment:

a. urgent dilatation and curettage


b. vital signs observation in the emergency room.
c. oral estrogen
d. GnRH agonist injection
e. Ikaclomin administrasion
13. Which of the following is considered dysfunctional uterine bleeding?

a. A bleeding in a female with hyperthyroidism


b. a female with uterine polyp
c. vaginal bleeding at the age of 60 years
d. atypical hyperplasia of the uterine mucosa
e. factor XI deficiency

14.Which of the following is NOT part of the manengment of shoulder dystocia?

a. Woods screw maneuver


b. fundal pressure
c. McRoberts maneuver
d. delivery of the posterior arm

15. Which of the following is correct regarding bacterial vaginosis?

a. it is a sexualy transmitted disease and therefore both partners need to be treated.


b. in contrast to trichomoniasis,the vaginal pH is lower than average.
c. there are high numbers of lactobacillus bacteria on culture plates.
d. treatment during pregnancy does not decreased the incidence of premature deliveries.
e. vaginal cultures are essential for the diagnosis.

18. Which is not use for therapy of uterine myomas:

a. Uterine artery embolizatione


b. Myoma extraction
c. Use focused Ultra sound
d. Cryolysis
e. Hysteroscopic excision

20. 23Year old female,7 month of amenorrhea, bilateral galattorea , normal breast and pelvic examination pregnancy
test-negative, which class of medications can cause this condition:

a. Anti emetics
b. Gonadotropin
c. Phenotiazin
d. Prostaglandins
e. GnRH analogs

22. Which of the following is true regarding endometrial polyps:

a. more common before age 50 years


b. common following tamoxifen treatment
c. malignant transformation
d. are always associated with symptoms
e. D & C is the treatment of choise
24. A 72 year old woman with vaginal hysterectomy for uterine prolapse and cystocele. Histopathologicaly
endometrial tumor is identified that limited to the endometrium and does not invade the cervix and no metastasis
.which of the following is the correct stage?

a. Stage I-a
b. Stage I -b
c. Stage II
d. Stage III
e. Can’t be determined

25. Which of the following statements regarding varicocele is not correct?

b. Improved fertility.

26. Which of the following parameters is normal in semen count ??

a. liquidation after 5 min


b. a count of 5 million cell/ml
c. 16% normal cell shape by kranger criteria
d. 2000,000 white cells per ml
e. 30% motility

27. Which of the following semen analysis is most sensitive:

a. postcoital analysis
b. Analysis of the antibody
c. Analysis penentratsii eggs Gamsteru
d. precise Kruger criteria
e. Analysis of sperm chromatin structure

28. Which of the following combination has the strongest association to decrease ovarian reserve ?

a. Higher than expected FSH level and low estradiol levels on cycle day 2-4
b. normal FSH levels and higher than expected estradiol level on cycle day 12-14
c. higher than expected FSH and estradiol levels on cycle days 2-4
d. higher than expected FSH levels and normal estradiol levels on cycle days 2-4
e. normal FSH and higher than expected estradiol levels on cycle days 2-4

29.Which of the following is correct regarding hirsutism ?

a. excessive vellus hair on the body


b. ovarian hyperandrogenism
c. clinical definition vary according to the population
d. it is secondary to hair follicle overreaction to normal androgen levels
e. its definition is based on facial hair alone .
30. What characterizes the follicular phase in the peri menopauses period ?

a. it is longer due to the difficulty in follicular recruitment


b. it is accompanied by increased inhibin secretion
c. it is shorter due to early follicular recruitment
d. substantial decrease in AMH levels
e. fertility is not affected

31. Which of the following is NOT associated with early menopause ?

a. polycystic ovarian disease


b. ionizing radiation exposure
c. fragile X syndrome
d. prior endometrial excision
e. cytotoxic exposure

32. Which of the following is TRUE regarding testosterone levels following oophorectomy?

a. there is an increase metabolic testosterone clearance from the plasma


b. there is a increase resistance to testosterone action in target tissue
c. testosterone levels is decreased by about 60%
d. testosterone levels is decreased by 5-8%
e. there is no change in testosterone levels following oopherctomy.

33. What is TRUE regarding the vaginal changes during menopause ?

a. the mucosa appears red and hyperaemic


b. the vaginal PH is basic (less acidic )
c. glucagen accumulates in the epithelial cells
d. increase tendency for infections due to increased lactic acid production
e. vaginal watery transparent discharge that is stretchable

34. What is correct about regarding ovum maturity ?

a. mature ovum has 2 polar bodies


b. mature ovum has wide extensive cumulus
c. an ovum that was aspirated prior to maturity cannot be fertilized and is useless
d. ovum maturation converts all ovum from immature to mature
e. mature ovum has complete and compressed cumulus

35. Which of the following is TRUE regarding sperm ovum fertilization in in-vitro fertilization ?

a. fresh sperm may be used directly with no preparation


b. fertilization is usually performed about 5 hours after ovum aspiration
c. in every fertilization there are at least 500,000 motile sperm cells used for every ovum
d. in ICSI one sperm cell is injected to the privitellin space
e. all ovum are mature and ready for sperm fertilization 36 hours after administration of hcg.
41. Patient 70 years old has developed angina tsilyulit konechnosti.davlenie 80/40 puls120 saturation temperature
95 39. In light hripy.Bolnoy received intravenous catheters 4litra saylayna was ustanovlen. Chto from treatment:

a. noradrenalin
b. adrenalin
c. dobutamin
d. milrinon.

42. Year-old child sovzdutiem abdomen with developmental problems from birth to difieatsiey using candles and
ochestitelnyh klizm.apatichen hernia zhivota.Pri percussion tempanit. Stul under pressure from the green and
zlovonnyy. na rengenne inflated loops of the small intestine. Which is the best treatment.

b. fluids and antibiotics plus an intestinal lavage.

43. Which of the following. Hernia 2 year old boy:

c. Recommended attempt manual reduction.

44. 30 y.o man had a motorcycle trauma fracture of the middle of the femoral kosti.Kakoe conventional treatment?

b. PO intromedulyarnymi Fixing nails for 6-8 hours.

45. What is true about the treatment of osteoaptrita:

c. The patient should be aware that effective surgical treatment for a limited period of time may
vozniknutpotrebnost in a second operation for several years.

46. 54 years old man had an accident, according to a urinary catheter -450 ml of bloody urine. Which test is needed?

a. cystoscopy
b. In / pyelography
c. CT without contrast
d. Retrograde urethrogram
e. PT tsistogrammoy

47. Patient after a surgery has small bowel obstruction caused by bezoar. Which surgery had he gone through?

a. vagotomy
b. Resection of the colon for cancer
c. Operation for obesity
d. multiple abdominal surgery
48. 88 years old man brought to the emergency room with severe pain in his groin. Two weeks ago he suffered of an
acute myocardial infarction, takes aspirin. temperature - normal, pulse = 115, no symptoms of irritation of the
peritoneum. right-sided incarcerated inguinal hernia was diagnosed. right move?

a. Emergency coronary angiography


b. Taliev load test
c. surgical correction of a hernia
d. reduction under deep sedation
e. CT abdomen and pelvis

49. 22 year female brought to ER after MVA injury . upon arrival is fully conscious , blood pressure 120/80 heart rate
120 regular , and room air 92% on exam .n there is abrasion on right chest wall and open fracture of right humerus on
chest x-ray a small to medium size pneumothorax is iden4tefied FAST in negative , next:

a. surgicall humerus fracture fixation


b. head CT
c.chest drain placement
d.repeat x-ray in 8 hours
e.right angiography

50. 38 y / f arrives to the ER with right upper abdominal pain , prior week she went laparoscopic Cholecystectomy , on
examination she is in no apparent distress without icterus ,wound iis clean and intact WBC 15,000 ,liver function
analysis is increased ,bilirubin is normal , what is the next step ??

a. ERCP + stent
b. Abdominal US
c. Liver scan
d. Diagnostic laparoscopy
e. Broad specific antibiotics and observation

51. Small bowel perforation in blunt trauma is reliably detected by:

a. Abdominal CT
b. Focused assessment with sonography for trauma (fast)
c. Diagnostic peritoneal lavage
d. Physical examination on admission
e. None of the above

52. 82 years old male, IDH, HTN, DM, hyperlipidemia, s/p carotid endarterectomy, arrives to the emergency room due
to left abdominal pain and bloody diarrhea for 4 days. On examination fever 37c with tenderness. After two days of
NPO, IV fluids and antibiotic treatments he develops severe abdominal pain, fever 39c and diffuse abdominal
tenderness. On x-ray no freelance air is evident. On CT- abdominal thickening of the colon and large amount of free
fluid in the pelvis, no free air. What is the treatment of choice?

a. Transfer the patient in ICU


b. Laparotomy and ileostomy surgery
c. Laparotomy and transverse loop colotomy surgery
d. Laparotomy and subtotal colectomy and ileostomy
e. Hartmann's procedure
53. 50 years old male arrives to the emergency room with right abdominal pain, vomiting and distention. On CT-
obstructing mass in the hepatic flexure without small bowel distention. What is the correct management?

a. NGT and IV fluids followed by further workup including tumor markers, colonoscopy and completion of CT chest
b. Urgent right hemicolectomy
c. Stent insertion to relieve the obstruction and preparation for later surgery
d. Hartmann's procedure

54. Air in the bile ducts on abdominal x-rays can be caused by all except:

a. Status post papillotomy


b. Small bowel necrosis secondary to mesenteric ischemia
c. Status post liver transplant and jejeunostomy
d. Gallstone ileus
e. 10 years following choledocoduodenostomy

55. 57 years old healthy female arrives to the emergency room, complaining of increasing abdominal distention and
constipation that began 2 months ago with recent worsening, in the past week symptoms worsened and the last
bowel movement was 10 days ago. There is no significant abdominal pain or vomiting, but the patient complains of
severe nausea. On physical examination the abdomen is distended, thympanic, without hernias or surgical scars,
laboratory - Hb- 10,2 WBC- 11,000 Na 132, on x-ray severe distention of the right and transverse colon and no small
bowel distention. What is the next step?

a. Fast bowel preparation followed by colonoscopy


b. CT scan of the abdomen and the pelvis with IV and oral contrast
c. Gastrographin enema
d. Laparotomy

56. Which is NOT associated with increased risk of breast cancer ?

a. smoking
b. atypia on previous breast Biopsy
c. 1st degree family relative with breast cancer
d. older ager
e. LCIS

57. Which has the greatest impact on the prognosis of breast cancer patient?

a. size of the tumour


b. histological tumour type
c. number of axillary lymphnode involvement
d. hormonal receptor in primary tumour

58. 46 y /F had bloody discharge of the left nipple for 3 months , on examination : no palpable mass , but left nipple
compression products minimal bloody discharge at 3o’c , mammography is normal and US identifies a minute mass
behind the left areola . what is the diagnosis?

a. intraductal papilloma
b. intraductal carcinoma
c. fat necrosis
d. cys
e. fibroadenoma
59. 50 y /m had colonoscopy and resection of polypes from the right colon few hours to the presentation to the ER ,
ex: diffuse abdominal tenderness with peritonitis is noted .chest x-ray shows free air under the diaphragm .what is
the next step ??

a. NTG ,antibiotics ,fluids


b. Colonoscopic closure of perforation
c. Right colectomy
d. Laparoscopy ex ,irrigation and drain placement

60. what is the main objective of hip or replacement ?

a. avoid deterioration of physiological load on joint above and below the replaced joint .
b. enable athletics especially professional , to achieve load on and stress that the native joint is capable of performing
c. avoid intra joint fx in patients that are prone for it
d. decrease pain and improve quality of life

61.All of the following are compensatory mechanism in hypovolemic shock exept:

a.increased peripheral resistance secondary to increased catecholamine levels.


b.increased DPG 2,3 levels.
c.right shift of the hemoglobin dissociation curve secondary to acidosis.
d.decreased cardiac contractility.

62.Which of the following is the gold standard diagnostic test for GERD?

a.esophageal manometry.
b.gastroscopy.
c.esophagogastric barium series.
d.esophageal PH-metry for 24 hours.

63. A useally healthy 65 year old female that is heavy smoker had a single event of painless bloddy Urine
(macrohematuria),a work up include cystoscopy,US,urinary culture,and urinary cytology-all negative.what should be
included in further evaluation?

a.there is no need for further evaluation.


b.US scan should be repeated with addition of doppler imaging of the renal blood vessels.
c.abdominal CT scan with IV contrast during bladder biopsy to rule out carcinoma in situ.

64.What is the recommended treatment for colorectal liver metastasis that are localized to one liver lobe?

a.lubectomy of the lobe containing the metastasis.


b.lubectomy of the lobe containing the metastasis and systemic chemotherapy.
c.systemic chemotherapy using cystotoxic and biologoc drugs.
d.palliative treatment only.
65.a 55 yr female with no medical historyis being evaluated for melena and is found to have 2cm gastrointestinal
stroma tumors(GIST)in the anterior wall of the stomach.the diagnosis is confirmed with immunohistochemical
staining.there is no adjacent organ invasion or distant metastasis.what is the recommended treatment?

a.open or laparoscopic margin free tumor excision and follow up.


b.subtotal gastrectomy and regional lymph node dissection.
c.treatment with tyxosine kinase inhibitors till complete response and after surgery in case of medical treatment
failure.
d.treatment with tyrosine kinase inhibitors imatinib.

66. in the ER a 60 year old male with no medical hist is found to have onstructing tumori in the sigmoid colon .
In combinative CT and gastrografian enema the cecum diameter is 15 cm what is the recomanded treatment ?

a. mecanech bowel preparation followed by surgery that includes sigmoidectomy and reg lymf nodes desection and
anastomosis
b. imediat surgery that includes includes sigmoidectomy and reg lymf nodes and anastomosis with no further
workup
c. immediate colonoscopy with biopsy and after dg is histologically confirmed sigmoidectomy
d. immediate surgery that includes sigmoidectomy and reg lymf nodes desection and correction of poximal stoma

67. 73 year old arrives to the ER with right upper abd pain physical ex the pacinet have icterus no fever right abd
tenderness WBC 7800 BILIRUBIN 65 elevated liver function test is 45 with distented gallbladder and mild extra
and intra hepatic ducts dilatation what is the next step:

a. percutaneus transhepatic colangiography


b. colycestoctomy with intraoperative colangiogram
c. MRCP
d. liver viral test
e. broad spectrum AB

68. 19 year old male is involved motor vechile accident on arrival BP 70/40 HR 130/MIN ROOM AIR SATURATION
92% with left lower abd abrasion the abd is soft without peritonitis and pelvis is stable there is left clavicular
fracture and left humerus .chest x ray left lung contusion and lower rib fracture , fast is positive after 2 litteri of I.V
fluids the BP is 75/40 heart rate is 120 what is the next step ?

a. chest , abd and pelvic CT


b. REPEAT FAST
c. laparotomy
d. DPL
e. left chest tube insertion

69. 78 year old male whit abd pain vomiting opstipation with no fever ,abd tenderness without peritonitis and x ray
reveals a number of distended small bowel loops with air fluids levels what is the next step ?

a. laparatomy
b. abd US
c. oral contrast study
d. gallbladder drainage
e. gastrografian enema
70 . 75 year old female with elevated in the ER for rectal bleeding post medical history is positive for diabetes treates
with glucophage and episode of diverticulitis she is pale BP 90/60 HR 120 her abd is soft , rectal ex with fresh
blood and no hemorrhoids HB 9.7 and there is bilious content in the NGT after I.V fluids and 2 units of rectum.
what is the next strep ?

a. urgent colonoscopy
b. sigmoidectomy
c. angiography
d. red blood cell scan
e. dg laparoscopy

71. 70 years old female follow lumpectomy and axillary dissection due to right breast cancer. On pathology the tumor
was found to be 2 cm, G2 infiltrating ductal carcinoma, ER (-), free margins and 1/19 lymph node (+). Which of the
following therapies is appropriate?

a. Radiation
b. Radiation + chemotherapy
c. Radiation + chemotherapy + Tamoxifen
d. Mastectomy + chemotherapy

72. 80 years old female with ischemic heart disease, diabetes mellitus, hypertension and exercise atherosclerotic
disease is admitted to general medicine with diarrhea and abdominal pain. A week ago she started with antibiotics for
pneumonia. Clostridium difficile (CDF) is positive on stool cultures. She is treated with IV Flagyl following oral
Vancomycin and IV fluids. Despite therapy - Temprature 40° C, abdominal distension, diarrhea, dyspnea, oliguria. Lab.
WBC 70,000, Hb 9.5, K+ 5.6, creatinin 2.6, pH 7.1, BE (-10), HCO3- 19. Abdominal CT - significant bowel wall thickness
thou the colon and moderate amount of free fluids with no air, the mesenteric blood vessels are patent. What is the
appropriate management?

a. Patent should be intubated + antibiotic therapy to cover resistant bacteria + ICU.


b. Urgent dialysis, to treat metabolic acidosis + hyperK+.
c. Fast bowel preparation followed by coloscopy to role out ischemic mucosa of the large bowel.
d. The patient should be taken to surgery that includes total colectomy and ileostomy.

73. 49 years old female with no family history of breast cancer, undergone open biopsy of 2 cm left breast mass. On
pathology - fibrosis, ductal atresia and several foci of LCIS that involve the surgical margins. Which of the following is
correct?

a. An extension of the margins is indicated.


b. The patient is at increased risk for developing breast cancer at the biopsy site.
c. If breast cancer develops, it will most likely be at the biopsy site.
d. If breast cancer develops, it will be lobular carcinoma.
e. There is no indication for Tamoxifen therapy in this patient.

74. A cirrhotic patient that is liver transplanted candidate is admitted for esophageal bleeding varicies. Therapy with
somatostatine + endoscopy including sclerotherapy and failed banding do not stop the bleeding. What is the next
step?

a. TIPS.
b. Repeat endoscopy.
c. Distal splenorenal shunt.
d. Surgical procedure (porto - azygos disconnection).
75. What is recommended approach to a patient with acute diverticulitis defined as Hinchey IV?

a. NPO + antibiotics + fluids.


b. Hartmann's procedure.
c. CT guided percutaneus drenage.
d. Urgent diagnostic colonoscopy.

76- A patient undergoes a segmental mastectomy and negative marging are obtained . two sentinel lymph nodes are
identified on final histologic evaluation 106 -mm region of macrometastases is found on H&E staining within two of
lymph nodes . the next step should be:

a. Completion mastectomy
b. Completion axillary dissection
c. Axillary dissection and breast radiation therapy
d. Systemic chemotherapy alone
e. No further axillary operation

77. A 65 old man has a 3 cm rectal polyp, the lower edge of this mass is located 6 cm from the anal verge. Biopsy of
the lesion shows a focus of invasive adenocarcinooma . ct of the abdomen and chest show no evidence of metastatic
disease the endorectal ultrasound reveals enlarged perirectal lymph nodes . the best manegment :

a. Local excision alone


b. Wide local excision , adjuvant chemoradiation therapy
c. Total mesorectal excision with coleanal anastomosis
d. Neoadjuvant chemoradiation therapy, total mesorectal excision , coeanal anastomosis
e. Neoadjuvant chemoradiation therapy, total mesorectal excision, abdominopneal resection

78. Which of the following statement about gastrointestinal stromal tumors (GIST) is true :

a. t1 tumors are confined to the mucosa


b. most GISTs stain positive for cd117 by immunohistochemistry
c. gastric GIST have a much worse prognosis than gastric adenocarcinoma
d. the second most common location of GISTis the colon
e. routine regional en-block lymphadenectomy improves regional control and conferes a survival advantage

79. A 55 year old woman with a history of an ileocecotomy for crohn disease undergoes cholecystomyfor
symptomatic gallstones , diarrhea and stool frequency increase markedly post-operatively . the single most effective
therapy is :

a.bismuth
b.octreotide
c.loperamide
d.cholestyramine
e. pancreatic enzymes

80. A 23 year woman undergoes laparoscopyic roux-en y gastric bypass for morbid obesity post- operatively she
develops tachycardia , oliguria , hypoxemia and anxiety . physical examination fails to demonstrates peritonitis . an
upper gastrointestinal series fails to demonstrate a leak . the next step should be :

a. ct -pulmonary angiogram
b. diuretics
c. laparotomy or laparoscopy
d. broad spectrum antibiotics , observation in the ICU
e. pulmonary artery catheter placement
81. 12 hours after colonoscopy with removal of a polyp 2.5 cm on a broad basis of the blind intestine, 55 year old
woman has abdominal pain, fever, leukocytosis, pain in the lower right quadrant.On abdominal radiography there is
no free air. What is the next step?

a. hospitalization for re-examination of the abdomen.


b. Diagnostic laparotomy
c. abdominal CT imaging
d. colonoscopy
e. gastrografin enema

82. What are the indications for Stricturoplasty in Chron's disease?

a. Previous resection
b. Stricture larger than 20 cm
c. Malnutrition
d. Perforation
e. Many strictures in a short segment

83. A man with cirrhosis and esophageal varices that are not bleeding , cannot tolerate β blockers. What will prevent
future bleeding from the varices?

a. Endoscopic ligation
b. Octreotid infusion
c. Tips
d. Endoscopic sclerotization
e. Portocaval shunt

84. In a 50 year old man with symptomatic GERD , on Endoscopic examination - evidence of barrett esophagus.
Which of the following is true?

a. Surgical treatment is not indicated


b. Treatment with PPI will reduce the size of the area involves with barrett
c. Surgical treatment of the GERD symptoms will not reduce the risk of cancer
d. Barrett esophagus is present in 50% of patients with GERD
e. Endoscopic ablation of the area involving the barrett will reduce the risk of esophageal cancer

85. Which of the following proposals for lobular carcinoma in situ is not true:

a. Tamoxifen reduces the risk of invasive breast cancer


b. patients are at increased risk of breast cancer on both sides
c. usually there are no calcifications or palpable tumor
d. required resection of cancer with clean boundaries
e. more frequently in women before menopause

86. A preterm bay 27 week 1050 gr, was ventilated post delivery due to RDS and his condition improved to 85%O2
after 1.5 hour of surfactant is given the exp[ecte change in lood gases is:

a. decrease bicaronate
b. increase pco2
c. increase po2
d. metaoliacidosis
e. respiratory acidosis
87. A five days baby is sleepy and refuse to be feed for several hours, in his anamnesis (past hstory (was noted that he
was orn to a mother with g6pd deficiency in the physical exam it was revealed that he is suffering from severe
jaundice specially arching around his neck during the exam it was revealed that he is suffering from apnea and
seiaures followed y asystole and all cpr failed, what is the clinical test that should e oerformed at the age of two days
to avod such complication?

a. bilrubin level
b. setum sodium
c. serum potassium
d. serum
e. seum calcium level

88.In asysmptomatic infant is diagnosed with congenital CMV infection y urine culture taken at the age of 3 days
,parente are requiring regarding future complication, which of the following correct?

a. autism is expected
b. possile progressive neural hearing impairment
c. blindness by age of year is expected
d. chronic liver failure is expected most prominent liver injury
e. cardiac defect is most common isolated finding

89. Tweelve year old child is eing followed in clinic for asthma, his treatment is inhale but he is still coughing you are
considering wether to increase the steroid dosing, which of the following will assisst with desicion making:

a. pulmonary function test


b. chest x-ray
c. seum IgE levels
d. CBC wit differential
e. skin test

90 .Which of the following is considered a risk for a sudden death syndrome :

a. pertusis vaccine
b. milk energy
c. lying down in cold eviorment
e. prone position

91.A 9 year old exammind in clinic for right otitis with purolent discharge, she has been treated 3 times previosly for
otitis in the past. Addisional history child has a cronic cough for years and even had pneumonia 3 times in the past.
Which of the folowing may assist in identifying the sendromm in which these findings are commen:

a. asses signs of maturity


b. urin analysis and urinary add blood protein
c. asses child is breathing with an open mouth
d. asses tonsil size
e. heart auscultation
92. A 2 year old has surgery for bowel obstruction, the cause is thick meconium with no other anatomic abnormality.
Which of thefollowing tests should be preformed?

a. sweat test
b. abddomen US to rule out other malformations
c. barium enema to rule out cronic malformations
d. barium swallow to rule out proximal malformation
e. rectal biopsy

93. A 9 year old child is being examind for bilateral knee and right elbow pain and mild sweling. On the posterior
aspect of the thighs and buttocks she has mildly raised rash that dos not dissapear when compressed. What should
the family be recommended:

a. there is no need for follow up if the rash resolves


b. follow blood press and urianalysis
c. complement and IgA levels should be tested
d. the child should be admited for treatment with high dose methylprednison
e. oral penecilin treatment for 10 days should be given

94. A 4 year old checked in clinic for paraorbital and paratibial swelling, on fisical examm there is pitting edema to the
level of the knees with normal blood pressure, urinalysis protein plus 4 , traces of blood. Lab normal, creatinin, low
albumin and elevated colesterol.which is correct regarding the childs diseas:

a. tow days of albumin infusion are recommended


b. kidney biopsy and steroids
c. macroscopic hematuria will follow
d. in most cases ne reacurance after treatment
e. no increesd risk for infectius diseases

95. The parents of a 5 year old notice he is snoring during sleep. The child has problems staying awake in prescool and
has behaverol problems, the father allsow snors. On fisical exam large tonsils with no exudat. What is the next step?

a. laryngoscopy
b. sleep lab
c. apnea ambulatory monitoring
d. telemetry
e.arterial blood gas testing

96.Regarding resuscitative efforts, the most important goal is ?

a. Restoration of age appropriate heart rate


b. Appropriate movement of the chest wall
c. Auscultation of equal breath sounds in both lung fields
d. Adequate oxygen delivery and utilization for the bode tissue
e. Palpation of equal pulses in all four extremities
97. A 5 year old is noted by the parents to snore at night . the child has also had problems staying awake in preschool
and has had behavioral problems. The father also snores. physical examination of the child reveals large , pink ,
nonexudative tonsils . the most appropriate next step is ?

a. Laryngoscopy .
b. Sleep laboratory
c. Ambulatory apnea monitoring
d. Telemetry
e. Arterial blood gas analysis

98. Infant botulism is suspected in a-2 month old infant what is the best test to confirm the diagnosis?

a. lumbar puncture
b. ct scan
c. muscle biopsy
d. electromyography
e. stool simple

99. an 8 year old boy is having repeated episodes of minimal change nephroitic syndrome one every episode was
treated prednisone till the proteinuria resolved however it recurred immediately after treatment cessation for which
of the following is the boy increased risk for?

a. reduced fertility
b. Chronic renal failure
c. Wilms tumor
d. Osteoporosis
e. E-deafness

100. A 1 year old infant is having an intermittent rash in the face in extremity folds that itches mostly at night , his
brother had a similar rash during infancy that has resolved . what is the most probably diagnosis ?

a. Atopic dermatitis
b. Scabies
c. Seborrheic dermatitis
d. Contact dermatitis
e. Psoriasis

101. Which of the following is correct regarding an immune deficient in a child?

a. antibiotics should be avoided


b. live attenuated vaccine can be given to a patient with agammaglobulinemia
c. agammaglobulinemia is clinicaly symptomatic at 1-3 month
d. Di george syndrom involves B cell deficiency
e. patient with only IgA deficiency may be asymptomatic

102. A 1.5 year old is brought to the clinic by his parents due to lack of appetite and palor. on physical examination his
weight is 7 kg HR-120, Hgb-7, MCV- 60 , MCH- 19, WBC-5300, PLT- 200000. what is the diagnosis?

a. gluten sensetivity (celiac)


b. parvovirus B19
c. ALL
d. gaucher disease
e. thallasemia major
103. An 8 day old infant is brought to the ER because of palor and increase bleeding following circumcision. Hgb-7,
MCV-90, WBC- 5600, PLT-340000, PT- 98%, PTT- 100 sec. which is the most likley diagnosis that cause the prolonged
PTT?

a. factor XIII
b. factor VII
c. factor X
d. factor VIII
e. factor XII

104. A 5 year old broght by his parents to the ER due to fever, weakness and abdominal pain for the last 2 weeks. The
boy is usually healthy. He looks ill, liver is palpated 4 cm and the spleen 5 cm below the rib-cage. On CBC: hgb-5, MCV-
80, WBC-34000, PLT- 24000, reticulocytes- 0.5%. What test will assist the diagnosis?

a. direct coombs test


b. coagulation panel
c. bone marrow aspiration
d. abdominal US/CT
e. chest X ray

105. A 4 year old boy is reffered for evaluation due to abdominal distension. Abdominal US demonstrate a mass of
the left kidney.what is the diagnosis?

a. burkitts lymphoma
b. neuroblastoma
c. clear cell sarcoma
d. adenocarcinoma
e. wilm's tumor

106.A 2.5-years-old toddler is bought to the ER by his parents due to general seizure at home that lasted about a
minute.at the ER his temperature is 39.5 and acute otitis is identfed.his physical examination is otherwise
normal(including neurologic examination)what is the next step at the ER.:

a. head CT
b. LP
c. EEG
d. Observation for a few hours
e. blood gases,lactate

107. Sever hypotonia whitout tendon reflex is identifiedon routine examination of a 2 months-old infant.whatis the
most probable diagnosis ?

a. Werding-tloffman(spinal muscular)atrophy
b. tay-sacki
c. canvan
d. krabbe
e. neurofibromatosis type 1
108. Which of the following is the most likely diagnosis of 3-years -old boy that run in circles,avoids eye contact ,and
does not talk?

a. ADHD
b. CD
c. Dyslexia
d. infantile autism
e. mental retardation

109. The treatment of cyanotic spell in a child with tetralogy of fallot include all of the following exept;

a. Oxygen delivery
b. Ice bag over the face
c. Calming down the child
d. morphine
e. Phenylephrine

110.A 3 years-old boy refered for evaluation of recurrent nasal bleeding for past 2 weeks,on CBC:PLT-3,000 which of
the following is not associated with the diagnosis of acute immune thrombocytopenia purpuia(ITP)?

a. Neutropenia (absolute neutrophil cout=200)


b. viral infecation a month prior to admission
c.Iron deficiency anemia(hgb=9)
d. mild spleen enlarge(2 cm below the rib cage)
e. blood urine
Part B:

1. A 12 year old boy is complaining of fever,fatigue,sore throat and abdominal pain for 10 days . on physical
examination there are tonsils exudates ,diffuse lymphadenopathy ,and enlarged spleen up to 3 cm under the rib cage.
2 days ago throat swab was sent for culture and moxypen(amoxicillin) treatment initiated, the cultures are negative
but in the meantime a diffuse rash appeared. What is the fastest and simplest test that will support the most
probable diagnosis?

a. blood cultures
b. blood smear
c. repeated throat swab
d. urinalysis

2.An 8 month infant arrives to the ER due to fever ,vomiting and somnolence that started today. The infant appears ill
and somnolent and on examination he has bulging fontanelle and nochal rigidity. There are no other abnormal
findings. On CBC -25000 mostly neutrophils, Hgb-11, PLT -320000. What are the most likely diagnosis, investigations,
and treatment?

a. viral meningitis , stool cultures,blood cultures ,L.P, antibiotics


b. bacterial meningitis ,L.P ,dexamethasone,immediately followed by antibiotics
c. bacterial meningitis with minigismus ,head CT, L.P, antibiotics according to results
d. the beginning of roseola infantum, L.P ,dicision based on results

3. A usually healthy with up to date immunizations 3,5 year old girl is having fever and cough for 2 days. On physical
examination, saturation is normal with decreased air sounds and crepitus over the base of the right lung. On CBC,
WBC-19000 with 80% PMN. Chest X RAY demonstrates right lower low infiltrate. Which of the following is the
recommended first choice treatment?

a. Azithromycin(azenil)
b. Ceftriaxone
c. Amoxicillin-clavulanate(Augmentin)
d. i/v Gentamycin
e. Amoxicillin (Moxypen)

4.a 3 year old child is having fever 40 c ,bloody diarrhea .Which of the following complications is not likely??

a. confusion,headache and seizures


b. hemolysis ,anemia and reduced kidney function
c. electrolytes abnormalities ,mainly hyponatremia
d. Gram negative bacteria ,growth on stool culture
e. Dyspnea and pneumonia

5.a baby is born at 40 weeks term following a pregnancy with no medical observation or tests performed by the
mother-the baby 1,9 Kg ,ead diameter 28 cm ,and there is purpuric rash and enlarged spleen and
liver,thrombocytopenia ,increased liver enzymes ,head U.S -calcification around the ventricles . what is the most
probable intra uterine infection??

a. EBV
b. Varicella
c. Parvovirus B 19
d. CMV
e. Syphilis
6. which of the following children should be admitted for urinary tract infection?

a. a 1 year child that also passed diarrhea X3 today


b. a 6 year girl complaining of flank pain
c. a 5 year girl with a history of uti at the age of 1 year
d. a 3 week old infant with fever 38.5c
e. a 4 year girl with fever of 39c, with burning sensation and urinary urgency

7. a 4 year old boy is brough to the ER with high fever and swelling of the right eye. The eye is shut with swelling and
redness of the eyelid. The ophthalmologist finds conjunctival edeme with yellowish discharge and the eye ball
movments are limited. What is the most appropriate therapeutic approach?

a. i.v. antibiotic and orbital and nasal cavities ct scan


b. oral antibiotic and antibiotic eye drops
c. i.v. antibiotic without imaging
d. i.v. antibiotic and right orbital us
e. urgent surgery to save the right eye

8. a 2 year child is admitted for high fever, difficulty in eating and drooling. On examination, there is a right torticolis.
With right submandibular lymphadenopathy. The parents describe his voice as “blurry”. Inspection of the right
pharynx reveals tonsils exudates. What is the appropriate treatment?

a. neck imaging and initiate in treatment


b. initiate oral penicillin
c. tube feeds due to difficulty in eating
d. initiate fluid support (for mononucliosis)
e. immediate intubation

9. a usually healthy 6 month old infant has a 5 days of fever up to 40c and no other complains. For the past 24 hours
there is a maculopapular rash over the face and body. Today the fever is decreasing . what is the most likely
diagnosis?

a. measles
b. varicella
c. roseola infantum
d. Kawasaki
e. scarlet fever

10. an infant has swallowed several pills at his grandmother’s house. He is found unconscious and later develops a
seizure. On ECG there is QRS widening. What did he most likely swallow?

a. ibuprofen
b. carbamazepine
c. tricyclic antidepressant
d. oral hypoglycemic agent
e. salicylates

12. Which of the following is correct about type 1 diabetes?

a. Incidence is constantly increasing.


e. simillar incidence in western countries
13. 5 years old boy diabetic. prot+1 ketons+4 gluc+4:

a. chetoacidosis
c. meckle diverticle

17. 1 month old comes to the ER with distended abdomen ,vomit... ... metabolic alcalosis:

a. Diagnosis with abdominal US


b. Diagnosis will show nefrologic disfunction
c. Examination will show a palp olive abdomen
d. The patient can be with out jaundice
e. Plain abdominal film is the best management choice

18. 10 year old child is going under a upper endoscopy test for a complain of epigastric pain , ulcers was found:

a. the ulcer are all a cause of H.pylori


b. pain will be showed immidiatly after meals
c. iron deficiency anemia can accompany
d. pain while sleeping is Atypical
e. no antibiotic therapy is needed

19. 2 year old male with FTT , diarrhea,distended abdomen, in blood test was found to be positive to
TransGlutaminas-Antibody:

a. higher incidence in trisomy 21


b. start gluten free diet
c. constipation cancelled the suspected diagnosis
d. avoid rice
e. biopsy will be normal

20. 15 year old with mononucleosis 2 weeks ago , comes now to your office the mom complaint of restless and
apathy and bilious vomiting of the child, for a correct diagnosis:

a. abdominal ulta sound


b. administer Ampicillin immediately
c. bowel restraction
d. 60% chance of recurrence
e. The more probable diagnosis is meckel ‘s diverticulum

23. which of the following is CORRECT regarding normal development of a 2 year old child?

a. independently copies an image


b. gets dressed without sentences
c. uses tow word sentences
d. identifies colours
e. draws a circulated human face

24. Which of the following that seem as advanced progress of neurological development is suspicious of significant
pathology and requires investigation??

a. steady seating at the age of 6 months


25. a 2 mounth old girl has a rapidly growing hemangioma involving the perioral region bilaterally. With symmetric
extension into the chin and neck. She was born 8 week premature with an other wise benign medical history. The
most appropriate next step in patient management is?

a. laser surgery
b. skin biobsy
c. reassurance that the lesion will go away on it’s own
d. inhaler prescription
e. ENT evaluation

28. person who experience other as either ” all good” or “all bad” and cannot experience an ambivalent relationship
are exhibiting which of the following ?

a.Denial
b.Identification
c.Projection
d.Regression
e.Splitting

30. which of the following brain regions appears abnormal in schizophrenia ?

a. medulla oblongata
b. optic chiasm
c. prefrontal cortex
d. reticular activity system (RAS)
e. third ventricle

31. common side effects of the selective serotonin reuptake inhibition includes :

a. constipation and orthostatic hypotension


b. sedation and priapism
c. restless ,tremors ,seizures .
d. nausea ,insomnia and sexual dysfunction
e. confusion and memory loss

32. posttraumatic stress disorder differ from adjustment disorder in that PTSD:

a. accurse in veterans
b. characterized by impairment of social functioning
c. persists long after stress has abated
d. is characterized by preoccupation with the stress
e. can be accompanied with depression

33. A 30y old man complains of panic attacks and anticipatory anxiety .which of the following drugs would be an
effective treatment

a. carbamazepin
b. fluoxeten
c. haloperidol
d. meprobamate
e. pentobarbital
35. which of the following side effects associated with antipsychotic medication can be lethal and requires the most
emergent treatment ?

a. oculogyric crises
b. torticollis
c. trismus
d. opisthotomus
e. laryngeal dystonia

38. borderline personality disorder :

b. prevalence in female
d. best treatment choice is antidepressant

39. most distinctive of malingering from factitious disorder is:

b. secondary gain
e.response to confront

40. what is the peak onset of schizophrenia in men?

a. 22 - 27 years
b. 24 - 31 years
c. 17-25 years
d. 26-45 years

41. which of the following is a good prognostic indicator for schizophrenia?

a. prominent affective symptoms


b. insidious onset
c. ventriculomegaly
d. negative symptoms

42. which of the following drugs has been shown to decrease the suicide rate ?

a. carbamazepine
b. olanzupine
c. sertraline
d. lithium
e. valproic acid

43. a 48 year -old male patient with difficult to treat depression has tried all the anti depressants except MAOIs. He
is currently on an SSRI. Which of the following SSRI s needs the longest wash out period before switching a patient to
an MAOI?

a. paroxetine
b. fluoxetine
c. citalopram
d. sertraline
e. escitalopram
44. Which of the following tricyclic anti depressants closely resembles an SSRI in its action mechanism?

a. amitriptyline
b. nortriptyline
c. imipramine
d. clomipramine
e. doxepine

45. a 70 year old man with a dementing disorder diese in a car accident during the previous 5 years his personality
had changed and he had caused much embarrassment to his family because of his intrusive and inappropriate
behavior .he had frontotemporal atrophy gliosis of the frontal lobe`s with matter , characteristic intracellular
inclusions, and swollen neurons . amgloide placks………………. Which of the following is the most likely diagnosis?

a. Alzheimer disease
b. pick disease
c. creutfeldt-jakob disease
d. B12 deficiency dementia
e. HIV dementia

48. most common cause of delirium in elderly?

c. hypoxia
d. multiple medications

49. 58 year old female with chronic mental disorder , comes to the physican with irregular choreathetoid movement
of her hands and the trunk she states the movement get worse under stress full condition , which medication to cause
this disorder:

a. fluxetin
b. clozapen
c. perphenazin
d. diazepam
e. phenobarbitol

50. 25 y / m has a chief complain of depression for 1 month , his mother who he was close to had died 1 month ago
and since then he is very tearful has difficulty concentrating ,lost 1.5kg , and has insomnia .What is the diagnosis?

a. major depression
b. dysthemia
c. PTSD
d. Adjustment disorder
e. Uncomplicated bereavement

51. A 27 years old male feeling blue for the past two weeks, he has little energy and troubles concentrating. Six weeks
ago he had been feeling very good, this pattern has been occurring for the past three years, though it never was so
severe, what is most likely the diagnosis?

a. Border line disorder


b. Seasonal affective disorder
c. Cyclothymic disorder
d. Major depression, reccurent
e. Bipolar disorder, depressed
52. A 32 years old male treated for major depression which of the following symptoms if present is one of the most
accurate indicators of a long term suicidal risk?

a. Revenge fantasies
b. Presence of range in the patient
c. Hopelessness
d. Presence of guilt
e. Patient has a need for punishment

53. A 33 years old male comes to his physician for an HIV test, the test is positive, he has no symptoms of AIDS. Which
of the following psychiatric diagnosis may develop in approximately 25 percent of patients informed of positive HIV
test?

a. Adjustment disorder with anxiety


b. PTSD
c. Bipolar disorder, manic
d. Panic disorder
e. Hypochondriasis

54. Dysthymic disorder:

a. Usually begins in adulthood


b. Does not respond to antidepressants
c. Presents with symptoms of lack of say in life and preoccupation with inadequacy
d. Usually limited to one or two episodes
e. Characteristically marked by psychomotor agitation or retardation

55. Most common cause of delirium within three days post operatively in a 40 years old man with a history of alcohol
dependence is:

a. Stress of surgery
b. Post operative pain
c. Pain medication
d. Infection
e. Delirium tremens

56. True anorexia (A.N) :

a. very common in older children and adolescent


b. medical complication include irregular heart beat constipation and osteoporosis
c. medical evaluation of pt with A.N doesn’t include ECG
d. delusions about food in schizophrenia are also usually concerned with the caloric content of food .
e. OC behaviour is very uncommon in ( A.N) patients

57. levodopa causes mania and psychosis supports which neurochemical hypothesis ??

a. norepinephrin
b. dopamine
c. glycine
d. serotonin
e. glutamine
58. After being struck on the head by a 4X4 piece of wood , a previously serious and dependable construction worker
starts inappropriate sexual remarks to his co-workers, is easily distractible and losses his temper on minor
provocation. What is the damaged part?

a. Occipital lobe
b. Temporal lobe
c. Limbic lobe
d. Basal gang.
e. Frontal lobe

59. 8 y /boy is constantly clearing his throat and blinking his eyes for 3 weeks ,he has symptoms intermittently for
several years never free of them for more than a day or tow . what is the best Tx ?

a. Alprazolam
b. methylephedrine
c. haloperidol
d. amitriptyline
e. lithium

60. 8y/boy complains of night bed wetting , behavioural interventions were not helpful , what is the best treatment of
choice ?

a. Ritalin
b. Paroxetine
c. Trazodone
d. Bezotropine
e. Imipramine

61. a 37 yr male is having dental calculus and severe gingivitis.his past medical history ispositive for anaphylactic
shock reaction to penicillin and a ventricular septal defectthat is completly recured during childhood.what is the most
appropiate prophylactic treatment perior to dental procedure?

a.there is no need for prophylactic treatment.


b.amoxicillin PO.2g. 1 hour perior to dental procedure.
c.vancomycin IV.1 g,1 hour perior to dental procedure.
d.azithromycin PO,500mg,1 hour perior to dental procedure.

64. VI delivery risk in woman with dispnea and orthopnea:

a. risk depends on cardiac size


d. life threatening CMD

66. which of the following is not associated with weight gain as common side effect in the following drugs :

a. aspartate insuline
b. glimepiride
c. metformin
d. rosiglitazone
67. Which of the following is true regarding troponin testing :

a. in acute coronary syndrome the mortality rate increases in proportion to troponin level.
b. minor increase in troponin level is diagnostic for MI even if no clinical symptoms of angina
c. troponin levels returns to normal in 24-48 hours and can be used to asses the time of the infarction
d. increase in troponin level can rule out pulmonary embolism or myocarditis

68. which of the following patients should be tested for bone mineral density :

a. 60 y.o. female that was involved in a motorcycle accident


b. 60 y.o. male without any history
c. 60 y.o. female with obesity
d. 60 y.o. female with L4 compressing found on incidental x ray

69. which of the following is true about UTI :

a. 2 weeks I.V. treatment in a young uncomlicated acute pyelonephritis


b. 2 or more episodes of UTI requires urological evaluation
c. 10x3 (ten in the power of 3 ) bacteria level is diagnostic .
d. pseudomonas aeroginosa is the most common community acquired UTI

70. which of the following increases the clinical suspect of ankilosing spondilitis in a patient with chronic back pain:

a. 40 y.o. female
b. reduced pain with excersize
c. lack of morning stiffness
d. normal MRI of sacroiliac joints

71. Which of the following results in hyponatremia with no osmolarity changes?

a. Hypoglicemia.
b. Hypertrigliceridemia.
c. Manitol treatment.
d. compulsive drinking.

72. a 72 years old male complaning of exercicional dyspnea. Echo - severe mitral regurgitation, left ventricle volume is
normal (end diastolic volume 35 ml) and ventricular function is preserved - EF - 55%.
Which of the following is correct?

a. Systolic murmur augments during valsalva.


b. Because bigger left ventricle is expected an EF of 55% is considered low.
c. medical therapy and observation should be sufficient since ventricular function volume and ejection function are
normal.
d. during surgery complete valve replacement should be attempted.
73. Usually healthy 40 years old male does not take medicine _____ complains of epigastric pain mainly at night, that
is allevied by food as well as recent diarrhea up to 4 times / day. Upper endoscopy demonstrates 3 gastric ulcers and
2 ulcers in the first portion of the duodenum. Which of the following tests will offer the highest probability of
diagnosis?

a. TSH
b. H. Pylory breast test.
c. 24 urine 5-HIAA
d. Gastrin levels.

74. The use of proton pump inhibitors can results in:

a. Vitamin C deficiency.
b. Vitamin D deficiency.
c. Vitamin B12 deficiency.
d. Folate deficiency.

75. 34 years old female is being evaluated for polydipsia and polyuria. Serum and urine glucose are normal, but urine
is highly dilluated - Volume 8 litters/day. Serum Na+ - 140 mEq/L, on water deprivation test - serum Na+ - 148 mEq/L,
even thou the patient does not drink at all - the urine output and osmolarity unchanged. 1 hour after sub cutis
administration of anti diuretic hormone (Vasopressin) - the hourly urinary volume returns to normal.
What is the most likely diagnosis?

a. Test is normal, no disease.


b. Nephrogenic diabetes insipidus.
c. Central diabetes insipidus.
d. Primary polydipsia.

76. A 60 year old female is a candidate for diagnostic /therapeutic PTCA due to chest pain . she has to fasting 12 hours
prior to the procedure . past medical history includes dm-type 2 and treated with metformin . simvastatin and
amlodipin which of the following medications should be discontinued n pre-operation for the procedure :

a. All
b. none
c. Metformin
d. Amlodipine

77- a 60 year old male with severe asthma is treated with ventolin inhalations and prednisone at a dose of 10 mg/day
for about a year the patient is involved with a motor accident on examination in the ER there is a central obesity .
moon face , a thin fragile skin , he is somnolent cold (rectal temp. 35c) and the blood pressure 80/40 the orthopedic
surgeon diagnosis a closed fracture of the ankle and abdominal ultrasound rules out abdominal bleeding .
which of the following should be included in this patient immediate treatment:

a. whole blood infusion


b. iv thiamin 100 mg
c. iv naloxone
d. iv hydrocortisone 100 mg
78. 86 year old female is independent in a nursery home . two years ago she fractured her hip and now she fell down
and fractured a ramus pubis . x-rays demonstrate lower back compression which is the recommended treatment:

a. Calcium only
b. B- vitamin d and biphosphonates
c. Calcium, vitamin D and biphosphonates
d. Calcium +vitamin D as biphosphonates were proven to increase mortality in the elderly

79. 17- years old male develops peripheral edema and dark urine, 10 days following severe tonsillitis , which of the
following laboratory results is expected:

a. Positive ANCA
b. Low c3 levels
c. Low c4 levels
d. High Ig-a levels

80. which of the following are correct regard EBV (ebstein barr virus):

a. Infection in adolescent is most commonly asymptomatic


b. A rash is present in most patients
c. Liver function tests are elevated in the minority of the patients
d. Infection may cause airway obstruction and spleen rupture

81. Which of the following is not true regarding Celiac disease?

a. It is possible to make a diagnosis based on serology alone


b. Diabetes type 1 is common in patients with celiac
c. The lesion is more commonly present at the proximal segment rather than the distant segment of the small bowel
d. The disease may present as an iron deficiency anemia

82. A 40 year old man, non smoker, is coughing for 9 weeks. Which one of the following will not cause the coughing?

a. Asthma
b. Interstitial lung disease
c. GERD
d. Post nasal drip

83. Which one of the following cannot be diagnosed with bronchoscopy?

a. Sarcoidosis
b. Mesothelioma
c. Lung infection
d. Lung cancer

84 A usual healthy 40 y.o .male that doesn't smoke is being evaluated by his physician due to coughing for 9 weeks.
All of the following are common causes of his complaint except:

a. Reflux (GERD)
b. Interstitial lung disease
c. Asthma
d. Posterior nasal drip
85. A 50 y.o. male is admitted due to severe lower extremities edema. Abdominal examination: Liver is not palpated,
spleen is enlarged and ascitis is present. Labs: pancytopenia, PT48 (normal 60-100%). Abdominal ultrasound identified
nodular liver. His evaluation includes all of the following except:

a. Hbs-Ag (HCV surface antigen)


b. Anti HCV (antibodies to HCV)
c. IgM-HAV (antibodies to HAV)
d. anti-smooth-muscule antibodies

86. thirty year old female evaluated for viral hepatitis blood test due to general weakness .labs :antiHBc positive
,antiHBs positive, HBs IgE negative, normal liver function test, which of the following is correct regarding this patient :

a. acute HBV INFECTION


b. HBV infection recovery
c. HBV recovery
d. HBV carrier
e. HCV carrier

87 .which of the following is correct regarding hypertension:

a. in the adult population the diastoli BP is better diagnostic predictor of moridity than systolic

88. 16 year old undergo routine blood test CBC normal, ALT 21, AST 18, direct bilirubine 4, total bilirubin , 40 LDH
280, alkaline phosphatase , 260 which of the following is corret:

a. Is most likely that this conditiond will not require any further treatment or evaluation

89. 71 year old develops dyspnea after infusionof one unit of FFP, CXR demonstrated bilateral interstitial infiltrates
,furosemide trial is not effective, the patient was treated with O2 and recovered after several hours, there is clinical
suspicion of transfusion related acute lung injury whats of the following is correct:

a. the cause of the reaction is an antibodyfor HRI PROTEIN of granulocytes surface

90. 75 year old female with osteoporosis due to compression fructure of the lumbar vertebra, the patient was
examined by othopedic surgeon that recommended immoilisation for two weeks and alendronate treatment
,aweeks later the patient complains of mid chest pain that worsens during meals, what is the most likely diagnosis:

e .esophagitis

91. a 55 year old male admited due to dry cough and fever up to 39 for 3 days. Several days ago he was discharged
from the oncology department were he was treated with chemotherapy , on exam respiratori rate 24 /min heart rate
98/min blood press 106/72, room air saturation is 88%. chest x ray demonstrated bilateral perihilar interstitial
infiltrats, broncoscopy with BAL is preformed with preliminary positiv silver staining result:

a. amphotericin
b. piperacillin/tazobactam
c. trimetoprin/metronidazol
d. penicilin
92. A 77 year old recently diagnosed with multiple mieloma awaiting treatment arives to the ER for sleepines ,
confustion , weaknes no feaver , lab total protein 57, albumin 30, deacres in kidney function, urianalysis ok. What is
the correct treatment?

a. hydration , panidronat and steroids


b. brod spectrum antibiotics to cover gram positiv bacteria
c. intensiv chemo in an age appropriat dose
d. plasmapheresis to correct blood viscosity

93. Which of the follow statements is correct regarding IgA nefropathy:

a. it dosnt apear after age 20


b. it has progresiv corse and in the half of the pathints will develop end stage renal failure
c. all tipes of IgA antibodies are increast
d. proteinuria or renal function deterioration are indications for treatment with ACE inibitors

94. A 68 year old female with prostetic valve endocarditis is treated 10 days with gentamycin and penicilin due to
strep viridant that is penicilin sensitiv [mic 0.01] dispite treatment she will has high fever and complaints of nocturnal
dispnea, on day 10 of treatment 2 blood colturs are still positiv for s.viridant. What is most appropriate treatment:

a. valve replacment surgery


b. chage antibiotics for ceftriaxon and gentamycin
c. continue treatment unless hemodynamics unstable
d. continue treatment if vegetation is smaller than 10 mm

95. A female with scleroderma being evaluated in clinic for the first time , she is found to have elevated BP 160/90
which will support diagnosis of scleroderma renal crisis:

a. normal renal funzion test


b. blood smear with RBC fragments
c. urianalysis without blood or protein
d.anti centromer antibodies

96. A 45 year old male with htn+ hyperlipidemia is diagnosed with diabetes type 2 creatinine 150Mmol\l 60\110
urinalysis – no blood - protein + 4 , urine sediment is with no abnormal findings , proteinuria 10g/d find examination
without diabetic retinopathy , which of the following is correct ?

a. In the case proteinuria is not due to diabetes because the time period till proteinuria in diabetic patient at least 10
years .
b. Diabetic nephoropatic does not exceed 3- 4g/d
c. Lacke of retinal eye changes roles out the diagnosis of diabetic nephropathy
d. Half of the patients with proeinuria of >500 mg/ dl will develop and stage renal failure with 5- 10 years

97- a usually healthy 32 year old female that is one week after normal delivery is having Melena and subcutaneous
bleeding cbc ,normal platelet count ,normal pt , ptt prolongation , what is the most diagnosis ?

a. Disseminated intravascular coagulation


b. Factor 8 acquired antibody
c. Congenital coagulation factor deficiency
d. Vit k deficiency
98. 30 year old diabetic, male is a hospital nurse he has an induration of 14 mm following , a.ppd (Monteux ) test, he
feels with no complains his physical examination and chest x-ray are normal what is the recommended ?

a. There is noncd for treatment as ,a ppd of less than 15 mm is considered negative


b. The test is positive and there for year is an indication isoniazid treatment for 9 months
c. The positive but there is no room for treatment since there aren-respiratory complaints and chest x-ray is normal
d. The test is borderline and therefore should be repeated

99. a 14 year old female requires urgent platelet infusion - on type and screen she is RH negative (d negative ) and
antibody screen
is (-) blood bank is currently lacking d- platelets which of the following is correct ?

a. D- platelets should not be given to a- d patient even emergencies


b. There are no side effects of giving d+ platelets since here anti bodies screen is (-)
c. Platelets infusion cannot cause the development of anti d -
d. Anti d injection should be given to the patient following d+ platelets infusion

100- a 30 year old female has 2 months of symmetrical poyarthitis involving small joints of the hands and wrist , one
examination normal author poyarthitis , labs - cbc , biochemistry urinalysis all normal crp elevated , rf - , ana+ , anti-
ccb + , which of the following is correct ?

a. The patient has overlap syndrome


b. Because rf - the patient should be treated with steroids alone
c.Test should be repeated anti count p + in the presence of - rf
d. The patient is at high risk for articular bone erosions

101. asthma. dispnea. ventolin x 2 / day:

b. cortison x 2 / day
d. prophilactic beta agonists

102. In which of the patients the potassium level may be low?

a. 27 year old male with HIV that has dyspnea and bilateral pulmonary infiltrate on prophylactic with Pentamidine
b. 50 year old male that has initiated treatment with statins for hyperlipidemia, and is complaining on severe pain and
shin tenderness
c. 47 year old male with weakness, abdominal pain, hypotension and hyperpigmentation on the palms
d. 20 year old bone marrow transplanted male with candida infection, treated with Amphotericin

103. 19 year old soldier is refered to the ER due to headaches, muscle pain and fever of 39 that appears 5 days after
outdoor hiking.On physical examination- maculopapular rash that involves the palms and the feet as well as tache
noir of the neck. Which treatment should be given?

a. penicillin
b. resprim
c. quinine
d. doxycycline
104. 75 year old female is treated for 7 days with ceftriaxone due to pneumonia. Treatment result in normal
temperature and respiratory status improvement. On the 9th day she has new fever (39) , abdominal distension and
diarreha. WBC-39000 ( normal 4000-10000). what sould be the treatment?

a. Discontinued ceftriaxone and initiate oral Metronidazole


b. Discontinued ceftriaxone and perform short colonoscopy
c. stool culture and add IV Vancomycin
d. Discontinued ceftriaxone and perform US of bile duct

105. NSTMI:

b. PTCA
c. Bypass

106.Which of the following is most common causes of pneumonia in AIDS patients:

a. pneumocysitis dirvecs
b. mycobacterium tuberculosis
c. Streptococos pneumonias
d. klabsiella pneumonia

107.A 21-years old male is brought to the ER after he was found unconscious following heroin injection.on physical
examnation –BP is 175/100 MMHG,heart rate is 110 per min,t-38.8 with tendrneas of the extremites muscle of
palpation.LAPS are:ceart-350 mmol/l(n60-110) BUN-15 mmol/l(N3.5-75).k 6.0 mmol/l,urine sg 1.0 PH 5.0,protein-
trace,blood -4 Urine sediment:1-3 RBCs/hpt,1-3 WBCs/hpf,many pigmented cagt. what is the most probable causes
for renal failure of this patients:

a. prerenal azotemia
b. chronic glumerulanephritis
c. Rhabdomyolsis
d. HIV nephropathy

108. A 25-years-old male is now treatment with amepradex only after he was diagnosed with doudenial ulcer a month
ago and was treated for h-pylori irradication .he interested in travelling next week to south pole for 6 month and he
want to confirm bacterial irradication. what the most appropriate method of doing in this setting?

a.Breath test
b.blood serology
c.gastric biopsy
d.gastric content culture

109. A 66-years-old diabetic male, with no know heart disease, exercises regularly (30 min x5ld)and does not smoking
on several blood pressure measurements his average BP. Is 145/89.LDL level is 65which of the following is correct
regarding this patient?

a. He should be encouraged to continue his healthy lifestyle with no change in his medication needed.
b. as for as blood pressures he is in prehypertinsion category and the is no indications for blood pressure medications
c. LDL level should be decreased and BP should be decreased to below 120/75
d. there is no need to decreases the LDL.BP should optimally be decreased to be below 130/80.

110. ulcerative colitis drug interactions:

b. allopurinol
d. prednisone
03.03.2011
By: Alhag Abu Anzeh Muhammad

Part A:

1. The most common neurological squela associated with bacterial meningitis in children is:

A. mental retaridation
B. chronic seizure disorder
C. impaired vision
D. impaired hearing
E. behavioral disturbance

2. The most common cause of viral meningoencephalitis in children is:

A. An enterovirus
B. Herpes simplex virus
C. An arbovirus
D. dmumps virus
E. A respiratory virus

3. A 12-year old girl experiences acute monocular blindness of 2 days duration past medical history reveals that she
has had headaches for the past 3 years that she cannot charatcerize one brief episode of diplopia, and one episode of
parasthesias of the feet. The episodes were hot related in time, did non occur in immediate proximity to the
headache, and resolved spontaneously. Finding of physical examination, including the funduscopic exmination, are
unremarkable other than reduced visual acuity. The most important diagnostic step is to perform:

A. CT scan
B. MRI
C. A electroencephalogram (EEG)
D. peripheral nerve conduction test's
E. A sural nerve biopsy

4. Which of the following is the etiology in most cases of mayasthenia gravis?

A. inheritance as a rcessive trait .


B. inheritance as an X- linked trait .
C. postinfectious usually after either influenza orchickenpox.
D. autoimmune disorder
E. idiopathic

5. A 3-year old boy presents to an urgent care clinic with a 3-day history of abdominal pain and difficulty walking,
Abnormal findings include blood pressure of 120 /80 mm Hg .diffuse abdominal tenderness and swelling of the
ankles. The most likely diagnosis is:

A. Systemic lupus erythematosus


B. Kawasaki's disease
C. Juvenile rheumatoid arthritis
D. Henoch-schonlein purpura
E. Stevens-johonson syndrome
6. A 12 year old boy presents with 1 year history of worsening polyuria and a 2 to 3 wk history nausea fatigue and
malaise serum creatinine is 4.0 mg dl bicarbonate is 15 mg dl. Urinalysis shows specific gravity of 1.004 trace
leukocytes trace blood and no protein with 3-5 WBCs per high-power field, 3-5 RBCs per high-power field and no
RBC casts which of the following is the most likely diagnosis?

A. acute poststreptococal glomerulonephritis


B. chronic interstitial nephritis
C. minimal-change nephritic syndrome
D. acute interstitial nephritis
E. chronic glomurulonephritis

7. Trisomy 21 is associated with?

A. malrotation
B. endocardial cushion defect
C. cleft palate
D. renal disease
E. sensonynural hearing loss

8. What is the most significant complication arising from Kawasaki disease?

A. coronary aneurysms
B. kidney failure
C. gallbladder hydrops
D. gastrointestinal bleeding
E. hypertension

9. Which of the following medication groupings most appropriate for a patient older than 5 years with moderate
persistent asthma?

A. none
B. a daily low dose inhaled corticosteroid
C. a daily low dose inhaled corticosteroid and a long acting inhaled beta 2 agonist
D. a daily oral corticosteroid
E. a daily medium dose inhaled corticosteroid and inhaled nedocromil

10. Peripheral pulmonic stenosis patent ductus arteriosus , retinopaty, cataracts, hepatosplenomagaly,
jaundice and nerve deafenss are the clinical manifestation typically associated with which of the following
congenital defects?

A. toxoplasmosis
B. syphilis
C. rubella
D. cytomegalovirus
E. hiv

11. A newborn has a dysmorphic feature. The pregnancy was complicated by a breach presentation. He shows
decreases movements, polyhydramnios, hypotonia, flat face, flat occiput, epicanthal folds and abdominal
distention. The couse of this is:

A. Trisomy 13
B. Trisomy 18
C. Edward syndrome
D. Trisomy 8
E. Trisomy 21
12. An 11 year old girl is examined because of fatigue, arthralgia and malar rash. Laboratory revels positive
anti nuclear anti bodies. You suspect SLE .What test if positive with be the most specific for diagnose SL:

A. Anti- smith antibodies


B. Anti double strand antibodies
C. Anti phospholipid antibodies
D. Anti cardiolipin antibodies
E. Antinuclear antibodies

13. An 18 month girls is examine because of a blood streak stools .the stool is
grossly positive for hemoccult blood test. She complain of constipation. What is the most likely the diagnosis:

A. Anal fissure
B. Peptic ulcer
C. Mallory Weiss syndrome
D. IBD
E. Necrotizing enterocolitis

14. Galactosemia, a disorder of carbohydrate metabolism, is inherited in an


autosomal recessive pattern .What are the risk of galactosemia in parents who are both carriers of the disorder:

A. 100
B. 75
C. 50
D. 25
E. 10

15. A 3 yr old present to the pediatrician with fever, pallor, anorexia, joints pain, petechiae and
hepatosplenomegaly. The most likely the diagnosis is:

A. Acute lympgenous leukemia


B. Acute Myelogenous Leukemia
C. Juvenile chronic myelogenous leukemia
D. Aplastic anemia
E. Osteosarcoma

16. Neonate born at 28 weeks gestation is now 2 weeks of age. Nasagastric feeds are started. 48 hours after
starting feeds the neonate develops a distended abdomen, bloody stool, pneumatosis intestinalis, and free air on
abdomen radiograph. Laboratory studies reveal thrombocytopenia. The child becomes hypotensive. The
most likely diagnosis is:

A. intestinal obstructions
B. aspiration pneumonia
C. malrotation
D. necrotizing enterocolitis
E. jejunal atresia
17. You are called to delivery room for a routine birth. The infant cries when the cord is cut. You examine the child
under the warmer and notice that when he stops crying, his chest heaves and he turns blue. You are unable to pass
the nasogastric tube through the nose for suctioning. Which condition is the most likely causing the
infants respiratory distress?

A. choanal atresia
B. vocal cord paralysis
C. subglottic stenosis
D. recurrent laryngeal nerve damge
E. laryngeal web

18. A 6-monthe old male infant presents to the pediatrician with arresting heart rate of 50 beat per minute. Physical
examination reveals no rash, and there is no history of rash. On chest radiograph there is no cardiomegaly. The
family history reveals maternal system lupus erythematosus. Which of the following diagnoses is the most likely cause
of bradycardia?

A. lyme disease
B. congenital complete heart block
C. sinus node dysfunction
D. cardiomyopathy
E. sinus bradycardia

19. A12 month-old infant present with hemoglobin level of 7.5 and hemotocrit 22% the mean corpuscular
volume 65 and the adjusted reticulocyte count is 1% . What is the most likely cause of the anemia in this child?

A. iron deficiency
B. chronic disease
C. transient erythrocytopenia of childhood
D. talassemia
E. parvovirus b19 aplastic crisis

20. Feeding between 6-12 months of age characterized by:

A. willing to be fed by a stranger


B. struggle for independence
C. the infant uses a spoon for feeding
D. poor weight gain reflectus a disease
E. the infant object to holding a spoon

21. Growth between 6 and 12 years is characterized by annual weight and height increments of:

A. 3.5 kg , 6 cm
B. 6 kg , 3.5 cm
C. 5 kg , 10 cm
D. 10 kg , 5 cm
E. 1.5 kg , 5 cm

22. Regarding resuscitative efforts in children, the most important goal is:

A. restoration of age appropriate heart rate


B. appropriate movement of the chest wall
C. auscultation of equal breath sounds in both lung fields
D. adequate oxygen delivery and utilization of the body tissues
E. palpation of equal pulses in all four extremities
23. A 9 month old boy is brought to the emergency room in limp and unresponsive
state. Initial examination shows a pulse rate of 35 /min and occasional irregular
breaths. After initiation of cpr (including tracheal intubation) delivary of oxygen via
positive - pressure breaths and chest compression , multiple attempts to insert an
intra - venous line fail. The most appropriate next step in manegment should be:

A. obtain an arterial blood gas sample


B. place in intraosseous needle and administer fluids and intropic agents
C. obtain a "state" head ct study to evaluate reasons for unresponsiveness
D. place transthoracic cardiac pacemaker
E. place a thoracostomy tube to evacuate a possible pneumothorax

24. The most recommended treatment for severe combined immunedeficiency is:

A. gene therapy
B. monthly iv gammaglobulin
C. monthly iv gammaglobylin and interferon-&(gama)
D. monthly iv gammaglobulin and il-8-monoclonal antibody
E. stem cell transplantation

25. Delayed separation of the umbilical cord after birth suggests which of the following types of
immune deficiency:

A. b-cell defect
B. t-cell defect c)combined b and t cell defect
C. phagocytic function defect
D. complement compenent deficiency

26. A 12-yr old white girl present with arthralgia of the knees and elbow and swollen hands of 6 months duration.
She has intermittent fever and has lost 7.5 kg in weight. Other than swollen joints, findings on physical
examination are normal. 3 years earlier, she was found to have thrombocytopenia and was diagnosed with ITP. In
addition, one summer, she had severe sunburn, and 2 tears ago she had mouth sores. Today she has a hematocrit of
25%, positive combs test, and the urinanalysis shows multiple RBC. The most common likely diagnosis is:

A. juvenile rheumatoid arthritis


B. ITP
C. evans syndrome
D. periarteritis nodosa
E. SLE

27. A 75-yr old boy develops progressive symptoms of fever, lassitude, arthralgias,
headache and abdominal pain. Physical examination shows hepatosplenomegally.
Further questioning discovers that he and his family live in a rural area and consume
unpasteurized dairy products in their diet. The the most likely etiologic agent of this
illness is:

A. actinomyces
B. bartonella hensallae
C. brucella
D. francisella tularensis
E. Yersinia enterocolitica
28. A 5-yr old boy is noted by the parents to snore at night. The child has also had problems staying awake in
preschool and has had behaivoral problems. The father also snores. Physical examination of the child reveals
large, pink, nonexudative tonsils. The most appropriate next step is:

A. laryngoscopy
B. polysomnography
C. ambulatory apnea analysis
D. telemetry
E. arterial blood gas analysis

29. The appropriate initial therapy of severe obstructive sleep apnea syndrome in a child is:

A. adenotonsillectomy
B. tracheostomy
C. parapharyngeal muscle surgery
D. theophylline
E. bilerel positive airway pressure

30. A 7 months-old girl present with temp. 38.3 BP 70/30mm, diffuse petechia first noted 4 hours before
presentation. Platelets count 88,000/mm3, and white blood cell count of 43,000/mm3 with 23% neutrophils and
42% bands. The infant has received all recommended vaccination. Which of the following is the most likely
bacterial etiology of this presentation?

A. staphylococcus aerus
B. streptococcus pneumonia
C. neiserria meningitides
D. haemophyllus influenza type b
E. coli o 157:h7

31. A 10 mo-old child presents to the emergency department with a 4 day history of fever, with temp. 39 c, and
watery diareea, just expirienced a generalized seziure. What is the most common syndrome?

A. Sallmonela gastroenteritis
B. Aeromonas gastroenteritis
C. Shigella gastroenteritis
D. Rotavirus gastroenteritis
E. Drug ingestion

32. A 10 years old child prezents to the office with unilateral conjunctivitis, that has been present for 5 days. You
palpate an enlarged periauricular node on the same side. She has no history of recent travel, but her family
introduced a kitten aproximately 2 monthes ago. The most likely etiologic agent responsible for her condition:

A. Borellia burgdorferi
B. Francisella tularensis
C. Bartonella hensalea
D. Staphylococcus aureus
E. Toxoplasma gondii
33. A 2 mo-old infant is suspected of having infant botulism. Which is the best means to confirm the diagnosis?

A. Lumber puncture
B. CT-scan
C. muscle biopsy
D. electoromyography
E. fecal specimen

34. Which of the following is true?

A. ulcerative colitis is typically characterized by rectal sparing


B. crohn’s disease is typically characterized by skip lesions
C. ulcerative colitis typically involves the terminal ileum
D. ulcerative colitis is typically characterized by transmural disease
E. crohn’s disease is typically characterized by crypt abscesses

35. A 9 mo-old infant accidentally ingests unknown quantity of digitalis. Which is the most significant noncardiac
manifestation of toxicity in this child?

A. fever
B. dizziness
C. vomiting
D. visual disturbances
E. urticaria

36. A12 years old boy comes to the emergency department at midnight with a complaint of severe scrotal pain since 7
p.m. there is no history of trauma. Which of the following is the most appropriate first step in his management?

A. order surgical consult immediately


B. order radioisotope scan as an emergency
C. order urinalysis and gram stain for bacteria
D. arrange for an elective ultrasound examination
E. order elective ultrasound examination

37. A 6 yr old boy whose past medical history is positive for three urinary tract infections, presents with a blood
pressure of 150/90 mmhg. He is likely to exhibit which of the following symptoms or signs?

A. multiple cranial nerve palsies


B. headaches
C. hyporeflexia
D. increased urinary output
E. right ventricular hypertrophy

38. A 6 yr old girl underwent removal of craniopharyngioma 3 months previously. She is on thyroid replacement
regimen but her mother reports she is very fatigued. The next step would be:

A. increase the dose of her thyroid medications


B. schedule a growth hormone stimulatiom test
C. measure ACTH and cortisol
D. obtain MRI study of her head
E. measure IGF - 1 and IGFBP - 3
39. During routine screening CBC, a 1 yr old child is noted to have pronounced eosinophilia. Which of the
following is the most likely explanation?

A. bacterial infection
B. chronic allergic rhinitis
C. helmith infestation
D. tuberculosis

40. A 6 month old infant has been exclusively fed a commercially available infant formula. Upon introduction of fruit
juices, the child develops jaundice, hepatomegally, vomiting, lethargy, irritability and siezures. Test for urine reducing
substances are positive. Which of the following is likely to explain the childs condition?

A. tyrosinemia
B. galactosemia
C. fructose intolerance
D. alpha 1 anti tripsin deficiency
E. glucose 6 phosphate dehydrogenase deficiency.

41. 1 mo child is admitted during hot weather after generalized seizure. Head CT is
normal. Family members state that recently they have strated to dilute the child's
formula in order to add water to his diet. Most probably the seizures was caused by:

A. Hyperthermia
B. Child neglect
C. Hypernatremia
D. Hyponatremia
E. Convulsions associated increased temperature

42. All of the following are important in the investigation of tall patients except:

A. Family analysis
B. Plasma factor IGF-1
C. Homocustin
D. Chromosomal analysis
E. Long fatty acids

43. A 12 y, girl, complains on muscle cramps and tingling in hands and feet that are not related to effort. When she
grasps a door handle she can’t release the grasp due to hand spasm. The most important lab analysis is:

A. Blood glucose determination


B. Blood calcium
C. EMG
D. Nerve conduction velocity
E. Arterial blood gas

44. A 16 y girl complains on headaches for the past 13 months, and visual changes for the past 2 weeks. Now she
has galactorrhea. Last menstrual period was 4 mo ago. The galactorrhea is most probably due to:

A. Stress of amenorrhea
B. Elevated prolactin level
C. Elevated estrogen level
D. Migren
E. Adrenal insufficiency
45. 14 y girl is admitted with vaginal bleeding which is longer and havier then her usual menstruation which is
irregular and started 13 mo ago. She is pale, tachycardic. Your next step in diagnosis:

A. Start medroxyprogesteron (provera)


B. Start conjugated estrogen (premarin)
C. Hct determination
D. Estrogen level determination
E. Platelets n’ determination

46. A 9 y, girl, was admitted with developmental delay and regression after normal development until 6 mo. Responds
with fear to loud noise, macrocephalus. On examination decresed visual contact, cheri red spots on retina. The
diagnosis is probably:

A. Tay sachs
B. Goucher’s disease
C. Fabry’s disease
D. Galactosemia
E. Glycogen storage disease type 1

47. A 5 mo, prominent motor delay, hypotinia and cardiomegaly. CK-860U/I. DIAGNOSIS:

A. Spinal muscular atrophy


B. Hypothyroidism
C. Prader willi
D. Pompe’s disease
E. Dawn syndrome

48. Which of the following is an advantage of skin tests (Skin prick test -SPT) compared with the RAST
to identify specific IgE?

A. SPT are not influenced by antihistamin administration


B. Sensitivity of SPT is higher than RAST
C. SPT are semi quantitative
D. SPT are associated with low risk of allergoc reaction
E. SPR are not influenced by dernographics

49. Which is LEAST LIKELY physical sign on examination of child with moderate asthma?

A. Thacypnea
B. Wheezing
C. Clubbing
D. Decresed air conduction on auscultation in right middle lung loe
E. Increased size of anterio-posterior chest

50. Nurse states that a new born baby girl, 3.5 kg after repeated C-sectionwas breathing heavily/hoarsely after 10
min. After birth, on examination there were no heavy breathing, breathing rate- 36/min. O2 saturation-96%. The child
is reactive. Next step:

A. Sepsis investigation
B. Chest x-ray
C. Observation, and if the heavy breathing will start again, hospitalization in new born ICU.
D. Surfactant therapy
E. Positive end expiratory pressure through nasal mask
56. A 72 years old man with acute urinary retention and blurred vision after taking antidepressions for 3 days,
which of these medication has these side effects?

A. venlafaxina
B. paroxetine
C. bupropion
D. nefazodone
E. amitriptyline

57. A8 year-old boy constantly cleaning thorat and blinking eyes for 3 weeks, symptoms intermittently for several
years. Never complete free from symptoms for more of one or two days. First medication in this case:

A. Alprazolam
B. methyphenidrate
C. Haloperidol
D. Amitriptyline
E. lithium

58. Hormones is most commonly used in the adjuvant treatment of depression:

A. progesterone
B. cortisol
C. ACTH
D. levothyrotine
E. Prolactin

59. A72 year-old with long story of recurrent psychotic depression, hospitalized during a relapse. He has prostatic
hypertrophy, coronary heart disease, recurrent orthostatic hypotension. Which of the following is the most
appropriate antipsychotic medication for this patient?

A. chlorpromazine
B. clozapine
C. thioridazine
D. haloperidol
E. olanzapine

60. The BZD actino depend on her interaction with which of the following receptors:

A. GABA
B. Serotonin
C. NMD
D. Dopamine
E. Acetylcholine

61. Which of the following statements is true about fire setter?

A. girls and boys are equally at risk for pathological fire setting
B. fire sartated by children rarely cause any serious damage
C. children younger than 6 are not likely to experiment with fire
D. the prognosis for treated children is exelant
E. a commonly associated feature is lower than average IQ
62. Alexithymia is:

A. an unpleasant mood
B. a loss of intrest in and withdrawal from pleasurable activities
C. an inability to describe or to be aware of emotion or mood
D. a normal range of mood implying absence of depressed or elevated emotional state
E. a state in which a person is easly annoyed and provokated to anger

63. Which of the following drugs is best used to treat acute delirium?

A. chlorpromazine(taroctyl)
B. b)diazepam (valium)
C. haloperidol (halodol)
D. amobarbital (amytal)
E. physostagmen salicylate (antilirium)

64. Which of the following is the most common symptom pattern associated with OCD?

A. obsession of doubt.
B. obsession of contamination.
C. intensive thoughts.
D. obsession of doubt.
E. compulsive hoarding.

65. Medical complication of eating disorder related to weight loss include all of the following except:

A. erosion of dental enamel with corresponding decay


B. braycardia
C. constipation and delayed gastric emptying
D. abnormal taste sensation
E. osteoporosis

66. Anorexia nervosa has mortality rate of up to approximately:

A. 1%
B. 18%
C. 30%
D. 42%
E. 50%

67. The first symptom of ADHD to remit as usually:

A. hyperactivity
B. distractibility
C. careless mistakes in school work
D. impulsivity
E. learning difficulties

68. When an examiner asks a patient to count backwards by 7 starting at 100…:

A. recent memory
B. remote memory
C. concentration
69. A 8 year old boy present for a routine visit, 1 mounth earlier you diagnosed tourette disease and prescribed him
medications. He and his parents reported that the medication is being helpful. Which of the following is the most
common initial symptom of tourette?

A. eye tics, such as blinking


B. facial tics, such as grimacing or licking
C. vocal tics such as throat clearing or grunting
D. whole body tics, such as body rocking or pelvic thrusting
E. self abusive tics, such hitting

70. A 8 year old child referred to the clinic because of bed wetting, several behavioral
interventions have been attempted, including eliminated fluid intake in the evening,
scheduled awakenings at nights to use the bathroom and urine alarm (bell and pad).
These techniques were unsuccessful and the child continued to urinate every night.
Which medication is the most appropriate to prescribe to treat the enuresis:

A. retalin
B. paroxetin
C. trazodone
D. imepramin (primonil)
E. benztropine (cogentin)

71. Which of the following is the most common side effect of mythylphenidate (Ritalin)?

A. tremor
B. Hypotension
C. weight gain
D. insomania
E. liver toxcity

72. A 15 years old is brought in to the pediatricians office by her mother, who is
concerned about her lack of apparent appetite. The patient is wearing baggy clothes
but her weight is in the 70-th percentile for her height. She admits that she "does not
eat much", but she claims to have a voracious appetite. She is active in school
musical productions, and she is very worried about "getting fat" and not getting the
lead part. Her medical history is unremarkable and although she started
menustruating at age 13, she has not had her period for at least 5 mounths. Which of
the following laboratory abnormalities would be the most likely found in patient?

A. Decreased CRH
B. Hyperholesteroemia
C. Hyperglycemia
D. Hyperthyroidism
E. Leukocytosis.

73. A patient reports to you that for the past week ot two he has had the belief that his intestines and his heart have
been removed. When asked about his lack of getting out in the world he responds "what world? There is no world".
This aspect of patients illness would be best reffered to as wich of the following?

A. Schizaaffective disorder
B. Capgras syndrome
C. Folie a' deux
D. Cotard syndrome
E. Major depression
74. A 52 yr old woman who has been treated with medication for 3 years for a
chronic mood disorder reports that although she feels well, she wonders if her
medication is cousing side effects. She complains of dry mouth, trouble urinating and occasional dizziness when
she gets out of the bed. Which of the following medications is the most likely being prescribed?

A. Fluoxetine
B. Imipramid (Premonil)
C. Phenelzine
D. Lithium
E. Divalproex sodium

75. A 38 year old woman presents to your clinic telling that she has had disturbing,
recurrent thoughts about harming her 7 mo old infant. She imagines using a knife and stab her child. Since having
these distressing thoughts, she had removed all sharp objects from her kitchen. Because of this, she has not been able
to prepare meals at home and has chosen to buy fast food or take-out food for family meals. She has not shared
these thoughts with her husband. The most accurate diagnosis for this condition is which of the following?

A. impulsive control disorder not otherwise specified


B. OCD
C. obsessive-compulsive personality disorder
D. psychosis not otherwise specific
E. schisotypal personality disorder

76. F 36 year old man is brought to the emergency department in respiratory arrest. On examination: he is
unresponsible, and the medical student rotating thought the emergency department observers pinpoint pupils and
antecubital track marks. There is suspicion that patient’s condition may be the result of a drugs overdose. The
patient most likely has overdosed on which of the following drugs?

A. cocaine
B. PCP
C. heroin
D. alcochol
E. inhales

77. A 60 year old man has a history of left middle cerebral stroke. Which of the following psychological
disturbances is the most common following such a neurologic effect?

A. anxiety
B. OCD
C. depression
D. mania
E. panic symptoms

78. Postmortem studies and CSF sampling from living patients have revealed a correlation between aggression,
impulsiveness, and suicide, and a low level in the metabolism of which of the following neurotransmitters compared
to control subjects?

A. dopamine
B. GABA
C. glutamine
D. norepinephrine
E. serotonin
79. A 25 y.o. man is brought into the emergency department lethargic and stuporous. He responds only to painful
stimuli, wakes up briefly and yells, then goes back to sleep. Ambulance personal report that they found him near a
house known for drug trafficking. There is no evidence of physical injury. Which of the following medications should
he receive first?

A. dextrose and flumaztnil


B. dextrose, flumazenil, and naloxone
C. dextrose, flumazenil, naloxone, and thiamine
D. dextrose and naloxone
E. dextrose, naloxone and thiamine

80. A social worker makes a routine visit to a 3 y.o. boy who has just been returned
to his biological mother after spending 3 month in foster care as a result of a severe
neglect.the child initially appears very shy and cling fearfully to his mother. Later on, he starts playing in a very
destructive and disorganized way. When a mother tries to stop him from throwing block at her, he starts kicking and
biting. The mother becomes enraged and start shouting. Which of the following is the most likely diagnosis for this
child?

A. oppositional defiant disorder


B. ADHD
C. reactive attachment disorder
D. posttraumatic stress disorder
E. major depression

86. A23year old,gravida1,para0,hase andergone Colposcopy for evaluation of high grade lesion found on pap
smear.The squamocolumnar junction was visible in its entirely,and the endocervical curettage was normal . A
direct biopsy of the cervix revealed a 1mm focus of invasion. The next best step is:

A. Radical trachelectomy
B. cryotheraby of cervix
C. cold Knife conization of cervix
D. simple hysterectomy
E. Radical hysterectomy

87. The Uterus and the fallopian tubes arise embryologically from which of the following?

A. Mullerian ducts
B. Wolfian ducts
C. urogenital sinus
D. mesonephric ducts

88. In Pregnancy. what is the Chadwick sign?

A. Bluish discoloration of the hyperemic vaginal mucosa


B. lower uterine Segment softening
C. Tenderness of breasts with enlargement
D. uterus palpable above the pubic symphysis

89. What is the average weight gain during normal pregnancy?

A. 5,5 kg
B. 9,5 kg
C. 12,5 kg
D. 15,5 kg
90. At what gestational age can examiner typically first detect fetal movements?

A. 15 weeks
B. 16 weeks
C. 20 weeks
D. 24 weeks

91. Duration of pregnancy from first day of LMP:

A. 250
B. 260
C. 270
D. 280

92. Pregnant women with which condition should be vaccinated against influenza no matter what stage of pregnancy:

A. Allergic rhinitis
B. Hyperthyroidism
C. Insulin dependent DM
D. ALL pregnant women

93. Severe maternal hypothyroidism has been linked to which:

A. Cretinism
B. Dwarfism
C. Hypogonadism in children
D. Limb reduction deformity

94. Definition of reactive non stress test:

A. 1 acceleration in 20 min
B. 2 acceleration in 20 min
C. 8 acceleration in 20 min
D. 15 acceleration in 20 min

95. During vibroacustic stimulation testing what fetal response is measured?

A. Breathing
B. Heart beating
C. Eye movement
D. Body movement

96. Which of the following is NOT part of the management of shoulder dystocia?

A. Woods screw maneuver


B. fundal pressure
C. McRoberts maneuver
D. delivery of posterior shoulder
97. What is the approximate incidence of breech presentation at term?

A. 0.5%
B. 3%
C. 7%
D. 12%

98. An 18 year old consults you for evaluation of disabling pain with her menstrual
periods. The pain has been present since menarche and is accompanied by nausea and
headache. History otherwise unremarkable, and pelvic examination is normal. You
diagnose primary dysmenorrheal and recommend initial treatment with which of the following?

A. Ergot derivatives
B. Anti-prostaglandin
C. Gonadotropin-realsing-hormone (GnRh) analogs
D. Danazol
E. Codein.

99. In an amenorrheic patient who has had pituitary ablation for a craniopharyngioma, which of the following
regimens is most likely to result in an ovulatory cycle?

A. Clomiphene citrate
B. Pulsatile infusion of GnRH
C. Continuous infusion of GnRH
D. Human menopausal or recombinant gonadotropin
E. Human menopausal or recombinant gonadotropin followed by human chorionic gonadotropin (hCG)

100. Which of the following medications is used first line therapy in the treatment of pelvic pain endometriosis?

A. unopposed estrogen
B. Dexamethasone
C. Danazol
D. Gonadotropins
E. Parlodel

101. The presentation of Asherman syndrome typically involves:

A. Amenorrhea
B. Oligomenorrhea
C. Menorrhagia
D. Metrorrhagia
E. Dysmenorrhea

102. During the evaluation of secondary amenorrhea in a 24-year-old woman, hyperprolactinemia is diagnosed.
Which of the following conditions could cause increased circulating prolactin concentration and amenorrhea in this
patient?

A. Stress
B. Primary huperthyroidism
C. Anorexia nervosa
D. Congenital adrenal hyperplasie
E. Polycystic ovarian disease
103. Luteal phase defects are ovulatory disorders. Which of the following studies performed in the second
half of the menstrual cycle is helpful in making this diagnosis?

A. Serum estradiol levels


B. Urinary pregnanetriol levels
C. Endometrial biopsy
D. Serum follicle-stimulating hormone (FSH) levels
E. Serum luteinizing hormone (LH) levels

104. A 22-year-old woman consults you for treatment of hirsutism. She is obese and has facial acne and hirsutism
on her face and periareolar regions and a male escutcheon. Serum LH level is 35 mIU/mL and FSH is 9 mIU/mL.
Androstenedione and testosterone levels are mildly elevated, but serum DHAS is normal.
The patient does not wish to conceive at this time.
Which of thefollowing single agents is the most appropriate treatment of her condition?

A. Oral contraceptives
B. Corticosteroides
C. GnRH agonist
D. Bromocriptine
E. Wedge resection

105. A 23-year-old woman presents for evaluation of a 7-month history of amenorrhea. Examination discloses
bilateral galactorrhea and normal breast and pelvic examination. Pregnancy test is negative. Which of the
following classes of medication may be the cause of her condition?

A. Antiestrogens
B. Gonadotropins
C. Phenothiazines
D. Prostaglandins
E. GnRH analogues

106. Which of the following findings characterizes a normal semen sample?

A. Leukocyte count > 1 milion per ml


B. Sperm concentration of 35 milion per ml
C. 4% normal sperm morphology
D. 10% progressive sperm motility
E. A volume of 1 ml

107. You see five postmenopausal patients in the clinic. Each patient has one of the conditions listed, and each
patient wishes to begin hormone replacment therapy today. Which patient would you start on therapy
immediately?

A. Mild essential hypertension


B. Liver disease with abnormal liver function tests
C. Personal history of brest cancer
D. Undiagnosed genital tract bleeding
E. Treated stage 3 endometrial cancer
108. The most important indication for surgical repair of a separate uterus is?

A. Habitual abortion
B. Dysmenorrhea
C. Menometrorrhagia
D. Dyspareunia
E. Premature delivery

109. Danazol used in the treatment of endometriosis causes which of the following changes within the
endometrium and endometriosis tissue?

A. Dysplasia
B. Hypoestrogenism
C. Hyperplasia
D. neoplasia
E. inflammation

110. A 25 yr old woman, gravida 4, para 4, with a history of leiomyomas, presents to the emergency department
reporting pelvic presure. She denies cardiac, renal, or hepatic symptoms. A pelvic ultrasound shows a 10 cm left
uterine mass that has the echogenicity of fibroid, pressure from the fibroid may also cause?

A. Leg ulcer
B. Femoral nerve pressure
C. superficial thrombophlebitis
D. petechial rash
E. varicose veins
Part B:

1. A 27 year old woman, gravida 2, para 1 at 30 week of gestation presents to clinc for a routine prenatal visit.
She has known to suffer from "serosal fibroids". Her fundus measures 37cm from the pubis.In discussing
possible complications of a fibroid uterus during pregnancy you mention that she is at highest risk for:

A. preterm premature rupture of membranes (PPROM)


B. Placental previa
C. Pregnancy induced hypertension (PIH)
D. Breech presentation
E. placental abruption

2. Which of the following patients is unlikely to have endometriosis:

A. A 19-years old with cyclic pain and bicornuate uterus with a non communication uterin hor
B. A 28 year- old patient with cyclic pelvic pain and who has a mother and sister with indometriosis
C. A 25 year old femal with a history of dysparunia, painful nodular masses in the rectovaginal septum
an a left adnexal mass
D. A 28 year old with menorrhagia a 4 cm submucosal myoma
E. A 32 year old with infertility and dysmenorrhea an a fixed and retroverted uterus on physical
examination

3. A 25year-old woman primigravida is in the emergency room comlaining of lower pelvic pain and spotting for
the past week.Her last normal menstrual period was 7 weeks ago.you have obtained a serum B-hCG. Which was
4000 IU\L and a transvaginal ultrasound was performed wich revealed no gestational sac in the endometrial cavity.
No adnexal masses and no free fluid in the cul de sac. The next step in the management of this patient is:

A. Reassurance and repeat B-hCG in 14 days


B. Laparoscopy
C. Laparotomy
D. methotreaxate,singl dose therapy
E. Dilatation and curettagge

4. A 14 year old nulligravid girl reports menstrural bleeding every 45 to 50 day and bleeding for 4 day.she
experienced menarche at age 13 she is not sexually active.Her physical examination is unremarkable and her
serum pregnancy test is negative.The next best step in management is:

A. Estrogen only pills


B. reassurance
C. NSAIDs
D. Hysteroscopy and dilatation an curettage
E. dilatation and curettage
F. coagulation profile

5. A 46-year old multiparous woman just underwent a hysterectomy becaus of a molar pregnancy.other than her
treatmente for gestational trophoblastic disease,you obtain a B-hCG two days after the operation.What is the next
step in management of this patient?

A. B-hCG in on week
B. Methotrexat treatmente
C. liver function test in two weeks
D. pelvic CTscan
E. chest radiograph in one month
6. A 26 year old woman gravid 4 , para 4 is happily married she is factor V leiden
mutation carrier she has regular periods that last 9-10 days are extremely heavy and
are associated with severe cramping she is fairly sure she has completed childbearing,
which of the following contraceptive metods will be the most reliable and suitable for her?

A. copper IUD
B. progesterone IUD-MIRENA
C. combined oral contraceptive pills
D. vaginal ring NUVARING
E. condom

7. A 32 year old woman gravid 3 para 3 just delivered a viabile female infant weighting 4000 grams via cesarian
section for nonreassuring fetal heart rate pattern, she recived intrathecal spinal anesthetic and nacratic for pain relief
durig the procedure , her foley catheter is left in place for several hours after the cesarian section this will prevent?

A. stress incontinence
B. urge incontinence
C. overflow incontinence
D. mixed incontinence
E. preoperative urinary tract infection

8. A 55 year old woman with her FMB at age 50 present with a history of 3 days of light vaginal bleeding you should?

A. give her vaginal estrogen for atrophic vaginitis and tell her to come back if the bleeding doesn’t
get better
B. perform a hysterectomy and bilateral salpingo-oophorectomy to rule out endometrial cancer
C. take a history perform a physical examination perform endometrial tissue sampling and order a pelvic
ultrasound or perform hysteroscopy
D. recommend she go on a diet since there is increased production of estrone in obese women
E. start ET instead of HT since a progestogen may make hor bleed

9. A 20 year old presente to you with a deep excavating painless lesion above the clitoris overlying the pubic
bone. Her serum VDRL is positive. A lumbar puncture and analysis of her cerebrospinal fluid also yields a
positive VDRL the best term to describe her lesion is:

A. condyloma acuminatum
B. condyloma latum
C. chancre
D. gumma
E. bubo

11. A 48 year old woman is being evaluated for cough that persist for 3 month. She
describe that it occur daily, Nonproductive, with no hemoptysis. She have no
dyspnea, wheezing, fever, weight los , no night sweats or recent illness. So travels
recently. She wasn`t exposed no anyone who have been ill. She never smoke. She
was diagnosed with essential hypertension 6 months ago and take lisinopril daily.
Physical examination is unremarkable. No oral /pharyngeal exudate or drainage.
Chest x ray is normal. Which is the most appropriate management option for this patient this time?

A. Discontinue lisinopril
B. Chest CT
C. Spirometry
D. Start antihistamine / decongestion combination
E. Start proton pump inhibitors
12. A 51 years old woman is evaluated during a routine exam. she have no history of hypertension , no tabaco use
, and no family history of heart disease .she take conjugated estrogen combined with medroxyprogesterone acetate
for intolerable hot flashes .physical examination is normal, BMI 31. Fast lipid panel: Total cholesterol 218 mg/dl, HDL
42 mg/dl, LDL 128 md/dl, triglyceride 240 mg/dl. The next appropriate step is:

A. Calculate the framingham risk factor


B. Calculate the non-HDL level
C. Prescribe atrovastin
D. Prescribe gemfibrozil

13. A 68 years old woman is being examined. Last year she had a painful rash in the
side of her back that was self-limited. She doesn’t recall a history of chickenpox. She
takes no medication, no allergy. Her vital sign are normal, and physical examination
is normal. Her CBC, liver enzyme, and serum chemistry are normal. She has
scheduled an influenza vaccine for today. The vaccine strategy for today is:

A. Zoster vaccine if negative for varicella anti body


B. Zoster vaccine if positive for varicella anti body
C. Zoster vaccine now
D. Zoster vaccine now + 6 months
E. Zoster vaccine is not indicated.

14. A 60 year old woman has a persistent pain after an episode of herpes zoster 3 month ago, involving the right side
T4 dermatome. She claim that the pain interfere with her sleep. History is normal. Vital sign are normal. The lesion is
crusted over and appears heals. The most appropriate treatment for this patient is:

A. Acyclovir
B. Codeine
C. Gabantin
D. Ibuprofen
E. Topical ….

15. A 36 yr old. With low HDL. No chest pain. History of 10 pack a year smoking. .
She is overwight with no coronary disease history. BP133/82 mm HG. BMI 29. Heart
examination is normal with no murmur or gallop rhythm. Total cholesterol is 198
mg/dl, HDL 33, LDL 129. Triglyceride 18. The management of this patient is:

A. Exercise stress test


B. Gemfibrozil
C. Nicotinic acid
D. Pravastatin
E. Lifestyle modification
16. A32-years-old man has a 5 day history of persistent nasal congestion and pain in
the right forehead area associated with a clear nasal discharge and mild cough. The
patient reports that he has had similar episodes in the past that were helped by
antibiotics. Medical history is otherwise unremarkable and he currently take no
medication .on physical examination, vital sings including temperature are normal.
Mild right suborbital ridge tenderness is present the nares are patent with a clear
mucoid discharge. There is no pharyngeal erythema or exudate. The lungs are clean
to auscultation which of the following is the beast initial management?

A. amoxicillin
B. CT scan of the sinuses
C. plain films of sinuses
D. symptomatic treatment
E. trimethoprim-sulfamathoxazole

17 .Which of thefollowing it true regarding hypovolemic shock?

A. loss of 20-40% of blood volume leads to shock physiology


B. loss of <20% of the blood volume will manifest as orthostasis
C. oliguria is a crutcial prognostic sign of impending vascular collapse
D. symptoms of hypovolemic shock differ from those of hemorrhagic shock
E. the first sign of hypovolemic shock is mental obtundation

18. A 28-years-old man is evaluated for 2 month history of recurrent confusion,


palpitation, and diaphoresis occurring whenever he misses breakfast. The symptoms are relieved with eating. His
medical history is otherwise unremarkabl and he takes no medications. On physical examination vital signs are
normal and BMI is 26.the rest of the general physical examination is normal .results of laboratory studies show a
fasting plasma GLU 52 mg/dl and insulin 18u/ml (normal 2-20). Which is the most appropriate next step in diagnosis?

A. CT of the abdomen
B. endoscopic ultrasonography of the pancreas
C. home glucose monitoring
D. octreotide scan
E. supervised 72-hour fast in the hospital

19. A 45 year old man is evaluated for a 3 month history of fatigue, constipation and poluria he also has a 5-year
history of hypertension .current medication are losatan and diltiazam. Physical examination finding including
vital signs are normal. Laboratory studies: ca 11.4 creatinine 1.1 mg/dl GLU fasting 88 mg/dl TSH 1.2 . Measurement
of which of the following should be done next?

A. calcitonin
B. 25 hydroxy vitamin d
C. parathyroid hormone
D. parathyroid hormone related protein

20. A 68 year old women is re-evaluated after laboratory studies show a fasting glu
113 she has a maternal family history of type 2 diabetes mellitus. On physical
examination ,blood pressure is 142/88 and BMI 29 other vital signs and examination
findings are normal .she undergoes an oral tolerans test during which her 2 hour
plasma glu increases 135 Hemoglobin A1C 5.8% LDL 110 HDL 48 Triglyc 172
Which of the folloing is the most appropriate treatment recommendation to control her glu ?

A. acrabose administration
B. diet and exercise
C. metformin administration
D. ramipril administration
E. rosiglitazone administration
21. A 45 year old woman is evaluated for a 6 - months history of weakness,
menstrual irregularities hirsutism, Insomnia and emotional lability. She also reprts an
8-kg weight gain during this period. She was previously healthy. She takes no
medications. On physical examination, temperature is 36, blood pressure is 172/190
mmhg, pulse rate is 68/min, respiration rate is 16 /min, and bmi is 32. The patient has
a rounded, plethoric face with increased supraclavicular and dorsal fat pads. There
are areas of unexplained eccymoses over the upper and lower extremities. Abdominal
examination reveals purpule striae. She has proximal muscle weakness. Results of
routine laboratory studies are normal except for serum potassium level of 3.4 meq/l.
Which of the following is the most appropriate next test for this patient?

A. cosyntropin stimulation test


B. high dose dexamethasone suppression test
C. measurement of morning serum cortisol level
D. measurement of 24 - hour urine free cortisol rxcretion

22. A 67 year old woman is transferred to the cardiothoracic intensive care unit (icu_ after undergoin repair of an
abdominal aortic aneurism. she has a -12 year history of type 2 diabetes mellitus. Her blood glucose level arraival
at the icu is 289 mg/dl . although no longer on cardiopulmonary bypass pump treatment to- control her blood
glucose level during her icu stay:

A. insulin glargine , once daily


B. intravenous insulin infusion
C. neutral protamine hagebro(nph)insulin , twice daily
D. regular insulin administered on sliding scale

23. A76 year old woman is reevaluated after results of thyroid function tests,
performed 2 weeks ago are abnormal. The patient otherwise feels well. She has a
history of hypertension, atrial fibrillation, gastroesophageal reflux disease and
depression. Current medication are metoprolol, amiodarone , warfarin , omeprazole, and sertraline. Respiration
rate 15/min .the thyroid gland is smooth and normal size. Cardiac examination reveals an irregular rhythm. Deep
tendon reflexes are normal.
Laboratory studies:
Tsh 6.5 u/ml
Thyroxine (t4), free 2.4 ng/dl (31 pmol /l)
Triiodothyronine (t3), free 0.8 ng/l (1.2 pmol/ l )

Which of the following medications is most responsible for the laboratory result?

A. amiodarone
B. metoprolol
C. omeprazol
D. sertraline
24. An 18 year old woman is evaluated for tachycardia, nervousness, decreased
exercise tolerance and weight loss of 6 months duration, she has otherwise been
healthy. Her sister has graves disease. She takes no medications. On physical
examination, blood pressure 128/ 78 mmhg. Pulse rate is 124/min, respiration rate is 16/min and bmi is 19.5.The
is no proptosis. An examination of the neck reveals a smooth thyroid gland that is greater than 1.5 times the
normal size. Cardiac examination reveals regular tachycardia with grade 2/6 early systolic murmur at the base. Her
lungs are clear to auscultation .laboratory studies:
Human chorionic gonadotropin - negative
Tsh < 0.01 u/ml
Thyroxine (t4), free 5.5ng/dl (71 pmol/l)
Triiodothyronine (t3) free 5.5ng/dl (71 pmol/l)

Which of the following is the most appropriate treatment regiment at this time?

A. atenolol only
B. methimazole only
C. atenolol and methimazole
D. radioactive iodine and methimazole

25. Crohn's disease is associated with which of the following:

A. inflammation limited to the superficial layer of the bowel wall


B. specific affinity to involve the rectosigmoid junction
C. continous mucosal areas of ulceration that affect the anus
D. fistula formation

26. An obese 75 old women is evaluated for persistent hyperglycemia. She has
followed a strict regimen of diet and exercise in an attempt to control her
hyperglycemia. Home blood glucose monitoring has shown preparendial levels
between 120 and 160 mg/dl and occasional postparendial levels exceeding 200 mg/dl. She has a history of
hypertension and hyperlipidemia. Current medication include lisinopril, hydrochlorothiazide, and pravastatin. Vital
signs and physical examination findings are normal. Except BMI of 30.the serum creatine level is 0.8 mg/dl and the
urine is negative for micro albuminuria.Which of the following is the most
appropriate step in treatment to improve her glycemic control:

A. continue the diet and exercise for additional 3 months


B. begin exenatide
C. begin glimepiride
D. begin metformine
E. begin pioglipazone

27. A 32 year old female physician is beginning a post graduate fellowship at a


university hospital and must undergo tuberculin skin testing. She is healthy, she grew
up in Africa and completed medical school and residency training in London. She
received BCG vaccine as a child. tuberculine skin testing results indicate a 16mm
area of induration at the tuberculine skin testing site. Physical examination is normal.
Which of the following us the most appropriate next step in the management of this patient?

A. chest radiograph
B. isoniazide, rifampin, pyrizonamide and ethambutol
C. repeat tuberculin kin testing in 2 weeks
D. no additional therapy or evaluation
28. A 75 year-old men with type 2 diabetes mellitus is evaluated in the emergency department for draining chronic
ulcer on the left foot, erythema and fever. Drainage initially began 3 weeks ago. Current medication include
metformin and biguanide. On physical examination he is not ill appearing. Temp is 37.9, other vital signs are
normal. The left foot is slightly warm and erythematous. A plantar ulcer that is
draining purulent material is present over the fourth metatarsal joint. A metal prob
makes contacts with bone. the reminder of the examination is normal. The leukocyte count is normal and an ESR
rate is 70mm /h. A plane radiograph of the foot shows numerous leukocytes, gram positive cocci in clusters, and
gram negative rods. Which of the following is the most appropriate management now?

A. Begin imipenem
B. Begin vancomycin and ceftazidime
C. Begin vancomycin and metronidazole
D. Perform bone biopsy

29. A 27 year old previously healthy woman is diagnosed with community acquired
pneumonia, for which she receives ceftriaxone therapy. Medical history is otherwise
non contributory. Routine and mycobacterial sputum clusters are performed. The
patient recovers uneventfully over the subsequenttwo weeks and a follow up
radiograph shows improvement. The routine sputum culture revels only normal flora,
however after 3 weeks the mycrobacterial cultures groves 2 colonies of
mycobacterium avium complex. Which is the most appropriate next step in management?

A. initiate clarithromycin, rifampin and ethambutol


B. initiate isoniazide
C. initiate rifampin, pyrizonamide and ethambutol
D. no further treatment

30. A19 years old male college student has 2 day history of urethral discharge and
burning sensation on urination. He has had 3 female sexual partners in the past
month. Physical examination discloses a scant mucoid discharge from the urethral
oriffice. No rashes or other lesions are seen. Gram stain of the urethral secretions
shows greater than 12 polymorphonuclear cells/ HFP. No intra - or extra-cellular
organism are seen. Which of the following is the most appropriate treatment?

A. acyclovir orally
B. azithromycin orally
C. benzathine penicillin G intramuscularly
D. cefixime orally
E. metronidazole orally

31. A 65 year-old man is evaluatesd in the ER for a 3 day history of gradualy


worsening low back pain and fever. He was evaluated by his internist 4 weeks ago for these symptomes, but his
condition has recently worsened. On phisical examination 38c, other vital signs are normal. General examination
including neurological examination is normal. MRI of the spine shows inhancement of the L3-L4 endplates
and inflamatory collections are seen. The patient is hospitalised. Blood cultures are
negative. The patient’s pain has worsened. The most appropriate next step, in the management of this patient is:

A. begin ceftriaxone
B. begin nafciline
C. CT-gaided percutaneus niddle biopsy
D. repeat blood cultures
32. A 65 year-old male is evaluated because a screen urine culture for insurance
policy grows more then 10⁵ colony forming ut/mL E coli. There is no fever, disuria, urinary frequency or other
symptomes. Medical history is unremarkble. There are no allergies, no medications. Physical examination is
normal. What is the most appropriate treatment for this patient?

A. amoxiciline
B. ciprofloxacine
C. trimetoprime- sulfamethoxazole
D. no treatment

33. A 25 years-old woman is evaluated in urgent care because of the recent onset of
heel pain, that is especially sever when jogging. She has been takin ibuprofen for the
past 7 days. Her additional medications are low-dose oral contrraception, that she has
been taking for the past 5 years and multivitamines. She does not smoke sigarettes, is
otherwise healthy, there is no history of hypertention. Physical examination reveals
162/102 mmHg, puls 90 beats/min, BMI 24. The remainder of the examination, including cardiopulmonary,
fundoscopic and neurological examination, is normal.
Which of the following is the most appropriate management of this patient’s hypertension?

A. begin captopril
B. begin hydroclorothiazide
C. begin labetolol
D. discontinue ibuprofen

34. A 35 years-old male comes for a new patient evaluation. He takes no


medications. His parents both have diabetes mellitus. On physical examination blood presure 160/100 mmHg,
BMI . The remainder of the examination is unremarkble. Lab studies, including serum electrolites, BUN, ceatinine
levels, urinalisis are normal. In addition to lifestyle modifications, which is the most appropriate next step in this
patient’s management?

A. lisinopril and hydroclorothiazide


B. metoprolol and hydroclorothiazide
C. terazosine
D. no therapy

35. A 22 years-old woman is evaluated at an on-site medical center after callapsing


while running a marathon. She’s disoriented. During the evaluation she expiriences a generalised tonico-clonic
seizure lasting 3 minutes. A wristband indicated diabetes mellitus. On physical examination, temperature is normal,
blood presure 120/60 mmHg, puls rate 100, respiratory rate 28. There is no evidence of hypovolemia or
edema. Cardiopulmonary examination is normal. On neurological examination she is confused, but has no
evidance of focal neurological deficit. On laboratory examination glucose is 120 mg/dl, sodium 118 mg/dl. What is the
most appropriate next step in this pacient’s management?

A. 3% saline infusion
B. 5% glucose by i.v. bolus
C. i.v. furosemide
D. normal saline infusion
36 A 56 years old man presents to your office with complains of “chronic” diarrhea.
He states that he has had loose stools for the last 2 days. He denies blood in the stool, fever, and has no weight loss
and no recent travel. Appropriate management at this time includes:

A. observation
B. check stool cultures
C. colonoscopy
D. stool fat studies

37. A 60 yr old man is evaluated during routine examination. The patient has smoked a pack of igarettes for 25
years. He has tried to quit smoking with nicotine gum and varenicline, but has been nsuccessful. He has mild
airway obstruction, but has good exercise tolerance and no cough, sputum roduction or hemoptysis. The
importance of smoking cessation is reviewed with the patient….

A. 18F- fluorodeoxyglucose and positron emission tomography (FDG-PET)


B. chest radiography
C. spiral CT of the chest
D. sputum cytology
E. no screening

38. A 44 years old is evaluated for a 6 month history of dyspnea, regurgitation of sour fluid and eructation. There is
not associated fever, chills, weight loss or vomiting. The condition failed to respond to 6 weeks of omeprazole
theraphy. The patients medical history includes hypertension, diabetes mellitus and obesity (BMI 36). Her
mediacations are lisinopril, methformin and insulin glargine. On examination vital
signs are normal, there is mild epigastric tenderness without rebound and stool is
negative for occult blood. Which of the following is the most appropriate next
diagnostic step in the evaluation of this patient?

A. ambulatory esophageal PH monitoring


B. barium esophagography
C. CT scan of the chest
D. esophageal manometry

39. A 25 year old man is evaluated for 6 month history of eight loose, non bloody
stools a day. He also has abdominal pain and small joint arthritis. The patient has a 5 year history of chrons
disease, initially treated with corticosteroids but then maintained on azathioprine therapy. Small bowel radiographic
series shows no stricturing disese but som, e active jejuno ileitis. Azathioprine metabolites were
recently measured and were found to be at therapeutic concentration. Which of the following is the most
appropriate additional therapy for this patient?

A. 5- aminosalycilates
B. anti tumor necrosis factor alpha inhubitor
C. calcineurin inhibitor
D. corticosteroids
40. A 72 year old man is evaluated for 2 month history of epigastric discomfort
associated in the past 6 weeks with 4.6kg weight loos. He has dark urine and light
stool. The patient has no significant medical history and takes no medications. On
physical examination vital signs are normal, there is scleral icterus, visible jaundiced
skin and mild epigastric tension. Laboratory studies are significant for total bilirubin
of 8.2 mg/dl and alkaline phospatase of 648 U/L. CT scan of the abdomen shows
fullness in the head of the pancreas and biliary dilation but no evident pancreatic
mass.which of the following is the most appropriate next step in the management of
this patient?

A. endoscopic ultrasonography
B. measurment of CA 19-9 concentration
C. surgical exploration
D. ultrasonography of the abdomen
E. visceral angiography

41. A 47 y W, is admitted for primary examination. No complaints and no


pathological findings on examination. Her father died from MI at the age of 71. The
mother is 81 y old and still alive. There is no cancer in the family history, but a friend of the patient was diagnosed
with cancer of the large intestine and she is concerned regarding her risk to develop the same. On examination the PB
and the pulse are normal, BMI=25.5. No other findings. Laboratory analysis is normal. Which of the
following is the most appropriate investigation for early detection of cancer of the
large intestine in this patient?

A. Annual digital rectal examination


B. Colonoscopy every 10 years
C. Barium enema with double contrast every 3 years
D. Flexible sigmoidoscopy every 10 years

42. 72 W, 72 years is being investigated for complaints about increasing pain in her
right knee for the past year. The pain is more severe in the medial part of the joint,
aggravated by stress and decreases during rest. She has no morning stiffness and
swelling. She felt no jamming or recoil in this joint. On examination blood pressure
and pulse are normal, there is an increase of bone at the proximal and distal
interphalangeal joints. No signs of effusion in the right knee joint. Passive flexion
and extension of the right knee cause pain. Tests, including complete blood count,
ESR and C-reactive protein were normal. X-ray of the right knee joint is normal. In
addition to acetaminophen as needed, which of the following is most appropriate next
step in treating this patient:

A. Arthroscopy
B. Knee joint puncture
C. MRI of the knee joint
D. physiotherapy
43. A 32 years male is undergoing investigation for pain and stiffness at the lower
back, which is (the pain) decreased by excersise and hot shower for the past 10 years.
No history of dermatological diseases, ophtalmological or intestinal as well. He did
not suffer from infections of the gastrointestinal or genitourinary systems. On
examination blood pressure and pulse -normal. Sacroiliac joints and lumbar spine are
painful upon in palpation. There is a complete loss of bending forward at the lower
spine. Standing straight against the wall, he cannot touch the wall with the back of his
head. X-ray images reveal complete fusion of the sacroiliac joints on both sides and a
square shape of vertebral bodies throughout the lumbar and thoracic spine. In
addition to initiation of NSAID’s and physiotherapy, the next most appropriate treatment for this patient is:

A. etanercept
B. low dose prednisone
C. metothrexate
D. sulfasalazine

44. Which of the following is most closely related with Julian Barre syndrome?

A. normal EMG at the late phase of the disease


B. descending asymmetrical paralysis
C. predisposing trauma
D. the symptoms usually begin at the lower extremities
E. low levels of protein in CSF (CEREBROSPINAL FLUID)

45. What can be caused with the use of Proton Pump Inhibitors (PPI)?

A. Vit. C deficiency
B. Vit. D deficiency
C. Vit. B12 deficiency
D. Folic acid deficiency

46. A 78 y/o arrives to urgent care clinic with new onset of chest pain. He describes it as crushing in theleft
substernal area. Non radiating, present for 14 hours. No prior episodes. He has history of hypertension and
hyperlipidemia. He is taking aspirin, hydrochlorothiazide, atorvastatine. b.p. 100/70 on right arm , 105/72 on left arm.
Pulse 100. Respiration rate 16/. no jugular vein distension , no murmurs , no rubs. lungs are clear , no peripheral
edema. Laboratory: platlets 120.000, serum creatinine kinase 450, troponin 60. Initial ecg shows st segment elevation
of 2mm in ii, iii, and avf also st segment depression in v2 v3. Chest x-ray appears
normal. Which of the following is the best managment for this patient?

A. chest c.t. with contrast


B. echocardiogram
C. primary percutaneous intervention
D. thrombolytic therapy
47. A 59 y/o male, evaluated for chest pain, sometimes accompanied by a burning
sensation, which begad about 4 weeks ago. The pain comes during periods of both
excertion and rest. He has a known history of hypertension for which he takes
nifedipine. The patient recently sustained a knee injury that prevented him to walk
without a cane. On physical examination he is found to be afebrile, b/p 138/86, pulse 74, respiratory rate 16, body
mass index 28.no signs of heart failure. Laboratory: cholesterol 210, ldl 140, hdl 40, creatinine 1.2, cardiac enzymes
negative. ecg shows LVH , with normal sinus rate. And no changes from prior ecg. Which is the most appropiate test
to perform next?

A. adenosin positron emission tomography stress test


B. cardiac catheterization with coronary angiography
C. coronary artery calcium score
D. c.t. coronary angiography
E. nuclear perfusion imaging stress test

48. Which of the following is not a diagnostic criteria for diabetes mellitus?

A. random plasma glucose > 200


B. fasting plasma glucose 140
C. abnormal glucose tolerance
D. HbA1C of 7.5

49. A 65 y/o male complains of 2 months of chest pain with excertion. Pain is
relieved upon rest. He has excertional dyspnea, orthopnea, lower extremity edema. He has 25 years history of
hypertension, he smokes 44 years and he take only ydrochlorothiazide. He is afebtile, b/p 118/80, pulse 95,
respiration rate 16/ he has mild jugular vein distension. Cardiac examination presents regular rate and rythm, S1 S2
sounds, no murmurs and no s3. Crackles are heared at both lungs bases. He has mild bilateral edema at his ankles.
Troponint less than 0.01 ecg is normal on echocardiogram he has ejection fraction of 40% he had global hypokinesia
and mild LVH. Which is the most appropriate diagnosis test?

A. cardiac catheterization
B. cardiac mri
C. radionuclide ventriculography
D. nuclear medicine stress test
E. standard excersize stress test

56. A 33 year-old woman trying to be pregnant, for 8 years is evaluated of her


receiving positive pregnancy test results. Medical history is significant for 3
miscarriages 6years, 3 years and 1 year ago. Each of which occurred early in her
pregnancies following her test miscarriage, laboratory studies indicated the presence of a lupus inhibitor and
antiphospholipid antibodies. No history of venous thromboembolism. Her last menstrual period was approximately 5
weeks ago. Physical examination is normal. Which of the following is the most appropriate anticoagulation for
duration pregnancy?

A. full dose unfractionated heparin


B. low dose aspirin
C. prophylactic LMWH plus low dose of aspirin
D. prophylactic dose LMWH
E. warfarin
57. A 50 year-old woman. In the ED for 4 days for history of pain, swelling and
erythema the left leg. No recent history of immobilization, cancer surgery or deep
venous thrombosis. Physical examination: temp.37.7°C PAO 132/82, CR 65/min, RR
16/min, left leg disclose warmth and circumference erythema and tenderness limited
on the posterior tibial portion of the leg. The circumference is 1 cm greater than the
right when measured 10 cm below the tibial tuberosity. Localized tenderness along
the distribution of the deep venous system and pitting edema are absent as are venous
varicosities. Which of the following is the most appropriate next step in diagnosis?

A. CT scan of the leg


B. D-dimer assay
C. Duplex ultrasonography
D. MRI of the leg
E. venography

58. A 28 year-old man is evaluated at a community center of health for 10 days


history of sore throat, headache, fever anorexia and muscle aches. 2 days ago a rash
developed on his trunk and abdomen. Previously healthy and has had no contact with
ill persons. He has had multiple male and female sexual partner and infrequently uses
condoms. He has HIV test several times, all were negative. Physical examination,
38.6°C, several small ulcers on the tongue and buccal mucosa, cervical and
sopraclavicular lymphoadenopathy faint maculopapular rash is present on the trunk
and the abdomen. Rapid plasma test ordered. Which diagnosis studies should also be done?

A. CD4 cell count measurement


B. EBV IgM measurement
C. HIV RNA viral load measurement
D. Skin biopsy

59. A 28 year old man, 6 month history of episodic dyspnea, cough and wheezing. As a child he has asthma and
allergies, but has been asymptomatic since his early teenage years. Symptoms started after upper resp. tract infection
and they are trigged by exercise or cold air. He awakened with asthma symptoms 5- 6 nights a month. No medication
and otherwise is healthy. Physical examination is normal, rx chest normal, FEV1 70% of predicted with 15%
improvement with inhaled albuterol. Which is the most appropriate therapy?

A. Aziathromycin
B. inhaled albuterol as needed
C. inhaled low-dose of corticosteroid plus inhaled albuterol as needed
D. long act β-agonist
E. long act β-agonist inhaled albuterol as needed

60. A 42 year-old man hospitalized evaluated for dyspnea plus pleuritic chest pain,
fracture of the right femur 3 weeks ago. Suffer of hypertension with the only medics is hydrochlorothiazide.
Physical examination, 38.1°C, HR 110/min, RR 22/min, PA 130/78, BMI 24; laboratory studies are normal,
toponins undetectable. EKG, elevation light of R waves in V4-V6, QRS has left ward axis. Contrastenhanced CT scan
shows emboli in the arteries perfusing lingual and posterior basal segment of left lower lobe. Most appropriate
treatment:

A. inferior vena cava filter


B. IV unfractionated heparin
C. IV tissue plasminogen activator
D. mechanical clot dissolution
E. surgical embolectomy
61. A patient undergoes a segmental mastectomy and negative margins are obtained, two sentil lymph node are
identified.on final histological evaluation a 6mm region of macrometastases is found on H&E staining within two
of the lymph node, the next step should be:

A. completion mastectomy
B. completion axillary dissection
C. axilary dissection and breast radiation therapy
D. systemic chemotherapy alone
E. no further axillary operation

62. A post-menopausal woman with new diagnosis of breast cancer in her left breast elects to undergo
mastectomy. Which of the following statement about contralateral prophylactic mastectomy is true?

A. overall mortality is improved


B. there is a survival advantage for patient with higher stage cancer
C. the incidence of contralateral breast cancer is significantly reduced
D. sentenil lymph node biopsy should be done bilaterally
E. reconstrication should be delayed until final histopathology is available

63. A patient with cirrhosis has large esophageal varices but has never had bleeding episode. He is unable to
tolerate a non selective beta blocker, prevention of an initial bleeding episode is best accomplished by:

A. endoscopy ligation
B. TIPS
C. endoscopic sclerotherapy
D. octreotide
E. a small diameter interposition portocaval shunt

64. A 50 y/o man with history of symptomatic GERD has Barrett’s esophagus diagnosed on upper
endoscopy, which of the following statement is true?

A. surgical treatment for symptom of reflux is not warranted


B. Barrett’s esophagus is seen in 50% of patient with GERD
C. PPI will reduce the area of barrett’s esophagus
D. surgical treatment of patient reflux symptoms will not reduce the risk of esophageal cancer
E. endoscopic ablation will reduce the risk of esophageal cancer

65. A 55 y/o man present with his first episode of diverticulitis he is hemodynamicaly normal and improve within
48 hrs receiving antibiotics and nothing by mounth. CT reveals stranding of sigmoid esentry. But no free air or
evidence of any abscess. The next step in treatment should be:

A. resection, anastamosis.
B. resection and colostomy.
C. discharge and elective colectomy.
D. discharge, oral antibiotics and observation.
E. colonscopic intramural stent
66. A12 hours after colonoscopy and removal of 2.5 cm sessile polyp from cercum, 55 years old woman as
abdominal pain, fever, leukocytosis, mind tenderness in the right lower quadrant. No free air in the bdomen. The next
step should be:

A. admission serial abdominal examination


B. exploratory laprotomy
C. abdominal CT
D. coloscopy
E. gastrograffin enema

67. A 48 year old man with left lower quadrant pain. Leukocytosis, low grad fever,
no peritoneal signs. Dmission of CT was interpreted showing acute diverticulitis, no
perforation or abscess. After 72 hours f treatment with i.v antibiotics and fluid the
pain resolved, bowel function and appetite became ormal, leukocytosis resolved, the
next step is:

A. sigmoid colectomy prior to discharge


B. elective colectomy 6-8 weeks after discharge
C. coloscopy prior to discharge
D. observation and medical management after discharge
E. follow-up CT

68. A 55 year old woman with ileocecectomy for chron disease undergos cholecystectomy for symptomatic
gallstones. Diarrhea and stool frequency are increased markedly postoperatively. Next step:

A. bismuth
B. octreotide
C. loperamide
D. cholestyramine
E. panceriatic enzyme.

70. A 65 years old patient has 3 cm rectal polyp. The lower age of the mass is 6 cm from anal verge. Biopsy
shows focus of adenocarcinoma. CT abdomen and chest shows no metastasis. The endorectal ultrasound shows
several perirectal lymph nodes. The best management is:

A. local excision
B. local excision adjuvant chemotherapy
C. total mesorectal excision with coloanal anastromosis
D. neoadjuvant chemotherapy and total mesorectal excision with coloanal anastromosis.
E. neoadjuvant chemotherapy and total mesorectal excision and abdomenoperineal resection.

71. A man presents with repeated episodes of bloody stools. Esophagogastroduodenoscopy and colonoscopy are
normal. The next step in managment should be:

A. Small bowel barium X-rays.


B. CT electrolysis
C. Capsule endoscopy
D. Push and pull enteroscopy
E. Mesenteric angiography
72. A 65 year old man remains in the ICU one mounth after coranary artery bypass
graftting. He has been treated for two episodes of pneumonia and continnues to
require ventilatory support. He develops watery diarrhea and abdominal distention.
Stool sent for clostridium difficile toxin is positive and oral metronidazole therapy is initiated. Five days later, he
developes hypotension requering vasopressors, renal insufficiency, and an elevated WBC count of 36,000/mm3. The
most appropriate management would be:

A. Intravenous Vancomycin
B. Vancomycin enemas
C. Cecostomy tube placement.
D. Diverting transverse loop colostomy
E.

73. A common physiologic characteristic of septic shock is:

A. Elevated central venous pressure


B. High systemic vascular resistance
C. Low cardiac output
D. Systemic vasodilation

74. Most sergical site infectious become clinecaly evident:

A. Whithin 2 days of operation


B. 5 or more days after operation
C. Because of fever immediately after operation
D. Becouse the patient did not have an adequate shave of the operative site
E. A mounth after operation

75. The most common couse of a small bowel obstruction is:

A. Tumor
B. Volvulus
C. Adhesions
D. Ileocolic intussusception

76. An abdominal compartment syndrome produces all of the following except:

A. acute renal failure


B. hypoxia
C. intestinal obstruction
D. elevated urinary bladder pressure
E. hypercarbia

77. A 40 y.o. woman with 1,5 sm palpable brest mass undergoes US guided core needle biopsy. Histology
showed cellular fibroadenoma with possibility of a phylloides tumor. Appropriate management includes:

A. a 6 month follow up US. If lesion size is stable, continue expectant management


B. excisional biopsy to differentiate between the 2 possibilities
C. wide local excision (lumpectomy) with the intent to achieve 1 to 2 sm negative margins around the lesion
D. wide local excision with sentinel biopsy
E. wide local excision with radiation
78. Paget’s disease is caused by which of the following processes:

A. atopic dermatitis on the nipple due to allergies


B. intraductal carcinoma that spreads into the epidermis of the basement membrane
C. invasive carcinoma that causes nipple inversion and then ulceration
D. invasion of the dermal lymphatics under the nipple called Sappy’s plexus
E. a papilloma in a large subareolar duct causing discharge and on the nipple

79. A 60 y/o man suddenly developed severe abdominal pain. In the emergence department 2 hours later his
abdomen was soft and nontender, his white blood cells were 18,300/mm3, and his arterial blood pH was
7,32. The probable diagnosis is?

A. acute appendicitis
B. acute pancreatitis
C. superior mesenteric artery embolism
D. perforated duodenal ulcer

80. A 60 y.o. man comes to the emergency department obtunded and hypotensive with melena and
hematochezia. He has no history of prior surgery or alcohol abuse. His physical examination is unrevealing with
stigmata of liver disease. The most likely source of bleeding is:

A. colonic diverticula
B. small bowel angiodysplasia
C. bleeding duodenal ulcer
D. aortoenteric fistula

86. A 38 old female who is a candidate for a mastectomy due to breast cancer arrives to the clinic seeking consult
regarding breast reconstruction .she is a candidate for adjuvant radiation and chemotherapy. What type of
reconstruction is most suitable for this Patient?

A. Despite the radiation ,she has an indication for immediate reconstruction with tissue expanders in order
to preserve the skin
B. this is an indication for immediate reconstruction using autologous flaps only
C. delayed reconsruction of about 6 month und using autologous flaps are recommended after the
completion of the radiation and chemotherapy
D. there is no option for early or delayed breast reconstruction in a patient treated with radiation

87. A 30 year old male with a gunshot to the abdomen arrives to the emergency room. His last meal was 8
hours ago .The patient is taken emergently to operating room. Which of the following is correct?

A. there is no need for delivering 100% O2 for 5 minutes prior to the induction of anesthesia
B. cricoid pressure should be applied prior to the induction of anesthesia to avoid aspiration
C. cricoid pressure is not warranted as the last meal was over 8 hours ago
D. surgery may be prformed under local anesthesia

88. Which of the following is correct regarding an incarcerated hernia in a 2 year old child?

A. this is indication fo urgent surgery


B. it is associated with undescendend testis
C. an attempt to reduce it is advisable
D. in most of the times it is a congenital direct hernia
89. What is the annual risk for rupture, dissection, and death when the diameter of the ascending aorta exceeds 5cm?

A. 1.5 %
B. 3%
C. 4.5%
D. 6.5%

90. A 36 year old male was involved in motor cycle accident ,he has an isolated injury that includes a fracture of the
right mid femur .what of the following statement is correct?

A. the accepted treatment for this fracture is plate and screws fixation within 24 hours
B. the accepted treatment for this fracture is intramedullary nails within 6-8 hours
C. the accepted treatment for this fracture is external fixation as soon as possiple to avoid the complication fat emboli
this should be followed by permanent fixation in 4-5 days
D. Timing of Surgery is of no importance. It is accepted to operate on such patients according to
operating room availability

91. A 4 y old boy, after fracture fixation with cast at the evening. At night felt pain and finger tingling. What is the next
treatment?

A. Another morphine dose to ease the pain


B. Elevate the arm on 2 pillows
C. Open cast
D. Urgent x-ray

92. A 15y boy, after scroatal injury during football. Complains on severe localized pain. On exam. Testicle firm
and tender. Next treatment/evaluation:

A. Local cooling and scroatal elevation


B. Operative exploration repair of tunica albuginea
C. Urgent Doppler
D. Admission for follow-up on hematoma development

93. 60 y, M, rectal carcinoma 5 cm above anal verge. No metastasis to distant sites. TRUS shows invasion to
perirectala fat (uT3) without regional L.N involvement. Treatment:

A. Radical resection (low anterior) and total mesorectal resection.


B. Combined treatment: neoadjuvant chemotherapy followed by Radical resection (low anterior) and total
mesorectal resection.
C. Radical resection (low anterior) and total mesorectal resection followed by chemotherapy
D. Neoadjuvant + TRUS, CT and surgery if residual tumor is defined in imaging

94. Accepted treatment of malignant cutaneous intermediate thickness melanoma: (Breslow 1-4) on right arm:

A. Wide local excision with 1cm margin from initial lesion


B. Wide local excision with 1cm margin from initial lesion +sentinel lymph node biposy
C. Wide local excision with 2cm margin from initial lesion
D. Wide local excision with 2 cm margin from initial lesion + sentinel lymph node biposy
95. A 55, M, comes to ER with severe epigastric pain. 2 months ago he has completed triple treatment of ulcer in first
part of duodenum with positive H.Pylori. On Chest x-ray air under the right diaphragm is detectible. Next step:

A. Urgent endoscopy
B. Omentopexy
C. Pelvic CT with contrast
D. Antrectomy

96. A 56-year-old female arrives to the ER because of Jaundice. She has no past
medical history. Laboratory finding include a bilirubin of 127 and significantly
elevated cholestatic and hepatocellular enzymes. On abdominal Ultrasound the
gallbladder is distended with no stones. The CBD is 18mm and there is extra and
intra-hepatic bile duct dilatation. Serum CA19-9 Levels are 1200. Abdominal CT
identifies a pancreatic head mass without SMV involvement. What is the next step in this patient’s Management?

A. Gallbladder drainage
B. Resection of the duodenum in the pancreatic head
C. PTC
D. Blood HBV and HCV testing
E. Neoadjuvant chemotherapy

97. An 88-year old male comes to the ER because of excruciating inguinal pain. Two weeks ago he suffered an
acute myocardial ischemia and is taking aspirin since. On examination he seems to be in pain, with no fever, heart
rate of 115 beats pro minute, regular. His abdomen is not distended and there is no evidence of peritonitis. He has an
incarcerated, non reducible, tight inguinal hernia. What is the next step in this patient’s Management?

A. PTCA
B. Thallium stress test
C. Surgery
D. Another attempt to reduce the hernia under deep sedation
E. Abdominal pelvic CT.

98. A 20-year-old male is injured a pedestrian in a motor-vehicle accident. At the ER


is fully conscious. BP is 110/70, her rate is 95. Room saturation is 92%. On
Examination there are abrasions on the right chest and abdomen, deformation of the
right arms and normal peripheral pulses. FAST indentifies small amount of
abdominal fluid, CT indentifies a stage 4 liver laceration with no active bleeding, and
significant right lung contusion. What is the next step in this patient’s Management?

A. Positive pressure ventilation


B. Laparotomy
C. Angiographic Embolization of the liver
D. Intensiv care observation
E. Surgical repair of arm fracture

99. A 38 year old female arrives to the ER with fever of 38.5 and right upper
abdominal pain. A week prior to her admission she underwent laparoscopic
cholecystectomy. On examination she is alert and oriented. Without scleral icterus, and normal appearing surgical
incisions. Libratory finding include Leucocystosis 15,000, slightly elevated liver enzymes, and normal bilirubin,
what is the next step in this patient’s Management?

A. ERCP and stent insertion


B. Abdominal Ultrasound
C. Liver scan
D. Exploratory laparotomy
E. broad-spectrum antibiotics treatment and observation.
100. A usual healthy 35 year old female in complaining of 2 months upper abdominal
pain that radiates to right shoulder after a fatty meal. The pain resolves 30 minutes
after she takes optalgin. No fever or vomiting is associateted. The patient is now in
the ER due to Jaundice for 3 days without fever or abdominal pain. On Examination
the abdomen is not tender. Laboratory finding include - no Leucocytosis, however
liver function tests are elevated with Bilirubin of 80 (normal <17) in abdominal
ultrasound there are stones in the gallbladder, the gallbladder is not distended,
gallbladder wall is not thickened. The CBD’s diameter is 9 mm. What is the next step in this patient’s Management?

A. NPO, IV fluids and antibiotics, follow up on liver enzymes and if they improve laparoscopic
cholecystectomy
B. NPO, IV fluids and antibiotics, and subsequent ERCP and surgery of gallbladder removal.
C. ERCP and surgery of gallbladder removal.
D. EUS or MRCP

101. A healthy 50-year-old male arrives to the ER with lower quadrant pain, vomiting, and abdominal distention.
Abdominal CT demonstrates mass at the hepatic flexure with no small bowel distention. What is the appropriate
treatment?

A. Nasogastric tube, IV fluids, followed by a work up and preoperative evaluation that includes tumor
markers, colonoscopy, and completion of chest CT
B. Urgent right hemicolectomy
C. Stent insertion to relieve the obstruction and preparation for surgery
D. Hartmann's procedure

102. A healthy 70-year-old male is brought to the trauma unit after being involved in a motor vehicle accident as a
seat belt-wearing driver. The collision on in an intercity highway. There are no direct injuries. On examination, BP
is 130/80, heart rate is 110 beats per minute, and there is a seat belt sign across the abdomen. FAST
demonstrates moderate amount of fluid in the pelvis. The patient is taken to a CT scan that identifies moderate
amount of fluid in the pelvis but the abdominal organs appear normal. What is the next step in this patient's
management?

A. Exploratory laparotomy
B. Repeat CT in 2 hours
C. NPO, IV fluids and antibiotics and admission to intensive care unit
D. Repeat FAST in 6 hours

103. A 102-year-old male is brought to the ER because of multiple vomiting and


abdominal distentions. Past medical history is positive for hypertension, benign
prostate hypertrophy, ischemic and valvular heart disease, Alzheimer and Parkinson disease. On examination, the
abdomen is very distended, with suprapubic fullness and tenderness. Stool stones are found on digital rectal
examination. On abdominal X-ray, the colon is full of stool and air without small bowel dilatation.
Which is the next step in this patient's management?

A. Abdominal CT scan
B. Colonoscopy
C. Urinary bladder catheterization and manual fecal disimpaction
D. Hartmann's procedure
104. A 57-year-old female physician that is usually healthy is admitted to the ER due
to progressively abdominal distention and constipation in the last two months. In the
last week her symptoms worsened and her last bowel movement was 10 days ago.
She complains of severe nausea with no abdominal pain or vomiting. On
examination, the abdomen is very distended, tympanic, with no hernias or scars. On
laboratory, hemoglobin - 10.2, WBC - 11,000, and chemistery is normal other than
hyponatremia of 132. On abdominal X-ray the right and transverse colon are much
dilated with no small bowel distention. Which is the next step in this patient's management?

A. Colonoscopy after fast bowel preparation


B. Abdomino-pelvis CT with IV and PO contrast
C. Gastrographin enema
D. Exploratory laparotomy

105. Which of the following factors has the strongest affect on the prognosis of breast cancer patient?

A. The size of the tumor


B. The tumor's histologic type
C. The number of involved lymph nodes
D. The presence of hormonal receptors in the primary tumor

106. A healthy 55 year old male is complaining of constipation and rectal bleeding. He is referred to
colonoscopy that identifies a rectal ulcerated mass 4 cm from the anal verge. Biopsy demonstrates
adenocarcinoma. What is the next step in this patint’s management/treatment?

A. Low anterior resection


B. Abdominoperinal resection
C. Refer the patient to CT and Trus as soon as posible
D. Refer the patient to an oncologist for radiation and chemotherapy

107. What is the most common etiology for small bowel obstruction in adults?

A. incarcerated inguinal hernia


B. incarcerated femoral hernia
C. post operative adhesions
D. crohn’s disease

108. All of the following are compensation mechanisms in hypovolemic shock except?

A. increased peripheral systemic resistance secondery to increased cathecholamins levels


B. increased levels of 2,3 DPG
C. a right shift of the hemoglobin dissociation curve secondery to acidosis
D. decreased cardiac contractility
109. What is correct regarding deep infection evolving a replaced knee or hip?

A. it always develops on the first few weeks following surgery and is usually treated with antibiotics for 10
days
B. it can develop moths*( ‫ ) שלהם<טעות‬and even years following the joint replacement surgery and surgical treatment
and IV antibiotics for 6 weeks are indicated
C. it is a rare complication (less than 0.01%) because smooth metals are involved with surface that repels
infections
D. because of its severity there is an indication for initiation of broad spectrum antibiotics on the most remote clinical
suspicion, even before any test performed

110. What is the main objective in hip or knee joint replacement?

A. avoid deterioration of physiological load on the joints above and below the replaced joint
B. enable athletes, especially professional, to achieve load and stress that the native joint is incapable of
performing
C. avoid intra-joint fracture in patient that are prove for it (diabetics, recurrent falls, dementia)
D. decrease pain and improve the quality of life of patients
04.10.2010
By: Alhag Abu Anzeh Muhammad

Part A:

1. A 5-mo-old infant develops signs of respiratory distress after coughing and sneezing for 3 days. He has marked
subcostal and intercostal retractions and a respiratory rate of 80 breaths/min. Breath sounds are markedly
diminished on both sides. Diffuse crackles can be heard bilaterally. There is no stridor. Arterial oxygen saturation in
100% oxygen by non-rebreather mask is 80%. The skin is pale and peripheral arterial pulses are weak.
Which of the following is the most appropriate immediate course of action?

A. Administration of corticosteroids
B. Intubation of the trachea and mechanical ventilation
C. Sampling of arterial blood and measurement of arterial pH and blood gases
D. Administration of normal saline
E. Administration of diuretics

2. A 2-yr-old boy is seen for his routine check-up, 4 weeks after an episode of bilateral acute otitis media that resolved
uneventfully with antibiotic treatment. He seems generally well, but his mother reports that he is not hearing as well
as usual. On pneumatic otoscopy, both his eardrums appear amber, opaque, and retracted, and both are immobile.
Otherwise his ENT examination is unremarkable.
Which of the following treatment regimens is the most appropriate to institute at this time?

A. A 10-day course of amoxicillin


B. A 10-day course of amoxicillin-clavulanate
C. Short-course treatment with azithromycin
D. Observation without treatment for at least 2 more mo
E. Referral for consideration of myringotomy and tube insertion

3.Conductive hearing loss is:

A. Common in children with chronic ear fluid


B. A permanent condition related to nerve damage
C. Always associated with ossicular abnormalities
D. An uncommon complication of otitis media
E. Unlikely when an intact tympanic membrane is present

4.A 2-mo-old girl has a rapidly growing hemangioma involving the perioral region bilaterally, with symmetric
extension onto the chin and anterior neck.She was born 8 wk premature with an otherwise benign medical history,
although her parents wonder if she may be developing asthma.
The most appropriate next step in patient management is:

A. Laser surgery
B. Skin biopsy
C. Reassurance that hemangiomas will go away on their own
D. Inhaler prescription
E. ENT evaluation
5. A 6-mo-old infant is presented for evaluation of a tuft of thick black hair located on the sacrum. The child's mother
states that the lesion has been present since birth and seems to be asymptomatic, and the child is otherwise healthy.
The most appropriate next step in management is:

A. Reassurance that the lesion will resolve spontaneously in time


B. MRI of the spinal cord
C. Referral to a plastic surgeon for excision
D. Counseling of the parents regarding melanoma risk
E. Watchful waiting

6. A blonde, blue-eyed girl was admitted at 2.5 mo of age with severe emesis to rule out pyloric stenosis. She had
poor development, eczema, and a musty odor. The most likely diagnosis is:

A. Wiskott-Aldrich syndrome
B. Galactosemia
C. Cystinosis
D. Phenylketonuria (PKU)
E. Biotinidase deficiency

7. A previously healthy 2-wk-old now has progressive lethargy. Physical examination reveals muscle rigidity,
opisthotonos posture, periods of hypertonicity,and flaccidity. Laboratory data reveal hypoglycemia, metabolic
acidosis, and cerebral edema. Plasma levels of leucine, isoleucine, and valine are elevated.
The most likely diagnosis is:

A. Hartnup disease
B. Maple syrup urine disease
C. Phenylketonuria
D. Homocystinuria
E. Galactosemia

8. Which of the following cancers has the highest incidence in young children (<7 yr of age)?

A. Ewing sarcoma
B. Hodgkin disease
C. Testicular cancer
D. Retinoblastoma
E. Osteosarcoma

9. Which of the following cancers is the most closely associated with Epstain-barr virus?

A. Osteosarcoma
B. Non-Hodgkin lymphoma (Burkitt lymphoma)
C. Ewing sarcoma
D. Wilms tumor
E. Hepatoblastor

10. An adolescent girl who is a cheerleader comes to you with a painful bump below her right knee. She denies fever
or trauma. Which of the following is the most likely diagnosis?

A. Legg-Calvé-Perthes disease
B. Osteoid osteoma
C. Osgood-Schlatter disease
D. Osteochondritis dissecans
E. Osteomyelitis of the tibial tubercle
11. A 9-mo-old boy is brought to the emergency room in a limp and unresponsive state. Initial examination shows a
pulse rate of 35/min and occasional irregular breaths. After initiation of CPR (including tracheal intubation), delivery
of oxygen via positive- pressure breaths, and chest compressions, multiple attempts to insert an IV line fail.
The most appropriate next step in management should be to:

A. Obtain an arterial blood gas sample


B. Place an intraosseous needle and administer fluids and inotropic agents
C. Obtain a "stat" head CT study to evaluate reasons for unresponsiveness
D. Place a transthoracic cardiac pacemaker
E. Place a thoracostomy tube to evacuate a possible pneumothorax

12. A 12-yr-old girl experienced diarrhea, which lasted for 3 days, 2 wk before manifesting progressive weakness and
inability to walk. She has intermittent tingling of her fingers and toes. Physical examination reveals marked peripheral
muscle weakness without atrophy or fasciculations. The deep tendon reflexes are absent in her ankles and 1+ at her
knees. Findings on the sensory examination are normal. Motor involvement is symmetric. The most likely diagnosis:

A. Transverse myelitis
B. Guillain-Barré syndrome
C. Polio
D. Myasthenia gravis
E. Mononeuritis multiplex

13. A 10 year-old girl has had diplopia and ptosis and weakness of her neck flexors for 2 mo. symptoms are worse in
the evening and are usually less severe on awakening in the morning. she has no fasciculations or myalgias, and her
tendon reflexes are 1-2+. The most likely diagnosis is?

A. Hysterical weakness
B. Muscular dystrophy
C. Spinal muscular atrophy
D. Botulism
E. Myastenia gravis

14. A 12-yr-old boy with cystic fibrosis experiences an acute exacerbation and is admitted for intravenous antibiotic
therapy. Sputum culture reveals Pseudomonas aeruginosa.The antibiotic recommended for treatment is:

A. Ampicillin-sulbactam
B. Ceftazidime
C. Cefotetan
D. Ceftriaxone
E. Ciprofloxacin

15. All of the following are features of scarlet fever except:

A. Pastia lines
B. Desquamation
C. White strawberry tongue
D. Red strawberry tongue
E. Impetigo (Preauricular lymphadenopathy)
16. Months old refuses to walk; began walking at the age of 10 months .Presents with 39 degrees fever, appears
moderately-ill, rotated left hip with limitation to passive and active movement; most likely causing pathogen:

A. Neisseria
B. Adenovirus
C. Staphylococcus
D. Group A streptococci
E. Group B streptococci

17. Sickle cell anemia become asplenic infection risk?

A. Gram negative
B. Encapsulate

18. A 7-days old infant is found to have a significant elevated total bilirubin, but is other wise well . The fractionated
bilirubin reports direct of 3mg/dI and indirect 6mg/dI .the most like diagnosis?

A. Breast milk jundice


B. Hereditary
C. Congenital infection.
D. Gilbert
E. Spherocytosis

19. A newborn develops sepsis and shock. The pathogen that most commonly causes systemic and focal infections in
the newborn is:

A. Staphylococcus aureus
B. Group A streptococci
C. Group B streptococci
D. Escherichia coli
E. Herpes simplex virus

20. Which of the following is associated with a poorer prognosis for persons presenting with meningococcal disease?

A. Presence of petechiae for <12 hr


B. Meningitis
C. Thrombocytosis
D. Leukocytosis
E. Low circulating levels of tumor necrosis factor

21. A previously healthy 18-month-old has been in a separate room from his family.the family notices the sudden
onset of coughing,which resolves in a few minutes.subsequently,the patient appears to be normal except for
increased amounts of drooling and refusal to take foods orally.
Which of the following is the most likely explanation for this toddler's condition?

A. Severe gastroesophageal reflux


B. Foreign body in the airway
C. Croup
D. Epiglottitis
E. Foreign body in the esophagus
22. You have just given a 10 year-old boy an injection of pollen extract as prescribed by his
allergist. You are about to move on the next patient when the boy starts to complain about nausea and a funny
feeling in his chest. You note that his face is flushed and his voice sounds muffled and strained.
Which of the following is the first priority in managing this episode of anaphylaxis?

A. Preparation for endotracheal intubation


B. administration of oxygen
C. subcutaneous injection of 1:1000 epinephrine
D. intramascular injection of diphenhydramine
E. administration of corticosteroids

23. A 15 year-old girl with short stature,neck webbing,and sexual infantilism is found to have.A chromosomal analysis
likely would demonstrate which of the following ?

A. Mutation at chromosome 15q21.1


B. Trisomy 21
C. XO karyotype
D. Defect at chromosome 4q16
E. Normal chromosome analysis

24. A 5-year-old is noted by the parents to snore at night. The child has also had problems staying awake in preschool
and has had behavioral problems. The father also snores.Physical examination of the child reveals large, pink,
nonexudative tonsils.The most appropriate next step:

A. Laryngoscopy
B. Polysomnography
C. Ambulatory apnea monitoring
D. Telemetry
E. Arterial blood gas analysis

25. A 4 year old boy is in the office for a routine health maintenance evaluation. his examination is normal except for
multiple deep dental cavities.you plan on referring him for dental evaluation.his mother tells you he has a "heart
condition". which of the following needs antibiotic as prophylaxis treatment?

A. VSD repaired 8 months ago


B. Tetrollogy Of Fallot
C. History of uncomplicated Kawasaki disease
D. Tranposition of great vessels
E. Ostium secundum arterial septal defect

26. A 4-year-old white girl has had joint swelling in multiple joints for over 6 mo. She is slow to move in the morning
and moves as if stiff for the first hours of the day. Thereafter, she is a very active child. She has no rash and very little
limitation of range of motion. Her erythrocyte sedimentation rate is 4. The most likely diagnosis is:

A. Hypermobility syndrome
B. Dermatomyositis
C. SLE
D. JRA
E. Henoch-Sch nlein purpura
27. Of the following signs of cancer, the most likely to occur in children is:

A. Abnormal discharge from a body orifice


B. Change in a mole or wart
C. Focal neurologic deficit
D. Cough, hoarseness, or difficulty swallowing
E. Change in urination or defecation pattern

28. A 12-yr-old white girl presents with arthralgias of the knees and elbow and swollen hands of 6 months' duration.
She has had intermittent fever and has lost 15 lb. Other than swollen joints, findings on physical examination are
normal. Three years earlier, she was found to have thrombocytopenia and was diagnosed with idiopathic
thrombocytopenic purpura (ITP). In addition,one summer she had severe sunburn, and 2 yr ago she had mouth
sores.Today she has a hematocrit of 25% and a positive result on a Coombs test,and the urinalysis shows multiple red
blood cells. The most likely diagnosis is:

A. JRA
B. ITP
C. Evans syndrome
D. Periarteritis
E. SLE

29. A 13-year-old develops fever,malaise,sore throat,and a dry,backing cough over several days. He does not appear
to be particularty sick,but his chest examination is significant for diffuse rales and rhonchi.The chest radiograph is
shown below. Which of the following is the most likely pathogen ?

A. Staphylococcus aureus
B. Mycobacterium tuberculosis
C. Haemophilus influenzae
D. Streptococcus pneumoniae
E. Mycoplasma pneumonia

30. A 9 year-old patient asthmatic patient uses albuterol 3 times per week,for the last 10 days she had wheezing day
and night ,increased the inhaler to 3-4 times a day,on examination there is a diffuse wheezing with moderate
subcostal retraction, next step in management:

A. order chest XR to asess for pneumonia


B. Systemic steroids
C. Lekutriens
D. Low dose inhalation steroids
E. Start 5 days course of systemic corticosteroid

31. Milk Cow allergy?

D. Check reducing agints in stool


E. Change his diet to an alternative protein source

32. A12 month-old female infant with failure to thrive is bronght to the office her parents note that she is very fussy
and offen spits up after feedings she also has two loose foul-smelling stool lach day.Which of the following foods can
she eat safety without aggravating or inducing her symptoms?

A. Rice
B. Wheat
C. Oats
D. Barley
E. Rye
33. A 6-yr-old girl underwent removal of a craniopharyngioma 3 mo previously. She is on a thyroid replacement
regimen, but her mother reports she is very fatigued. The next step would be to:

A. Increase the dose of her thyroid medication.


B. Schedule a growth hormone stimulation test
C. Measure ACTH and cortisol
D. Obtain an MRI study of her head
E. Measure IGF-1 and IGFBP-3

34. A 10-yr-old boy receiving vincristine for treatment of a malignancy has developed the syndrome of inappropriate
antidiuretic hormone secretion (SIADH), a known complication of this therapy.
All of the following parameters would be decreased in this patient except:

A. Urine production rate


B. Serum osmolality
C. Intravascular volume
D. Uric acid
E. Serum sodium

35. A mother and her 14½-yr-old daughter come to you because the girl has not begun to menstruate. Her medical
history and findings on the complete physical examination are normal. Breast development and pubic hair have been
present for 18 months and are normal.
Which of the following would be the most appropriate next step in the management of this patient?

A. Reassurance that she probably will begin menstruating within the year
B. Laboratory evaluation for systemic disease
C. Urinary estriol determination
D. Buccal smear
E. Referral for psychologic counseling

36. A newborn infant spends about 40 min with the mother but then falls asleep and does not respond to the
mother's voice. Which of the following statements regarding this lack of activity is true?

A. It is suggestive of sepsis
B. It is suggestive of sedation
C. It is normal
D. It represents a seizure
E. It is due to apnea

37. A 4-yr-old boy is noted to have stereotypic body movements and poor verbal and nonverbal communication,with
absence of empathy. At daycare, he has not made any friends.The most likely diagnosis is:

A. Attention deficit hyperactivity disorder


B. Dysthymic syndrome
C. Deaf-mutism
D. Autism
E. Cerebral palsy
38. The probable age of a child who rolls back to front, has a thumb-finger grasp, self-inhibits to "no" and bangs two
cubes is:

A.10-12 mo
B.7-8 mo
C.12-15 mo
D.3-4 mo
E.15-18 mo

39. m/o, feeding intolerance, poor-weight gain and large tongue; large posterior fontanelle and umbilical herni.
Next step:

A. Abdominal radiography
B. CBC and blood culture
C. Barium swallow test
D. Serum TSH level
E. Admission for FTT work-up

40. Small mandible on x-ray no thymos:

B. Di-George syndrome

41. A mother brings her 3 years old son with Down-syndrom to the clinic because his gums have been bleeding in the
last week.she reports that he has been less energetic than usual-Exam reveuls that the child has a temperature of
37.8 c orally pallor, splenomegaly, gingival, which of the following is most likely ?

A. Aplastic anemia
B. Left shift
C. Leukemia
D. Leukmoid rxn

42. A 30 hour-old full term infant has face and chest jaundice,he is breast feeding well. And has an otherwise normal
exam.his bilirubin level is 16.5mg/dl. which is the following is the most appropriate course of action?

A. Stop breast feeding and give formoula


B. Start phototherapy
C. Phototx
D. Plasma exchange

43. Cystic Fibrosis (CF) vitamin supplement?

A. Vitamin B
B. Folate
C. Vitamin C
D. Vitamin D
E. Vitamin B12

44. A 18 mo-old already walked now stopped walking,blow legs cause?

A. Primary hyperParathyroidsm
B. Rickets
45.

D. Obtain a urine culture by catheter and start antibody

46. A 14 year-old healty girl has a urinalysis a part of her well child visit.Urinc dipstick testing show specific gravity
1.014 ,Ph-6 ,and 2+ proteinuria and is negative for blood,microscopic examination of the urine is unreveuling the
most appropriate next step in diagnosis is to?

B. Dip stick mid stream morning


C. 24 hours protein collection
D. Measure the urine protein to creatinine ratio an a first morning voided sampl

47. Child came to ER with an epileptic seizure, he is known to have epilepsy and had recieved valproic acid 30 min at
home, he is still seizing on arrival- what the next management?

A. Intubation
B. That u should make an IV access and give anticonvulsant,
C. Lorazepam
D. Stabilize airway and provid 100% O2

48. You are seeing 10 years old boy for his will-child checkup and notice that he is Hypertensive. he has otherwise
been doing weel when looking back throngh his old chart, tou realize that his previous blood pressure were normal.
what would by your next course of action ?

B. Urinanlisis
C. Ultrasound
D. Rechecking his blood pressure in 1 week
E. Basic hormonal and metabolic panel

49. Which of the following autoantibodies is most specific for SLE? ((which of the following tests would be most
specific in the diagnosis of SLE)):

A. Anti-ribonucleic acid (RNA)


B. Anti double standed DNA antibody
C. Anti-nuclear
D. Anti-double-stranded deoxyribonucleic acid (dsDNA)
E. Anti-ribonucleoprotein

50. A 14-yr-old girl presents with vaginal bleeding that is more prolonged and profuse than her usual periods, which
are irregular and first started 13 mo ago. She has pallor and tachycardia. The next step in her evaluation is to:

A. Administer medroxyprogesterone (Provera)


B. Administer conjugated estrogens (Premarin)
C. Determine the hematocrit
D. Determine the estrogen level
E. Determine the platelet count
51. A 54-year-old woman undergoes a laparotomy because of a pelvic mass. At exploratory laparotomy, a unilateral
ovarian neoplasm is discovered that is accompanied by a large omental metastasis. Frozen section diagnosis confirms
metastatic serous cystadenocarcinoma.
Which of the following is the most appropriate intraoperative course of action?

A. Excision of the omental metastasis and ovarian cystectomy


B. Omentectomy and ovarian cystectomy
C. Excision of the omental metastasis and unilateral oophorectomy
D. Omentectomy and bilateral salpingo-oophorectomy
E. Omentectomy, total abdominal hysterectomy, and bilateral salpingo-oophorectomy

52. A 51-year-old woman is diagnosed with invasive cervical carcinoma by cone biopsy. Pelvic examination and rectal-
vaginal examination reveal the parametrium to be free of disease, but the upper portion of the vagina is involved with
tumor. Intravenous pyelography and sigmoidoscopy are negative, but a CT scan of the abdomen and pelvis shows
grossly enlarged pelvic and periaortic nodes. This patient is classified at which of the following stages?

A. IIa
B. IIb
C. IIIa
D. IIIb
E. IV

53. Varicella-zoster infection is typically most severe in which of the following?

A. adolescents
B. children
C. nonpregnant adults
D. pregnant female

54. A woman is found to have a unilateral invasive vulvar carcinoma that is 2 cm in diameter but not associated with
evidence of lymph node spread. Initial management should consist of which of the following?

A. chemotherapy
B. radiation therapy
C. simple vulvectomy
D. radical vulvectomy
E. radical vulvectomy and bilateral inguinal lymphadenectomy

55. A patient is receiving external beam radiation for treatment of metastatic andometrial cancer. The treatment
field include the entire pelvis. Which of the following tissues within this radiation field is the most radiosensitive?

A. vagina
B. ovary
C. rectovaginal septum
D. bladder
E. rectum
56. A 24-year-old woman presents with new onset right lower quadrant pain, and you palpate an enlarged, tender
right adnexa. Which of the following sonographic characteristics of the cyst in this patient suggests the need for
surgical exploration now instead of observation for one menstrual cycle?

A. lack of ascites
B. unilocularity
C. papillary vegetation
D. diameter of 5 cm
E. demonstration of arterial and venous flow by Doppler imaging

57. A 70-year-old woman presents for evaluation of a pruritic lesion on the vulva. Examination shows a white, friable
lesion on the right labia majora that is 3 cm in diameter. No other suspicious areas are noted. Biopsy of the lesion
confirms squamous cell carcinoma. In this patient, lymphatic drainage characteristically would be first to the:

A. External iliac lymph nodes


B. Superficial inguinal lymph nodes
C. Deep femoral lymph nodes
D. Periaortic nodes
E. Internal iliac nodes

58. A 17-year-old girl is seen by her primary care physician for the evaluation of left lower quadrant pain. The
physician felt a pelvic mass on physical examination and ordered a pelvic US. You are consulted because an ovarian
neoplasm is identified by ultrasound. Which of the following, the most common ovarian tumor in this type of Patient?

A. Germ cell
B. Papillary serous epithelial
C. Fibrosarcoma
D. Brenner tumor
E. Sarcoma botryoides

59. Which of the following may be used for the successful treatment of migraine headaches during pregnancy?

A. atenolol
B. acupuncture
C. sumatriptan
D. all of the above

60. Transvaginal Ultrasaunde with Doppler color flow imagining is used to detect malignant ovarian tumors on the
basis of the?

A. Different temperature of tumor tissue


B. Ultrasaunde pattern of ovarian tumors
C. Increased blood flow of ovarian arteries
D. Neuvascularity of tumor blood supply
E. Discordance of ovarian artery blood sopply between the left and right ovaries

61. At the time of annual examination, a patient expresses concern regarding exposure to sexually transmitted
diseases. During your pelvic examination a singular, indurated, nontender ulcer is noted on the vulva. Venereal
Disease Research Laboratory (VDRL) and fluorescent treponemal antibody (FTA) tests are positive. Without
treatment, the next stage of this disease is clinically characterized by which of the following?

A. Optic nerve atrophy and generalized paresis


B. Tabes dorsalis
C. Gummas
D. Macular rash over the hands and feet
E. Aortic aneurysm
62. 59 year-old woman undergoes vaginal hysterectomy and anteropost repair for uterine
prolapse, which of the following is a complication of this procedure that often develops within 2 weeks of surgery:

A. dysparunia
B. Stress urinary incontinence
C. Non fistulous fecal incontinence
D. Enterocele
E. Vaginal vault prolapsed

63. A 36-year-old woman presents to your office for contraception. She has had three vaginal deliveries without
complications. Her medical history is significant for hypertension, wellcontrolled with a diuretic, and a seizure
disorder. Her last seizure was 12 years ago. Currently she does not take any antiepileptic medications. She also
complains of stress-related headaches that are relieved with an over-the-counter pain medication. She denies any
history of surgeries. She is divorced, smokes one pack of cigarettes per day, and has three to four alcoholic drinks per
week. On examination, her vital signs include weight 90 kg, blood pressure 126/80 mm Hg, pulse 68 beats per minute,
respiratory rate 16 breaths per minute, and temperature 36.4°C (97.6°F). Her examination is normal except for some
lower extremity nontender varicosities. She has taken birth control pills in the past and wants to restart
them because they help with her cramps.

Which of the following would contradict the use of combination oral contraceptive pills in this patient?

A. Varicose veins
B. Tension headache
C. Seizure disorders
D. Obesity and Smoking in a woman over 35 years of age
E. Mild essential hypertension

64. Which of the following adverse fetal effects has been reported to be caused by maternal par-vovirus infection?

A. microcephaly
B. hydrocephaly
C. hydrops
D. cardiac defects

65. Which of the following is most strongly associated with peripartum stroke?

A. Hypertension
B. Chorioamnionitis
C. sickle cell disease
D. Placental abruption

66. Endemic post natal cretinism is seen in countries with hay incidences of which dietary problems:

A. Low protein intake


B. Iodine deficient
C. Elevated lithium level in
D. Magnesium deficient

67. What is the most common etiology of glycosuria in pregnancy?

A. false positive from lactose


B. gestational diabetes
C. augmented glomerular filtration of glucose
D. insulin resistance
68. Which of the following may occur frequently in pregnancies complicated by anemia in 2d trimester:

A. Pregnancy induces (HTN) Hypertension


B. Gestational diabetes
C. Preterm labor
D. Urinary tract infection

69. An intrauterine pregnancy of approximately 10 weeks gestation is confirmed in a 30-year-old G5P4 woman with
an IUD in place. The patient expresses a strong desire for the pregnancy to be continued. On examination, the string
of the IUD is noted to be protruding from the cervical os.
Which of the following is the most appropriate course of action?

A. Leave the IUD in place without any other treatment


B. Leave the IUD in place and continue prophylactic antibiotics throughout pregnancy.
C. Remove the IUD immediately.
D. Terminate the pregnancy because of the high risk of infection.
E. Perform a laparoscopy to rule out a heterotopic ectopic pregnancy

70. Which of the following is a risk factor for placenta accreta?

A. young age
B. primigravidity
C. fundal placenta
D. previous cesarean delivery

71. Typical permanent anatomical characteristics of the cervix following delivery include which of the following?

A. wider external os
B. scarred, narrowed external os
C. cervical length is doubled from pregnancy
D. cervical length is halved from pregnancy

72. What percentage of term labors present with a longitudinal lie?

A.20
B.50
C.70
D.99

73. What part of the fetal anatomy is palpated easily with extreme asynclitism of the vertex?

A. nose
B. mouth
C. ear
D. shoulder

74. Which of the following characterizes true labor?

A. Painless contractions with rupture of membranes


B. Engagement of the fetal head
C. Progressive cervical dilatation and effacement
D. Rhythmic lower abdominal pain
75. In the presence of uterine contractions, what cervical dilatation reliably represents the onset of active labor?

A. 0 to 2 cm
B. 3 to 5 cm
C. 4 to 6 cm
D. 6 to 7 cm

76. The first evidence of pubertal development in the female is usually?

A. Onset of menarche
B. Appearance of breast buds
C. Appearance of axillary and pubic hair
D. Onset of growth spurt

77. A 9-year-old girl presents for evaluation of regular vaginal bleeding. History reveals thelarche at age 7 and
adrenarche at age 8. The most common cause of this condition in girls is:

A. Idiopathic
B. Gonadal tumors
C. McCune-Albright syndrome
D. Hypothyroidism
E. Tumors of the central nervous system

78.

B. Menarche delayed begun 16 years of age

79. Normal stature with minimal or absent pubertal development may be seen in ?

A. Testicular feminilization
B. Kallman syndrome
C. Pure gonadal dysgenesis
D. Turner
E. Intermittent athletic training

80. A 7 year-old girl is brought in to see you by her mother because the girl has developed breast and has a few pubic
hairs starting to show up. Which of the following is the best treatment for the girl's condition ?

A. Exogenous gonadotropins
B. Ethinyl estradiol
C. GnRH agonists
D. Clomophene citrate
E. No treatment, reassure the mother that pubertal symptoms at age 7 are normal

81. While evaluating a 30-year-old woman for infertility, you diagnose a bicornuate uterus. You explain that additional
testing is necessary because of the woman’s increased risk of congenital anomalies in which organ system?

A. Skeletal
B. Hematopoietic
C. Urinary
D. Central nervous
E. Tracheoesophageal
82. In the evaluation of a 26-year-old patient with 4 months of secondary amenorrhea, you order serum prolactin and
.-hCG assays. The pregnancy test is positive, and the prolactin comes back at 100 ng/mL (normal <25 ng/mL in this
assay). This patient requires

A. Routine obstetric care


B. Computed tomography (CT) scan of her sella turcica to rule out pituitary adenoma
C. Repeat measurements of serum prolactin to ensure that values do not increase over 300 ng/mL
D. Bromocriptine to suppress prolactin
E. Evaluation for possible hypothyroidism

83. A patient presents to you for evaluation of infertility. She is 26 years old and has never been pregnant. She and
her husband have been trying to get pregnant for 2 years. Her husband had a semen analysis and was told that
everything was normal. The patient has a history of endometriosis diagnosed by laparoscopy at age 17.At the time
she was having severe pelvic pain and dysmenorrheal. After the surgery, the patient was told she had a few small
implants of endometriosis on her ovaries and fallopian tubes and several others in the posterior cul-de-sac. She also
had a left ovarian cyst, filmy adnexal adhesions, and several subcentimeter serosal fibroids. You have recommended
that she have a hysterosalpingogram as part of her evaluation for infertility.
Which of the patient's following conditions can be diagnosed with a hysterosalpingogram?

A. Endometriosis
B. Hydrosalpinx
C. Subserous fibroids
D. Minimal pelvic adhesions
E. Ovarian cyst

84. A 19-year-old patient presents to your office with primary amenorrhea She has normal breast and pubic hair
development, but the uterus and vagina are absent. Diagnostic possibilities include which of the following?

A. XYY syndrome
B. Gonadal dysgenesis
C. Mullerian agenesis
D. Klinefelter syndrome
E. Turner syndrome

85. What are the antigens of the rhesus blood group system?

A. CDE
B. cCDeE
C. CDEcde
D. CDe

86. A 4-year-old girl is brought to her pediatrician because her parents think she does not seem to be “developing
normally.” The girl’s mother states that her daughter seemed normal for at least the first 2 to 3 years of her life. She
was walking and beginning to speak in sentences. She was able to play with her mother and older sister. The mother
has been noticing that over the past 2 months her daughter has lost these previously acquired abilities. She will no
longer play with anyone else and has stopped speaking entirely. She has lost all bowel control, when previously she
had not needed a diaper for at least a year. Which of the following is the most likely diagnosis?

A. Rett disorder
B. Childhood disintegrative disorder
C. Autism
D. Asperger disorder
E. Pervasive developmental disorder
87. Which of the following has been considered common finding in the history of fire- letters Stuttering?

A. Stuttering
B. Trichtillomania
C. Auditory hallucinations
D. Echolalia
E. Enuresis

88. A 7-year-old boy avoids sleepovers because he wets his bed one to two times per week and is afraid his friends
would tease him. He has never achieved a year-long period of dryness throughout the night. A physiologic work-up
shows no evidence of an organic cause for this problem, and there is no evidence of a psychiatric disorder in any
other category. Which of the following treatments is likely to be effective and should be tried first?

A. Pharmacotherapy with imipramine


B. Bladder training (reward for delaying micturition during daylight hours(
C. Classic conditioning with a bell-and-pad apparatus
D. Pharmacotherapy with desmopressin
E. Psychotherapy

89. A social worker makes a routine visit to a 3-year-old boy who has just been returned to his biological mother after
spending 3 months in foster care as a result of severe neglect. The child initially appears very shy and clings fearfully
to his mother. Later on, he starts playing in a very destructive and disorganized way. When the mother tries to stop
him from throwing blocks at her, he starts kicking and biting. The mother becomes enraged and starts shouting.
Which of the following is the most likely diagnosis for this child?

A. Oppositional defiant disorder


B. ADHD
C. Reactive attachment disorder
D. Posttraumatic stress disorder (PTSD)
E. Major depression

90. Concerning selective mutism which is correct:

A. only 1-3% history of delay onset of speach


B. is commonly associated with malformation of broca's area
C. first characteristic of it is school phobia
D. common in boy > girls
E. children with selective mutism speak freely at home

91. A 4 year-old boy experiences episodes of waking up screaming in the middle of the night thrashing wildly with his
eyes wild openining ,when his parents try to wake him up and secure him he pushes them away,2 min later he had
suddenly fallen asleep .the next day he has no memory about the event.
correct statement about his condition include:

A. behavioral therapy and relaxation technique are the treatment of choice


B. specific treatment is seldom required for this disorder
92. A 12-year-old boy is brought to the psychiatrist because his mother says the boy is driving her “nuts.” She reports
that he constantly argues with her and his father, does not follow any of the house rules, and incessantly teases his
sister. She says that he is spiteful and vindictive and loses his temper easily. Once he is mad, he stays that way for
long periods of time. The mother notes that the boy started this behavior only about 1 year previously. While she
states that this behavior started at home, it has now spread to school, where his grades are dropping because he
refuses to participate.The patient maintains that none of this is his fault--his parents are simply being unreasonable.
He denies feeling depressed and notes that he sleeps well through the night.
Which of the following is the most likely diagnosis?

A. oppositional defiant disorder (ODD)


B. Antisocial personality disorder
C. Conduct disorder
D. Childhood-onset schizophrenia
E. Mania

93. Which of the following statement concerning disorder is true associated with physical cruelty to animals?

A. This disorder is more common in children of parents with avoiding personality with avoidant
B. This disorder is defined as apart of developmental stage
C. Conduct disorder is freguently assoiated with physical cruelty to animals
D. ?
E. Traditional individual psychotherapy is the most effective treatment for this disorder

94. Conduct disorder coexist often with:

A. ADHD
B. Dysthymic disorder
C. Major depressive disorder;
D. Early onset bipolar I;
E. Anxiety disorder

96. Childhood schizophrenia :

A. is not diagnosed using the same symptoms as are used for adult schizophrenia
B. tends to have a better prognosis that adult schizophrenia;
C. Boys became symptomatic at the same age as girls;
D. tends to have an abrupt onset;
E. all of the above

97. An 18-year-old girl comes to the psychiatrist because she pulls out her hair in patches when she is anxious or
upset. She is taught to make a tight fist whenever she has this impulse rather than pull out her hair.Which of the
following techniques is this?

A. Habit reversal training


B. Extinction
C. Simple conditioning
D. Flooding
E. Desensitization
98. A 24-year-old woman, 5 days after delivery of a normal ,full-term infant, is brought to the obstetrician because
she is so tearful. She states that her mood is quite labile, often changing within minutes. She has trouble sleeping,
both falling asleep and awakening early. She notes anhedonia, stating she doesn’t enjoy “much of anything” right
now. Which of this patient’s symptoms point preferentially to a postpartum depression?

A. Time--that is, 5 days post delivery


B. Tearfulness
C. Labile mood
D. Insomnia
E. Anhedonia

99. A 35-year-old woman is seeing a psychiatrist for treatment of her major depression. After 4 weeks on fluoxetine
at 40 mg/day, her psychiatrist decides to try augmentation. Which of the following is the most appropriate
medication?

A. Lithium
B. Sertraline
C. An MAO inhibitor
D. Clonazepam
E. Haloperidol

100. A 37-year-old woman is admitted to an inpatient treatment program for withdrawal from heroin. Eighteen
hours after her last injection of heroin, she becomes hypertensive, irritable, and restless. She also has nausea,
vomiting, and diarrhea. Which medication would be best to treat some of the symptoms of opioid withdrawal?

A. Chlordiazepoxide
B. Haloperidol
C. Paroxetine
D. Phenobarbital
E. Clonidine

101. A 30-year-old woman is diagnosed as bipolar. At the same time that this illness is diagnosed, it is discovered that
she is pregnant .Which of the following drugs has the highest risk to the fetus if used?

A. Valproic acid
B. Lithium
C. Chlorpromazine
D. Haloperidol
E. Fluoxetine

103. A 36-year-old woman is diagnosed with a paranoid delusional disorder after she repeatedly called police to her
home, convinced the neighbors were about to harm her by electrocuting her in her sleep. She is started on
olanzapine. Which of the following side effects is this patient at greater risk for while on this medication?

A. Increased sweating
B. Neuroleptic malignant syndrome
C. Insomnia
D. Glucose abnormalities
E. Hyponatremia
104. A 35 year old woman is reffered to a psychiatric outpatient clinic by a plastic surgeon. According to the referral
letter, this person has consulted 3 cosmetic surgeons in the last 6 months about what she perceives to be a deformity
in the upper lip, all of the physicians told her that there is no deformity and there is no need for any surgery but the
woman firmly believes that there is a deformity, and this believe has hampered her socially and occupationally.
What is the diagnosis?

A. Hypochondriasis
B. Somatization
C. Body dismorphic disorder
D. Delusional disorder ,somatic type

105. It is NOT prescribed for the express train of psychoanalytic psychotherapy:

C. Cognitive deficiencies;
D. Ability to verbalization of deficiencies

106. Which of the following is common in the childhood history of patients with borderline personality disorder?

A. Shyness
B. Psychosis
C. Conduct disorder
D. Sexual abuse
E. Magical and odd beliefs

107. A 35-year-old man tells his therapist that he has difficulty describing his emotions accurately.He says that he
can't find the words to describe how he is feeling. What is the phenomenon that is causing this inability?

A. Neurasthenia
B. Alexithymia
C. Mental fatigue
D. Dysphoria

108. A 30 year-old woman begins to see a therapist because of loneliness and feelings of being unloved and
unwanted with some mild depressive symptoms .she flirts constantly with the therapist and is hurt when the
therapist douse not reciprocate. Which of the following disorders is she most likely to have?

A. Antisocial personality disorder


B. Borderline personality disorder
C. Schizotypal personality disorder
D. Obsessive - compulsive personality disorder
E. Histrionic personality disorder

110. A 57 year-old man whom ….have treated for 20 years forbipolar disorder is admitted to the general hospital
with chest pain,the hospital physician aske whether the lithium he is receiving can affect his electrocardiogram
(ECG),your inform her that lithium has which at the following ?

A. Ventricular dysrhythmia
B. ST-segment depression
C. T-wave flattening
D. Prolong the QT interval
E. Appearance of U waves
Part B:

1. What's right according to Anorexia Nervosa:

A. Very often occurs in adolescents and tanagers ;


B. System's complications may include: heart rate arrhythmia ;constipation and osteoporosis

2. What's right according to Tourette’s syndrome :

A. Has been shown to possibly have a familial and genetic relationship with OCD;
B. Social phobia
C. Generalized anxiety disorder
D. The behavioural technics is most effective therapy for this disorder;
E. None of the above

3. Which of the following statements is true of suicide in adolescence?

A. Completed suicide decreases in incidence with increasing age.


B. Completed suicide is more common in girls than in boys.
C. Suicide attempts are always associated with a mood disorder.
D. Suicide attempts often precipitated by arguments with family members, girlfriends, or boyfriends.
E. The most common method used in completed suicide among adolescents in the United States is toxic ingestion.

4. A 23 year-old patient with schicophrcuin Who was recently started on neuroleptic complains that he cannot still
and feels constantly restless the nursing staff has noted that he often paces.
which of the following is this known as ?

A. Parkinsonism
E. Akathisia

5. If a patient receiving clozapine shows a white blood count (WBC) of 2,000 per cc, the clinician should:

A. Increase the dosage of clozapine at once


B. Terminate any a...ntibiotic therapy
C. Stop the administration of clozapine at once
D. Monitor the patient’s WBC every 10 days
E. Institute weekly complete blood count (CBC) tests with differential

6.The most common cause of delirium within 3 days postoperatively in a 40-year-old man with a history of alcohol
dependence is :

A. Stress of surgery
B. Postoperative pain
C. Pain medication
D. Infection
E. Delirium tremens

7. Mouth ulceration is associated with which of the following types of withdrawal?

A. Cocaine
B. Opioids
C. Nicotine
D. Alcohol
E. Benzodiazepines
8. All of the following statements regarding cyclothymic disorder are true except?

A. Symptoms must be present for at least 2 years


B. Occurs at the same rate in men and women
C. Symptoms may satisfy criteria for major depression
D. Consists of hypomania alternating with depressed mood
E. Lifetime prevalence rate is about 0.4 to 1 percent

9. Dysthymic Disorder:

A. Usually begins in adulthood


B. Does not respond to anti-depressants
C. Presents with symptoms of lack of say in life and preoccupation with inadequacy
D. Is usually limited to one or two episodes
E. Is characteristically marked by psychomotor agitation or psychomotor retardation

10. Hypothyroidism in the elderly may commonly present with all of the following except?

A. Lassitude
B. Fatigue
C. Cognitive impairment
D. Anorexia
E. Constipation

11. All the following are vitamin K-dependent coagula¬tion factors except?

A. Factor X
B. Factor VII
C. Protein C
D. Protein S
E. Factor VIII

12. A 36-year-old African-American woman with systemic lupus erythematosus presents with the acute onset of
leth¬argy and jaundice. On initial evaluation, she is tachycardic, hypotensive, appears pale, is dyspneic, and is
somewhat dif¬ficult to arouse. Physical examination reveals splenomegaly. Her initial hemoglobin is 6 g/dL, white
blood cell count is 6300/µX, and platelets are 294,000/µL. Her total bilirubin is 4 g/dL, reticulocyte count is 18%, and
haptoglobin is not detectable. Renal function is normal, as is urinalysis.
What would you expect on her peripheral blood smear?

A. Macrocytosis and PMN's with hypersegmented nuclei


B. Microspherocytes
C. Schistocytes
D. Sickle cells
E. Target cells

13. A healthy 62-year-old woman returns to your clinic after undergoing routine colonoscopy. Findings included two
1.3-cm sessile (flat-based), villous adenomas in her ascending colon that were removed during the procedure.
What is the next step in management?

A. Colonoscopy in 3 months
B. Colonoscopy in 3 years
C. Colonoscopy in 10 years
D. CT scan of the abdomen
E. Partial colectomy
F. Reassurance
14. A 23-year-old woman is diagnosed with a lower extremity deep venous thrombosis. Which of the following
medical conditions represents a contraindication to therapy with low-molecular-weight heparin (LMWH)?

A. Pregnancy
B. Obesity
C. Dialysis-dependent renal failure
D. Uncontrolled diabetes mellitus
E. Jaundice

15. Which of the following statements about lead-time bias occurrence is true?

A. A test does not influence the natural history of the disease; patients are merely diagnosed at an earlier date.
B. Slow-growing, less aggressive cancers are detected during screening; aggressive cancers are not detected by
screening, due to death.
C. Screening identifies abnormalities that would never have caused a problem during a person's lifetime.
D. The screened population differs significantly from the general population in that they are healthier.
E. A test detects disease at an earlier and more curable stage of disease.

16. The triad of portal vein thrombosis, hemolysis, and pancytopenia suggests which of the following diagnosis?

A. Acute promyelocytic leukemia


B. Hemolytic-uremic syndrome (HUS)
C. Leptospirosis
D. Paroxysmal nocturnal hemoglobinuria (PNH)
E. Thrombotic thrombocytopenia purpura (TTP)

17. Which of the following symptoms is most suggestive of an esophageal mass?

A. Early satiety
B. Liquid phase dysphagia only
C. Odynophagia with chest pain
D. Oropharyngeal dysphagia
E. Solid phase dysphagia progressing to liquid phase dysphagia

18. Which of the following should prompt investigation for hereditary nonpolyposis colon cancer screening in a 32-
year-old man?

A. Father, paternal aunt, and paternal cousin with colon cancer with ages of diagnosis of 54, 68, and 37 years,
respectively
B. Innumerable polyps visualized on routine colonoscopy
C. Mucocutaneous pigmentation
D. New diagnosis of ulcerative colitis
E. None of the above

19. A 56-year-old patient inquires about screening for colon cancer. He has no risk factors for colon cancer, other than
age. Which of the following statements is true regarding which screening test you recommend for this patient?

A. 50% of patients with a positive fecal occult blood testing have colon cancer.
B. One-time colonoscopy detects more advanced lesions than one-time fecal occult blood testing with
sigmoidoscopy.
C. Perforation rates for sigmoidoscopy and colonoscopy are equivalent.
D. Sigmoidoscopy has not been shown to reduce mortality.
E. Virtual colonoscopy is as effective as endoscopic colonoscopy for detecting polyps <5 mm in size.
20. A 73-year-old man is admitted to the hospital with 3 weeks of malaise and fevers. His past medical history is
notable only for hypertension controlled with a thiazide diuretic. He smokes one pack of cigarettes per day and works
as an attorney. His physical examination is notable only for a new systolic heart murmur heard best in the mitral
region. His laboratory examination is notable for mild anemia, an elevated white blood cell count, and oc¬casional red
blood cells on clean catch urine. Blood cul¬tures grow Streptococcus bovis and echocardiogram shows a <l-cm
vegetation on the mitral valve. What additional evaluation is indicated for this patient?

A. Colonoscopy
B. Head CT scan
C. Pulmonary embolism protocol CT scan
D. Renal biopsy
E. Toxicology screen

21. A 32-year-old man presents with jaundice and malaise. He is found to have acute hepatitis B with positive
hepatitis B virus (HBV) DNA and E antigen. Which of the following antiviral agents are approved as part of a
therapeutic regimen for mono-infection with hepatitis B?

A. Efavirenz
B. Ganciclovir
C. Lamivudine
D. Rimantadine
E. Tenofovir

22. A 72-year-old woman is admitted to the intensive care unit with respiratory failure. She has fever, obtundation,
and bilateral parenchymal consolidation on chest imaging.
Which of the following is true regarding the diagnosis of Legionella pneumonia?

A. Acute and convalescent antibodies are not helpful due to the presence of multiple serotypes.
B. Legionella can never be seen on a Gram stain.
C. Legionella cultures grow rapidly on the proper media.
D. Legionella urinary antigen maintains utility after antibiotic use.
E. Polymerase chain reaction (PCR) for Legionella DNA is the "gold standard" diagnostic test.

23. In the inpatient setting, extended-spectrum β-lactamase (ESBL)-producing gram-negative infections are most
likely to occur after frequent use of which of the following classes of antibiotics?

A. Carbapenems
B. Macrolides
C. Quinolones
D. Third-generation cephalosporins

24. A 30-year-old female is seen in the clinic before undergoing an esophageal dilation for a stricture. Her past
medical history is notable for mitral valve prolapse with mild regurgitation. She takes no medications and is allergic to
penicillin. Her physician should recommend which of the following?

A. Clarithromycin 500 mg PO 1 h before the procedure


B. Clindamycin 450 mg PO 1 h before the procedure
C. Vancomycin 1 g intravenously before the procedure
D. The procedure is low-risk, and therefore no prophy laxis is indicated.
E. Her valvular lesion is low-risk, and therefore no prophylaxis is indicated
25. Which of the following favors a diagnosis of acute bacterial epididymitis?

A. A solid nontender testicular mass


B. Absence of blood flow on Doppler examination
C. Concurrent urethral discharge
D. Elevation of the testicle within the scrotal sac
E. Lack of response to ceftriaxone plus doxycycline therapy

26. What is the most common side effect of oral ribavirin when used with pegylated interferon for the treatment of
hepatitis C ?

A. Drug-associated lupus
B. Hemolytic anemia
C. Hyperthyroidism
D. Leukopenia
E. Rash

27. A 52-year-old female presents with a community-acquired pneumonia complicated by pleural effusion. A
thoracentesis is performed, with the following results: Appearance Viscous, cloudy
pH 7.11
Protein 5.S g/dL
LDH 285IU/L
Glucose 66 mg/dL
WBC 3800/mm3
RBC 24,000/mm3
PMNs 93%
Gram stain Many PMNs; no organism seen
Bacterial cultures are sent, but the results are not currently available.

Which characteristic of the pleural fluid is most suggestive that the patient will require tube thoracostomy?

A. Presence of more than 90% polymorphonucleocytes (PMNs)


B. Glucose less than 100 mg/dL
C. Presence of more than 1000 white blood cells
D. pH less than 7.20
E. Lactate dehydrogenase (LDH) more than two-thirds of the normal upper limit for serum

28. A 23-year-old male is climbing Mount Kilimanjaro. He has no medical problems and takes no medications. Shortly
after beginning the climb, he develops severe shortness of breath. Physical examination shows diffuse bilateral
inspiratory crackles. Which of the following is the most likely etiology?

A. Acute interstitial pneumonitis


B. Acute respiratory distress syndrome
C. Cardiogenic shock
D. Community-acquired pneumonia
E. High-altitude pulmonary edema

29. Which of the following conditions would be expected to increase the residual volume of the lung?

A. Bacterial pneumonia
B. Cryptogenic organizing pneumonia
C. Emphysema
D. Idiopathic pulmonary fibrosis
E. Obesity
30. Which of the following is true regarding hypovolemic shock?

A. Loss of 20-40% of the blood volume leads to shock physiology.


B. Loss of <20% of the blood volume will manifest as orthostasis.
C. Oliguria is a crucial prognostic sign of impending vascular collapse.
D. Symptoms of hypovolemic shock differ from those of hemorrhagic shock.
E. The first sign of hypovolemic shock is mental obtundation.

31. A 23-year-old hospital worker is evaluated for a known contact with a patient with active tuberculosis. One year
ago his intermediate-strength PPD had 3 mm of induration; now it has 13 mm of induration at 48 h. He has no
significant past medical history and is on no medications. Subsequent management should include:

A. Chest radiography
B. Isoniazid 300 mg/d for 3 months
C. Measurement of baseline liver function tests
D. Measurement of liver function tests every 3 months
E. Repeated intermediate-strength PPD testing in 2 weeks

32. A 72-year-old female with severe osteoporosis presents for evaluation of shortness of breath. She is a lifetime
nonsmoker and has had no exposures. On physical examination you note marked kyphoscoliosis.
All the following pulmonary abnormalities are expected except?

A. Restrictive lung disease


B. Alveolar hypoventilation
C. Obstructive lung disease
D. Ventilation-perfusion abnormalities with hypoxemia
E. Pulmonary hypertension

33. Angioneurotic edema is associated with the use of:

A. ACE inhibitors
B. Beta blockers
C. Loop diuretics
D. Alpha-receptor blockers
E. Calcium-channel blockers

34. Goodpasture's syndrome is associated with:

A. Osteoporosis and renal lithiasis


B. Pathologic fractures and thyroiditis
C. Hepatitis and recurrent cystitis
D. Pulmonary hemorrhage and glomerulonephritis
E. Pica and angioedema

35. Which of the following forms of hepatitis does NOT have a chronic state?

A. Hepatitis A
B. Hepatitis B
C. Hepatitis C
D. Hepatitis D
36. A 54-year-old woman presents to your office with complaints of frequent sweating episodes, palpitations,
nervousness, and sensitivity to heat with increased appetite and weight loss. The most likely diagnosis is:

A. Hypothyroidism
B. Menopause
C. Addison's disease
D. Hyperthyroidism
E. Cushing's disease

37. Which of the following is not indicated in the emergent treatment of thyroid storm?

A. Propylthiouracil
B. Supersaturated potassium iodine
C. Propranolol
D. Aspirin
E. Acetaminophen

38. Which of the following statements about hyperglycemic hyperosmolar nonketotic coma is true?

A. It is usually associated with type I adult-onset diabetes mellitus.


B. It is associated with fluid overload.
C. Associated laboratory findings include elevated serum lactate.
D. Treatment involves intravenous administration of glucose.
E. Treatment involves fluid administration.

39. Chron's disease is associated with which of the following?

A. Inflammation limited to the superficial layer of the bowel wall


B. The affinity to involve the rectosigmoid junction
C. Decreased risk of colon cancer
D. Continuous mucosal areas of ulceration that affect the anus
E. Fistula formation

40. Dermatomyositis is associated with which of the following?

A. Generalized morbilliform rash


B. Underlying malignancy
C. Elevated lipids
D. Distal muscle weakness
E. Inflammatory bowel disease

41. Which of the following statements about hereditary angioedema is true?

A. It is related to excessive amyloid deposition.


B. It is caused by a deficiency of the C1 esterase inhibitor.
C. Attacks are triggered by antihistamines.
D. Treatment involves dehydroepiandrosterone (DHEA) administration.
42. A 56-year-old man presents to your office with complaints of "chronic" diarrhea. He states that he has had loose
stools for the last 2 days. He denies blood in the stool, fever, and has no weight loss and no recent travel .
Appropriate management at this time includes:

A. Observation
B. Check stool cultures
C. Colonoscopy
D. Stool fat studies

43. The presence of polymorphonuclear cells in stool samples most likely supports the diagnosis of a:

A. Bacterial infection
B. Viral infection
C. Parasitic infection
D. Fungal infection

44. Acromegaly is associated with which of the following factors?

A. Excessive cortisol secretion


B. Lack of adequate parathyroid hormone
C. Excessive growth hormone
D. Thyroid dysfunction
E. Excessive gastrin secretion

45. Which of the following conditions is a contraindication to influenza vaccination?

A. Allergy to eggs
B. Allergy to red dye
C. Allergy to penicillin
D. Allergy to milk
E. Allergy to dust mites

46. A construction worker presents with pain over the lateral elbow. He reports that he has been using a hammer
more often, and this seems to aggravate his discomfort. The most likely diagnosis is:

A. Carpal tunnel syndrome


B. Rotator cuff dysfunction
C. Lateral epicondylitis
D. Ulnar nerve entrapment
E. Biceps tendonitis

47. Which of the following has been shown in multiple cohort studies to reduce the risk of colon cancer?

A. Folic acid
B. B complex vitamin
C. Aspirin
D. Vitamin C
E. Vitamin E
48. Which of the following statements about Charcot-Marie-Tooth syndrome is true?

A. Decreased pain, temperature, and vibratory sense are usually seen.


B. Symptoms include the development of a foot drop.
C. Symptoms are commonly noted at birth.
D. Hypertrophy of the distal leg muscles is common.
E. Hyperactive reflexes are noted.

49. The most effective drug for the treatment of traveler's diarrhea is:

A. Metronidazole
B. Tetracycline
C. Ciprofloxacin
D. Trimethoprim-sulfamethoxazole (TMP-SMX)
E. Doxycycline

50. Guillain-Barré syndrome is most closely associated with:

A. Normal electromyogram (EMG) findings found late in the course


B. Descending asymmetric paralysis
C. Previous trauma
D. Symptoms that usually begin in the lower extremities
E. Low levels of protein in the cerebrospinal fluid (CSF)

51. Which of the following best describes migraine headaches?

A. Aura following the resolution of the headache


B. Recurrent headaches lasting less than 4 hours
C. Unilateral, throbbing headache
D. Bilateral band-like headache
E. Rhinitis with facial pain

52. A 30-year-old woman from Minneapolis presents to your office complaining of paresthesias, weakness, lack of
coordination, and difficulty with gait. Her symptoms are worse after a hot shower. Examination of the cerebral spinal
fluid shows oligoclonal bands of immunoglobulin G (IgG). The most likely diagnosis is:

A. Multiple sclerosis
B. Huntington's disease
C. Parkinson's disease
D. Neurofibromatosis
E. Amyotrophic lateral sclerosis (ALS)

53. Which of the following medications is the most appropriate to use in the emergent treatment of anaphylaxis?

A. Diphenhydramine
B. Isoproterenol
C. Epinephrine
D. Prednisone
E. Atropine
54. Which of the following statements about rheumatic fever (RF) is correct?

A. It is a complication of a group B streptococcal infection.


B. It most commonly affects the aortic valve.
C. It is most commonly seen in patients younger than 4 years.
D. Prophylaxis against recurrent attacks with monthly penicillin administration is indicated after an acute attack.
E. It usually follows a streptococcal skin infection.

55. The use of proton pump inhibitors can result in:

A. Vitamin C deficiency
B. Vitamin D deficiency
C. Vitamin B12 deficiency
D. Folate deficiency

56. The drug of choice for the treatment of trigeminal neuralgia is:

A. Naproxen
B. Prednisone
C. Carbamazepine
D. Valproic acid
E. Phenobarbital

57. Which of the following is NOT in the diagnostic criteria for the presence of diabetes mellitus?

A. Random plasma glucose > 200 mg/dL


B. Fasting plasma glucose of 140 mg/dL
C. Abnormal glucose tolerance test
D. Hemoglobin A1C (HbA1c) of 7.5

58. Which of the following is the treatment of choice for pyoderma gangrenosum?

A. Steroid therapy
B. Topical antibiotics
C. Oral antibiotics that treat methicillin-resistant S. aureus
D. Methotrexate
E. Plasmapheresis

59. Which of the following statements about myasthenia gravis is true?

A. Symptoms usually improve with administration of norepinephrine-inhibiting medications.


B. Symptoms include ptosis, diplopia, dysarthria, dysphagia, and proximal muscle weakness of the limbs.
C. Symptoms are aggravated by the administration of edrophonium.
D. Symptoms rarely fluctuate and usually spare the facial nerves.

60. Type I diabetes mellitus is associated with:

A. Hypersensitivity to glucose
B. Overproduction of glucagon
C. Lack of insulin production by the pancreas
D. Tissue resistance to insulin
E. Excessive growth hormone secretion
61. A 32 year old man is in a high speed motorcycle collision and present with an obvious pelvic fracture. On
examination, he has a scrotal hematoma and blood at his urethral meatus.
Which of the following is the most appropriate next step in his management?

A. Placement of a Foley catheter


B. Cystoscopy
C. CT of the pelvis
D. Retrograde urethrogram
E. Nephrostomy tube placement

62. A 52 year old woman sees her physician with complains of fatigue headache, flank pain, hematuria, and
abdominal pain. She undergoes a sestamibi scan that demonstrates persistent uptake in the right superior
parathyroid gland at 2 hours. Which of the following laboratory values is most suggestive of her diagnosis?

A. Serum acid phosphatase above 120 IU/L


B. Serum alkaline phosphatase above 120 IU/L
C. Serum calcium above 11 mg/dl
D. Urinary calcium below 100 mg/day
E. Parathyroid hormone levels below 5 pmol/l

63. A patient with spontanious eye opening,who is conFused and localizes pain has a Glasgow Coma Score (GCS) of:

A. 9
B. 11
C. 13
D. 15

64. Which of the following tumor markers is the best to monitor recurrence of colon cancer?

A. α fetoprotein
B. Carcinoembryonic antigen
C. Cancer antigen 15-3
D. 5-hydroxyindoleacetic acid

65. Hyperacute rejection is caused by:

A. Performed antibodies
B. B-cell-generated Antidonor antibodies
C. T-cell-mediated allorejection
D. Nonimmune mechanism

66. The treatment of choice for a 1.8 cm in diameter,N0,M0 invasive breast cancer is:

A. Lumpectomy alone
B. Lumpectomy,sentinel node biopsy,and radiation
C. Mastectomy with sentinel node biopsy and radiaton
D. Mastectomy with axillary node dissection and radiation

67. Which of the following increase the risk of breast cancer:

A. Multiple gestations
B. Late menarche
C. Late menopause
D. Prolonged and multiple episodes of lactation
68. The most appropriate treatment with DCIS involving 2 quadrents is:

A. Observation
B. Lumpectomy
C. Lumpectomy with radiation
D. Mastectomy

69. Which of the following is a true surgical emergency in a newborn:

A. Tetralogy of Fallot
B. Truncus arteriosus
C. Total anommaly pulmonary venouse connection
D. Coarctaion of the aorta

70. Which of the following is the best predictor of a singnificantly increased operative risk for coronary artery bypass
surgery:

A. Electrocardiography (ECG)
B. The degree of proximal arterial stenosis
C. The ventricular Ejection Fraction
D. A history of multiple myocardial infarcts

71. The most effective treatment of achalasia is:

A. Antispasmodic medication
B. Dilatstion of the lower esophageal sphincter
C. Esophagomyotomy
D. Resection of the cardioesophageal junction

72. Dieulafoyw’s lesion most commonly results in:

A. Progression to gastric cancer


B. Gastroparesis
C. Gastrin outlet obstruction
D. Upper gastrointestinal bleed

73. The initial diagnostic examination in a 50-year-old patient who is febrile and tender in the left lower quadrent of
the abdomen should be:

A. Flexible sigmoidoscopy
B. Water soluble contrast enema
C. Ultrasound
D. Computed tomography scan

74. Which of the following is the most likely to harbor a colorectal cancer:

A. Hamaromatous polyp
B. Tubular adenomatous polyp
C. Tubuluvillous adenomatous polyp
D. Villouse adenomatous polyp
75. The most common site of volvulus is:

A. Cecum
B. Proximal jejunum
C. Sigmoid colon
D. Stomach

76. Which of the following is the most commonly confused with appendicitis in children:

A. Meckel’s diverticulitis
B. Mesentric adenitis
C. Pelvec inflammatory disease
D. Acute gastroenteritis

77. An 80 year old man is admitted to the hospital complaining of nausea, abdominal pain, distension and diarrhea. A
cautiously performed transanal contrast study reveals an apple core configuration in the rectosigmoid area. Which of
the following is the most appropriate next step of the management?

A. Colonoscopic decompression and rectal tube placement


B. Saline enemas and digital disimpaction of fecal matter from the rectum
C. Colon resection and proximal colostomy
D. Oral administration of metronidazole and checking a Clostridium difficile titer
E. Evaluation of an electrocardiogram and obtaining an angiogram evaluate for colonic mesenteric ischemia.

78. The most common bening lesion in the liver is:

A. Simple (congenital) hepatic cyst


B. Hamartoma
C. Hepatic adenoma
D. Focal nodular hyperplasia

79. Approximately 2 weeks after viral respiratory illness, an 18 month old child complains of abdominal pain passes
some bloody mucus perrectum. A long, thin mass is palpable in the right upper quadrant of the abdomen.
Intussusceptions is suspected. Correctstatements concerning Intussusception in infants include which of the
following?

A. Recurrence rates following treatment are high


B. It is frequently preceded by a gastrointestinal viral illness.
C. A 1 to 2 week period of parenteral alimentation should precede surgical reduction when surgery is required
D. Hydrostatic reduction without surgery rarely provides successful treatment
E. The most common type occurs at the junction of the descending and the sigmoid colon.

80. The most common cause of acute cholecysitits is:

A. Cyctic duct obstuction


B. Escherichia.coli infection
C. Multiple gallstones
D. Salmonella infection
81. The ideal time to give immunizations to patients undergoing elevtive splenectomy is:

A. A least 2 monthes before surgery


B. At least 2 weeks before surgery
C. In the holding area just prior to surgery
D. On the day of discharge from the hospital after splenectomy

82. Spigelian hernias occur:

A. On the lateral border of the rectus abdominis


B. In the linea alba
C. In the medial wall of the inguinal canal
D. In the femoral triangle

83. A newborn infant is born from a mother with polyhydramnion presents with excessive salivation along with
coughing and choking with the first oral teeding.An x-ray of the abdomen shows gas in stomach and a nasogastric
tube coiled in the esophagus.Which of the following is the most likely diagnosis?

A. Esophageal atresia
B. Tracheoesophageal fistula
C. Esophageal atresia and tracheoesophageal fistula
D. Omphalocele
E. Eastroshcisis

84. Which of the following is the most sensitive test for a pheochromocytoma?

A.24 h urine vanillymandelic acid


B.24 h urine metanephrine
C.24 h urine epinephrine
D.24 h urine norepinephrine

85. A double bubble on an air contrast upper gastrointestinal series in an infant is characteristic of?

A. Duodenal atresia
B. Jejunal atresia
C. Meconium ileus
D. Pyloric stenosis

86. After being injured by a bull on his mother's farm, a young man is placed in a cast for a supracondylar fracture of
his humerus. He begins to experience intense pain and weakness in the ipsilateral hand. Compartment pressures are
measured and are markedly elevated.
Which of the following statements regarding compartment syndromes following orthopedic injuries is true?

A. The first sign is usually loss of pulse in the extremity


B. Passive flexion of the extremity proximal to the involved compartment will aggravate the pain.
C. Surgical decompression is necessary only as a last resort.
D. These syndromes are most commonly associated with supracondylar fractures of the humerus and tibial shaft
E. The syndrome is often painless.
87. Appropriate treatment for a single episode of microscopic hematuria is:

A. Reassurance and serial urinalysis to confirm resolution


B. Urine culture and intravenous pyelogram alone
C. Cystoscopy alone
D. Imaging of upper tract and cystoscopy <=right

88. Transurenthral resection syndrome after endoscopic resection of the prostat occurs because of:

A. Air embolism during the procedure


B. Blood loss from postoperative irrigation
C. Absorption of irrigant
D. Relaxation of the urethral sphincter from spinal anesthesia

90. DIVD characterize by:

B. Decreased fibrinogene.
D. Decreased fibrin split product

91. A 65 yo woman has a life-threatening pulmonary embolus 5 days following removal of a uterine malignancy.she is
immediately heparinized and maintained in good therapeutic range for the next 3 days, then passes blood from her
vagina and develops tachycardia, hypotension, oliguria.Following resusitation, abdominal CT reveals a major
retroperitoneal hematoma, which of the following is the best next step in Management?

A. Immediately reverse heparin by calculated dose of protamine and place a vena cava filter (eg. A Greenfield filter)
B. Reverse heparing with protamin, explore and evacuate the hematoma and ligate the vena cava below the renal
veins
C. Switch to low-dose heparin
D. Stop heparin and observe closely
E. Stop heparin, give Fresh-Frozen plasma (FFP) and begin Warfarin therapy

92. A 42 year-old man sustains a gunshot wound to the abdomen and is in shock. Multiple units of packed red blood
cells (PRBC) are transfused in an effort to resuscitate him.He complains of numbness around his mouth and displays
carpopedal spasm and positive Chvostek sign. An ECG demonstrate a prolonged QT interval,
Which of the following is the most appropriate treatment?

A. Intravenous bicarbonate
B. Intravenous potassium
C. Intravenous calcium
D. Intravenous digoxin
E. Intravenous parathyroid hormone

93. A 59 years old woman undergoes an exploratory laparotomy for peritonitis and is found to have perforated
diverticulitis. She undergoes a sigmoid resection whith an end colostomy.she is administrated a third generation
cephalosporin within 1 hour prior to the incision and the antibiotic is continued postoperativly. One week later she
develops an intra abdominal abscess which is percutaneusly drained.bacteroid fragilis is isolated from the culture.
Which of the following statements regarding her perioperative antibiotic regimen most accurate?

A. The perioperative dose of antibiotics should have been given closer to the time of incision
B. The patient should have resieved several doses of antibiotics prior to laparotomy
C. The patient should have received 1 generation cephalosporin
D. The patient did not have adequate gram negative coverage
E. The patient did not have adequate anaerobic coverage
94. The standard surgical treatment of gastrointestinal stromal tumor (GIST) of the stomach is:

A. Endoscopic ablation
B. Wedge resection with clear margins
C. Subtotal gastrectomy
D. Total gastrectomy

95. A 32 years old medical student notes an asymptomatic mass in his testicle on examination the mass cannot be
transilluminated. The urolog says it is a seminoma.seminoma is accurately described by which of the following
statements?

A. It is the most common type of testicular cancer


B. Metastasis to the liver and bone are frequently found
C. It does not respond to radiation
D. The 5- year survival rate approaches to 50 %
E. Common presentation is that of painful lump that transilluminates

96. What percentage of breast cancers are caused by mutation in the BRCA gene?

A.<3%
B.5-10%
C.15-20%
D.25-30%

97. The most appropriate treatment of a woman with lobular carcinoma in situ (LCIS) involving one quadrent is:

A. Observation
B. Lumpectomy
C. Lumpectomy with radiation
D. Mastectomy

98. Which is of the following is a cause of cardiogenic shock in a trauma patient?

A. hemothorax
B. penetrating injury to the aorta
C. air embolism
D. iatrogenic increased afterlaod due to pressors

99. A 24 year-old man is brought into the ER after a fall from a ladder His breathing is labored, and be is cyanotic. No
breath sounds can be heard,even in the right lung field which is resonant to percussion.
The first step in the Management should be:

A. Cricothyroidotomy
B. Obtaining a stat chest X-ray
C. Passing an oral endotracheal tube
D. Tube thoracostomy.
100. A 19 years old man is admited to the emergancy department with a stab wound just below the right inguinal
ligament. There is a profuse bleeding from the wound and he is in shock.
The first step in local wound control should be to?

A. Apply comprresion of the bleeding vessel whith a gloved finger


B. Place a torniquiet on the right tight above the wound
C. Use clamps and ligaturs to control the bleeding
D. Wrap the wound and upper tighth in a bulky presure dressing

101.A 72 year-old man undergoes an aortobifemoral graft for symptomatic aortoiliac occlusive disease.the inferior
mesenteric artery (IMA) is ligated at its aortic attachment.24 hours aftar surgery the patient has abdominal
distention,fever,and bloody diarrhea.Which of the following is the most appropriate diagnostic study for this patient?

A. Aortogram
B. Magnetic resonance imaging (MRI)
C. Computer tomographic (CT) scan
D. Sigmoidoscopy
E. Barium enema

102. A 59 year-old women presents with right lower puadrant pain,nausea,and vomiting,she undergoes an
uncomplicated laparoscopic appendectomy.Postoperatively the pathology reveals a 2.5-cm mucinous
adenocarcinoma with lymphatic invasion. Staging workup, including colonoscopy,chest x-ray. And computed
tomography (CT) scan of the abdomen and pelvis,is negative.
Which of the following is the best next step in her management?

A. No further intervention at this time,follow-up every 6 months for 2 years


B. Chemotherapy
C. Neoadjuvant chemotherapy followed by right hemicolectomy
D. Ileocecectomy
E. Right hemicolectomy

103. A39 year-old previously healthy male is hospitalized for 2 weeks with epigastric pain radiating to his
back,nausea,and vomiting.Initial laboratory values revealed an elevated amylase level consistent with acute
pancreatitis.Five weeks following discharge,he complains of early satiety,epigastric pain,and fevers,On
presentation,his temperature is 38.9 c and his heart rate is 120 beats per minute,his white blood cell (WBC) count is
24.000/mm³ and his amylase level is normal.He undergoes a CT scan demonstrating a 6-cm by 6-cm rim-enhancing
fluid collection in the body of the pancreas.
Which of the following would be the most definitive management of the fluid collection?

A. Antibiotic therapy alone


B. CT-guided aspiration with repeat imaging in 2 to 3 days
C. Antibiotics and CT-guided aspiration with repeat in 2 to 3 days
D. Antibiotics and percutaneous catheter drainage
E. Surgical internal drainage of the fluid collection with a cyst-gastrostomy or

104. A 70 year-old woman has nausea,vomiting,abdominal distention,and episodic crampy midabdominal pain.She
has no history of previous surgery but has a long history of cholelithiasis for which she has refused surgery.Her
abdominal radiograph reveals a spherical density in the right lower quadrant.
Which of the following is the definitine treatment for this patient's bowel obstruction?

A. Ileocolectomy
B. Cholecystectomy
C. Ileotomy and extraction
D. Nasogastric (NG) tube decompression
E. Intravenous antibiotics
105. A 47 year-old asymptomatic woman is incidentally found to have a 5-mm polyp and no stones in her gallbladder
on ultrasound examination. Which of the following is the best management option?

A. Aspiration of the gallbladder with cytologic examination of the bile


B. Observation with repeat ultrasound examinations to evaluate for increase in polyp size
C. Laparoscopic cholecystectomy
D. Open cholecystectomy with frozen section
E. En bloc resection of the gallbladder,wedge resection of the liver,and portal lymphadenectomy

106. A 39 year-old Female with no significant medical history and whose only medication is oral contraceptive pills
(OCP), presents to the emergency room with right upper quadrant (RUQ) pain. CT demonstrates a 6-cm hepatic
adenoma in the right lobe of the liver. Which of the following is a definitive treatment of this lesion?

A. Cessation of oral contraceptive and a serial of CT scans


B. Intra-arterial embolization of the hepatic adenoma
C. Embolization of the right portal vein
D. Resection of hepatic adenoma
E. Systemic chemotherapy

107.A 40 year-old woman notices a skin rash around her areola.There is an erosive area about the nipple.There is
tenderness,itching,and intermittent bleeding. Which of the following statement is true regarding this entity?

A. It usually precedes development of Paget disease of bone


B. It presents with nipple-areolar eczematous changes
C. It does not involve axillary lymph nodes because it is a manifestation of intraductal carcinoma only
D. It accounts for 10% to 15% of all newly diagnosed breast cancers
E. It is adequately treated with wide excision when it presents as a mass

108. A 61 year-old woman with history of unstable angina complains of hematemesis after retening and vomiting
following a night of binge drinking.Endoscopy reveals a longitudinal mucosal tear at the gastroescophageal
janction,which is not actively bleeding which of the following is the next recommended step in the management of
this patient ?

A. Angiography with embolization


B. Baloon tamponede
C. Expbratory laparotomy gastroesotomy and oversewing the tear
D. Systemic vasopressin infusion
E. Expectant management
109. A 55 year-old man present with fever and pain in the perineal region upon farther questioning he also complains
of fregvence, urgency,dysuria,and a decreased urinary stream.on physical exam his abdominal is soft, non-distended
and non-tender, digital rectal exam. Demonstrates exquisite tenderness on the anterior aspect. Laporatory. exam
reveals leukocytosis and finding on urinary consistent with a bacterial infection which of the following is the most
likely diagnosis?

A. UTI
B. BPH (benign prostatic hyperglasis)
C. Prostatitis
D. Pylonephritis
E. Peri-anal abscess

110. 55 year old man complains of chronic intermittent epigastric pain. A gastroscopy demonstrates a 2 cm prepyloric
ulcer. Biopsy of the ulcer yields no malignant tissue. After a 6 weeks trail of medical therapy, the ulcer is unchanged.
Which of the following is the best next step in his management?

A. Repeat trail of medical therapy


B. Local excision of the ulcer
C. Highly selective vagotomy
D. Partial gastrectomy with vagotomy and Billroth I reconstruction
E. Vagotomy and pyloroplasty
01.03.2010
By: Alhag Abu Anzeh Muhammad

Part A:

1- an 8-mo-old girl is brought to your office with complaint that she has development delay. her head is enlarged, and
she feeds poorly and does not focus. you examine that baby and find her hypotonic and unable to sit. she has
considerable head lag. ad head circumference is above the 95 th percentile.
which of the following disease would consider the most probable diagnosis ?

a.birth asphyxia
b. familial hydrocephalus
c.rubella infection in utero
d.canavan disease
e.hunler disease

2- a serios complication of the treatment of hypernatremic dehydration is ?

a. cerebral thrombosis
b. cerebral edema
c. hypoglycemia
e. none of the above

3- the best feeding protocol for a temperamentally irregular infant is ?

a. a fixed schedule
b.one based on the parent's schedule
c. every 1-2 hr
d. one based on demand
e. 60 min for each feeding

4- during examination of an 8-mo-old child, the difficulty of optimal chest auscultation is due primarily to ?

a. recent meal with gastric distention


b. rapid respiratory rate
c. stranger anxiety
d. transmited nasal sounds
e. pliable chest wall

5- which of the following is a known complication of vascular ring?

a. renal failure
b. heart failure
c. cardiac arrhythmia
d. tracheomalacia
e. constipation
6- a 5-yr-old with AIDS who is receiving zidovudine, lamivudine, and ritonavir presents whith rapid respiratory
rate progressive dyspnea of 2 days duration. there is no fever. the chest radiograph reveals bilateral diffuse alveolar
disease. the most important diagnostic test whoud be?

a. CT scan of the chest


b. pharyngeal swab for viral and bacterial cultures
c. blood gas profile
d. serum lactate dehydrogenase level
e. bronchoalveolar lavage for cytology

7- A 6-yr-old girl who was previously healthy presents with a 1-wk history of nocturnal perianal itching
there are no other symptoms , and findings on physical examination are normal. the most appropriate therapy is ?

A. Bacitracin ointment to the perianal area


B. Diphenhydramine orally as needed for itching
C. Single oral dose of mebendazole repeated in 2 weeks
D. A 2 week course of amoxicillin/clavulanate
E. Ketoconazole in a single dose

8- A 4-mo-old child with vitamin D deficiency rickets would be expected to show all of the following except?

a. craniotabes
b. bowleg
c. rosary
d. low serum phosphate levels
e. high alkaline phosphatase leveles

9- A 5-yr-old boy who lives in an urban area is hospitalized because of low grade fever, flaccid
paralysis of both leges, sensory changes, and absent ankle deep tendon reflexes. the child received
only two immunizations with oral poliovaccine (OPV) , at the ages of 2 and 6 mo. the most probable diagnosis is?

a. acute paralytic poliomyelitis due to wild poliovirus


b. paralysis due to nonpolioenteroviruses
c. vaccine- associated poliomyelitis
d. guillain-barre syndrome
e. tick-bite paralysis

10- which of the following may be a manifestation of enterovirus infection?

a. ataxia
b. intususception
c. parotitis
d. herpangina
e. renal failure

21- a20-month-old child is brought to the ed because of fever and irritability and refusal to move his right knee
that resists-passive motion.
which of the following is the most important test to confirm the impression of septic arthritis:

A .x-ray of the knee


b. examination of the joint
c. erythrocyte sedimentation rate
d. complete blood count and differential
e. blood culture
22- as apart of your anticipatory guidance to new parents of healthy newborn you suggest putting the child in
which of the following position for sleep?

a.supine position
b.prone position
c.seated postion
d.trendelenburg postion
e.a hammock

23- amentally retarted 14-year old boy has along face, large ears ,micropenis,and large testes.
chromosome analysis is likely to demonstrate which of the following?

a.trisomy 21
b.trisomy 18
c.trisomy 13
d.fragile x syndrome
e.williams syndrome

24- amother wisches to breast-feed her newborn infant ,but is worried about medical conditions that would
prohibit her from doing so.you counsel her that of her listed conditions which of the following is acontraendication
to breast-feeding?

a.upper respiratory tract infection


b.cracked and bleeding nipples
c.mastitis
d.inverted nipples
e.hiv infection

25- afull-term infant is born after a normal pregnancy delivery however is complicated by marginal placental
separation .at 12 hours of age ,the child although appearing to be in good health ,passes a bloody meconium
stool .for determining the cause of the bleeding.
which of the following diagnostic procedures should be performed first?

a.a barium enema


b.an apst test
c.gastric lavage with the normal saline
d.an upper gastrointestinal series
e.a platelet count prothrombin time and partial thromboplastin time

26- during a regular checkup of an 8-year -old child you note a loud first heart sound with a fixed and
widely split second heart sound at the upper sternal border that does not change with respirations.the
patient is otherwise active and healthy.which of the following heart lesions most likely explains these findings?

a.atrial septal defect(ASD)


b.ventricular septal defect(VSD)
c.isolated tricuspidal regurgition
d.tetralgy of fallot
e.mitral valve prolapse
27- a2 year -old child has a 2 week history of splitting fevers which have been as high as 40. she has spindle-
shaped swelling of finger joints and complains of upper sternal pain.when she has fever , the parents note a faint
salamon -colored rash that resolves with the resolution of the fever.she has had no conjunctivitis or mucositis , but
her heart sound are muffled and she has increased pulsus paradoxus .
which of the following is the most likely diagnosis?

a.acute rheumatic fever


b.juvenile rheumatoid arthritis
c.toxic synovitis
d.septic arthritis
e.osteoarthritis

28- a 12 -yr-old girl is hospitalized in intensive care.she has a foley catheter and a urine culture is
positive for enterococcus .in addition to removing the catheter.
which antibiotics is the recommended agent for treatment of enterococcus infections?

a.penicillin
b.ampicillin
c.clindamycin
d.erthromycin
e.vancomycin

29- a 10 month -old infant has poor weight gain ,a persistent cough and a history of the several bouts of
pneumonitis . the mother describes the child as having very large foul -smelling stools for months. Which of the
following diagnostic maneuvers is likely to result in the correct diagnosis of this child?

a.ct of the chest


b.serum immunnoglobins
c.tb skin test
d.inspiratory and expiratory chst x-ray
e.sweeat chloride test

30- previously healthy active 18 month -old child presents with unilateral nasal obstruction and foul - smelling
discharge .the child's examination is otherwise unremarkable .which of the following is the most likely diagnosis?

a.foreign body
b.nasal polyps
c.frontal sinusitis
d.devaited septum
e.choanal atresia

31. A 5year old child with iron deficiency and severe episodes of grossly, bloody (maroon) stools.
Which is the most likely explanation for his anemia?

B: Meckel diverticulum.

32. A 7month old child recieves chron. total parenteral nutrition for short gut syndrom.
Which is the most typical complication of total parenteral nutrition?

A: Sepsis
33. A 14y old girls with a 9 month history of diarrhea, abdominal pain (periumbilical and postprandial), weight loss
and blood in stool. What is the most likely diagnose?

C: Chron’s disease

34. Two weeks after a viral syndrome a 2year old develops bruising and petechia more prominent
over both legs. Examination shows no hepatosplenomegaly or lymph node enlargement. Blood tests
show normal haemoglobin, haematocrit and WBC count and 15.000 Thrombocytes. What is the most likely diagnosis?

D: Idiopathic thrombocytopenic purpura

35.On a routine screening for complet bloodtest a 1year old noted to have a microcytic anemia. A follow up
haemoglobin electrophorese demonstrates increased cencentration of haemoglobin A2.
The child is most likely to have?

B:Beta thalassemia trait.

36. Given folid acid to a pregnant woman prevents the child from having which defect?

D:Neural tube defect.

37. A 5year old boy has bloody urin, puffy eyes and headache for one day. He had fever and a sore throat 10
days ago which resolved without treatment. What is the most likely diagnosis?

D:Post infectious glomerulo nephritis.

38. A 18month old has 10 hours of fever and irritability. CSF 2000 WBC 90% polymorphnuklear. Spinal fluid
is depressed. He is treated with Cefotaxim and Vancomycin. 8 hours after treatment he gets general convulsions.
What is the next step?

D:Order sodium serum concentration.

39. A 6year old girl comes to the hospital after convulsion. She is alert without distress. On her skin you notice
caf‫י‬-au-lait spots, a Shagreen patch and periungal fibromas.
Which is the most likely diagnosis?

E:Addison disease

40. Which of the following is an effective screening test for T-cell function?

a. Absolute lymphocyte count


b. Flow cytometry for CD4 (helper) and CD8 (cytotoxic) T cell
c. Respiratory burst assay
d. Candida skin test
e. Mumps antibody titer after mumps vaccination
41. Which of the following is a complication of intravenous immunoglobulin (IVIG) Therapy ?

a.Anaphylaxis
b.Renal failure
c.Trasmission of HIV
d.Hepatic toxicity
e.Hyperglycemia

42. The treatment of choice for a 3-years old boy suffering from acute otitis media is :

a.Azythromycin
b.Gentamycin
c.Chloramphenicol
d.Erythromycin
e.Amoxieillin

43. The state laboratory calls your office telling you that a newborn infant , now 8 days old, has an elevanted
thyroid stimulating hormone (TSH) and low thyroxin (Tn) on his newborn screen, If this condition is left
untreated , the infant is likely to demonstrate which of the following in the first few months of life?

a.Hyperreflexia
b.Hyperirritability
c.Diarrea
d.Prolonged jaundice
e.Hyperphagia

44. A 18-years old girl has hepatosplenomegaly, an intention tremor, disarthria, dystonia and
deterioration of her school performance, she also developed abnormal urine with excess glucare ,
protein and uric acid, she has a several-year history of elevated liver enzymes of unknown etiology .
Which of the following best explains her condition?

a.Galactosemia
b.α1-antitrypsin deficiency
c.Gilbert syndrome syndrome
d.Dubin Johnson
e.Wilson disease

45. A 12-years-old boy has scent, long, slightly pigmented pubic hairs, slight enlargement of his penis , a pink testure
and enlarged scrotum, He is most likely at Which tanner stage?

a.1
b.2
c.3
d.4
e.5

46. An 18-years old male collage student is seen in the student health clinic for urinary frequency, dysuria, and
urethral discharge . Which of the following is likely to explain this condition?

a.Herpex simplex
b.Escherichia coli urinarytract infection
c.Clamydial urethritis
d.Syphilis
e.Hiv infection
47. Allergy to Which of the following food products is the most common in children?

a.Peanut
b.Sesame
c.Egg
d.Fish
e.Cow's milk

48. An 8-years old -----------next action ?

a.Explain that is a mild side effect of the drug .


b.Give the child oral anti histamine .
c.Inject corticosteroids IV
d.Inject epinefrin IM
e.Intubate and start mecchenical ventilation

49. The parents -----------next action ?

a.Secondary bacterial infection


b.Bathing with tepid water
c.Application of steroid scream before the emollient Cool,
d.humid ambient temperatures
e.Excessive use of Lubrification

50. A 1-month old infant who is otherwise and well and asymptomatic , has been brought in because of
a rash . the child has inflammatory scalp scale which has a combination of greasy, yellow and
erythematous features , as well as infiammation of the neck, axillary and inguinal creases.
the most likely diagnosis is?

a.Atopic dermatitis
b.Seborrheic dermatitis
c.Psoriasis
d.Langerhans cell histiocytosis
e.Candidiasis

51. a 17-year-old girl is seen by her primary care physician for the evaluation of left lower quadrant pain. The
physician felt a pelvic mass on physical examination and ordered a pelvic ultrasound. You are consulted because an
ovarian neoplasm is identified by the ultrasound.
Which of the following is the most common ovarian tumor in this type of patient?

a. Germ cell
b. Papillary serous cyst epithelial
c. Fibrosarcoma
d. Brenner tumor
e. Sarcoma botryoides

52. Which is the narrowest diamerter of the pelvic inlet through which the fetal head must pass?

a. True conjugate
b. Diagonal conjugate
c. Transverse conjugate
d. Obstetrical conjugate
53. The HCG receptor is also the receptor for ehat other hormone?

a. LH
b. FSH
c Estriol
d. Progesterone

54. Highest levels of hPL (human placental lactogen) can be found in which of the following?

a. Fetal urine
b. Fetal serum
c. Amniotic fluid
d. Maternal serum

55. Which of the following is decreased during normal pregnancy?

a. Glomerular filtration rate


b. Renal plasma flow
c. Creatinine clearance
d. Serum concentration of urea nitrogen

56. At what level does compression of the ureters by the gravid uterus occur?

a. Bladder trigone
b. Pelvic brim
c. Sacrospinous ligaments
d. Ureterovescical j unction

57. Where oxytocin is primarily synthesized?

a. Adrenal gland
b. Placenta
c. Posterior pituitary
d. Ovary

58. A 21-year-old woman returns to your office for evaluation of an abnormal Pap smear. The Pap smear
showed squamous abnormality suggestive of a high-grade squamous intraepithelial lesion (HGSIL).
Colposcopy confirms the presence of a servical lesion consistent with severe cervical
dysplasia (CIN III).
Which of the following human papilloma virus (HPV) types is most often associated with this type of lesion?

a. HPV type 6
b. HPV tybe 11
c. HPV tybe 16
d. HPV tybe 42
e. HPV tybe 44

59. Isolated fetal structural defects are most commonly found in which of the following organ?

a. Liver
b. Heart
c. Bladder
d. Neural tube
60. What is the lie if the fetal and maternal axes cross at a 45-degree angle?

a. Longitudinal
b .Breech
c. Oblique
d. Transverse

61- what is the most common position of the fetal vertex as it enters the pelvis?

A.Right occipitoanterieor (ROA)


B.Right occipitotransverse (ROT)
C. Left occipitoanterieor (LOA)
D. Left occipitotranverse ( LOT )

62-what is the primary mechanism of placental site hemostasis?

A.Vasoconstriction by contracted uterus


B.Prostaglandins secretion
C.Maternal hypotension
D.Decreased cardiac output

63- what happens to perinatal mortality after 42 weeks gestation?

A.Markedly decreased
B. Slightly decreased
C. No change
D. Increased

64- A woman with chronic hypertension was prescribed alpha-methyldopa at 24 weeks gestation.
what is the mechanism of action of alpha-methyldopa?

A.Relaxes arterial smooth muscles


B.Increased sodium and water diuresis
C.Increased peripheral vascular resistance
D.Acts centrally to decrease vascular tone

65- which of the following techniques is best for the diagnosis of fetal cytomegalovirus infection?

A. Amniocentesis or cordocentesis
B. Chorionic villus sampling
C. Ultrasound
D. Magnetic resonance imaging
66- A 32-year-old woman presents to the emergency room complaining of severe lower abdominal
pain. On bimanual pelvic examination. Bilateral adnexal masses are palpated. The patient is sent to
the ultrasound department, and a transvaginal pelvic ultrasound demonstrated bilateral tubo-ovarian
abscesses. Which of the following is the most appropriate next step in the management of this patient?

A.Admit the patient for emergent laparoscopic drainage of the abscesses


B.Call interventional radiology to perform CT-guided percutaneous drainage of the abscesses
C.Send the patient home and arrange for intravenous antibiotics to be administered by a home health agency
D.Admit the patient for intravenous antibiotic therapy
E.Admit the patient for exploratory laparotomy,TAH/BSO

67- A55-year-old G3P3 with a history of fibroids presents to you complaining of irregular vaginal
bleeding. On physical examination, her uterus is 12 weeks in size and irregularly shaped. Her ovaries are not
palpable, A urine pregnancy test is negative.
Which of the following is the most reasonable next step in the evaluation of this patient?

A.Schedule her for hysterectomy


B.Insert a progesterone-containing intrauterine device (IUD)
C.Arrange for outpatient endometrial ablation
D.Perform an office endometrial biopsy
E.Arrange for outpatient conization of the cervix

68- A couple present to your office to discusspermanent sterilization. They have three children and are sure they do
not want any more. You discuss the risk and benefits of surgical sterilization.
Which of the following statements is true regarding surgical sterilization?

A.They cannot be performed immediately postpartum


B. They have become the second most common method of contraception
C.They can be considered effective immediately in females (bilateral tubal ligation)
D.They can be considered effective immediately in males (vasectomy)
E.Tubal ligation should be performed in the secretory phase of the menstrual cycle

69- Patients with Turner’s syndrome will most commonly display which of the following endocrine profiles?

A.Elevated TSH, normal FSH, normal prolactin


B.Elevated FSH, normal TSH, normal prolactin
C.Elevated prolactin, normal TSH, normal FSH
D.Normal FSH, normal TSH, normal prolactin

70- A 15 year old patient with a diagnosis of Mayer-Rokitansky-Kuster-Hauser syndrome is referred to you
because of cyclic pelvic pain. You may counsel the patient that:

A.The pain cannot be real because she does not have any pelvic structures.
B.She does not require any imaging of the urinary tract.
C.Skeletal abnormalities are uncommon with this disorder.
D.Her pain may be related to the presence of a rudimentary mullerian structure with functioning endometrium.
71- A 46 year old woman comes to your office worried about her decreased sexual desire and
perimenopausal symptoms. Her medical and surgical history are significant only for right salpingo-
oophorectomy 10 years ago. Upon performing her annual exam, you feel left adnexal fullness, and
pelvic sonogram shows an 8-cm cyst. After discussion of potential removal of the ovary, the patient is
very worried that removal of her remaining ovary will leave her with no testosterone. You tell her that:

A.Nearly all testosterone production is from her remaining ovary and her concerns are justified.
B.One fourth of her testosterone production is fron her ovary, one fourth from her adrenals, and the remaining
C.half from peripheral conversion.
D.Testosterone production is not linked to a woman’s ovaries.
E.Given that she is perimenopausal, she likely has little testosterone circulating anyway.

72- Which of the following is the best explanation for the mechanism of the action of the intrauterine device
(IUD)?

a. Hyperperistalsis of the fallopian tubes accelerates oocyte transport and prevents fertilization
b. A subacute or chronic bacterial endometritis interferes with implantation
c. Premature endometrial sloughing associated with menorrhagia causes early abortion
d. A sterile inflammatory response of the endometrium prevents implantation
e. Cervical mucus is rendered impenetrable to migrating sperm

73-A 50 year old healthy woman presents to the office for a routine gynecologic visit. She
complains of new onset episodes of intense heat through her neck and chest followed by periods of
profuse sweating. She is concerned because these episodes interfere with her ability to work and also
wake her up from sleep. What is the most effective therapy for the symptoms the patient is describing?

a.Venlafaxine
b.Clonidine
c.Estrogen replacement
d.Black cohosh

74- A 62 year old woman presents to your office complaining of discomfort and pain with
intercourse, which she notes has developed over the past few years. A physical exam reveals atrophic vaginal
mucosa. What initial therapy would you recommend for this patient?

a.A vaginal estrogen


b.Transdermal estrogen
c.Marriage counseling
d.Oral hormone replacement therapy

75- While evaluating a 30-year-old woman for infertility, you diagnose a bicornuate uterus. You explain that
additional testing is necessary because of the woman’s increased risk of congenital anomalies in which organ system?

a. Skeletal
b. Hematopoietic
c. Urinary
d. Central nervous
e. Tracheoesophageal
76- In an amenorrheic patient who has had pituitary ablation for a craniopharyngioma, which of the following
regimens is most likely to result in an ovulatory cycle?

a. Clomiphene citrate
b. Pulsatile infusion of gonadotropin-releasing hormone (GnRH)
c. Continuous infusion of GnRH
d. Human menopausal or recombinant gonadotropin
e. Human menopausal or recombinant gonadotropin followed by human chorionic gonadotropin (hCG)

77- During the Ovulation of infertility in 25 year old female .... showed evidium of of Asherman syndrome.
which one of the following smptoms would be expect in the Patient:

a. Hypomenorrhea
b. Oligomenorrhea
c. Menorrhagia
d. Metrorrhagia
e. Dysmenorrhea

78- A 22-year-old woman consults you for treatment of hirsutism. She is obese and has facial acne and
hirsutism on her face and periareolar regions and a male escutcheon. Serum LH level is 35 mIU/mL and FSH
is 9 mIU/mL. Androstenedione and testosterone levels are mildly elevated, but serum DHAS is normal. The
patient does not wish to conceive at this time.
Which of the following single agents is the most appropriate treatment of her condition?

a. Oral contraceptives
b. Corticosteroids
c. GnRH
d. Parlodel
e. Wedge resection

79- A 19-year-old patient presents to your office with primary amenorrhea. She has normal breast and pubic hair
development, but the uterus and vagina are absent. Diagnostic possibilities include:

a. XYY syndrome
b. Gonadal dysgenesis
c. Mullerian agenesis
d. Klinefelter syndrome
e. Turner syndrome

80 - A 54-year-old woman undergoes a laparotomy because of a pelvic mass. At exploratory laparotomy, a unilateral
ovarian neoplasm is discovered that is accompanied by a large omental metastasis. Frozen section diagnosis confirms
metastatic serous cystadenocarcinoma. The most appropriate intraoperative course of action is:

a. Excision of the omental metastasis and ovarian cystectomy


b. Omentectomy and ovarian cystectomy
c. Excision of the omental metastasis and unilateral oophorectomy
d. Omentectomy and bilateral salpingo-oophorectomy
e. Omentectomy, total abdominal hysterectomy, and bilateral salpingooophorectomy
81) which of the following is the major cause of unplanned pregnancies in women using oral contraceptives?

a) break through ovulation at midcycle


b) high frequency of intercorse
c) incorrect use of oral contraceptives
d) gastrointestinal malabsorption
e) development of antibodies

82) A postmenopausal woman presents with pruritic white lesions on the vulva. Punch biopsy of a representative area
is obtained. Which of the following histologic findings is consistent with the diagnosis of lichen sclerosis?

a. Blunting or loss of rete pegs


b. Presence of thickened keratin layer
c. Acute inflammatory infiltration
d. Increase in the number of cellular layers in the epidermis
e. Presence of mitotic figures

83) A 65-year-old woman presents to your office for evaluation of genital Prolapse.
Which of the following factors is least important in the subsequent development of genital prolapsed in this patient?

a. Chronic cough
b. Chronic constipation
c. Chronic hypertension
d. Childbirth trauma
e. Menopause

84) An intrauterine pregnancy of approximately 10 weeks gestation is confirmed in a 30-year-old G5P4 woman with
an IUD in place. The patient expresses a strong desire for the pregnancy to be continued. On examination, the string
of the IUD is noted to be protruding from the cervical os.
Which of the following is the most appropriate course of action?

a. Leave the IUD in place without any other treatment.


b. Leave the IUD in place and continue prophylactic antibiotics throughout pregnancy.
c. Remove the IUD immediately.
d. Terminate the pregnancy because of the high risk of infection.
e. Perform a laparoscopy to rule out a heterotopic ectopic pregnancy.

85) A 62-year-old woman presents for annual examination. Her last spontaneous menstrual
period was 9 years ago, and she has been reluctant to use postmenopausal hormone replacement because of a strong
family history of breast cancer. She now complains of diminished interest in sexual activity.
Which of the following is the most likely cause of her complaint?

a. Decreased vaginal length


b. Decreased ovarian function
c. Alienation from her partner
d. Untreatable sexual dysfunction
e. Physiologic anorgasmia
86) A 70-year-old man with a dementing disorder dies in a car accident. During the previous 5 years, his personality
had dramatically changed and he had caused much embarrassment to his family because of his intrusive and
inappropriate behavior. Pathological examination of his brain shows frontotemporal atrophy, gliosis of the frontal
lobes’ white matter, characteristic intracellular inclusions, and swollen neurons. Amyloid plaques and neurofibrillary
tangles are absent. Which of the following is the most likely diagnosis?

a. Alzheimer disease
b. Pick disease
c. Creutzfeldt-Jakob disease
d. B12 deficiency dementia
e. HIV dementia

87) 24-year-old previously healthy man is brought to the emergency room after he began yelling that people on the
bus were out to hurt him. In the emergency room, he is agitated, hypervigilant, and anxious. He is unable to give
much history other than to say that he is a graduate student and nothing like this has ever happened before.
Which of the following is the most likely cause of this behavior?

a. Delirium
b. Pick disease
c. Dissociative disorder
d. Vitamin B12 deficiency
e. Cocaine intoxication

88) emaciated and lethargic 16-year-old girl arrives at the emergency room. Her blood pressure is 75/50, her heart
rate is 52 beats per minute, her potassium is 2.8 meq/L, and her bicarbonate is 40 meq/L. The girl’s parents
report that she has lost 35 lb in 3 months but is still convinced that she is overweight. She eats only very small
amounts of low-calorie food, and she runs 2 to 3 hours every day.
What other activities is this patient most likely to have engaged in?

a. Sexual promiscuity
b. Ethanol abuse
c. Purging
d. Wearing tight clothes
e. Shoplifting

89) A 23-year-old man comes to the physician with the complaint that his memory has worsened over the past 2
months and that he has difficulty concentrating. He has lost interest in his friends and his work. He has difficulty with
abstract thoughts and problem solving. He has also felt depressed. MRI scan shows parenchymal abnormalities.
Which of the following is the most likely diagnosis?

a. Alzheimer disease
b. Vascular dementia
c. HIV-related dementia
d. Lewy body disease
e. Binswanger disease

90) which of the following is the most common cause of delirium in elderly?

a) substance abuse
b) accidental poisining
c) hypoxia
d) use of multiple medication
91. a58 year old woman with chronic mental disorder ,comes to the physician with irregular choreoathetoid
movements of her hands and trunk .she states that the movments get worse under stressful conditions.
which of the following medications is most likely to have caused this disorder:

a-fluoxetine
b-perphenazine
c-diazepam
d-phenobarbitol
e-clozapine

92. 62 year old man with chronic schizophrenia is brought to the ER after he is found wandering
around his halfway house confused and disoriented.his serum Na is 123 meq/l and urine Na is 5 meq/l the patient
has been treated with risperidone 4mg/day for the past 3 years with good symptoms control .his roommate reports
that the patient often complain of feeling thirsty .which of the following is most likely cause of this symptoms:

a-renal failure
b-inappropriate ( ADH ) secretion
c-addison disease
d-phsychogenic polydipsia
e-nephrotic syndrome

93. 75 years old man is being cared for in hospic setting. He has widely spread prostatic carcinoma and is
considered terminal. Which of the following psychiatric symptoms are seen in 90% of all terminal patients:

a-delusions
b-hallucinations
c-flight of ideas
d-anxiety
e-depression

94. a 25 year old man comes to the psychiatrist with chef complaint of depressed mood for I month . his mother ,to
whom he was very close died 1 month ago and since that time he has felt sad and been very tearful . he has
difficulty concentrating, has lost 1.5 kg and is not sleeping soundly through the night.
which of the following is the most likely diagnosis:

a-major depression
b-disthymia
c-post traumatic stress disorder
d-adjustment disorder
e-uncomplicated bereavement

95.how long after stroke is a patient at higher risk of developing a depressive disorder:

a-2 weeks
b-2months
c-6months
d-1year
e-2years
96. a 27 years old woman has been feeling blue for the past 2 weeks. She has little energy and has
trouble concentrating. She states that 6 weeks ago she had been feeling very good with lots of energy
and no need for sleep. She says that this pattern has been occurring for at least the past three years
though the episodes have never been so sever that she couldn’t work.
which of the following is the diagnosis:

a- border line personality disorder


b-seasonal affecting disorder
c-cyclothymic disorder
d-major depression,recurrent
e-bipolar disease, depressed

97. which of the following is a relative contraindication for ECT:

a-space occupaing lesion in the brain


b-pregnancy
c-hypertension
d-seizure disorder
e-post myocardial infarction 6 months earlier

98. 32 year old man is being treated for major depression. Which of the following symptoms. If present is one of the
most accurate indicators of long term suicidal risk:

a-revenge fantasies
b-presence of rage in the patient
c-hopelessness
d-presence of guilt
e-the patient has need for punishment

99. 28 years old taxi driver is chronically consumed by fears of having accidentally run over a
pedestrian . although he tries to convince himself that his worries are silly, his anxiety continues to mount until he
drives back to the scene of the accident and proves to himself that no body lies hurt in the street . this behavior
exemplifies which of the following:

a-a compulsion secondary to obsession


b-an obsession triggered by a compulsion
c-a delusional ideation
d-a typical manifestations of obsessive compulsive personality disorder
e-a phopia

100. 33 years old man comes to his physician for an HIV test ,the test is positive. The patient has no signs or
symptoms of AIDS.
which of the following psychiatric diagnosis may develop in as many as 25% of patient informed of positive HIV test:

a-adjustment disorder with anxiety


b-PTSD
c-bipolar disorder, manic
d-panic disorder
e-hypochondriasis
101. every 4 to 5 weeks a usually well functioning few day irritation:

d. pre menstrual

102. A young woman presents to the emergency room vomiting bright red blood. Once she is medically stable, the
intern who performs her physical examination notices that the enamel of her front teeth is badly eroded and her
parotid glands are swollen. Which of the following best describes the source of these medical complications?

a. Inadequate caloric intake


b. Purging
c. Laxative abuse
d. Diuretic abuse
e. Ipecac toxicity

103. A 52-year-old woman is diagnosed with breast cancer that is metastatic to her bones. She comes to the
psychiatrist for help in managing her depressed mood and anxiety secondary to this diagnosis. Which of the following
would most likely indicate an increased vulnerability to suicide if found in this patient, in addition to her anxiety and
depressed mood?

a. The extent of the cancer’s spread to her bones


b. The location of the bone metastases to her bones
c. A feeling of a loss of control
d. How much pain she has with the metastases (even though it is currently wellcontrolled)
e. A history of social phobia

104. A 36-year-old woman comes to the emergency room after she is found unresponsive at a party. Urine
toxicology is positive for cocaine. Upon awakening, the patient is interviewed and is found to have a history
consistent with cocaine abuse. Which of the following is this patient at an increased risk for having?

a. Death by suicide
b. Avoidant personality disorder
c. ADHD
d. Bipolar disorder--mania
e. Mild mental retardation

105. A 16-year-old male with a long record of arrests for breaking and entering, assault and battery, and drug
possession is found dead in his room with a plastic bag on his head. For several months he had been experiencing
headaches, tremors, muscle weakness, unsteady gait, and tingling sensations in his hands and feet. These symptoms
(and the manner in which the boy died) suggest that he was addicted to which of the following substances?

a. PCP
b. Cocaine
c. Methamphetamine
d. An inhalant
e. Heroin
107. A 28-year-old business executive sees her physician because she is having difficulty in her new
position, as it requires her to do frequent public speaking. She states that she is terrified she will do or say something
that will cause her extreme embarrassment. The patient says that when she must speak in public, she becomes
extremely anxious and her heart beats uncontrollably. Based on this clinical picture.
which of the following is the most likely diagnosis?

a. Panic disorder
b. Avoidant personality disorder
c. Specific phobia
d. Agoraphobia
e. Social phobia

108. Benzodiazepines, barbiturates, and many anticonvulsants exert their influence through which of the following
types of receptors?

a. Muscarinic
b. Dopaminergic
c. Glutamic
d. Adrenergic
e. Aminobutyric acid (GABA)-allergic

109. After being struck on the head by a four-by-four piece of wood, a previously serious and
dependable construction worker starts making inappropriate sexual remarks to his coworkers, is easily distracted,
and loses his temper over minor provocations. What part of his brain has most likely been damaged?

a. Occipital lobe
b. Temporal lobe
c. Limbic system
d. Basal ganglion
e. Frontal lobe

110. A 17-year-old boy is brought to the emergency room by his friends after he “took a few pills” at a
party and developed physical symptoms, including his neck twisting to one side, his eyes rolling upward, and his
tongue hanging out of his mouth. The patient responds immediately to 50 mg of diphenhydramine intramuscularly
with the resolution of all physical symptoms. Which of the following substances is most likely to have caused the
symptoms?

a. Methamphetamine
b. Meperidine
c. Alprazolam
d. Methylphenidate
e. Haloperidol
Part B:

1. A 43-year-old man is admitted to the neurology service after he went blind suddenly on the morning of
admission. The patient does not seem overly concerned with his sudden lack of vision. The only time he
gets upset during the interview is when he is discussing his mother’s recent death in Israel--he was
supposed to bring his mother to the United States, but did not because he had been using drugs and did
not save the necessary money. Physical examination is completely negative.
Which of the following is the most likely diagnosis?

a. Conversion disorder
b. Hypochondriasis
c. Factitious disorder
d. Malingering
e. Delusional disorder

2. A 69-year-old man with a diagnosis of delirium has symptoms of psychosis which include: frightening auditory and
visual hallucinations and paranoid delusions.
Which of the following medications should be chosen first for this man’s symptoms?

a. Haloperidol
b. Quetiapine
c. Valium
d. Olanzepine
e. Ziprasidone

3. A 34-year-old man recurrently perceives the smell of rotten eggs. This kind of hallucination is most commonly
seen in patients with which of the following diagnoses?

a. Parietal tumors
b. Narcolepsy
c. Grand mal epilepsy
d. Partial complex seizures
e. Wilson disease

4. A 25-year-old woman is diagnosed with schizophrenia when, after the sudden death of her mother, she
begins complaining about hearing the voice of the devil and is suddenly afraid that other people are out to
hurt her. Her history indicates that she has also experienced a 3-year period of slowly worsening social
withdrawal, apathy, and bizarre behavior. Her family history includes major depression in her father.
Which of the following details of her history leads the physician to suspect that her outcome may be poor?

a. She is female.
b. She was age 25 at diagnosis.
c. She had an acute precipitating factor before she began hearing voices.
d. She had an insidious onset of her illness.
e. There is a history of affective disorder in her family

5.The first symptoms of ADHD to remit is usually:

a. hyperactivity
b.Distractivity
c. carless mistacks in schoolwork
d. Impulsivity
e. learning difficulties
6. A 45-year-old man is diagnosed as having diabetes and will require insulin. His physician explains the use of the
medication and tells the patient that he will need to be seen at frequent intervals until his
glucose levels come under good control. The patient has always been somewhat hostile with the
physician, but upon hearing this news, he says angrily, “You doctors are always the same! You always want control--of
my time, of my money, and now of my every action!” As far as the physician knows, this patient has never had an
unpleasant encounter with a physician before.
Which of the following is the most likely explanation for the patient’s reaction to his doctor?

a. The patient is becoming delusional.


b. The patient is experiencing transference to this authority figure.
c. The patient is splitting.
d. The patient is becoming manic.
e. The patient is anticipating being rejected by his physician

7. A 24-year-old woman comes to the psychiatrist with a 2-month history of short episodes of “feeling
like I am going to die.” During these episodes, she also notes feelings of dizziness and nausea, along with a
feeling of choking. She describes these episodes as very frightening and she is terrified of having another. She denies
substance use of any medical problems. Which of the following treatment regimens should be started?

a. Imipramine
b. Fluoxetine
c. Phenelzine
d. Paroxetine and alprazolam
e. Buspirone and citalopram

8.Which of the following is not an appropriate pharmacologic treatment for tic disorder?

a. methylphenitate
b. guanfacine
c. risperidone
d. clonidine
e. haloperidol

9. A demanding 25-year-old woman begins psychotherapy stating that she is both desperate and bored. She
reports that for the past 5 or 6 years she has experienced periodic anxiety and depression and has made several
suicidal gestures. She also reports a variety of impulsive and self-defeating behaviors and sexual promiscuity. She
wonders if she might be a lesbian, though most of her sexual experiences have been with men. She has abruptly
terminated two previous attempts at psychotherapy. In both cases she was enraged at the therapist because he was
unwilling to prescribe anxiolytic medications. Which of the following is the most likely diagnosis?

a. Dysthymia
b. Histrionic personality disorder
c. Antisocial personality disorder
d. Borderline personality disorder
e. Impulse control disorder not otherwise specified

10. A 30-year-old woman is diagnosed as bipolar. At the same time that this illness is diagnosed, it is discovered
that she is pregnant. Which of the following drugs has the highest risk to the fetus if used?

a. Valproic acid
b. Lithium
c. Chlorpromazine
d. Haloperidol
e. Fluoxetine
11) five days after an uneventful cholecystectomy,an asymptomatic middle aged woman is found to have a
serum level of 125 mEq/L. which of the following is the most appropriate management strategy for this patient?

a.Administration of hypertonic saline solution


b.Restriction of free water
c.Plasma ultrafiltration
d.Hemodialysis
e.Aggressive diuresis with furosemide

12) patient with a nonobstructing carcinoma of the sigmoid colon is being prepared for eective resection .
Which of the following reduces the risk of postoperative infectious complications?

a.A single preoperative parenteral dose of antibiotic effectiv e against aerobes and anaerobes
b.Avoidance of oral antibiotics to prevent emergence of Clostridium duffucile
c.Postoperative administration for 48 h of parenteral antibiotics effective against aerobes and anaerobes
d.Postoperative administraion of parenteral antibiotics effective against aerobes and anaerobes until the patient's
intravenous lines and all other drains are removed
e.Redosing of antibiotics in the operating room if the case lasts for more than 2 h

13) 68 year old man is admitted to the coronary care unit with an acute myocardial infraction.His
postinfraction course is marked by congestive heart failure and intermittent hypotention .On the fourth day in
hospital,he develops severe midabdominal pain.On physical examination,blood pressure is 90/60 mm Hg and pulse is
110 beats per minute and regular;the abdomen is soft with mild generalized tenderness and distention.Bowel sounds
are hypoactive:stool Hematest is positive.
Which of the following is the most appropriate next step in this patient's management?

a.Barium enema
b.Upper gastrointestinal series
c.Angiography
d.Ultrasonography
e.Immediate celiotomy

14)A 65 year old manundergoes a low anterior resection for rectal cancer. On the fifth day in hospital,his
physical examination shows a temperature of 39 C(102 F),blood pressure of 150/90 mm Hg,pulse of 110
beats per minute and regular,and respiratory rate of 28 breaths per minute.Acomputed tomografhy (CT) scan
of the abdomen reveals and abscess in the pelvis.
Which of the following most accurately describes his present condition?

a.SSi-surgical site infection


b.Sepsis
c.Svere sepsis
d.Sptic shock
e.Svere septic shock

15)A cirrotic patient with abnormal coagulation studies due to hepatic synthetic dysfunction requires an urgent
cholecystectomy. A transfusion of FFP is planned to minimize the risk of bleeding due to surgery.
What is the optimal timing of the transfusion?

a)The day before surgery


b)The night before surgery
c)On call to surgery
d)Intraoperatively
e)In the recovery room
16)A 42 year old man sustains a gunshot wound to the abdomen and is in schok.Multiple units of packed red blood
cells are transfused in an effort to resuscitate him.He complains of numbness around his mouth and
displays carpopedal spasm and a positive Chvostek sign.An electrocardiogram demonstrates a prolonged OT interval.
Which of the following is the most appropriate treatment?

a)Intravenous bicarbonate
b)Intravenous potassium
c)Intravenuos calcium
d)Intravenous digoxin
e)Intravenous parathyroid hormone

17)A teenage boy falls from his bicycle and is run over by a truck.On arrival in the emergency room(ER),he is awake
and alert and appears frightened but in no distress. The chest radiograph suggests an air-fluid level in the left lower
lung field and the nasogastric tube seems to coil upward into the next best step in his management?

a)Placement of a left chest tube


b)Thoracotomy
c)Laparotomy
d)Esofagogastroscopy
e)Diagnostic peritoneal lavage

18)A 65 year old man who smokes cigarettes and has chronic obstructive pulmonary disease falls and
fractures the third, fourth, and fifth ribs in the left anterolatral chest. Chest x-ray is otherwise normal.
Which of the following would be the most appropriate next step in his management?

a)Strapping the chest with adhesive tape


b)Admission to the hospital and treatment with oral analgesia
c)Tube thoracostomy
d)Placement of an epidural for pain management
e)Surgical fixation of the fractured ribs

19)A 36 year old man who was hit by a car presents to the ER with hypotension.On examination,he has tenderness
and bruising over his left lateral chest below the nipple.An ultrasound examination is performed and reveals free fluid
in the abdomen. What is the most likely organ to have been injured in this patient?

a)Liver
b)Kidney
c)Spleen
d)Intestine
e)Pancreas

20) An 18 year old male was assaulted and sustained significant head and facial trauma.
Which of the following is the most common initial manifestation of increased intracranial pressure?

a)Change in level of consciounsness


b)Ipsilateral(side of hemorrhage) pupillary dilation
c)Contralateral pupillary dilation
d)Hemiparesis
e)Hypertension
31) what statement is regarding the treatment of pancreatic carcinoma?

a. is more common in the body of the gland


b. surgery in well slected patients is the most effective treatment.
c. in case of painless jaundice the carcinoma is not resecteble.
d. it can be resectble only if localized in body or tail.
e. is associated with diabetes insipidus.

32) 34 years old woman with complicated appendicectomy after acute appenditis the biopsy demonstrate a 1 cm
carcinoid tumor. The best management in this case?

a. hemycolectomy only
b. hemycolectomy with chemiotherapy
c. chemiotherapy only
d. wide resection only
e. no further treatment

33) 88 years old with terminal renal failure, coronary disease and brain metastasis complains acute colecistitis.
What is the best treatment of the colecistitis in this patient?

a. tube colecistostomy
b. open colecistectomy
c. laparoscopic colecistectomy
d. intravenous antibiotics and colecistectomy
e. lithotripsy and long term bile acid therapy.

34) what complication is associated with pancreasectomy?

a. diabetes mellitus
b. hypercalcemia
c. hyperphosphatemia
d. constipation
e. weight gain

35) what statement is true regarding hepatic hemangioma?

a. is the most common benign tumor in liver


e. the primary treatment is surgery

36) in 75 years old man with asymptomatic carotid built. what is the best test for is evaluation?

a. transcranial Doppler
b. eco Doppler
c. computerized tomography angiography
d. arch aortogram
e. magnetic resonance angiography.

37) 64 years old man with a story of a triple bypass 2 years ago actually treated with thiazide diuretic and aspirin.
The correct statement regarding antiplatelet therapy?

a. aspirin is not an effective antiplatelet therapy


b. antiplatelet work mechanism is by enhancing prostaglandin sintesis
d. aspirin is also used for venous thrombosis
e. antiplatelet effect is permanent and last for all platelet life generally around 20-25 days
38) 36 years old man present in ER with renal colic. Rx demonstrate 1.5 cm stone in upper urinary track.
What is best treatment?

a. hydratation and analgesic therapy only is non effective


b. serial Rx are useful to follow the stone
c. it is associated with microematuria
d. after the resolution it is probable a recurrence of another episode of kidney stone.
e. increased level of BUN and creatinine are expected.

39) 25 years old with a painful full penis erection for several hours. Which of the next statements are true?

a. it can be associated with sickle disease.


e. it can be treated primarily with surgical corpospongiosal shunt

40) in 65 years old man what is the best test for differential diagnosis between septic arthritis and gout?

a.white blood cell count


e. evaluation of synovial fluid aspiration.

41.A 60 years old male was admitted for severe right abdominal pain,body temperature is 37 c.ultrasounography
shows an aconstic shadow in the common bile duct.tis condition is commonly associated with:

a.Elevated conjugated bilirubin


b.Elevated unconjucated bilirubin
c.decrease in alkaline phosphatase
d.very high levels of SGOT and SGPT

42.when choledocholithiasis is suspected by abdominal ultrasound the next most appropriate test or procedure will
be:

a.computed tomography (CT)


b.biliary scintigraphy
c.Endoscopic retrograde pancreaticography(ERCP)
d.Emergency surgery

43.a 70 year old man had abdominal ultrasound for the follow up of an abdominal aortic aneurism .the
presence of stones in the gallbladder is indentified.the patient suffers from some low back discomfort only, which is
thought to be attributed to the aneurysm. the appropriate next step in managing the gallstones is:

a.observation
b.laparoscopic cholecystectomy
c.open cholecystectomy
d.ultrasound of gallbladder

44.a 49 year old man was the restrained driver in a motor vehicle collision, he decelerated rapidly in order to avoid
hitting another car and swerved into a ditch. he complains of chest pain.
which of the following finding on chest X-Ray would be most suspicious for an aortic injury?

a.multiple right-sided rib fractures


b.a left pulmonary contusion
c.widening of the mediastinum greater than 8 cm
d.pnemomediastinum
45.The optimal time to give immunization to patients undergoing elective splenectomy is:

a.at least 2 months before surgery


b.at least 2 weeks before surgery
c.in the holding area just prior to surgery
d.on the day of discharge from the hospital after splenectomy

46.Spigelian hernias occur:

a.on the lateral border of the rectus abdominis


b.in the lines alba.
c.in the medial wall of the inguinal canal
d.in the femoral triangle
e.in the epigastrium

47.a 16 year old girl undergoes total colectomy for familial adenomatous polyposis (FAP).two years later ,there is a 4
cm asymptomatic abdominal wall mass in the incision. the appropriate management is:

a.Acourse of decarbazine and doxorubicin


b.a course of vincristine,actinomycin ,and cyclophosphamide
c.Radiation therapy to the area.
d.wide resection of the mass

48.symptoms of retroperitoneal sarcoma are usually produced by:

a.bleeding into the tumor mass.


b.compression of adjacent tissue
c.invasion of retroperitoneal organs.
d.metastasis to retropertoneal organs
e.metastasis to retroperitoneal lymph nodes

49. Twelve hours after having undergone a subtotal thyroidectomy ,a 30 year old woman develops agitation and
difficulty breathing.examination reveals tachycardia and anterior cervical swelling. the surgical dressing is dry.
the most appropriate treatment at this point would be immediate:

a.insertion of an orotracheal tube.


b.Re-opening of the cervical wound
c.determining the serum calcium concentration
d.administration of morphine
e.chest x-ray

50.The treatment of choice for a 1.8 cm in diameter N0,M0 invasive breast cancer in 50 years old women is:

a.lumpectomy alone.
b.lumpectomy,sentinel lymph node biopsy, and radiation.
c.mastectomy with sentinel lymph node biopsy and radiation.
d.mastectomy with axillary lymph node dissection and radiation
61) you are investigating the cause for a patient's anemia. he is a 50-year-old man who was found to have a
hematocrit of 25%on routine evaluation. his hemat ocrit was 47%1 year ago. mean corpuscular volume is 80,mean
corpuscular hemoglobin concentration Is 25.reticulocyte count is 5%.review of the peripheral blood smear shows
marked numbers of polychromatophilic macrocdytes. ferritin is 340mg/l. what is the cause of this patient's anemia?

A. defective erythroid marrow proliferation


B. extravascular hemolysis
C. intravascular hemolysis
D. iron-deficiency anemia
E. occult gastrointestinal BLEEDING

62) A 73-year-old man is admitted to the hospital with 3 weeks of malaise and fevers. His past medical
history is notable only for hypertension controlled with a thiazide diuretic. He smoke one pack of cigarettes per day
and works as an attorney. His physical examination is notable only for a new systolic heart murmur heard best in the
metral region. His laboratory examination is notable for mild anemia, an elevated white blood cell count, and oc-
casional rad blood cells on clean catch urine. Blood cul-tures grow Streptococcus bovis and echocardiogram shows a
>1-cm vegetation on the mitral valve. What additional evaluation is indicated for this patient?

A. Collonoscopy.
B. Head CT scan.
C. pulmonary embolism protocol CT scan.
D. Renal biopsy.
E. Toxicology screen.

63)A healthy 62_year_old woman returns to your clinic after undergoing routine colonoscopy. findings included
two 1.3_cm sessile(flat-based),villous adenomas in her ascending colon that were removed during the
procedure. what is the next step in management?

A. colonoscopy in 3 month
B. colonoscopy in 3 years
C. colonoscopy in 10 years
D. ct scan in the abdomen
E. partial colectomy
F. Reassurance

64) Which of the following statements about cardiac toxicity from cancer treatments is true?

a.Doxorubicin-based cardiac toxicity is idiosyncratic and does-independent.


b.Anthracycline-induced congestive heart failure is re-versible with time and control of risk factors.
c.Mediastinal irradiation often results in acute peri-carditis during the first few weeks of treatment.
d.Chronic constrictive pericarditis often manifests symptomatically up to 10 years after treatment.
e.The incidence of coronary atherosclerosis in pa-tients who have a history of madiastinal irradiation is the same as
that in age-matched controls.

65)The triad of portal vein thrombosis, hemolysis, and pancytopenia suggests which of the following
diagnoses?

a.Acute promyelocytic leukemia.


b.Hemolytic-syndrome (HUS).
c.Leptospirosis .
d.Paroxysmal nocturnal hemoglobinuria (PNH).
e.Thrombotic thrombocytopenia purpura (TTp).
66) Which of the following best describes the mechanism of action of clopidogrel?

A. Activates antithrombin and inhibits clotting enzymes.


B. binds to the activated GPIIb/IIIa receptor on the platelet surface to block binding of adhesive molecules.
C. inhibits cyclooxygenas 1 (COX-1) on platelets to decrease production of thromboxane A2.
D. inhibits phosphodiesterase to block the breakdown of cyclic adenosine monophosphate(cAMP)to inhibit platelet
activation.
E. irreversibly blocks p2y12 to prevent adenosine diphosphate(ADP)-induced platelet aggregation

67)which of the following symptoms is most suggestive of an esophageal mass?

A. Early satietly
B. liquid phase dysphagia only
C. Odynophagia with chest pain
D. Oropharyngeal dysphagia
E. solid phase dysphagia progressing to liquid phase dysphagia

68)which of the following carries the best disease prognosis with appropriate treatment?

A. Burkitt's lymphoma
B. diffuse large B cell lymphoma
C. follicular lymphoma
D. mantle cell lymphoma
E. nodular sclerosing Hodgkin's disease

69) A 48 year -old woman is admitted to the hospital with anemia and thrombocytopenia after
complaining of profound fatigue. her initial hemoglobin is 8.5 g/dl , hematocrit 25.7%, and platelet count
42,000/ml. her leukocyte count is 9540/ml, but 8% blast forms are noted on peripheral smear. A chromosomal
analysis show a reciprocal translocation of the long arms of chromosomes 15 and 17, t(15;17),and a diagnosis of acute
promyelocytic leukemia is made. the induced regimen of this patient should include which of the following drugs:

A. All-trans-retinoic acid(ATRA,or triretinoin)


B. arsenic
C. cyclophosphamide,daunorubicin,vinblastin,and prednisone
D. rituximab
E. whole-body irradiation

70)A 56-year-old patient inquires about screening for colon cancer. He has no risk factors for colon concer, other the
age. Which of the following statements is true regarding which screening test you recommend for this patient?

A. 50% of patients with a positive fecal occult blood testing have colon cancer.
B. One-time colonoscopy detects more advanced le-sions than one-time fecal occult blood testing with
sigmoidoscopy.
C. Perforation rates for sigmoidoscopy and colons-copy are equivalent.
D. sigmoidoscopy has not been shown to reduce mortality.
E. Virtual colonoscopy ia as effective as endoscopic colonoscopy for detecting >5 mm in size. (

71) A 46-year -old woman presents with new onset ascites and severe abdominal pain : a hepatic Doppler
examination reveals hepatic vein thrombosis . she also reports tea colored urine on occasion ,particularly in the
morning ,as well as recurrent worsening abdominal pain .On further evaluation ,she is found to have an
undetectable serum haptoglobin ,elevated serum lactase dehydrogenase ,hemoglobinuria and elevated reticulocyte A
peripheral smear shows no schistocytes .What is the most likely diagnosis ?

A- adenocarcinoma of the ovary


B- antiphospholipid syndrome
C- aplastic anemia
D- factor V leiden deficiency
E- paroxysmal nocturnal hemoglobinria
72) A 17-year-old woman presents to the clinic complaining of vaginal itchiness and malodorous discharge.She is
sexually active with multiple partners ,and she is interested in getting tested for sexually transmitted diseases .A wet-
mount microscopic is performed ,and trichomonal parasites are identified.
Which of the following statements regarding trichomoniasis is true?

A- a majority of women are asymptomatic


B- no treatment is necessary as disease is self-limited
C- the patient's sexual partner need not be treated
D- trichomoniasis can only be spread sexually
E - tricomoniasis is 100% sensitive to metronidazole

73) A 32-year-old man presents with jaundice and malaise .He is found to have acute hepatitis B with positive
hepatitis B virus (HBV) DNA and E antigen . Which of the following antiviral agents are approved as part of a
therapeutic regimen for mono-infection with hepatitis B ?

A- efavirenz
B- ganciclovir
C- lamivudine
D- rimantadine
E- tenofovir

74) the most common cause of traveler’s diarrhea is:

A-campylobacter jejuni
B- entamoeba histolytica
C- enterotoxigenic Escherichia coli
D- giardia lamblia
E- vibrio cholera

75) In the inpatient setting ,extended-spectrum beta-lactamase (ESBL)-producing gram-negative infection are most
likely to occur after frequent use of which of the following classes of antibiotics ?

A- carbapenems
B- macrolides
C- quinolones
D- third-generation cephalosporine

76) which of the following is true regarding influenza prophylaxis ?

A- patients receiving an intramuscular influenza vaccine should be warned of the increased of


GuillainBarre syndrome
B- patients with hypersensitivity to eggs should not receive the intramuscular vaccine
C- the intramuscular influenza vaccine is a live ,attenuated strain of influenza that is based on isolates from the
previous year's strains of influenza A and B
D- the intramuscular influenza vaccine should not be given to immunocompromised hosts
E- the intranasal spray ."flu-mist" ,is an inactivated virus preparation based on the previous year's strains of influenza
77) A patient presents to the clinic complaining of nausea, vomiting ,crampy abdominal pain ,and
markedly increased flatus . the patient has not experienced any diarrhea or vomiting but notes that he has been
belching more than usual and he describes a "sulfur-like" odor when he does so . he returned from a 3-
week trip to peru and ecuador several days ago and notes that his symptoms began about a week ago. Giardiasis is
considered in the differential .Which of the following is true regarding Giardia ?

A- boiling water prior to ingestion will not kill Giardia cysts


B- Giardia is a disease of developing nations ;if this patient had not travelled ,there would be no likelihood of
giardiasis
C- hematogenous dissemination and eosinphilia are common
D- ingestion of as few as 10 cysts can cause human disease
E- lack of diarrhea makes the diagnosis of Giardia very unlikely

78) Which of the following statements is true regarding ulcerative colitis ?

79) In a patient with bacterial endocarditis , which of the following echocardiographic lesions is most to lead to
embolization ?

A- 5-mm mitral valve vegetation


B- 5-mm tricuspid valve vegetation
C- 11-mm aortic valve vegetation
D- 11-mm mitral valve vegetation
E- 11-mm tricuspid valve vegetation

80) Several family members present to a local emergency room 2 days after a large family summer picnic where deli
meats and salads were severed . They all complain of profuse diarrhea ,headaches ,fevers ,and myalgias .Their
symptoms began ~24 h after the picnic . It appears that everyone who ate Aunt Emma's bologna surprise was afflicted
.Routine cultures of blood and stool are negative to date .
Which of the following is true regarding Listeria gastroenteritis ?

A- antibiotic treatment is not necessary for uncomplicated cases


B- carriers are asymptomatic but can easily spread infection via the fecal-oral route
C- gastrointestinal illness can result from ingestion of single organism
D- illness is toxin-mediated ,and organisms are not present at the time of infection
E-person-to-person spread is a common cause of out-breaks

81) A 19-years-old woman comes to your office after being bitten by a bat on the ear while camping in a primitive
shelter .she is unable to produce a vaccination record. On physical examination , she is afebrile and appears well .
There are two small puncture marks on the pinna of her left ear .
What is an appropriate vaccination strategy in this context ?

a.Intravenous ribavirin
b.No vaccination
c.Rabies immunoglobulins
d.Rabies inactivated virus vaccine
e.Rabies inactivated virus vaccine plus immunoglobulins

82) In a patient who has undergone a traumatic splenectomy, what test can be ordered to establish lack of splenic
function ?

a.CT scan of abdomen


b.Neutrophil migration studies
c.Peripheral blood flew cytometry
d.Peripheral blood smear
83) A person with liver disease caused by Schistosoma ansoni would be most likely to have:

a.ascites
b.esophageal varices
c.gynecomastia
d.jaundice
e.spider nevi

84) Which of the following represents an emergent (same day) indication for cardiac surgery in a patient with
infective endocarditis ?

a.Culture-proven fungal endocarditis


b.Culture-proven resistant organism with septic pulmonary emboli
c.Prosthetic valve endocarditis 4 months after surgery
d.Sinus of Valsalva abscess ruptured into right heart
e.Staphylococcus lugdunensis in a patient with previous history of endocarditis

85) A 30-years-old female is seen in the clinic before undergoing an esophageal dilation for a stricture. Her past
medical history is notable for mitral valve prolapse with mild regurgitation. She takes no medications and is allergic to
penicillin. Her physician should recommend which of the following?

a.Clarithromycin 500 mg PO 1 h before the procedure


b.Clindamycin 450 mg PO 1 h before the procedure
c.Vancomycin 1 g intravenously before the procedure
d.The procedure is low-risk , and therefore no prophylaxis is indicated
e.Her valvular lesion is low-risk , and therefore no prophylaxis is indicated

86) A 78-years-old male presents to the clinic complaining that every time he shaves with a straight razor,
he passes out. His symptoms have been occurring for the last 2 months. Occasionally, when he puts on a
tight collar, he passes out as well. The loss of consciousness is brief, he has no associated prodrome, and
he feels well afterward. His past medical history is notable for hypertension and hypercholesterolemia.
His only medication is hydrochlorothiazide. On physical exam his vital signs are normal, and his cardiac
exam is normal with the exception of a fourth heart sound. Which of the following is the most appropriate
next diagnostic test?

a.Stress echocardiography
b.Adenosine thallium scan
c.Computed tomogram of the neck
d.Carotid sinus massage
e.Tilt table test

87) A 37-years-old male with Wolff-Parkinson-White syndrome develops a broad-complex irregular tachycardia
at a rate of 200 beats per minute. He appears comfortable and has little hemodynamic impairment. Useful
treatment at his point might include:

a.Digoxin
b.Amiodarone
c.Propranolol
d.Verapamil
e.Direct-current cardioversion
88) Which of the following patients meets criteria for the diagnosis of the metabolic syndrome ?

a.A man with waist circumference of 110 cm , wellcontrolled diabetes mellitus with fasting plasma glucose of 98
mg/dL, and blood pressure of 140/75 mmHg
b.A woman with triglycerides of 180 mg/dL , waist circumference of 75 cm , and polycystic ovary syndrome
c.A man with nonalcoholic liver disease , obstructive sleep apnea , and blood pressure of 135/90 mmHg
d.A woman with high-density lipoprotein (HDL) of 54 mg/Dl , blood pressure of 125/80 mmHg , and fasting plasma
glucose of 85 mg/dL

89) When treating a patient with a non -ST-segment elevation myocardial infarction (NSTEMI), risk
stratification and timely administration of anti-ischemic and antithrombotic therapies are paramount. For a patient
with unstable angina with negative biomarkers.
which medication regimen is most appropriate an initial treatment?

a.Aspirin , beta blocker, spironolactone , HMG-CoA reductase inhibitor (statin)


b.Aspirin , clopidogrel , nitroglycerin , beta blocker, heparin
c.Aspirin , nitroglycerin , beta blocker, heparin , glycoprotein IIB/IIIa inhibitor
d.Aspirin , morphine , oxygen , nitrates

90) Which of the following associations correctly pairs clinical scenarios and community-acquired pneumonia
( C A P ) pathogens?

a.Aspiration pneumonia: Streptococcus pyogenes


b.Heavy alcohol use: atypical pathogens and Streptococcus aureus
c.Poor dental hygience: Chlamydia pneumoniae, klebsiella pneumoniae
d.Structural lung disease: Pseudomonas aeruginosa , S. aureus
e.Travel to southwestern United States: Aspergillus spp

91. Which of the following forms of hepatitis does NOT have a chronic state?

A) Hepatitis A
B) Hepatitis B
C) Hepatitis C
D) Hepatitis D

92. Which of the following organisms is responsible for the development of pseudomembranous colitis?

A) Escherichia coli
B) Clostridium difficile
C) Pseudomonas aeruginosa
D) Methicillin-resistant Staphylococcus aureus
E) Enterococcus faecalis

93. Which of the following medications is used for the treatment of pseudomembranous colitis?

A) Metronidazole
B) Amphotericin B
C) Ketoconazole
D) Acyclovir
E) Intravenous vancomycin
94. Which of the following statements about diabetic retinopathy is true?

A) Proliferative retinopathy is associated with a poorer prognosis than nonproliferative retinopathy.


B) Nonproliferative retinopathy is associated with neovascularization.
C) Symptoms of retinopathy usually begin with eye pain.
D) Diabetics should have eye examinations every 3 years.
E) Unfortunately, there is no treatment for diabetic retinopathy

95.Which of the following signs and symptoms are associated with Sjogren's syndrome?

A) Hepatomegaly, chronic rhinitis, and palmar erythema


B) Keratoconjunctivitis, parotid gland enlargement, and xerostomia
C) Confusion, tremors, and peripheral neuropathies
D) Polycythemia, leukocytosis, and negative rheumatoid factor
E) Hyperextensibility of joints, iriditis, and glossitis

96.Which of the following laboratory results best support the diagnosis of subclinical hypothyroidism?

A) Normal T4, low TSH


B) Normal T4, high TSH
C) Low T4, high TSH
D) Normal T4, normal TSH
E) Low T4, borderline low TSH

97. A high urinary sodium concentration in the presence of a low plasma osmolality is most closely associated with:

A) syndrome of inappropriate antidiuretic hormone (SIADH)


B) third-degree burns
C) hyperglycemia
D) prolonged diarrhea
E) water overload

98.Side effects of 3-hydroxy-3-methylglutaryl coenzyme A (HMG CoA) reductase inhibitors include:

A) chronic rhinitis
B) infertility
C) rhabdomyolysis
D) orthostatic hypotension
E) cataracts

99.A 24-year-old poorly controlled diabetic patient presents to your emergency room with ketoacidosis, fever, pain,
and purulent drainage of the sinuses with black eschar formation affecting the nasal septum. The most likely
diagnosis is:

A) mucormycosis
B) cocaine use
C) chronic sinusitis
D) Pseudomonas infection
E) Staphylococcus infection
100.A 62-year-old business executive with a history of migraines is noted to have hypertension; otherwise, he
is healthy. Which of the following is the best medication for the treatment of his mhypertension?

A) Beta Blocker
B) Angiotensin-receptor blocker
C) Alpha Blocker
D) ACE inhibitor
E) Thiazide diuretic
F) alcohol withdrawal

101. Which of the following is an effect of niacin use?

A) Increased LDL cholesterol


B) Increased triglyceride levels
C) Increased HDL cholesterol
D) Hypoglycemia
E) Decreased uric acid levels

102. A 60-year-old alcoholic man, currently being treated for gastritis, presents to the office with painful breasts that
appear enlarged. The most likely cause is:

A) breast cancer
B) excessive calcium carbonate ingestion
C) cimetidine use
D) trauma
E) prolactinoma

103. A 60-year-old man presents with pain, swelling, and redness of the first metatarsal phalangeal joint. His
cardiologist recently prescribed hydrochlorothiazide for his hypertension. The most likely diagnosis is:

A) podagra
B) degenerative joint disease
C) rheumatoid arthritis
D) osteomyelitis
E) Morton's neuroma

104. Which of the following values is acceptable for a 65-year-old man who had a previous coronary artery bypass
graft?

A) Total cholesterol of 215 mg/dL


B) HDL cholesterol of 32 mg/dL
C) LDL cholesterol of 68 mg/dL
D) Triglycerides of 228 mg/dL
E) Blood glucose of 130 mg/dL

105. A blood urea nitrogen (BUN): creatinine level greater than 20 is associated with:

A) dehydration
B) renal stones
C) bladder outlet obstruction
D) hypercalcemia
E) renal artery stenosis
106. Which of the following is the most appropriate medication for the treatment of hypertension in a diabetic
patient?

A) Beta Blocker
B) ACE inhibitor
C) Diuretic
D) Calcium-channel blocker
E) Alpha Blocker

107. A 60-year-old woman presents with complaints of diffuse proximal muscle pain, low-grade fevers, and
generalized fatigue. Laboratory findings include an elevated ESR and mild anemia.
The most likely diagnosis is:

A) influenza
B) dermatomyositis
C) polymyalgia rheumatica
D) systemic lupus erythematosis
E) rheumatoid arthritis

108. Which of the following is associated with pseudogout?

A) Calcium pyrophosphate crystal deposition in the large joints


B) Lack of response with the use of colchicine
C) High uric acid levels
D) Lack of response with the use of anti-inflammatory medication
E) Negative birefringence seen with polarized microscopy

109. a 65 year old man complains of gynecomastia and galactorhea with erectile dysfunction.
the most likely diagnosis is:

a)breast cancer
b)testicular cancer
c)prolactinoma
d)adrenal adenoma
e)diabetes mellitus

110. Which of the following conditions is often the first sign of amyloidosis?

A) Proteinuria
B) CHF
C) Cardiac arrhythmias
D) Rheumatoid arthritis
E) Night blindness

You might also like